Fisica General i Una

384

Transcript of Fisica General i Una

Page 1: Fisica General i Una

3

Page 2: Fisica General i Una

4

CAV QC21.2 Física general I / [Elaborado por] Rafael López Frontado. - - 2da ed. F582 . - - Caracas : UNA, 2011. 2011 389 p. : il. ; 29 cm. ISBN 978-980-236-699-6 Carrera de Ingeniería / Carrera de Matemática

1. Física. 2. Educación a distancia - - Modulo de estudio. I. Universidad Nacional Abierta. II. López Frontado, Rafael.

Todos los derechos reservados. Prohibida la reproducción total o parcial por cualquier medio gráfico, audiovisual o computarizado, sin previa autorización escrita. Universidad Nacional Abierta Apartado Postal Nº 2096 Caracas 1010 A, Carmelitas, Venezuela Copyright © UNA 2011 Primera edición, 2008 Segunda edición, 2011 ISBN 978-980-236-699-6 If 13520115302683 Registro de Publicaciones de la Universidad Nacional Abierta Nº UNA – 2011 – 5961

Page 3: Fisica General i Una

5

Page 4: Fisica General i Una

Cómo citar este documento:

López Frontado, R. (2011). Física general I, Texto UNA. 2da edición. Caracas: UNA

Page 5: Fisica General i Una

7

INDICE MÓDULOS y UNIDADES

CONTENIDO PÁGINA

INTRODUCCIÓN 9 MÓDULO I

Cinemática de la partícula

19

UNIDAD 1.-

Movimiento de una partícula

21

UNIDAD 2.-

Tipos de movimientos

63

MÓDULO II

Dinámica de la partícula

103

UNIDAD 3.-

Leyes de newton y aplicaciones

105

UNIDAD 4.-

Trabajo y energía

151

MÓDULO III

Dinámica de sistemas de partículas

179

UNIDAD 5

Momento lineal y angular

181

UNIDAD 6

Trabajo y energía para sistemas de partículas

227

UNIDAD 7

Cuerpos rígidos

255

UNIDAD 8

Movimiento gravitatorio

293

UNIDAD 9

Movimiento oscilatorio

323

UNIDAD 10

Fluidos

359

BIBLIOGRAFÍA 389

APÉNDICE A Sistemas de medidas 391

APÉNDICE B Como trabajar con Unidades en Física 395

GLOSARIO 397

Page 6: Fisica General i Una

9

I.- INTRODUCCIÓN En el proceso de actualización del libro texto para la asignatura Física

General I, código 300, se ha escogido una metodología para la elaboración del presente texto, tomando en cuenta diversas razones.

Entre estas su antigüedad, siendo conveniente hacerle ciertas revisiones . Otra de las razones, es que en varios centros locales, y quizás en algunas

oficinas de apoyo, no existen asesores de la asignatura, lo cual hace que el estudiante se encuentre desasistido, sin poder recurrir a alguien con quien consultar sobre determinado tema.

En este texto de la asignatura Física General I se proporciona al estudiante

una secuencia de seguimiento del curso a través de los diferentes temas, para que así pueda comprender mejor los conceptos y principios contenidos en los diversos tópicos de la asignatura, facilitándole de esta manera el aprendizaje global de la misma.

También se le ofrece al estudiante una serie de ejercicios resueltos en cada

unidad, que lo ayudan en la adquisición de destrezas para resolver los diferentes problemas, brindándosele recomendaciones o sugerencias sobre la forma de enfrentar y dar solución a situaciones problemáticas inherentes a esta asignatura. En tal sentido se le proporcionan una serie de ejemplos resueltos, ejercicios propuestos, ejercicios de autoevaluación y preguntas conceptuales, cuya lectura y comprensión, ayudará al estudiante en el proceso de aprendizaje.

Como complemento de esta impresión se han incluido al final del texto los

apéndices A,B y un glosario. En el apéndice A se le proporciona al estudiante tablas de valores de algunas unidades y equivalencias de las mismas, y en el apéndice B se muestra la manera de resolver un problema, tomando en cuenta las unidades a usar. En el glosario se le presenta una lista de términos a utilizar a lo largo del texto.

En el texto se resaltan en negrita aquellos conceptos donde se quiere que el

estudiante ponga mayor atención durante su instrucción En la elaboración de este texto, otro de los aspectos tomados en cuenta es

despertar la curiosidad del estudiante para que pueda adquirir con más profundidad los conceptos y principios esbozados en los diferentes temas de la asignatura, contenidos en el Plan de curso de la materia. Para ello se han seleccionado los siguientes textos:

1. Física para Ciencias e Ingeniería, Serway R. A., Beichner R. J., (2001),

Tomo I, (4ª Edic) México, McGraw Hill.

López Frontado, R. (2011). Física general I, Texto UNA. 2da edición. Caracas: UNA

Page 7: Fisica General i Una

10

2. Física universitaria, Young H. D., Freedman R. A., (Sears y Zemansky) (1999), Tomo I, (9ª Edic), México, Addison Wesley Longman.

3. Física, Resnick, Halliday y Krane, (1996), Tomo I, (4ª Edic), México, Compañía Editorial Continental S.A. (CECSA). La presente selección de estos textos se ha realizado tomando en cuenta el

nivel académico que presentan, el cual es muy aceptable para la asignatura, ya que ofrecen una presentación didáctica y pedagógica muy apropiada, además de resaltar en los diferentes temas lo más importante, con variados colores y tonalidades. Proporcionan también un gran número de ejemplos, relacionados con cada uno de los tópicos tratados en cada capítulo. De igual manera están bastante actualizados en lo referente a los avances científicos y tecnológicos que tienen relación con esta asignatura.

Con el fin de que el estudiante sea más eficaz en el uso del texto, se

presentan además una serie de íconos y sus significados se explican a continuación. Estos íconos se muestran en la siguiente tabla:

Iconos empleados en el material instruccional

AMPLIACIÓN DE CONOCIMIENTOS: está dirigido al estudiante que desea profundizar más en sus conocimientos en determinado tema.

ATENCIÓN: se presenta cuando se quiere hacer una aclaratoria, una advertencia o una reflexión sobre algún aspecto del contenido.

CASO DE ESTUDIO: es la exposición de una situación muy similar a la realidad a la cual se le dará solución.

CONSULTA EN LA WEB: indica referencias a páginas Web

CONSULTA EN OTROS LIBROS: se refiere a un llamado a consulta en libros que no figuran como textos de carácter obligatorio para el curso.

EJERCICIOS Y ACTIVIDADES PROPUESTAS: son ejercicios o actividades sugeridas a manera de práctica sobre algún tema de la unidad.

López Frontado, R. (2011). Física general I, Texto UNA. 2da edición. Caracas: UNA

Page 8: Fisica General i Una

11

EJERCICIOS DE AUTOEVALUACIÓN: ejercicios que debe realizar el estudiante y posteriormente verificar contra los resultados aquí presentados.

EJEMPLO: es la exposición de un caso alusivo al tema en cuestión y su resolución.

RECORDATORIO: indica algún aspecto a ser enfatizado, relacionado con los conocimientos adquiridos previamente por el estudiante.

Como una recomendación adicional para facilitarle al estudiante el proceso de

aprendizaje, en cuanto al de la Física se refiere, es conveniente tomar en cuenta las ideas esbozada por el profesor Mark Hollabaugh, del Normandale Community Collage, cuyo extracto se presenta a continuación:

Preparación para este curso Para el que haya cursado física en Bachillerato, es recomendable que prepare

un glosario de los nuevos términos que se presenten, asegurándose de comprender cómo se utilizan en física.

Aprender a aprender Como cada uno de nosotros tiene un estilo propio y una forma de aprendizaje

preferido, es conveniente comprender cuál de su propio estilo de aprendizaje le ayudará a concentrar su atención en los aspectos de la física que podrían ofrecer dificultad, y utilizar los componentes de su curso que le permitirán superar un escollo con más facilidad:

a) Si aprende escuchando, le será provechoso trabajar con el asesor de

su centro local para que le explique. b) Si aprende explicando, le será provechoso trabajar con otros

estudiantes. c) Si se le dificulta resolver problemas, dedique más tiempo a aprender

cómo resolver problemas. d) Es importante comprender y adquirir buenos hábitos de estudios

López Frontado, R. (2011). Física general I, Texto UNA. 2da edición. Caracas: UNA

Page 9: Fisica General i Una

12

e) Quizás lo más importante que puede hacer por sí mismo es dedicar un

tiempo adecuado de estudio y programado con regularidad en un ambiente libre de distracciones.

Responda las preguntas siguientes con respecto a su persona:

• ¿Soy capaz de utilizar los conceptos matemáticos fundamentales de álgebra, geometría y trigonometría? (De no ser así, organice un programa de repaso con ayuda de su profesor asesor).

• En cursos similares, ¿qué actividad me ha causado mayor dificultad?¿Cuál ha sido la más fácil para mi?

• ¿Comprendo mejor el material si leo el libro antes o después del asesoramiento?

• ¿Dedico suficiente tiempo al estudio de la física? ¿Estudio física todos los días?

• ¿Trabajo en un lugar tranquilo donde puedo mantener la atención? Trabajar con otros Los científicos o los ingenieros rara vez trabajan aislados unos de otros, más

bien suelen colaborar entre ellos. Usted aprenderá mejor y se divertirá más al trabajar con otros estudiantes.

Secciones de clases y notas Un componente importante de todo curso universitario es la sesión de

asesoramiento. En física esto reviste una importancia especial, porque su profesor asesor le puede hacer con frecuencia demostraciones de los principios físicos y ejecutará una serie de actividades que faciliten la comprensión de los principios básicos de la física. Procure no faltar a las asesorías.

No olvide hacer preguntas sobre los temas que le presenten mayor dificultad cuando vea a su asesor en las horas previstas. Recuerde que la única pregunta “tonta” es la que no se hace.

Exámenes Presentar exámenes provoca tensiones. Pero si usted se siente bien

preparado y descansado, el estrés será menor. Prepararse para los exámenes es un proceso continuo, que se inicia en el momento que se ha concluido el último examen. Debe revisar de inmediato el examen y entender los errores que haya cometido.

López Frontado, R. (2011). Física general I, Texto UNA. 2da edición. Caracas: UNA

Page 10: Fisica General i Una

13

Cómo usar el texto Al inicio de todo texto se encuentra una parte llamada “Introducción” o

“Prefacio”, donde el autor o los autores indican cómo usar el texto para obtener mejor provecho en el proceso de aprendizaje. Así se incluyen preguntas iniciales de capítulo y preguntas de evalúe su comprensión; ejemplos resueltos; estrategias para resolver problemas; términos clave, resumen y problemas.

ESTRATEGIAS (O SUGERENCIAS) PARA RESOLVER PROBLEMAS Los profesores de física abordan los problemas de una manera sistemática y

lógica. El estudiante se ayudará siguiendo una estrategia sistemática para resolver problemas llamada IPEE (Identificar, Plantear, Ejecutar y Evaluar). Estudie esta sugerencia con gran detenimiento y póngalas en práctica. Este sistema IPEE consta de:

ESTRATEGIA PARA RESOLVER PROBLEMAS

IDENTIFICAR los conceptos relevantes: Primero decida qué ideas de la física son relevantes para el problema. Este proceso no implica cálculos, pero a veces resulta la parte más difícil; es muy importante, no lo omita. También se debe identificar la incógnita del problema, aunque algunas veces se presentan más de una incógnita. Esta variable es la meta del proceso de resolución de problemas; asegúrese no perderla de vista durante los cálculos. PLANTEAR el problema: Si resulta apropiado, dibuje la situación descrita en el problema. Con base en los conceptos que escogió en el paso de Identificar, selecciones las ecuaciones que usará para resolver el problema y decida cómo la usará. EJECUTAR la solución: En este paso, se hacen las cuentas. Antes de hacer los cálculos, haga una lista de las cantidades conocidas y desconocidas, e indique cuál o cuáles son las incógnitas a determinar. Después despeje las incógnitas de las ecuaciones. EVALUAR la repuesta: La meta de la resolución de problemas en física no es sólo obtener un número o una fórmula; es entender mejor. Ello implica examinar la repuesta para ver que nos dice. En particular pregúntese: ¿es lógica esta repuesta? Revise su trabajo y modifique la solución según sea necesario.

López Frontado, R. (2011). Física general I, Texto UNA. 2da edición. Caracas: UNA

Page 11: Fisica General i Una

15

II.- DESCRIPCIÓN DEL CURSO

El curso de Física General I constituye la primera parte del bloque de las

ciencias básicas relacionadas con la Física, y como tal es la ciencia que estudia los fenómenos naturales, analizando los componentes de la materia y sus interacciones mutuas.

Es un curso teórico-práctico, que le proporciona al estudiante de ingeniería los

conceptos básicos fundamentales necesarios para una mejor comprensión de los fenómenos a estudiar durante su carrera.

El curso proporciona una explicación sistemática de los fenómenos naturales,

aplicando los conceptos y principios básicos fundamentales de la mecánica clásica, necesaria para que el estudiante pueda comprender los conocimientos que vaya a adquirir en otras asignaturas subsiguientes del pensum de su carrera.

En la Universidad Nacional Abierta, el curso de Física General I está incluido

en el pensum de las carreras de Ingeniería: de Sistemas e Industrial; y en las carreras de Licenciatura en Matemática y Licenciatura de Educación mención Matemática, como un curso básico obligatorio, una vez que el estudiante haya adquirido un conocimiento previo y la destreza del cálculo diferencial e integral.

OBJETIVO DEL CURSO

Aplicar de manera analítica sistemática, creativa y lógica, los conceptos y principios básicos de la mecánica clásica a los fenómenos naturales.

CONTENIDO

MÓDULO I.- CINEMÁTICA DE LA PARTÍCULA. MÓDULO II.- DINÁMICA DE LA PARTÍCULA. MÓDULO III.- DINÁMICA DE SISTEMAS DE PARTÍCULAS y

APLICACIONES.

López Frontado, R. (2011). Física general I, Texto UNA. 2da edición. Caracas: UNA

Page 12: Fisica General i Una

17

MÓDULO I CINEMÁTICA DE LA PARTÍCULA

MÓDULO II DINÁMICA DE

LA PARTÍCULA

MÓDULO III SISTEMAS DE PARTÍCULAS

Unidad 1 MOVIMIENTOS DE

PARTÍCULAS

Unidad 2 TIPOS DE MOVIMIENTOS

Unidad 3 LEYES DE NEWTON

Unidad 7 CUERPOS RÍGIDOS

Unidad 8 GRAVITACIÓN

UNIVERSAL

Unidad 9 MOVIMIENTOS OSCILATORIOS

Unidad 10 FLUIDOS

FÍSICA GENERAL I

Unidad 4 TRABAJO Y ENERGÍA

Unidad 5 DINÁMICA DE SISTEMAS

DE PARTÍCULAS

Unidad 6 TRABAJO Y ENERGÍA PARA SISTEMAS DE

PARTÍCULAS

ESQUEMA DE LA ASIGNATURA

López Frontado, R. (2011). Física general I, Texto UNA. 2da edición. Caracas: UNA

Page 13: Fisica General i Una

19

MÓDULO I

CINEMÁTICA DE LA PARTÍCULA

En este módulo se estudia, a través de la Cinemática, el movimiento de los cuerpos sin tomar en cuenta las causas que lo producen. Para una mejor comprensión consideramos a los cuerpos representados como Cuerpos puntuales, llamados simplemente partículas.

Para el estudio del movimiento de la partícula se incluyen temas necesarios

para el mejor entendimiento y desarrollo de todo el curso. Se inicia el módulo refiriéndose a la manera de cuantificar y medir las

magnitudes físicas, para lo cual se exponen los conceptos fundamentales sobre medidas, tales como conceptos de unidad, sistemas de unidades, etc.

Existen muchas magnitudes que poseen un carácter vectorial, así como

también se incluyen temas relacionados con el álgebra y análisis vectorial; aunque el estudiante ya conoce algunos de cursos anteriores, otros son nuevos pero necesarios para una descripción apropiada de los fenómenos mecánicos.

El estudio del movimiento, que es el propósito de la cinemática, supone que se

considere la descripción geométrica en el transcurrir del tiempo, y es donde “nacen” los conceptos de velocidad y aceleración de un cuerpo.

Una vez concluido este módulo, el estudiante estará en capacidad de

comprender los siguientes temas de la dinámica.

OBJETIVO

Aplicar los conceptos de sistemas, modelos, magnitudes, medidas, unidades, velocidad y aceleración en la resolución de problemas inherentes al movimiento de una partícula.

ESTRUCTURA DEL MÓDULO

UNIDAD 1.- MOVIMIENTO DE UNA PARTÍCULA UNIDAD 2.- TIPOS DE MOVIMIENTOS

López Frontado, R. (2011). Física general I, Texto UNA. 2da edición. Caracas: UNA

Page 14: Fisica General i Una

21

UNIDAD 1

MOVIMIENTO DE UNA PARTÍCULA

Esta unidad se refiere al estudio de la naturaleza de la Física, y el uso de

modelos idealizados para representar sistemas físicos. En todos los fenómenos inherentes a los fenómenos físicos intervienen

magnitudes cuyas cantidades han de compararse, razón por la cual se establecen medidas que se pueden interpretar con facilidad a través de un lenguaje universal. Por eso se definen los sistemas de unidades, necesarios para la descripción cuantitativa de los fenómenos físicos.

Muchas de las magnitudes estudiadas en física tienen la característica

vectorial, tales como velocidad, aceleración, fuerza, campos, etc., las cuales necesitan que se especifiquen su dirección y sentido. Por estas razones se han introducido los conceptos de magnitudes vectoriales.

El cálculo vectorial implica relaciones numéricas y geométricas, permitiendo

así resolver eficazmente multitud de problemas que plantean la ciencia y la técnica de nuestro tiempo.

Existen movimientos en todo nuestro alrededor. Lo vemos en muchas de las

actividades cotidianas de las personas: en el caminar de las mismas a lo largo de una calle, en los autos que pasan por la carretera, en el vuelo de los aviones, en el vuelo de las aves. El movimiento está en todas partes.

Es fácil reconocer el movimiento, pero no lo es así el describirlo. Para ello es

importante entender el concepto de razón de cambio. La razón de cambio es una cantidad dividida entre el tiempo, y nos dice qué tan rápido ocurre un fenómeno, o cuánto cambia una cantidad en cierto intervalo de tiempo. En esta unidad aprenderás que el movimiento se describe en función de las razones de cambio, conocidas como rapidez, velocidad y aceleración.

Para el estudio del movimiento de los cuerpos, los cuales se representan

como cuerpos puntuales (llamados también partículas), se usan los conceptos vectoriales, de posición, de velocidad y de aceleración. También se explican los conceptos de vector desplazamiento y distancia entre dos puntos, aclarándose la diferencia entre desplazamiento y distancia.

Al estudiar el vector velocidad, se explican los conceptos de velocidad media,

velocidad instantánea y rapidez (magnitud de la velocidad), aclarándose la diferencia entre velocidad media y velocidad instantánea.

Al explicar el concepto de vector aceleración, se incluyen los conceptos de

aceleración media y de aceleración instantánea.

López Frontado, R. (2011). Física general I, Texto UNA. 2da edición. Caracas: UNA

Page 15: Fisica General i Una

22

Por último se explican lo que se conoce como problemas directos y problemas inversos.

CONOCIMIENTOS PREVIOS: para que el estudiante comprenda correctamente el contenido de esta unidad, debe tener los siguientes conocimientos:

1.- Operaciones algebraicas elementales, inclusive con vectores. 2.- Conceptos de modelo, hipótesis y teorías físicas. 3.- Conocer las operaciones trigonométricas. 4.- Aplicar la semejanza de triángulos y el teorema de Pitágoras. 5.- Conocer las ecuaciones de una recta en forma continua. 6.- Comprender los conceptos de magnitud. 7.- Comprender los conceptos de límite. 8.- Integración inmediata de las funciones elementales. 9.- Vector derivada.

1.1 OBJETIVO:

Obtener la posición, velocidad y aceleración de una partícula a partir de sus condiciones iniciales. 1.2 RECOMENDACIÓN PARA EL ESTUDIO DEL CONTENIDO

El estudiante en esta unidad debe comprender claramente los conceptos

básicos siguientes: Magnitudes. Unidades. Sistemas de Unidades. Cantidades Vectoriales.

Posición de la Partícula. Vector posición. Vector desplazamiento. Vector velocidad. Vector aceleración.

López Frontado, R. (2011). Física general I, Texto UNA. 2da edición. Caracas: UNA

Page 16: Fisica General i Una

23

Lo relacionado con las mediciones en Física, Sistemas de Unidades y Vectores y sus aplicaciones está bien explicado en los temas correspondientes de la bibliografía recomendada. Para una mejor comprensión del contenido de la unidad, se recomienda al estudiante que realice la lectura de los conceptos relacionados con el movimiento de una partícula y de los ejemplos presentados, y utilice las técnicas de lectura que le facilita la comprensión del contenido. Dé respuesta a las preguntas propuestas y consulte a los profesores que administran y asesoran presencialmente en su Centro Local, o en su defecto, a través del correo electrónico (suministrado por la carrera).

1.3 CONTENIDO: 1.3.1 NATURALEZA DE LA FÍSICA. 1.3.2 MEDICIONES FÍSICAS, SISTEMAS DE UNIDADES. 1.3.3 VECTORES. 1.3.4 MOVIMIENTO DE UNA PARTÍCULA: VECTORES POSICIÓN, VECTOR

DESPLAZAMIENTO, VELOCIDAD Y ACELERACIÓN. ECUACIÓN DE LA TRAYECTORIA.

1.3.5 OPERACIONES INVERSAS. 1.3.1 NATURALEZA DE LA FÍSICA

La Física es una ciencia fundamentada en las observaciones experimentales y mediciones cuantitativas. El principal objetivo de la física es determinar el limitado número de leyes que gobiernan los fenómenos naturales para desarrollar teorías que permitan predecir los resultados de futuros experimentos. Las leyes de la física se expresan en lenguaje matemático, herramienta que proporciona un puente entre la teoría y el experimento.

Toda la física puede dividirse en: física clásica y física moderna. La física clásica está constituida por la física desarrollada hasta antes de 1900. El aporte más importante a la física clásica fue hecha por Isaac Newton con sus leyes aplicada a la mecánica clásica (llamada también simplemente mecánica), siendo además uno de los creadores del cálculo como una herramienta matemática. Los desarrollos de la termodinámica, la electricidad y magnetismo se realizaron a fines del siglo XIX.

López Frontado, R. (2011). Física general I, Texto UNA. 2da edición. Caracas: UNA

Page 17: Fisica General i Una

24

La física clásica está constituida por

las teorías, conceptos, leyes y experimentos en mecánica clásica, la termodinámica, óptica y electromagnetismo clásico.

Como efecto del uso de los principios que constituyen la física clásica, la

tecnología correspondiente produjo aparatos mecánicos y algo de aparatos eléctricos y electromecánicos.

Se aprovecharon las fuentes de energía convencionales, tales como el carbón,

petróleo y energía hidráulica, desarrollándose así el transporte rápido por tierra, aire y mar, incluyéndose el viaje a través del espacio.

La física moderna corresponde a la era de la física que se inicia a partir de los últimos años del siglo XIX, siendo los desarrollos más importante de la física moderna, la teoría de la relatividad de Einstein y la mecánica cuántica.

La teoría de la relatividad de Einstein y la física cuántica son teorías con grandes éxitos, los más efectivos que se conocen en la extensa gama de fenómenos que abarcan. La teoría de la relatividad revolucionó por completo los conceptos tradicionales de espacio, tiempo y energía. La teoría cuántica ofrece los mismos resultados que la teoría de Maxwell sobre la electricidad y magnetismo en los casos frecuentes en que muchos electrones y fotones toman parte; también produce los mismos resultados que las leyes de Newton en los casos en que h (constante de Plank) es considerara muy pequeña y la naturaleza ondulatoria de la materia no afecte los resultados.

Por lo tanto, la teoría relativista de Einstein y la física cuántica pueden producir el resultado de muchos experimentos con gran exactitud; razón por la cual ninguna otra teoría, que sea tan amplia y sencilla, puede comparársele; no obstante, a juzgar por la historia de las teorías científicas, no se les pueden considerar como la última palabra.

Como aplicaciones de los conocimientos adquiridos en la física moderna se tiene: los procesos nucleares, manejados como grandes fuentes de energía (aunque deben manejarse con precaución); el uso de los principios cuánticos en el microscopio electrónico y el láser; el uso de aparatos compactos debido a los transistores y circuitos electrónicos; el desarrollo de la informática debido al uso de la computadora; las predicciones estadísticas y la toma de decisiones por control automático.

! !" "

"" #"" "# "$"" !" #" $$

MecánicaTermodinámica

ClásicaÓptica

FÍSICAElectromagnetismo

Teoría de la relatividadModerna

Mecánica Cuántica

López Frontado, R. (2011). Física general I, Texto UNA. 2da edición. Caracas: UNA

Page 18: Fisica General i Una

25

Basado en los conceptos planteados hasta el momento, responda la siguiente

pregunta.

Pregunta 1.1.-

¿Cuáles, entre las siguientes actividades, están más cerca de ser tecnología (T) y cuáles de ser ciencia (C)?

a) reparar un televisor, b) diseñar un televisor, c) investigar las causas del

cáncer, d) tratar el cáncer de un paciente, e) estudiar el comportamiento de una rata, f) manejar un automóvil. 1.3.2 MEDICIONES FÍSICAS

Las leyes de física se expresan en cantidades fundamentales que requieren una definición clara. En mecánica, las cantidades fundamentales son: longitud (L), masa (M), y tiempo (s). Las otras cantidades de la mecánica, llamada derivadas, se pueden expresar en función de estas tres. En 1960 un comité internacional (DGPM) estableció un conjunto de patrones para la longitud, la masa y otras cantidades fundamentales. Este sistema se integró como una adaptación del sistema métrico, y se denominó Sistema SI, llamado Sistema Internacional. En este sistema SI, las unidades de longitud, masa y tiempo son: metro (m), kilogramo (kg) y segundo (s), respectivamente. Otras unidades patrón establecidas por el comité son: grado kelvin (K) para la temperatura, ampere (A) para la corriente, y mol para la cantidad de moléculas de una sustancia.

En el sistema SI, las definiciones de las unidades patrón son:

Longitud La unidad patrón es el metro (m), y se define como la distancia recorrida por la

luz en el vacío durante un tiempo de 1 s.2 997 924 458

Masa La unidad patrón es el kilogramo (kg), y se define como la masa de un cilindro

determinado de aleación de platino-iridio que se conserva en el Laboratorio Internacional de Pesas y Medidas en Sèvres, Francia.

López Frontado, R. (2011). Física general I, Texto UNA. 2da edición. Caracas: UNA

Page 19: Fisica General i Una

26

Tiempo La unidad patrón es el segundo (s), y se define como 9 192 631 770 veces el

período de vibración de la radiación del átomo de Cesio 133.

Se observa que el Sistema Internacional de unidades (SI) usa como base de unidades al kilogramo (kg) para la masa, el metro (m) para la longitud y el segundo (s) para el tiempo; y dispone de un método para designar unidades mayores o menores, a través de prefijos unidos a las unidades básicas y derivadas por factores de varias potencias de diez. La tabla (1.1) ofrece un resumen de estas unidades y sus abreviaturas.

Tabla 1.1

Prefijos para unidades SI

Símbolo Nombre Valor T tera 1012

G giga 109

M mega 106

k kilo 103

↓ ↓ ↓ d deci 10-1

c centi 10-2

m mili 10-3

µ micro 10 6

n nano 10-9

p pic 10-12

Así se tiene, como ejemplo, que un microcoulomb (1 µC) es igual a 1x10-6 C,

y un megawatt (1 MW) es igual a 1x106 W.

Ejemplos de cantidades derivadas:

Densidad Como ejemplo de las cantidades derivadas, se tiene a la densidad de

cualquier sustancia, la cual se representa por la letra ρ (rho), y se define su masa m por unidad de volumen V. Matemáticamente se representa por

m= (1.1)V

!

López Frontado, R. (2011). Física general I, Texto UNA. 2da edición. Caracas: UNA

Page 20: Fisica General i Una

27

Rapidez

Otro ejemplo de las cantidades derivadas es la rapidez de un cuerpo, el cual se define como la distancia recorrida por el cuerpo en un intervalo de tiempo dado. Esto es,

distancia recorridarapidez= (1.2)intervalo de tiempo

ANÁLISIS DIMENSIONAL

La dimensión de una magnitud es una propiedad que la define, y denota la naturaleza física de una cantidad.

Así se tiene que x = y, no tiene sentido si x representa una distancia mientras que y representa un tiempo; esto quiere decir que para que x = y, es necesario que tanto x como y deben representar cantidades que tienen las mismas dimensiones.

Igualmente se tiene que si y = x + z, entonces x y z deben tener la misma dimensión de y. Cuando se usan las dimensiones, sólo se asignan dimensiones a las variables, a los números no se le asignan dimensiones.

Las dimensiones adaptadas a las cantidades fundamentales son: L para la longitud, M para la masa y T para el tiempo. Las dimensiones para algunos de las

cantidades derivadas, tales como la de la velocidad es LT

, la de la aceleración es

2

LT

. Para dar una idea del uso de las dimensiones, veamos el siguiente: ejemplo, sea

la siguiente ecuación 2atx=vt+2

, al ser expresada dimensionalmente, se tiene que

2

2

L L T LL= T+ L=L+T T 2 2

! , se observa que las dimensiones de todos los términos

de la ecuación son iguales.

Ahora veamos este otro ejemplo, sea la ecuación 2 2 2ov =v +2ax . Al sustituir sus

dimensiones, se tiene que

2 2 2 2 3

22 2 2 2 2 2

L L L L L L= + L = +T T T T T T

→ , de donde se observa que todos los términos de

la ecuación dimensional no son iguales; por lo tanto se puede comprobar que la ecuación 2 2 2

ov =v +2ax no es correcta.

López Frontado, R. (2011). Física general I, Texto UNA. 2da edición. Caracas: UNA

Page 21: Fisica General i Una

28

De estos ejemplos se puede concluir que: el uso de las dimensiones permite comprobar si una ecuación física es correcta o no.

UNIDADES

Los términos de una ecuación, además de tener las mismas

dimensiones, se deben medir con la misma escala. Esto significa que si cada sumando de una ecuación puede ser longitud, todos los sumandos deben estar medidos en metro (m) o en centímetro (cm), pero no medir unos en metro (m) y otros en cm. 1.3.3 VECTORES

En la física existen algunas cantidades que se pueden describir plenamente con un número (o magnitud) y una unidad, tales como tiempo, temperatura, masa, densidad, carga eléctrica, etc.; estas cantidades físicas se denominan cantidades escalares. Existen también otras cantidades que para ser descritas plenamente deben estar asociadas con un número (o magnitud) con una dirección y con un sentido, y no pueden describirse tan sólo con un número, tales como posición, velocidad, aceleración, fuerza, etc.; estas cantidades reciben el nombre de cantidades vectoriales.

Para darnos una idea de la diferencia entre cantidades escalares y cantidades vectoriales, veamos los siguientes ejemplos:

Cuando el termómetro indica que la temperatura de un cuerpo es de 30º C, con ese valor se establece completamente la temperatura del cuerpo. Sin embargo, cuando se dice que un cuerpo se ha desplazado 10 m, esta expresión queda un poco vaga, debido a que es necesario indicar cómo se ha desplazado (la dirección del desplazamiento) y a qué punto se ha desplazado (el sentido del desplazamiento).

Si una cantidad física se describe con un solo número, decimos que es una cantidad escalar. Mientras que si la cantidad física es descrita por una magnitud, una dirección en el espacio, y sentido, decimos que es una cantidad vectorial. Un vector se denota por medio de una letra negrita cursiva con una flecha arriba A

r,

como se indica en la figura (1.1). Los símbolos manuscritos se subrayan o escriben con una flecha. Un vector se dibuja como una flecha, que va desde un punto inicial O (o cola de la flecha) hasta un punto final P (punta de la flecha) (figura 1.1). La longitud de la flecha indica la magnitud del vector, y su dirección es la del vector.

López Frontado, R. (2011). Física general I, Texto UNA. 2da edición. Caracas: UNA

Page 22: Fisica General i Una

29

La dirección de un vector se indica por medio del ángulo (por ejemplo, el ángulo θ) que forme dicho vector con un eje, tomado como referencia.

Las características de una cantidad escalar son su magnitud y su unidad; mientras que de una cantidad vectorial son: su magnitud, su dirección y su sentido.

Figura 1.1

Representación del vector A

r, donde θ es

el ángulo que indica su dirección respecto al eje de referencia.

Escriba siempre los símbolos vectoriales de una de estas formas. Ya que si Ud. no distingue entre vectores y escalares en su notación, tampoco lo hará en su mente, y se confundirá.

Si dos vectores tienen la misma dirección, son paralelos. Si tienen la misma magnitud y dirección, los vectores son iguales (figura 1.2a). Si tienen la misma magnitud pero su dirección difiere en 180º, los vectores son opuestos (figura 1.2b).

La magnitud de una cantidad vectorial, se representa por la misma letra del vector sin la flecha arriba (A). Otra forma de representar la magnitud es encerrando al símbolo vectorial entre dos barras verticales ( A

!). La magnitud de una cantidad

vectorial es una cantidad escalar y siempre es positiva.

Si A = A' y = 'θ θ

! !, los vectores

A y A'r r

son iguales, esto esA=A'

! !.

(a)

A = B y - =180º! "

! !, los

vectores A y B! !

son opuestos, esto es A= -B

r r.

(b) Figura 1.2

López Frontado, R. (2011). Física general I, Texto UNA. 2da edición. Caracas: UNA

Page 23: Fisica General i Una

30

SUMA DE VECTORES

Para sumar dos vectores, supongamos que una partícula primero sufre un desplazamiento A

! y luego sufre otro desplazamiento B

!. Se tiene entonces que el

desplazamiento final de la partícula es el vectorCr

, como si la partícula hubiese partido del mismo punto y hubiese tenido un solo desplazamientoC

r, como se

muestra en la figura (1.3). Se tiene entonces que el vector C!

es el vector sumatoria o resultante, de los desplazamientos A

! y Br

Simbólicamente se puede expresar por:

C=A+B (1.3)! ! !

Esta expresión implica que la suma de vectores es un proceso geométrico, y no es lo mismo que sumar dos escalares. Cuando se suman vectores, regularmente se coloca la cola del segundo vector en la punta del primer vector (como se muestra en la figura 1.3a).

Figura 1.3

Suma de dos vectores.

Si invertimos el orden de los desplazamientos A!

y Br

, esto es, colocamos primero B

r y luego A

! (como se muestra en la figura 1.3b), el resultado es el mismo.

Así se tiene que

C=A+B=B+A! ! ! ! !

Esto indica que la suma de vectores obedece la ley conmutativa. Si dibujamos Ar

y B!

con sus colas en el mismo punto (como se muestra en la figura 1.3c), el vector C

! es la diagonal de un paralelogramo construido con A

! y B

! como lados

adyacentes. La diferencia D=A-B

! ! ! se obtiene como la suma de A

r

y -Br

(el vector opuesto de B!

), como se muestra en la figura (1.4), así se tiene que

D=A-B=A+(-B)! ! ! ! !

Figura 1.4

Diferencia de vectores.

López Frontado, R. (2011). Física general I, Texto UNA. 2da edición. Caracas: UNA

Page 24: Fisica General i Una

31

Cuando se suman más de dos vectores, el procedimiento gráfico consiste en escoger un punto como origen; luego se dibuja un primer vector, colocando en dicho punto original su cola, respetando su dirección y magnitud; luego se dibuja un segundo vector, colocando su cola en la punta del primero; así se continúan dibujando los siguientes vectores, tal que la cola de cada uno se coloca en la punta del anterior, hasta dibujar el último vector. El vector suma será aquel vector que se obtiene uniendo el origen inicial con la punta del último vector. Como ejemplo, supongamos los vectores A, B y C

!! ! mostrados en la figura (1.5a), y el vector suma de

los mismosR!

, mostrado en la figura (1.5b).

Figura 1.5 Suma de varios vectores.

COMPONENTES DE VECTORES

Las componentes de un vector se obtienen mediante la descomposición de un vector, que consiste en buscar dos o más vectores que produzcan el mismo efecto de un vector dado. Por ejemplo supongamos que se tiene un vectorA

r, y sean x e y

dos ejes cualesquiera, como se muestra en la figura (1.6).

Para obtener las componentes x yA y Ar r

, desde

la punta del vector Ar

, se trazan paralelas a los ejes x e y. Así se tiene que

x yA=A + A (1.4)! ! !

Cuando los ejes x e y son perpendiculares entre sí, como se muestra en la figura (1.7), se tiene que

x yA=A + A! ! !

Figura 1.6

Descomposición de un vector.

López Frontado, R. (2011). Física general I, Texto UNA. 2da edición. Caracas: UNA

Page 25: Fisica General i Una

32

Si se conocen las magnitudes de x yA y Ar r

, la

magnitud del vector Ar

se obtiene usando el teorema de Pitágoras, esto es

2 2x yA= A +A (1.5)

La dirección del vector A

r con respecto al eje x

está dada por

y y-1

x x

A Atan = , =tan (1.6)

A Aφ φ

⎛ ⎞→ ⎜ ⎟

⎝ ⎠

Si se conocen la magnitud del vector A

r y su

dirección ! , las componentes x yA y Ar r

están dadas por

x yA =Acos y A =Asen (1.7)! !

Cuando se tienen más de dos vectores, cuyo vector suma (o vector resultante) se quiera obtener, se procede de la siguiente manera: Cada uno de los vectores a sumar se descompone en sus componentes a lo largo del eje x y del eje y., como se muestra en la figura (1.8). Por ejemplo: sean los vectores A, B y C.

!! !

Al descomponerse, se tienen:

Componentes:

x y

x y

x y

A=A +A

B=B +B

C=C +C

r r r

r r r

r r r

El vector resultante está dado por

x yR=R +R! ! !

y sus componentes son:

x x x x

y y y y

R =A +B +CR =A +B +C

La magnitud y la dirección del vector resultante, respectivamente son:

y2 2 -1x y

x

RR= R +R , =tan

⎛ ⎞⎜ ⎟⎝ ⎠

Figura 1.7

Descomposición de un vector en coordenadas rectangulares.

Figura 1.8

Suma de vectores, usando sus componentes en las direcciones de los ejes Ox y Oy.

Figura 1.9

Vector suma (o vector resultante).

López Frontado, R. (2011). Física general I, Texto UNA. 2da edición. Caracas: UNA

Page 26: Fisica General i Una

33

VECTOR UNITARIO

Un vector unitario u (algunas veces llamado versor), es un vector de magnitud igual a la unidad que tiene la misma dirección del vector dado. El vector unitario, matemáticamente se expresa por:

Aˆ ˆu= , de donde se tiene que A=Au (1.8)A

rr

En el sistema de coordenadas rectangulares x, y y z, son: ˆ ˆ ˆi, j y k , respectivamente, como se muestra en la figura (1,10). Así se tiene que un vectorA

r, escrito en función de sus componentes,

usando los vectores unitarios correspondientes, está dado por:

x y

x y z

ˆ ˆA=A i+A j (en dos dimensiones)ˆ ˆ ˆA=A i+A j+A k (en tres dimensiones)

!

!

Figura 1.10 Vectores unitarios en las direcciones Ox, Oy y Oz.

Para darnos una idea de la suma de vectores, veamos el siguiente ejemplo: Ejemplo 1.1

Dados los vectores ˆ ˆ ˆA=(12i+6j-2k) m

! ˆ ˆ ˆy B=(8i-10j+16k) mr

, obtenga la magnitud del vector resultante.

SOLUCIÓN:

El vector resultante está dado por:

x x y y z zˆ ˆ ˆ ˆ ˆ ˆR=(A +B )i+(A +B )j+(A +B )k=(12+8)i+(6-10)j+(-2+16)k

ˆ ˆ ˆ R=(20i-4j+14k) m!

r

r

La magnitud de R es

2 2 2R= (20) +(-4) +(14) R=24,74 m!

López Frontado, R. (2011). Física general I, Texto UNA. 2da edición. Caracas: UNA

Page 27: Fisica General i Una

34

Basado en los conceptos presentados hasta los momentos, responda la

siguiente pregunta. Pregunta 1.2.-

¿Todas las cantidades fundamentales son vectores?

OPERACIONES CON VECTORES:

PRODUCTO DE UN ESCALAR POR UN VECTOR

Sea un vector x y zˆ ˆ ˆA=A i+A j+A k

r, que se multiplica por un escalar cualquiera ! ,

se tiene

x y zˆ ˆ ˆA= A i+ A j+ A k (1.9)λ λ λ λ

r

Se obtiene que el resultado es un vector donde cada componente está

multiplicado por el escalar ! .

Para darnos una idea del producto de un vector por un escalar, veamos el

siguiente ejemplo: Ejemplo 1.2.-

Dado el vector ˆ ˆ ˆA=2i-4j+3k, y el escalar =2λ

r, determine: a) el vector V= Aλ

r r, b) la

magnitud del vectorV!

.

SOLUCIÓN: a) El vector V= A!

r r está dado por:

ˆ ˆ ˆ ˆ ˆ ˆ ˆ ˆ ˆV=2(2i-4j+3k)=2 2i-2 4j+2 3k V=4i-8j+6k× × × ⇒r r

b) La magnitud del vector V!

está dada por: 2 2 2V= 4 ( 8) +6 = 16+64+36= 116 V=10,77+ − ⇒

López Frontado, R. (2011). Física general I, Texto UNA. 2da edición. Caracas: UNA

Page 28: Fisica General i Una

35

Ejercicio propuesto 1.1.-

Dados los vectores ˆ ˆ ˆ ˆ ˆ ˆ ˆ ˆ ˆA=3i+4j-5k , B=-i+j+2k y C= 2i-3j+4k

!! !. Encuentre: a) La

magnitud del vector resultante, b) El vectorV=2A-3B+5C!! ! !

, c) La magnitud del vectorV!

.

PRODUCTO ESCALAR ENTRE VECTORES El producto escalar entre dos vectores A y B

r r, se

expresa por A Br ri (también se denomina producto

punto), es una cantidad escalar que se define como el producto de sus magnitudes por el coseno del ángulo formado entre ellos, como se muestra en la figura (1.11). Expresada matemáticamente, se tiene A B=AB cos = A B cos (1.10)θ θ

r r r ri

Donde ! es el ángulo formado entre los vectores, el cual está siempre entre 0º y 180º.

Figura 1.11

Producto escalar A Br ri .

Tomando en cuenta la definición del producto escalar, se tiene que si los

vectores son paralelos, se tiene que ! = 0, y el cos 0º = 1, por lo tantoA B=ABr ri . Si

los vectores son perpendiculares, se tiene que ! = 90º, y el cos 90º = 0, por lo tantoA B=0

r ri .

Si los vectores A y Br r

los expresamos en función de sus componentes, esto es

x y z

x y z

ˆ ˆ ˆA=A i+A j+A kˆ ˆ ˆB=B i+B j+B k

!

!

Los productos escalares de los vectores unitarios ˆ ˆ ˆi, j y k son:

ˆ ˆ ˆ ˆ ˆ ˆ ˆ ˆ ˆ ˆ ˆ ˆi i=j j=k k=1 y i j=j k=k i=0 (1.11)i i i i i i

López Frontado, R. (2011). Física general I, Texto UNA. 2da edición. Caracas: UNA

Page 29: Fisica General i Una

36

El producto escalar entre A y Br r

, será

x y z x y z

x x x y x z y x y y

y z z x z y z z

ˆ ˆ ˆ ˆ ˆ ˆA B=(A i+A j+A k) (B i+B j+B k)ˆ ˆ ˆ ˆ ˆ ˆ ˆ ˆ ˆ ˆ =(i i)A B +(i j)A B +(i k)A B +(j i)A B +(j j)A Bˆ ˆ ˆ ˆ ˆ ˆ ˆ ˆ +(j k)A B +(k i)A B +(k j)A B +(k k)A B

r ri i

i i i i i

i i i i

Aplicando los resultados (1.11), se tiene

x x y y z zA B=A B +A B +A B (1.12)r ri

Notemos que

2 2 2 2 2 2 2x y z x y zA =A A=A +A +A A= A +A +A!

r ri

Expresión que nos permite obtener la magnitud de un vector, conocidas sus

componentes en tres dimensiones.

La definición del producto escalar, dada por la ecuación (1.10) permite determinar directamente el ángulo entre los vectores A y B

r r. Así se tiene que

-1A B A Bcos = =cos (1.13)AB AB

θ θ⎛ ⎞

→ ⎜ ⎟⎝ ⎠

r r r ri i

El uso del producto escalar entre vectores nos permitirá comprender el concepto de Trabajo, el cual será estudiado en la unidad 4.

Basado en los conceptos emitidos hasta los momentos, responda la siguiente

pregunta. Pregunta 1.3.-

¿Cuál es para Ud. el significado físico de un Producto Escalar?

López Frontado, R. (2011). Física general I, Texto UNA. 2da edición. Caracas: UNA

Page 30: Fisica General i Una

37

Para darnos una idea del producto escalar entre vectores y su significado,

veamos el siguiente ejemplo: Ejemplo 1.3.- Dado los vectores ˆ ˆ ˆ ˆ ˆ ˆA=5i-2j+k, y B=-3i+j-7k

! ! . Encuentre: a) El producto escalar

entre A y B! !

. b) El ángulo formado entre los vectores A y B! !

.

SOLUCIÓN:

a) El producto escalar entre A y Br r

está dado por:

ˆ ˆ ˆ ˆ ˆ ˆA B=(5i-2j+k) (-3i+j-7k)=5(-3)+(-2)+(-7) A B= -24!r r r ri i i

b) Para determinar el ángulo formado entre los vectores A y B! !

, se aplica la definición del producto escalar entre vectores, así se tiene:

-1

A Bcos = , A= 25+4+1=5.48, B= 9+1+49=7.68AB

-24cos = =cos (-0,570)=124,75º5,48 7,68

!

! !"#

r ri

PRODUCTO VECTORIAL ENTRE VECTORES

El producto vectorial entre dos vectores A y B.r r

, se representa por el símboloA B×

! !, y está definido como el vector perpendicular al plano formado por

A y B.r r

en la dirección de avance de un tornillo de rosca derecha que ha sido rotado de A

! hacia B

r, como se muestra en la figura (1.12), está dado por:

AxB=C (1.14)

!! !

Otra regla sencilla para establecer la dirección del vectorC=A B!

! ! !, es la regla de la

mano derecha, donde los dedos índice y el mayor apuntan en las direcciones de A y B

! !,

respectivamente, y el dedo pulgar apunta en la dirección de C

r.

Figura 1.12

López Frontado, R. (2011). Física general I, Texto UNA. 2da edición. Caracas: UNA

Page 31: Fisica General i Una

38

La magnitud del producto vectorial A B×

! ! está dado por

A B =AB sen = A B sen (1.15)! !"

r r r r

Donde θ es el ángulo formado entre A y B.

r r. De la definición del producto

vectorial se tiene que AxB=-BxA! ! ! !

Si los vectores A y B

! ! son paralelos, se tiene queAxB=0

r r; y si los vectores

son perpendiculares, se tiene que AxB =ABr r

.

Los productos vectoriales de los vectores unitarios son:

ˆ ˆ ˆ ˆi j=- j i=kˆ ˆ ˆ ˆ ˆ ˆ ˆ ˆ ˆ ˆ ˆj k=-k j=i , i i=j j=k k=0 (1.16)ˆ ˆ ˆ ˆ ˆk i=-i k=j

! " "## " " " " "$# " "#%

!

Si A y B.r r

están expresadas en función de sus componentes rectangulares, se tiene: x y z x y z

ˆ ˆ ˆ ˆ ˆ ˆA B=(A i+A j+A k) (B i+B j+B )k! !r r

x x x y x z y x

y y y z z x

z y z z

ˆ ˆ ˆ ˆ ˆ ˆ ˆ ˆA B=(i i)A B +(i j)A B +(i k)A B +(j i)A Bˆ ˆ ˆ ˆ ˆ ˆ +(j j)A B +(j k)A B +(k i)A Bˆ ˆ ˆ ˆ +(k j)A A +(k k)A B

× × × × ×

× × ×

× ×

r r

De donde se obtiene que

y z z y z x x z

x y y x

ˆ ˆA B=(A B -A B )i+(A B -A B )jˆ +(A B -A B )k (1.17)

!r r

La ecuación (1.17) también se puede escribir en la forma más compacta de determinante,

x y z

x y z

ˆ ˆ ˆi j kA B= A A A (1.18)

B B B

!! !

López Frontado, R. (2011). Física general I, Texto UNA. 2da edición. Caracas: UNA

Page 32: Fisica General i Una

39

El producto vectorial entre vectores nos permite comprender los conceptos correspondientes al movimiento circular de la partícula (Unidad 2), los conceptos de momento angular (Unidad 5) y, más adelante, el comportamiento de una partícula en presencia de un campo magnético, etc.

Basándose en los conceptos presentados hasta los momentos, responda la

siguiente pregunta. Pregunta 1.4.-

¿Cuál es para Ud, el significado físico de un producto vectorial?

Para darnos una idea del producto vectorial entre vectores, veamos el

siguiente ejemplo: Ejemplo 1.4.- Dado los vectores ˆ ˆ ˆ ˆ ˆ ˆA=4i-3j+2k y B=-5i+2j-7k

! !, determine: a) el producto

vectorial entre A y B! !

, b) la magnitud de A B!r r

. SOLUCIÓN:

IDENTIFICAR los conceptos pertinentes: La definición del producto vectorial entre dos vectores, nos proporciona otro

vector que es perpendicular al plano formado por los vectores factores.

PLANTEAR Y EJECUTAR la solución del problema: a) Para determinar el producto vectorial entre A y B

! !, se puede usar las

determinantes, como se muestra a continuación:

[ ] [ ] [ ]ˆ ˆ ˆi j k

ˆ ˆ ˆA B= 4 -3 2 =i (-3)(-7)-(2)(2) -j (4)(-7)-2(-5) +k 4(2)-(-3)(-5)-5 2 -7

ˆ ˆ ˆ ˆ ˆ ˆA B=i(21-4)-j(-28+10)+k(8-15) A B=17i+18j-7k

×

× ⇒ ×

! !

! ! ! !

López Frontado, R. (2011). Física general I, Texto UNA. 2da edición. Caracas: UNA

Page 33: Fisica General i Una

40

b) La magnitud de A B!! !

está dada por:

2 2 2A B = (17) +(18) +(-7) = 289+324+49= 662 A B =25,73! " !! ! ! !

EVALUAR los resultados: como se observa, el producto vectorial es un vector, expresado con sus tres componentes en las direcciones de las coordenadas x, y y z. La magnitud del producto vectorial se representa únicamente por un número. 1.3.4 MOVIMIENTO DE UNA PARTÍCULA

En esta unidad se estudia el movimiento de un cuerpo. Pero si se considera el cuerpo como tal, resultaría bastante complicado por los momentos, ya que el movimiento de un cuerpo puede ser de traslación, o también puede ser una combinación de traslación más rotación, lo cual resultaría más complejo su estudio. Para evitar alguna complejidad, consideraremos al cuerpo en dimensiones bastante pequeñas, denominado cuerpo puntual o simplemente partícula. De esta forma estudiaremos el movimiento de traslación de la partícula. Para iniciar este estudio de los cuerpos, es necesario que estudiemos previamente algunos conceptos, fundamentales para su comprensión. Por esta razón, empezaremos por conocer la posición de una partícula en un momento determinado; para ello definiremos los que se conoce como vector posición.

VECTOR POSICIÓN Es un vector que define la posición de un punto, con respecto a un sistema de coordenadas preestablecido, cuyo origen es O. Sea r

!el vector que

indica la posición de A con respecto al sistema O, cuyas coordenadas son x, y, y z, como se muestra en la figura (1.13). El vector posición matemáticamente está expresado por la siguiente ecuación: ˆ ˆ ˆr xi yj zk (1.19)= + +

!

VECTOR DESPLAZAMIENTO Cuando una partícula se traslada durante su movimiento, desde un punto A hasta un punto B, como se muestra en la figura (1.14), se dice que la partícula se ha desplazado, esto es que el cuerpo ha cambiado de posición. Para representar este cambio de posición se hace uso del llamado vector

Figura 1.13

Vector posición

Figura 1.14 Vector desplazamiento

López Frontado, R. (2011). Física general I, Texto UNA. 2da edición. Caracas: UNA

Page 34: Fisica General i Una

41

desplazamiento, el cual lo representamos por r!!

. El vector desplazamiento se expresa por la ecuación por B Ar r r (1.20)Δ = −

! ! !

donde Ar

! es el vector posición del punto A, y Br

r es el

vector posición del punto B. Es importante no confundir el concepto denominado desplazamiento con el concepto denominado distancia, ya que el desplazamiento es un vector que va desde una posición inicial a una posición final, mientras que la distancia es escalar, y es el camino recorrido desde la posición inicial hasta la posición final, como se muestra en la figura (1.15). La partícula en su movimiento traza una curva denominada trayectoria, que de acuerdo a su forma puede ser rectilínea, parabólica o circular, cuando la trayectoria es una línea recta, una parábola o un círculo, respectivamente. Cuando la posición de una partícula cambia a medida que transcurre el tiempo, el vector posición se expresa por r(t)

!,

ˆ ˆ ˆr(t) = x(t) i+ y(t) j+ z(t)k (1.21)

!

Donde x(t), y(t) y z(t) son las coordenadas de la posición en función del tiempo, también llamados parámetros del movimiento, y sus ecuaciones respectivas ecuaciones paramétricas. ˆ ˆ ˆi, j y k , son los vectores unitarios en las direcciones de x, y, y z, respectivamente. VECTOR VELOCIDAD Otro concepto importante por definirse es la denominado rapidez, la cual es una cantidad escalar que representa la distancia recorrida por el cuerpo en un intervalo de tiempo dado; esto es la razón de cambio a la que se recorre la distancia. La rapidez se represente por la letra v, y se expresa en metros por segundos (m/s) en el SI. Cuando se divide la distancia recorrida (Δx) entre el intervalo de tiempo

Figura 1.15

Desplazamiento, vector que va desde A hasta D. Distancia, camino recorrido desde A hasta D, pasando por B y C, esto es, Distancia = AB + BC + CD

López Frontado, R. (2011). Física general I, Texto UNA. 2da edición. Caracas: UNA

Page 35: Fisica General i Una

42

(Δt), se obtiene lo que se denomina como rapidez media, esto es,

m

distancia recorridarapidez = , intervalo de tiempo

x v = (1.22)t

!!

Cuando el intervalo de tiempo Δt se toma lo más pequeño posible, tal que (Δt→0) se obtiene lo que se denomina rapidez instantánea, y se representa por la letra v; así se tiene que

0

x dxv= lim = (1.23)t dtx! "

!!

Considerando los conceptos presentados hasta los momentos, responda la siguiente pregunta.

Pregunta 1.5.- ¿Qué mide el velocímetro de un automóvil, la rapidez media o la rapidez instantánea? La velocidad es un vector que representa el cambio de posición de un cuerpo en un intervalo de tiempo dado, esto es la razón de cambio de posición de la posición de un cuerpo. La velocidad se representa por la letra v

!, y al igual que la rapidez se

expresa en metro por segundo (m/s) en el SI. Al dividirse el vector desplazamiento r!

! entre el

intervalo de tiempo transcurrido Δt, si la partícula se desplaza desde A hasta B, se obtiene lo que se denomina velocidad media, la que se represente por mvr

(Figura 1.16). Así se tiene

B Am

B A

r rrv (1.24)t t t

−Δ= =Δ −

! !!!

El vector velocidad media es un vector que tiene la misma dirección del desplazamiento, y se expresa en m/s (en el sistema de unidades SI).

Rapidez media Rapidez instantánea

Figura 1.16

Velocidad media

Figura 1.17

La velocidad v!

es tangente a la curva en el punto A.

López Frontado, R. (2011). Física general I, Texto UNA. 2da edición. Caracas: UNA

Page 36: Fisica General i Una

43

Cuando el intervalo de tiempo, se hace lo más pequeño posible, es decir, cuando Δt tiende a cero, se obtiene lo que se denomina velocidad instantánea, o simplemente vector velocidad, representada por el vector v

!, y expresada

matemáticamente con la ecuación:

t 0

r rv lim (1.25)t

ddtΔ →

Δ= =Δ

r rr

El vector velocidad siempre es tangente a la curva que representa la trayectoria de la partícula. Al igual que la velocidad media, se expresa en m/s (en el sistema de unidades SI).Lo que se denomina como simplemente rapidez es la magnitud de la velocidad instantánea.

NOTA Es importante aclarar que la denominación de Velocidad promedio (utilizada en algunos textos como el de Serway), tiene el mismo significado que la velocidad media (usada en el presente texto). Aclaratoria que se hace extensible al término de rapidez promedio = rapidez media.

Basándose en los conceptos emitidos hasta los momentos, responda la

siguiente pregunta.

Pregunta 1.6.-

El velocímetro de un automóvil que se desplaza hacia el este indica 80 km/h. El auto pasa al lado de otro automóvil que viaja hacia el oeste a 80 km/h. ¿Viajan los automóviles con la misma velocidad? ¿Viajan los automóviles con la misma rapidez?

Para darnos una idea del concepto de velocidad, veamos los siguientes

ejemplos: Ejemplo 1.6

Un caballo se aleja de su entrenador galopando en línea recta una distancia de 160 m en 17,0 s. Luego regresa abruptamente y galopa la mitad de la distancia en 6,8 s. Calcule para todo el viaje del caballo: a) la velocidad media y b) la rapidez media. Tome “alejándose de su entrenador” como el sentido positivo del galope.

DATOS: AB = 160 m, t1 = 17,0 s, BC = 80 m, t2 = 6,8 s.

López Frontado, R. (2011). Física general I, Texto UNA. 2da edición. Caracas: UNA

Page 37: Fisica General i Una

44

SOLUCIÓN: IDENTIFICAR los conceptos pertinentes: En este problema se considera la

diferencia entre la velocidad media y la rapidez media. Para ello se usarán los conceptos de desplazamiento y de distancia recorrida por el caballo.

PLANTEAR el problema: La figura 1.18 representa el movimiento del caballo cuando se aleja del entrenador, por el recorrido desde A hasta B, a partir del punto original O, y el movimiento de regreso desde B hasta C.

Figura 1.18

La figura (1.18) representa el movimiento del caballo cuando se aleja del entrenador, por el recorrido desde A hasta B, a partir del punto original O, y el movimiento de regreso desde B hasta C

EJECUTAR la solución del problema: a) Para determinar la velocidad media se tiene que tC = t1 + t2 = 23,8 s. Por lo tanto,

C Am m

C A

x -x 80,0-0 mˆ ˆ ˆv = i= i v =3,36i t -t 23,8-0 s

⇒! !

b) La rapidez media está dada por:

AB BC

m mC A

x + x 160 + -80 mv = = v =10,08 t -t 23,8 s

Δ Δ⇒

EVALUAR el resultado: En el resultado se obtiene que la velocidad media mv

! es diferente a la rapidez

media, debido a que la velocidad media mx ˆv = it

!!

r es una cantidad vectorial (e igual al

desplazamiento entre el tiempo transcurrido), mientras que la rapidez media vm =

mdistancia recorrida v =tiempo transcurrido

, es una cantidad escalar.

VECTOR ACELERACIÓN Cuando la velocidad de una partícula cambia (ya sea en magnitud, la dirección o ambas) se dice que ese cuerpo está acelerado. Se define, entonces, la aceleración como el cambio de la velocidad de un cuerpo durante un intervalo de tiempo, esto es la razón de cambio de la velocidad de un cuerpo. La aceleración se

Figura 1.19

Cambio de velocidad

López Frontado, R. (2011). Física general I, Texto UNA. 2da edición. Caracas: UNA

Page 38: Fisica General i Una

45

representa por la letra ar

, y se expresa en metros por segundos al cuadrado (m/s2) en el sistema SI. A medida que una partícula se desplaza a lo largo de su trayectoria, su velocidad puede cambiar, esto es,

B Av = v - v (1.26)!! ! !

Si se toma en cuenta el intervalo de tiempo transcurrido durante este cambio, se obtiene lo que se denomina aceleración media, la cual se expresa por

B Am

B A

v - vva = = (1.27)t t - t

!!

! !!!

La aceleración media es un vector que tiene la misma dirección del cambio de velocidad, y se expresa en m/s2 (En el sistema de unidades SI). Si ahora hacemos el intervalo de tiempo lo más pequeño posible, esto es, que Δt tienda a cero, se obtiene lo que se denomina aceleración instantánea (o simplemente aceleración). Esto es,

t 0

v va lim (1.28)t t

ddΔ →

Δ= =Δ

r rr

El vector aceleración está dirigido hacia la concavidad de la curva.

Figura 1.21

Figura 1.20 La aceleración media ma

r

es paralela al cambio de velocidad vΔ

r.

En la figura (1.21), se tiene que la velocidad Av

!punto A

es tangente a la curva mientras que la aceleración Aa

! está dirigida hacia la

concavidad de la curva. De igual forma en el punto B.

Basándose en los conceptos emitidos hasta los momentos, responda las

siguientes preguntas.

Pregunta 1.7.-

¿La aceleración es la razón de cambio de qué?

López Frontado, R. (2011). Física general I, Texto UNA. 2da edición. Caracas: UNA

Page 39: Fisica General i Una

46

Pregunta 1.8.-

¿Puede un objeto que se acelera conservar una rapidez constante pero no una velocidad constante?

1.3.5 PROBLEMAS INVERSOS

A partir de los conceptos de vector posición, vector velocidad y vector aceleración, los problemas se pueden catalogar como problemas directos y problemas inversos.

Problemas directos son aquellos que al conocerse la función de la posición

r(t).!

de un partícula en función del tiempo, se deriva la función con respecto al tiempo y se obtiene la velocidad de la partícula v(t)

! en función del tiempo, y al

continuarse derivando con respecto al tiempo, se obtiene la aceleración a(t)!

de la partícula en función del tiempo. Esto es,

dr(t) dv(t)r(t) v(t) = a(t) = (1.29)dt dt

! !r r

r rr

Problemas inversos son aquellos problemas en los que, al conocerse la

aceleración a(t)!

de la partícula en función del tiempo, se determina la velocidad v(t)!

y por último a determina la posición r(t).

! de la partícula en función del tiempo. Esto

es, a(t) v(t) r(t)! !! !!

A continuación le presentamos el procedimiento para obtener la velocidad en función del tiempo v(t)

!. Usando la definición del vector aceleracióna(t)

!, de donde se

despeja el diferencialdvr

, luego se integran ambas partes de esta ecuación, aplicando las condiciones iniciales, obteniéndose así la ecuación (1.30), la cual expresa la velocidad en función del tiempo v(t)

!. Matemáticamente, este

procedimiento se describe a continuación:

o o o

o

v t t

ov t t

t

ot

dva(t) = dv = a(t)dt v- v = a(t)dtdt

v = v + a(t)dt (1.30)

! !

"

# # #

#

r

r

rr r rr r r

rr v

López Frontado, R. (2011). Física general I, Texto UNA. 2da edición. Caracas: UNA

Page 40: Fisica General i Una

47

Para determinar la posición de la partícula en función del tiempo r(t).!

, a partir de la definición del vector velocidad v(t)

! en función del tiempo, se despeja el

diferencialdrr, integrándose luego, tomando en cuenta las condiciones iniciales, para

obtener así la ecuación (1.31), la cual expresa la posición de la partícula en función del tiempo r(t).

!. Matemáticamente este procedimiento se describe a continuación:

o o o

o

r t t

or t t

t

ot

drv(t) = dr = v(t)dt r- r = v(t)dtdt

r = r + v(t)dt (1.31)

! !

"

# # #

#

!

!

!! ! !! ! !

! ! !

Analizando la ecuación (1.30), tenemos que: si la aceleración a(t)

! es positiva,

el movimiento de la partícula se denomina Movimiento Acelerado; si la aceleración a(t)!

es negativa, el movimiento de la partícula se denomina Movimiento Desacelerado (o Retardado).

Otros casos particulares que podemos estudiar son los siguientes:

a) Cuando la aceleración a!

es constante, la ecuación (1.30) queda expresada matemáticamente por

o v = v +at (1.32)!

!! !

Y la ecuación de la posición se obtiene sustituyendo este resultado en la ecuación (1.31), esto es

2

o o1 r=r +v t+ at (1.33)2

!! ! !!

b) Otro caso particular, se tiene cuando la aceleración a!

es constante y además es positiva (a>0)!

. Analizando la ecuación (1.32) se observa que la velocidad v

r aumenta

proporcionalmente a medida que el tiempo transcurre, se dice entonces que estamos en presencia de un Movimiento Uniformemente Acelerado. Las ecuaciones para la velocidad (ec. 1.32) y la posición (ec. 1.33) quedan tal como están escritas. Esto es:

o

2o o

v = v + at y1r = r + v t+ at2

!! !

! ! !!

Figura 1.22 Gráfica de v vs t, para un Movimiento Uniformemente Acelerado.

López Frontado, R. (2011). Física general I, Texto UNA. 2da edición. Caracas: UNA

Page 41: Fisica General i Una

48

c) Si ahora la aceleración a

!, además de ser constante

es negativa (a<0)r

, entonces la velocidad de la partícula disminuye proporcionalmente a medida que transcurre el tiempo, se dice que el movimiento de la partícula es Uniformemente Desacelerado (o Retardado). Las ecuaciones para la velocidad y la posición quedan expresadas:

o

2o o

v = v - at1r = r + v t- at2

!! !

! ! !!

d) Si ahora la aceleración a

! es igual a cero (a=0)

!,

entonces se observa que la velocidad de la partícula se mantiene constante durante su movimiento, y el movimiento se denomina Movimiento Uniforme. Esto es:

o

o o

v = vr = r + v t

! !

! ! !

Figura 1.23

Gráfica de v vs t para un Movimiento Uniformemente Retardado.

Figura 1.24

Gráfica v vs t, para un Movimiento Uniforme.

Tomando en cuenta los conceptos presentados hasta los momentos,

responda la siguiente pregunta.

Pregunta 1.9.- ¿Un objeto con aceleración constante puede invertir la dirección en la que se

mueve? Explique.

A continuación se presentan unos ejemplos de problemas donde Ud. puede visualizar las aplicaciones de los diferentes conceptos tratados en esta unidad. Pero antes vean las Sugerencias, presentadas como Recomendaciones, para resolver problemas.

López Frontado, R. (2011). Física general I, Texto UNA. 2da edición. Caracas: UNA

Page 42: Fisica General i Una

49

RECOMENDACIÓN PARA RESOLVER PROBLEMAS DE MOVIMIENTO DE UNA PARTÍCULA. Para que pueda resolver los problemas relacionados con el movimiento de una partícula, se recomienda:

IDENTIFICAR los conceptos pertinentes: el concepto clave en el movimiento

de un cuerpo es la aceleración, la cual puede ser constante o variable con el tiempo, y su comportamiento respecto al movimiento del cuerpo. Las ecuaciones de movimiento sólo son válidas a partir de las condiciones iniciales del mismo.

PLANTEAR el problema con los pasos siguientes:

a) Definir su sistema de coordenadas, dibujando sus ejes. Tomando en cuenta

las condiciones iniciales del problema, considerando estas condiciones en forma vectorial, de acuerdo a su sistema de coordenadas. En algunos problemas se recomienda tomar el eje x como horizontal y al eje y como vertical.

b) Hacer una lista de las cantidades conocidas y desconocidas, decidiendo luego cuáles son las incógnitas del problema. Asegúrese de tener tantas ecuaciones como incógnitas pedidas por determinar.

c) Suele ser útil plantear el problema con variables literales y luego traducirlo a símbolos.

EJECUTAR la solución: use las ecuaciones planteadas anteriormente para

obtener los valores de las incógnitas pedidas; desde el inicio y use los mismos ejes y escala de tiempo durante todo el problema. Al hacerlo, resista la tentación de dividir la trayectoria en segmentos y analizarlos individualmente. No hay que volver a comenzar, con nuevos ejes y nueva escala de tiempo. (Por ejemplo, cuando se trabaja con movimientos de cuerpos lanzados verticalmente, no se necesita dividir el problema en dos segmentos: uno cuando el cuerpo se mueve hacia arriba y el otro segmento cuando el cuerpo se mueve hacia abajo). Lo más fácil es configurar las ecuaciones necesarias desde el inicio y usar los mismos ejes y escala de tiempo durante todo el problema

EVALUAR la respuesta: examine si sus resultados para ver si son lógicos y si los valores numéricos son razonables. En caso de que se le presenten dudas, pregúntele a su asesor del Centro Local.

López Frontado, R. (2011). Física general I, Texto UNA. 2da edición. Caracas: UNA

Page 43: Fisica General i Una

50

Recuerde que el manejo eficiente del contenido de esta unidad es

importante, por su repercusión en el contenido de las subsiguientes unidades del curso de Física General I.

Para darnos una idea del concepto de posición, desplazamiento y distancia,

veamos los siguientes ejemplos: Ejemplo 1.7.- (Vector posición, vector desplazamiento, distancia)

Un niño camina hacia el Norte recorriendo una distancia de 100 m, luego

cruza hacia el Este y recorre una distancia de 80 m, y por último se desvía en dirección Sur Oeste 30º, caminando una distancia de 150 m. Determinar: a) el desplazamiento del niño, desde su posición inicial hasta el punto final (indicando su magnitud y su dirección); b) la distancia total recorrida por el niño durante su movimiento. SOLUCIÓN:

IDENTIFICAR: En este problema se utilizan los conceptos de vector posición, vector

desplazamiento y distancia recorrida por un cuerpo.

PLANTEAR el problema: Para resolver este problema, se escoge un sistema de coordenadas, donde se

hace un bosquejo de cada uno de los movimientos realizados por el niño. Luego se expresan cada uno de los vectores en función de sus componentes en la dirección de los ejes de coordenadas.

EJECUTAR la solución del problema:

a) Al expresar cada uno de los vectores en función de sus componentes,

se tiene que,

ˆ ˆA = 100 j = +100 jˆ ˆB = 80 i = 80 i

ˆ ˆ ˆ ˆC = -150sen30º i-150cos30º j = -75 i-129,9 j

!

!

!

El vector resultante será ˆ ˆR = 5 i- 29,9 j mr

Figura 1.25

López Frontado, R. (2011). Física general I, Texto UNA. 2da edición. Caracas: UNA

Page 44: Fisica General i Una

51

El vector desplazamiento es el vector resultante

ˆ ˆR = 5 i- 29,9 j m!

La magnitud del vector desplazamiento se obtiene aplicando el teorema de Pitágoras, esto es

R2 = (5)2 + (-29,9)2 ⇒ R = 30,3 m

La dirección se obtiene usando la siguiente función trigonométrica:

-29,9tan = = -80,5º5

! !"

Como el vector resultante está ubicado en el cuarto cuadrante, la dirección de este vector será: Dirección ϕ = 360º - 80,5º = 279,5º

b) La distancia total recorrida por el niño será :

Distancia = 100 + 80 + 150 ⇒ Distancia = 330 m.

EVALUAR el resultado: con estos resultados se demuestra la diferencia que

existe entre el concepto de desplazamiento de un cuerpo en su movimiento, y la distancia recorrida por el mismo. Esto es que desplazamiento se refiere a un vector con magnitud y dirección, mientras que distancia es una cantidad escalar que indica el camino recorrido por la persona.

Ejemplo 1.8.- (Velocidad media)

Un automóvil se traslada por la autopista central desde Caracas a Valencia. En el tramo comprendido entre Caracas y Maracay, distantes 110 km, tarda 1 hora y 15 min; mientras que en el trayecto de Maracay y Valencia, distantes 80 km, tarda 45 min. Calcular la velocidad media de ese automóvil: a) en cada uno de esos tramos, y b) en el recorrido total. SOLUCIÓN: IDENTIFICAR los conceptos pertinentes:

La velocidad media, por definición, es igual al desplazamiento del automóvil entre el tiempo transcurrido. Además se tiene que la velocidad media es una cantidad vectorial.

López Frontado, R. (2011). Física general I, Texto UNA. 2da edición. Caracas: UNA

Page 45: Fisica General i Una

52

PLANTEAR el problema: Por definición sabemos que la velocidad media entre dos puntos es una

cantidad vectorial, y si designamos una dirección como positiva (movimiento desde Caracas hasta Maracay, y luego hasta Valencia), la podemos expresar por:

B Am

B A

x -xx ˆ ˆv = i= it t -t

!!

!

EJECUTAR la solución del problema:

a) Entre Caracas y Maracay, la velocidad media es

m1 m1110 km 110 km kmˆ ˆ ˆv = i = i v = 88,0 i

175 min h h75×60

⇒r r

Entre Maracay y Valencia, la velocidad media es

m2 m280×60 km kmˆ ˆv = i v =106,7 i

45 h h!

! !

b) La velocidad media en el recorrido desde Caracas hasta Valencia, es:

mT mT(110 + 80)×60 km kmˆ ˆv = i v = 95,0 i

(75 + 45 h h!

! !

Observe que m1 m2

mTv + v 88,0 106,7 kmv = 97,4

2 2 h+≠ =

¿Puedes explicar por qué?

EVALUAR la solución del problema: la última ecuación del inciso (b) es

incorrecta, debido a que no se refiere a la velocidad media, sino al valor promedio de dos velocidades.

Ejemplo 1.9.- (Problemas directos del Movimiento de partículas, conocida la posición de la partícula en función del tiempo, calculo de desplazamiento, distancia, velocidad media, rapidez media y velocidad) La posición de una partícula que se mueve a lo largo del eje x, viene dada por

2x=2t -8t+10 , en donde x está expresada en metros (m) y t en segundos (s). Calcular:

López Frontado, R. (2011). Física general I, Texto UNA. 2da edición. Caracas: UNA

Page 46: Fisica General i Una

53

a) el tiempo para el cual la velocidad es nula, b) el desplazamiento y la distancia total recorrida por la partícula entre t = 0 y t = 5 s, c) la velocidad media y la rapidez media entre el mismo lapso de tiempo, d) la velocidad de la partícula en t = 5 s.

SOLUCIÓN: IDENTIFICAR los conceptos pertinentes: este problema se refiere al tipo de

problema que se denomina directo, esto es se da la posición de la partícula, la varía en relación al tiempo, de acuerdo a la ecuación dada; y de ahí se pide obtener otros conceptos tales como desplazamiento, velocidad, y a la vez permite observar la diferencia entre velocidad y rapidez y entre el desplazamiento y la distancia.

PLANTEAR Y EJECUTAR la solución del problema:

a) Para determinar el tiempo en el cual la velocidad de la partícula es nula, se obtiene una expresión para la velocidad en función del tiempo, esto es,

dx mv = = 4t- 8 dt s

Considerando el momento en el cual la velocidad es nula, se tiene v = 0 = 4 t – 8, de donde se despeja el tiempo t, así se obtiene que, → t = 2 s.

b) El desplazamiento está dado por:

2

x = x(5 s) - x(0) x(0) = 10 m,x(5 s) = 2(5) - 8(5) +10 x(5 s) = 20 m

x = 20 -10 x = 10 m

Δ

→ Δ ⇒ Δ

Para calcular la distancia, observemos que existe un cambio en la dirección del movimiento en t = 2 s, como se muestra en la figura (1,26), esto es,

Figura 1.26

Es necesario determinar cuál es la posición de la partícula cuando t = 2 s, esto es :

x(2 s) = 2(2)2 – 8 (2) + 10 x(2) = 2 m

La distancia = ⎢Δx1⎥ + ⎢Δx2⎥ = ⎢x(2 s) – x(0)⎥ + ⎢x(5 s) – x(2 s)⎥

López Frontado, R. (2011). Física general I, Texto UNA. 2da edición. Caracas: UNA

Page 47: Fisica General i Una

54

La distancia = ⎢2 - 10⎥ + ⎢20 - 2⎥ ⇒ La distancia = 26 m.

c) La velocidad media entre t = 0 y t = 5 s, está dada por

m mx x(5 s) - x(0) 20-10 mˆ ˆ ˆ ˆv = i = i = i v = 2 it 5 - 0 5 0 s

!"

! #

! !

mdistancia total 26 mRapidez media = = , rapidez = 5,2 tiempo total 5 s

d) La velocidad en t = 5 s, será

mv(5 s) = 4(5) - 8 v(5 s) =12,0 s

EVALUAR los resultados: los resultados están de acuerdo con la aplicación

de los conceptos previstos.

Ejemplo 1.10.- (Problema inverso, donde se tiene la aceleración en función

del tiempo, y se calcula la velocidad y la posición en función del tiempo)

La aceleración de una partícula que se mueve sobre el eje Ox, viene dada por: a = 18 t – 4, donde a está expresada en m/s2, y t en segundos. Si inicialmente la partícula estaba en reposo, en el origen de coordenadas, calcule:

a) La velocidad y posición de la partícula en función del tiempo.

b) ¿Cuál es la posición cuando la velocidad es nula?

c) ¿Cuál es el desplazamiento y la distancia total recorrida entre t = 0 y t = 2 s?

SOLUCIÓN:

IDENTIFICAR los conceptos pertinentes: Este problema es del tipo denominado inverso, esto es que al tener la

aceleración que varía en función del tiempo se pide determinar la velocidad y la

López Frontado, R. (2011). Física general I, Texto UNA. 2da edición. Caracas: UNA

Page 48: Fisica General i Una

55

posición en función del tiempo, para lo cual se usan las definiciones de aceleración y de velocidad.

PLANTEAR Y EJECUTAR la solución del problema: a) Para determinar la velocidad, aplicando la definición de la aceleración

dva=dt

, se tiene

2

ma = (18t - 4)s

, v t t

2

0 0 0

mdv = a(t)dt = (18t - 4)dt v(t)=(9t -4t) s

!" " "

Para determinar la posición de la partícula en función del tiempo, a partir de la definición de la velocidad dxv=

dt, se tiene que

x t t

2 3 2

0 0 0dx = v(t)dt = (9t -4t)dt x(t)=(3t -2t ) m!" " "

b) Para determinar la posición de la partícula cuando la velocidad es nula,

primero se obtiene el tiempo cuando la velocidad sea nula. Esto es,

12

2

t =0 x(0)=0v=0=9t -4t=t(9t-4)=0, 4t = =0,44 s x(0,44)=-0,13 m

9

!"#$

!#%

c) Para determinar el desplazamiento y la distancia total recorrida,

observamos que el movimiento de la partícula tiene un cambio de dirección en t = 0,44 s, o sea cuando la velocidad es cero. Para darnos una idea más clara al respecto, hacemos una gráfica de la posición en función del tiempo. Calculemos en primer lugar la posición de la partícula en diferentes tiempos.

t=0, x(0)=0

Para t=0,44 s x(0,44)=-013 mt=2 s x(2,0)=16,0 m

!"# !$# !%

Ahora hacemos una gráfica de la posición en función del tiempo, como se muestra en la figura (1.27).

Figura 1.27

López Frontado, R. (2011). Física general I, Texto UNA. 2da edición. Caracas: UNA

Page 49: Fisica General i Una

56

Con estos valores podemos determinar el desplazamiento. Así se tiene que

1 2 B A C BDistancia total = x + x = x -x x -x

Distancia total = (-0,13)-0 16-(-0,13) Distancia total=16,26 m

Δ Δ +

+

EVALUAR el resultado: los resultados están de acuerdo a lo previsto en el

problema.

RESUMEN Las cantidades fundamentales de la mecánica son longitud, masa y tiempo, siendo sus unidades en el sistema SI el metro (m), kilogramo (kg) y segundo (s), respectivamente. La densidad de una sustancia se define como la masa por unidad de volumen de dicha sustancia. Diferentes sustancias tienen diferentes valores de densidades. El método de análisis dimensional nos permite resolver problemas de física. Las dimensiones se pueden tratar como cantidades algebraicas. Cuando calcule el resultado a partir de varios números medidos, medidos cada uno con determinada precisión, deberá dar el resultado con el número correcto de cifras significativas. Cuando multiplique varias cantidades, el número de cifras significativas de la repuesta debe ser igual al de cifras significativas de la cantidad que tenga el número más bajo de cifras significativas. Cuando se sumen o resten números, el número de lugares decimales del resultado debe ser igual al número más pequeño de lugares decimales de cualquier término de la suma. La posición de la partícula en movimiento se expresa por usando el vector posición r(t)! . Cuando una partícula se mueve desde una posición inicial ir

r hasta una posición final fr

! , su desplazamiento es

f ir=r -r (1.20)!! ! !

López Frontado, R. (2011). Física general I, Texto UNA. 2da edición. Caracas: UNA

Page 50: Fisica General i Una

57

La velocidad media de una partícula durante algún intervalo es el desplazamiento r!r dividido entre el intervalo Δt durante el cual se realiza ese desplazamiento.

f i

mf i

r -rrv = = (1.24)t t -t

!!

! !!!

La rapidez media de una partícula a la distancia total recorrida dividida entre el intervalo total durante el cual recorre esa distancia.

distancia recorridarapidez=intervalo tiempo

La velocidad instantánea de una partícula es igual al límite de la razón r

tΔΔ

r

cuando Δt se aproxima a cero. O sea, que es igual a la derivada de rr con respecto a t, o mejor dicho es igual a la rapidez de cambio de la posición en el tiempo:

t 0

r drv= lim = (1.25)t dt! "

!!

! !!

La rapidez instantánea de una partícula es la magnitud de su velocidad instantánea. La aceleración media de una partícula se define como el cambio de su velocidad v!

r dividida entre el intervalo Δt durante el cual se produce ese cambio, esto es,

f im

f i

v -vva = = (1.27)t t -t

ΔΔ

r rrr

La aceleración instantánea de una partícula es igual al límite de la razón vt

!!

r cuando Δt se aproxima a cero. O sea, es igual a la derivada de v! con respecto

al tiempo t, o mejor igual a la rapidez de cambio de la velocidad en el tiempo, esto es,

t 0

v dva= lim = (1.28)t dt! "

!!

! !!

Cuando la velocidad y la aceleración de la partícula están en la misma dirección, la partícula está acelerando. Si la velocidad y la aceleración están en direcciones opuestas, la partícula está reduciendo su velocidad.

López Frontado, R. (2011). Física general I, Texto UNA. 2da edición. Caracas: UNA

Page 51: Fisica General i Una

58

Las ecuaciones de cinemática para una partícula que se mueve con aceleración uniforme (aceleración con magnitud y dirección constante) son:

o

2o o

v=v +at (1.32)1r(t)=r +v t+ at (1.33)2

!! !

! ! !!

Los problemas se abordan mejor en una forma organizada. El estudiante debe recordar y aplicar los pasos de conceptualizar, clasificar, analizar y finalizar de la estrategia general para la solución de problemas.

EJERCICIOS DE AUTOEVALUACIÓN

Instrucciones: En su libreta de apuntes desarrolle los siguientes ejercicios, y compare sus resultados con los presentados a final de la unidad. 1.1.- Para los dos vectores ˆ ˆ ˆ ˆA=3i+2j y B=4i+5k

! !, obtenga: a) el vector V=3A-4B

! ! ! (su

magnitud); b) el producto escalarA Br ri ; c) el producto vectorialA B×

r r; d) el ángulo

formado entre A y B! !

.

1.2.-

En la figura se muestra la gráfica velocidad-tiempo para un objeto que se mueve a lo largo del eje x. a) Trace una gráfica de la aceleración contra el

tiempo. b) Determine la aceleración media del objeto en el

intervalo t = 5 s a t = 15 s. c) Determine la aceleración media del objeto en el

intervalo t = 0 a t = 20 s. 1.3.- Si el movimiento de una partícula a lo largo del eje x, se describe por medio de la ecuación x (t) = 3 t2 – 2 t + 4, donde x está expresado en metros (m), y t en segundos (s). a) ¿Cuál es la posición de la partícula en t = 1 s, y en t = 3 s? b) ¿Cuándo la velocidad es nula? c) ¿Cuál es la distancia total recorrida y el desplazamiento entre t = 0 y t = 3 s? d) ¿Cuál es la velocidad media en el intervalo de tiempo de (c)?

López Frontado, R. (2011). Física general I, Texto UNA. 2da edición. Caracas: UNA

Page 52: Fisica General i Una

59

1.4.- La velocidad de una partícula que se mueve a lo largo del eje Ox, varía con el tiempo de acuerdo a v(t) = (15 – 8 t) m/s. Suponiendo que la partícula está en el origen en el instante inicial. Determine: a) Su aceleración. b) Su velocidad en t = 3 s. c) Su velocidad media en el intervalo de tiempo entre t = 1 s y t = 3 s. d) Su desplazamiento en ese intervalo de tiempo. 1.5.- Una partícula se desplaza a lo largo de una línea recta bajo la influencia de una aceleración que tiene la forma a (t) = - 6 t , donde a (t) está expresada en m/s2 y t en segundos. a) Calcule la posición en función del tiempo de esta partícula, si en t = 0 la

partícula se encuentra en el origen con una velocidad inicial de 3 m/s. b) ¿Cuál es el desplazamiento entre t = 0 y t = 4 s? c) ¿Cuál es la distancia recorrida en ese lapso de tiempo? 1.6.- Una partícula se mueve a lo largo del eje x. Su velocidad como función del tiempo viene dada por la ecuación v (t) = 5 + 10 t, donde v está expresada en m/s. La posición de la partícula en t = 0 es 20 cm. Determine: a) La posición de la partícula en función del tiempo. b) Su velocidad en t = 3 s. RESPUESTA A LAS PREGUNTAS:

Estas respuestas se corresponden con las preguntas presentadas en el desarrollo de la unidad.

Respuesta a la pregunta 1.1.-

Más cerca de ser Ciencias están: investigar las causas de un cáncer; estudiar comportamiento a una rata. Más cerca de ser Tecnología están: reparar un televisor, diseñar un televisor, tratar el cáncer de un paciente, manejar un automóvil.

Respuesta a la pregunta 1.2.-

No. Por ejemplo, la masa de un cuerpo, el tiempo y la carga eléctrica de una partícula, son cantidades escalares, debido a que estas cantidades están

López Frontado, R. (2011). Física general I, Texto UNA. 2da edición. Caracas: UNA

Page 53: Fisica General i Una

60

completamente definidas con el valor (o magnitud) de ellas, no necesita que se les defina dirección ni sentido.

Respuesta a la pregunta 1.3.-

Proporciona una definición más amplia del concepto trabajo, debido a que el trabajo realizado sobre un cuerpo se obtiene al multiplicar escalarmente la proyección de la fuerza aplicada sobre el cuerpo por el desplazamiento del cuerpo.

Respuesta a la pregunta 1.4.-

Proporciona una comprensión más amplia del movimiento rotacional de los cuerpos, debido a que el efecto de torsión (o de rotación) que adquiere un cuerpo está dado por el producto vectorial la fuerza aplicada y la posición del punto donde se aplica dicha fuerza.

Respuesta a la pregunta 1.5.-

La rapidez instantánea, debido a que el intervalo de tiempo en el cual se toma la rapidez es muy pequeño.

Respuesta a la pregunta 1.6.-

Los automóviles tienen diferentes velocidades, debido a que las velocidades son cantidades vectoriales, lo cual nos indica que son opuestas; pero tienen la misma rapidez, por tener la misma magnitud.

Respuesta a la pregunta 1.7.-

La aceleración de un cuerpo se define como el cambio de velocidad en un intervalo de tiempo, dicho de otra forma como la razón de cambio de la velocidad.

Respuesta a la pregunta 1.8.-

Sí, por ejemplo en el movimiento circular uniforme, debido a que la rapidez es la magnitud de la velocidad, lo cual se mantiene constante, mientras que la velocidad, como cantidad vectorial que es, cambia tan solo con cambiar cualquiera de sus características, las cuales son: magnitud, dirección o sentido.

Respuesta a la pregunta 1.9.-

Sí, por ejemplo el moviendo de un cuerpo que es lanzado verticalmente hacia arriba, el cual está bajo el efecto de la aceleración de gravedad, invierte su dirección una vez que alcanza su máxima altura; mientras que la aceleración de gravedad se mantiene constante.

López Frontado, R. (2011). Física general I, Texto UNA. 2da edición. Caracas: UNA

Page 54: Fisica General i Una

61

RESPUESTA DEL EJERCICIO PROPUESTO:

Estas respuestas se corresponden con los ejercicios propuestos en el desarrollo de la unidad.

Respuesta al ejercicio propuesto 1.1.-

a) El vector resultante está dado por:

ˆ ˆ ˆ ˆ ˆ ˆR=A+B+C, R=(3-1+2)i+(4+1-3)j+(-5+2+4)k, R=4i+2j+k⇒rr r r r r

Su magnitud está dado por:

2 2R= 4 +2 +1 R= 21=4,58⇒

b) El vector V=2A-3B+5C!! ! !

está dado por:

ˆ ˆ ˆ ˆ ˆ ˆ ˆ ˆ ˆ ˆ ˆ ˆ ˆ ˆ ˆ ˆ ˆ ˆV=2(3i+4j-5k)-3(-i+j´+2k)+5(2i-3j+4k´)=6i+8j-10k+3i-3j+6k+10i-15j+20kˆ ˆ ˆ V=19i-10j+16k!

!

!

c) La magnitud del vector V

! está dada por:

2 2 2V= (19) +(-10) +(16) = 361+100+256= 717 V=26,78!

RESPUESTAS A LOS EJERCICIOS DE AUTOEVALUACIÓN:

Respuesta al Ejercicio de Autoevaluación 1.1.- a) ˆ ˆ ˆV=-7i+6j-20k, V=22,03

r; b) 12; c) ˆ ˆ ˆA B=10i-15j-8k×

! !; d) 58,71°.

Respuesta al Ejercicio de Autoevaluación 1.2.-

a)

b) 1,6 m/s2; c) 0,8 m/s2.

López Frontado, R. (2011). Física general I, Texto UNA. 2da edición. Caracas: UNA

Page 55: Fisica General i Una

62

Respuesta al Ejercicio de Autoevaluación 1.3.-

a) 5 m; 25 m; b) 0,33 s; c) 21,66 m; 21 m; d) 7 m/s.

Respuesta al Ejercicio de Autoevaluación 1.4.-

a) -8 m/s2; b) -9 m/s; c) -1 m/s; d) -2 m.

Respuesta al Ejercicio de Autoevaluación 1.5.-

a) 3x(t)=3t-t m ; b) -52 m; c) 56 m.

Respuesta al Ejercicio de Autoevaluación 1.6.-

a) 2x(t)=5t +5t+0,2 m; b) 35 m/s.

López Frontado, R. (2011). Física general I, Texto UNA. 2da edición. Caracas: UNA

Page 56: Fisica General i Una

63

UNIDAD 2

TIPOS DE MOVIMIENTOS

En esta unidad se estudian diferentes tipos de movimiento de partículas, empezando con el movimiento de la partícula en una dimensión, denominado movimiento rectilíneo, el cual se puede orientar en dirección horizontal o en dirección vertical. Luego se tratará el movimiento de la partícula en el plano, el cual puede ser parabólico o circular.

Se continuará considerando que tanto la posición como la velocidad y la

aceleración son cantidades vectoriales. Para algunos problemas es conveniente hacer uso de las coordenadas rectangulares: usando las coordenadas a lo largo del eje Ox u Oy, si el movimiento es en una dimensión, sea el movimiento horizontal, vertical y usando las dos coordenadas cuando es un movimiento parabólico. Cuando el movimiento es circular, las coordenadas más convenientes son las polares.

Cuando la partícula se mueve a lo largo de una trayectoria circular, se

estudian los conceptos de posición, velocidad y aceleración angulares.

CONOCIMIENTOS PREVIOS:

El estudiante, para la correcta comprensión del contenido de esta unidad,

debe tener los siguientes conocimientos: 1.- Conceptos y unidades de velocidad y aceleración. 2.- Integración inmediata de las funciones elementales. 3.- Vector derivada.

2.1 OBJETIVO

Resolver problemas de diferentes tipos de movimientos de una partícula.

López Frontado, R. (2011). Física general I, Texto UNA. 2da edición. Caracas: UNA

Page 57: Fisica General i Una

64

2.2 RECOMENDACIÓN PARA EL ESTUDIO DEL CONTENIDO

En esta unidad, el estudiante debe comprender claramente los siguientes conceptos básicos: La ecuación que describe la trayectoria de una partícula. Las componentes tangenciales y normales (o perpendiculares) de la aceleración cuando el movimiento sea curvilíneo.

CONCEPTOS BÁSICOS DE LA UNIDAD: Ecuación de la trayectoria, Componentes tangenciales y normales de la aceleración en movimiento curvilíneo.

Para una mejor comprensión del contenido de la unidad, realice la lectura de

los conceptos relacionados con los diferentes tipos de movimientos de partícula y de los ejemplos presentados en esta unidad, respondiendo las preguntas insertas a lo largo del desarrollo de la unidad y resolviendo los problemas propuestos al final de la misma, para ello utilice las técnicas de lectura que le faciliten la comprensión del contenido como, por ejemplo: subrayado, búsqueda del significado de términos desconocidos, elaboración de esquemas, entre otros.

2.3 CONTENIDO

2.3.1 MOVIMIENTO EN UNA DIMENSIÓN. 2.3.2 MOVIMIENTO EN EL PLANO:

PARABÓLICO CIRCULAR.

2.3.1 MOVIMIENTO EN UNA DIMENSIÓN

El Movimiento en una Dimensión es el movimiento más simple de una partícula. Este movimiento se puede realizar en una trayectoria horizontal, para lo cual lo denotaremos con las componentes de los vectores de posición, velocidad y aceleración, a lo largo del eje Ox; o se puede realizar en una trayectoria vertical, para lo cual lo denotaremos con las componentes a lo largo del eje Oy.

Las ecuaciones que expresan la posición, la velocidad y la aceleración,

respectivamente, de una partícula en un movimiento horizontal, son:

dx dvx(t), v(t)= , a(t)= (2.1)dt dt

López Frontado, R. (2011). Física general I, Texto UNA. 2da edición. Caracas: UNA

Page 58: Fisica General i Una

65

Si la aceleración que tiene la partícula, durante su movimiento, es constante, entonces la posición x(t) de ella en función del tiempo se puede expresar por la siguiente ecuación:

2o o

1x(t)=x +v t+ at (2.2)2

y la velocidad v(t) de la partícula en función del tiempo está dada por

ov(t)=v +at (2.3) También se tiene una expresión que relaciona la posición, la velocidad y la

aceleración, sin necesidad de conocer el tiempo. Esta es:

2 2o ov =v +2a(x-x ) (2.4)

NOTA: Es importante notar que la posición, la velocidad y aceleración, son

cantidades vectoriales, y las ecuaciones escritas representan las componentes correspondientes de dichas cantidades, a lo largo del eje Ox. Si el movimiento de la partícula fuese vertical, las ecuaciones representarían las componentes a lo largo del eje Oy

Antes de proporcionarle algunos ejemplos de problemas, en los que Ud.

pueda visualizar las aplicaciones de los diferentes conceptos tratadas en esta unidad, es conveniente que lea las Sugerencias presentadas como Recomendaciones para resolver problemas.

RECOMENDACIÓN EN LA RESOLUCIÓN DE PROBLEMAS Para que pueda resolver los problemas relacionados con los diferentes tipos

de movimiento, se recomienda: IDENTIFICAR los conceptos pertinentes: en casi todos los problemas de

movimiento de una partícula, ya sea en una dimensión o en dos dimensiones, se podrán las ecuaciones de aceleración constante, aunque algunas veces se le presentarán problemas en donde la aceleración no es constante.

López Frontado, R. (2011). Física general I, Texto UNA. 2da edición. Caracas: UNA

Page 59: Fisica General i Una

66

PLANTEAR el problema con los pasos siguientes:

a) Seleccione un sistema de coordenadas y dibuje sus ejes, de acuerdo con las condiciones iniciales del problema. Es preciso decidir dónde está el origen de las coordenadas. El criterio suele ser la comodidad. Recuerde que la dirección positiva de los ejes de coordenadas determina la dirección positiva de los vectores posición, velocidad y aceleración. Dibuje un gráfico que muestre el movimiento de la partícula, expresado en el problema.

b) Replantee el problema en forma literal y luego traduzca su descripción a símbolos y ecuaciones. ¿Cuándo llega la partícula a cierto punto (cuánto vale t)? ¿Dónde se encuentra la partícula cuando tiene cierta velocidad?

c) Haga una lista de las cantidades conocidas y de las incógnitas, tomando en cuenta cualquier información implícita.

EJECUTAR la solución: despeje la incógnita usando sólo símbolos, sustituya

luego los valores conocidos y calcule el valor de la incógnita. Algunas veces tendrá que resolver ecuaciones simultáneas con dos incógnitas.

EVALUAR la repuesta: Examine sus resultados para ver si son lógicos. ¿Están los resultados

dentro del rango general de valores esperados? En las dudas que tenga sobre el tema, solicite ayuda al asesor de su Centro Local.

Recuerde que el manejo eficiente del contenido de esta unidad es importante

por su repercusión en el contenido de las subsiguientes unidades del curso de Física General I.

Para darnos una idea del movimiento de un cuerpo en una dirección, veamos

los siguientes ejemplos

Ejemplo 2.1.- (Movimiento en dirección horizontal)

Un cuerpo se mueve a lo largo de una pista horizontal con una velocidad inicial de 3 m/s, y una aceleración constante de 4 m/s2, en la dirección opuesta a la de la velocidad.

a) ¿Cuál es la velocidad del cuerpo 7 s después?

López Frontado, R. (2011). Física general I, Texto UNA. 2da edición. Caracas: UNA

Page 60: Fisica General i Una

67

b) ¿Cuál es el desplazamiento y la distancia recorrida en ese lapso de tiempo?

DATOS: o o 2

m mx =0, v =3 , a=-4 s s

SOLUCIÓN: IDENTIFICAR los conceptos pertinentes: De acuerdo al enunciado del problema nos damos cuenta de que el

movimiento del cuerpo durante cierto tiempo es uniformemente retardado, hasta que llega a un punto donde cambia de dirección. Por lo tanto debemos determinar el tiempo que tarda el automóvil en cambiar la dirección de movimiento.

PLANTEAR y EJECUTAR la solución del problema: a) Como no nos indican en qué sentido se mueve el cuerpo, tenemos la

libertad de escoger este sentido del movimiento, así podemos suponer que el cuerpo se está moviendo inicialmente hacia la derecha (en el mismo sentido de la velocidad inicial), y la aceleración, la cual es constante, está dirigida hacia la izquierda (debido a que la aceleración lleva sentido contrario a la velocidad). El gráfico, mostrado en la figura (2.1), da una idea de cómo se realiza el movimiento del cuerpo, esto es:

Figura 2.1 La ecuación de la posición en función del tiempo tomando en cuenta las condiciones iniciales, está dada por

2 2o

1x = v t+ at x = 3t- 2t2

y la ecuación de la velocidad, está dada por

ov = v +at v = 3 - 4t

La velocidad a los 7 s, será mv 3 4(7) v 25s

= − ⇒ = −

Como puede observarse, la velocidad tiene un signo negativo, lo que nos indica que en este instante, la partícula se está moviendo hacia la izquierda.

López Frontado, R. (2011). Física general I, Texto UNA. 2da edición. Caracas: UNA

Page 61: Fisica General i Una

68

b) El desplazamiento está dado por Δx = x(7 s) – x(0)

Pero x(7 s) = 3(7) – 2(7)2 → x( 7 s) = - 77 m y x(0) = 0

Δx = - 77 – 0 ⇒ Δx = - 77 m

Para determinar la distancia recorrida por el cuerpo en ese lapso de tiempo es necesario observar que el cuerpo en el instante inicial se estaba moviendo hacia la derecha; y a los siguientes 7s se estaba moviendo hacia la derecha; esto nos indica que el cuerpo en un instante dado ha debido cambiar el sentido de su movimiento. Esta posición corresponde al instante en el cual la velocidad del cuerpo se hace cero, esto es:

3v=0=3-4t t= s=0,75 s4

Ahora se determina la posición donde se encuentra el cuerpo en ese instante,

sustituyendo este tiempo en la ecuación de x (t), x(0,75 s) = 3 (0,75) – 2 (0,75)2 → x(0,75) = 1,125 m Entonces, la distancia recorrida = d =⎮Δx1⎮+ ⎮Δx2 ⎮, donde Δx1 = x(0.75) – x(0) → Δx1 = 1,125 m Δx2 = x(7) – x(0,75) → Δx2 = - 78,125 m Por lo que d = ⎮1,125⎮+ ⎮- 78,125⎮ ⇒ d = 79,25 m Para tener una idea de cómo se realizo el movimiento, dibujemos el siguiente

gráfico,

Figura 2.2

EVALUAR la solución del problema:

De acuerdo al resultado del problema, se observa que el automóvil durante un tiempo de 0,75 s se mueve hacia la derecha en un movimiento uniformemente retardado, al final del cual cambia de dirección su movimiento después del recorrer

López Frontado, R. (2011). Física general I, Texto UNA. 2da edición. Caracas: UNA

Page 62: Fisica General i Una

69

una distancia de 1,125 m. A partir de ese instante su movimiento es hacia la izquierda, durante el cual el movimiento es uniformemente acelerado recorriendo una distancia de 78,125 m.

Ejemplo 2.2.- (Movimiento en Dirección Vertical)

Se lanza un cuerpo verticalmente hacia arriba con una velocidad de 50 m/s, desde el techo de un edificio de 120 m de altura. Despreciando el efecto del aire, encuentre:

a) La máxima altura que alcanza el cuerpo con respecto a la base del

edificio.

b) El tiempo necesario para alcanzar esa altura.

c) La velocidad del cuerpo al llegar a la base del edificio.

d) El tiempo total transcurrido desde que fue lanzado el cuerpo hasta que alcanza el suelo.

DATOS: Ho = 120 m, vo = 50 m/s, g = 9,8 m/s2

SOLUCIÓN: IDENTIFICAR los conceptos pertinentes: En este problema se aplican los conceptos de movimientos de un cuerpo,

tomando en cuenta que la aceleración que actúa sobre el cuerpo cuando está en movimiento es la aceleración de gravedad, la cual es constante para pequeñas alturas, despreciando el efecto del aire.

PLANTEAR el problema: De acuerdo al enunciado, el movimiento es vertical, como se muestra en la

figura (2.3). La aceleración que actúa sobre la partícula es la aceleración de gravedad, la cual está dirigida verticalmente hacia abajo. Así se tiene que la ecuación de movimiento está dada por:

2

o o atr(t) = r + v t+2

rr r r

López Frontado, R. (2011). Física general I, Texto UNA. 2da edición. Caracas: UNA

Page 63: Fisica General i Una

70

Figura 2.3

Las condiciones iniciales son :

o o

o o

ˆr = H jˆv = v jˆa = -gj

r

r

r

Sustituyendo las condiciones iniciales en la ecuación de movimiento, se tiene

2

o o

2

gtˆ ˆ ˆr(t) = H j+ v tj- j2

y(t) =120 + 50t- 4,9t (1)!

r

Y la ecuación de la velocidad será,

o yˆ ˆ ˆ ˆv(t) = v j- gtj = 50 j- 9,8tj v = 50 - 9,8t (2)!

! EJECUTAR la resolución del problema:

Aplicando las ecuaciones (1) y (2) se procede a resolver el problema.

a) Para determinar la altura máxima alcanzada por el cuerpo (en B), se obtiene primero el tiempo que tarda la partícula en llegar al punto B, cuando la velocidad es cero (vB = 0).

B 1 150v =0=50-9.8t t = t =5,1 s9,8

!

Entonces, la altura máxima será:

ymax = 120 + 50(5,1) – 4,9 (5,1)2 ⇒ ymax = 247,5 m

b) El tiempo que tarda la partícula en llegar a su altura más alta, ya la obtuvimos en la pregunta (a), y es 1 t =5,1 s! .

c) Para determinar la velocidad de la partícula cuando llega a la base del

edificio, es necesario obtener previamente el tiempo de vuelo de la partícula, o sea el tiempo que tarda la partícula en desplazarse desde el punto inicial hasta llegar a la base del edificio. Esto es,

2

2C v

50 ± (50) + 4×4,9×120y = 0 = 120 + 50 t- 4,9 t t =

9,8

→ tv = 10,62 s

López Frontado, R. (2011). Física general I, Texto UNA. 2da edición. Caracas: UNA

Page 64: Fisica General i Una

71

Sustituyendo en la ecuación (2), se tiene

C Cmv = 50 - 9,8(10,62) v = -54,15 s

El signo negativo de la velocidad nos indica que la partícula se está moviendo verticalmente hacia abajo.

d) El tiempo total es el tiempo de vuelo, o sea el tiempo que tarda el cuerpo de haber sido lanzado hasta que cae al suelo, esto es

vt 10,62s⇒ = .

EVALUAR el resultado: los resultados obtenidos nos dan una idea de cómo, aplicando los conceptos pertinentes y las condiciones iniciales, se pueden obtener los resultados pedidos.

SUGERENCIA: Ahora resuelva este problema, tomando como origen del sistema de coordenadas al punto A, posición de donde fue lanzada la partícula, y aplique las condiciones iniciales correspondientes.

Ejercicio propuesto 2.1.-

Un auto se mueve a una rapidez constante de 30 m/s, y súbitamente se

apaga el motor en la parte inferior de una colina. El auto experimenta una desaceleración constante de 2 m/s2, mientras asciende la colina.

a) Escriba las ecuaciones de la posición y la velocidad como funciones del

tiempo, tomando x = 0 en la parte inferior de la colina, en donde vo = 30 m/s.

b) Determine la distancia máxima recorrida por el auto hacia la parte

superior de la colina, después de haberse apagado el motor.

López Frontado, R. (2011). Física general I, Texto UNA. 2da edición. Caracas: UNA

Page 65: Fisica General i Una

72

Ejercicio propuesto 2.2.-

Se lanza un cuerpo verticalmente hacia arriba, desde una altura de 50 m, y se observa que tarda 15 s en llegar al suelo.

a) ¿Con qué velocidad se lanzó el objeto?

b) ¿Qué velocidad tiene 2 s antes de llegar al suelo?

c) ¿Cuál es la máxima altura alcanzada por el objeto?

Ejemplo 2.3.- (PROBLEMA DE ENCUENTRO EN 1D)

Desde el techo de un edificio lo suficientemente alto, se lanza una piedra

verticalmente hacia arriba con una velocidad de 30 m/s. Desde el mismo punto, se deja caer otra piedra, 4 s después de haberse lanzado la primera. Determine:

a) ¿Cuánto tiempo después de haberse lanzado la primera, se encuentran

las piedras?

b) ¿En qué punto se encuentran?

c) Calcule la velocidad de cada piedra cuando se encuentran.

DATOS: Ao Bo A Bmv =30 , v =0, t =t +4 ss

SOLUCIÓN IDENTIFICAR los conceptos relacionados con el problema: en este problema,

llamado de encuentro, se relaciona el movimiento de dos cuerpos que se encuentran después de un cierto tiempo, los cuales se realizan bajo el efecto de una aceleración constante.

PLANTEAR el problema: En este problema se establecen las ecuaciones de movimiento para cada

cuerpo, tomando en cuenta sus condiciones iniciales. Se tiene presente que el

López Frontado, R. (2011). Física general I, Texto UNA. 2da edición. Caracas: UNA

Page 66: Fisica General i Una

73

encuentro se produce cuando la posición de los dos cuerpos es la misma. Así se tiene que conviene designar al primer cuerpo, como cuerpo A, y al segundo, como cuerpo B. Luego se hace un diagrama, que nos dé idea de cómo será el movimiento de los cuerpos, como se muestra en la figura (2.5). Se selecciona un sistema de coordenadas para los dos cuerpos. Así se tiene que,

Las ecuaciones del movimiento y de velocidad, son:

20 0 0

1r(t) = r + v t+ at y v(t) = v +at2

r r r rr r r

Las condiciones iniciales para cada cuerpo son:

Ao Bo

Ao Bo

A B

r = o r = 0ˆv = 30 j v = 0

ˆ ˆa = -gj a = -gj

r r

r r

r r

Figura 2.4

Al sustituir las condiciones iniciales se obtienen las ecuaciones de movimiento, correspondiente para cada cuerpo, así se tiene que, 2 2

A A A B B1 1ˆ ˆ ˆr = 30t j- gt j r = - gt j2 2

! !

De donde se obtiene las correspondientes ecuaciones

paramétricas, esto es,

2 2

A A A B B

A A B B

1 1y = 30t - gt (1), y = - gt (2)2 2

v =30-gt (3), v =-gt (4)

EJECUTAR la resolución del problema:

a) El momento de encuentro de las piedras se produce cuando yA = yB =

yE, esto es,

2 2 2A A A A A A A

A A

1 1 1 130t - gt =- g(t -4)2, 30t - gt =- gt +4gt -8g2 2 2 28gt = t =8,52 s

4g-30!

b) Para determinar el punto de encuentro, se sustituye tA en cualquiera de

las ecuaciones de y(t), sea la del cuerpo A o la del cuerpo B, esto es, 2

E A A B B1y =y (t =8,52)=y (t =4,52)=- (9,8)(4,52)2

E y =-100,2 m!

López Frontado, R. (2011). Física general I, Texto UNA. 2da edición. Caracas: UNA

Page 67: Fisica General i Una

74

c) Para determinar la velocidad de cada cuerpo en el instante del encuentro, se sustituyen las condiciones iniciales del problema en las ecuaciones de las velocidades, esto es,

A A B B

A B

A B

v =30-9,8t y v =-9,8tm mv =30-9,8(8,52)=-53 y v =-9,8(4,52)=-44,3 s s

m mˆ ˆ v =-53j y v =-44,3j s s

!r r

EVALUAR el resultado del problema: el instante del encuentro se produce a

los 8,52 s después de haberse lanzado la piedra A. Su punto de encuentro está situado a 100,2 m de altura. Las velocidades de cada piedra en ese instante están dadas por A Bv y v .

r r

Ejercicio propuesto 2.3.-

Un auto está esperando que cambie la luz roja del semáforo. Cuando la luz cambia a verde, el auto acelera uniformemente durante 6 s a razón de 2 m/s2, después de lo cual se mueve con velocidad constante. En el instante que el auto comienza a moverse, un camión que se mueve en la misma dirección con movimiento uniforme lo pasa. Determine a qué distancia del semáforo, y en qué tiempo después se encontrarán nuevamente el auto y el camión, si la velocidad del camión es: a) 5 m/s; b) 10 m/s; y c) 15 m/s.

2.3.2 MOVIMIENTO EN EL PLANO: PARABÓLICO Y CIRCULAR

MOVIMIENTO DE UN PROYECTIL Un proyectil es un cuerpo que recibe una velocidad inicial, y su movimiento

sigue una trayectoria determinada por los efectos de la aceleración de gravedad y la resistencia del aire.

Analizaremos este tipo de movimiento, suponiendo que la partícula se

desplaza con una aceleración constante, despreciando el efecto del aire, la curvatura y la rotación de la Tierra. El movimiento de la partícula está limitado a un plano vertical, determinado por la velocidad inicial.

Para ilustrar este tipo de movimiento, consideremos un proyectil que se lanza

desde un punto situado a una altura Ho, con una velocidad inicial ov!

, la cual forma un

López Frontado, R. (2011). Física general I, Texto UNA. 2da edición. Caracas: UNA

Page 68: Fisica General i Una

75

ángulo θ con la horizontal. En la figura (2.5) se muestra un esquema del movimiento de este proyectil. Esto es,

La ecuación que describe el movimiento es 2o o

1r(t)=r +v t+ at2

! ! !! y la velocidad

es ov=v +at!! !

Figura 2.5

Las condiciones iniciales son:

o o

o ox oy ox o

oy o

ˆr =H jˆ ˆv =v i+v j, v =v cosθ

v =v senθˆa=-gj

!

!

!

Sustituyendo estas condiciones iniciales en las ecuaciones de movimiento y

de la velocidad, se tiene,

2o ox oy

ox oy

1ˆ ˆ ˆ ˆr(t)=H j+v t i+v tj- gt j (2.5)2

ˆ ˆ ˆv(t)=v i+v j-gtj (2.6)

!

!

De donde se tiene que las ecuaciones paramétricas y de las componentes de

la velocidad, son respectivamente:

oxx ox

2y oyo oy

x(t)=v t (2.7) v =v (2.9) , y 1 v =v -gt (2.10)y(t)=H +v t- gt (2.8)

2

! !" "# #

"$"$

A partir de las ecuaciones paramétricas, se puede obtener la ecuación de la

trayectoria que describe el proyectil. Para esto, se despeja el tiempo de la ecuación (2.7):

ox o

x xt= =v v cosθ

y luego se sustituye en la ecuación (2.8), obteniéndose

2 2

oo o2 2 2 2

o o o

v senθ g x g xy=H + x- =H +tanθ x - v cosθ 2v cos θ 2v cos θ

López Frontado, R. (2011). Física general I, Texto UNA. 2da edición. Caracas: UNA

Page 69: Fisica General i Una

76

Al simplificarse, se obtiene 2

o 2 2o

gxy=H +tanθ x- (2.11)2v cos θ

La ecuación (2.11) nos indica que la trayectoria del proyectil es una parábola, razón por la cual este movimiento se denomina MOVIMIENTO PARABÓLICO. En este tipo de movimiento, la velocidad en cualquier instante está dada por:

x y o oˆ ˆ ˆ ˆv(t)=v i+v j=v cosθ i+(v senθ -gt)j (2.12)

r

La rapidez del proyectil (la magnitud de su velocidad) en cualquier instante

está dado por 2 2x yv= v +v (2.13)

Y la dirección de la velocidad, en función del ángulo ϕ que forma con el eje x+, está dado por

y

x

vtan = (2.14)

v!

En este tipo de movimiento, la altura máxima se obtiene cuando la

componente de la velocidad a lo largo del eje y se hace cero, esto es vy = 0. El desplazamiento horizontal máximo se obtiene conociendo primero el tiempo de vuelo tv del proyectil.

Estos resultados son válidos cuando:

a) El alcance horizontal es tan pequeño como para despreciar la curvatura de la Tierra.

b) La altura es suficientemente pequeña como para despreciar la variación de

la aceleración de gravedad con la altura.

Además se considera que la velocidad inicial es tan pequeña como para despreciar la resistencia del aire.

López Frontado, R. (2011). Física general I, Texto UNA. 2da edición. Caracas: UNA

Page 70: Fisica General i Una

77

Para darnos una idea del movimiento de un cuerpo a lo largo de una

trayectoria parabólica, veamos los siguientes ejemplos:

Ejemplo 2.4.-

Se lanza una botella de refresco, desde una ventana de un edificio situada a una altura de 40,0 m, con una velocidad de 30 m/s en dirección horizontal. Encuentre:

a) ¿A qué distancia horizontal de la base del edificio cae la botella?

b) ¿Cuál es la velocidad de la botella cuando llega al suelo?

c) ¿Cuál es la velocidad de la botella, cuando pasa por un punto situado a

una altura de 2 m?

DATOS: Ho = 40,0 m, vo = 30,0 m/s.

SOLUCIÓN; IDENTIFICAR los conceptos pertinentes al problema: en este problema se

describe el movimiento de una partícula, tomando en consideración que la velocidad inicial forma un ángulo con el eje horizontal, y la partícula describe una trayectoria parabólica.

PLANTEAR el problema: para darnos una idea acerca del movimiento de la

botella, hagamos un esquema referente al mismo, como se muestra en la figura (2.6). Además consideremos que la aceleración que actúa sobre la botella es la aceleración de gravedadgr , la cual podemos considerar que se mantiene constante. Así se tiene que las ecuaciones de movimiento y de la velocidad de la partícula son:

2o o o

1r(t) = r + v t+ at , v = v +at2

! ! ! !! ! !

Figura 2.6

Las condiciones iniciales son:

o o

o o

2

ˆ ˆr =H j=40,0 m jmˆ ˆv =v i=30,0 is

mˆ ˆa=-gj=-9,8 js

!

!

!

López Frontado, R. (2011). Física general I, Texto UNA. 2da edición. Caracas: UNA

Page 71: Fisica General i Una

78

Al sustituir las condiciones iniciales en las ecuaciones de movimiento y de velocidad, se tiene:

29,8ˆ ˆ ˆ ˆ ˆr=40j+30ti- t j y v=30i-9,8tj

2! !

Las ecuaciones paramétricas y las de las componentes de la velocidad son:

x2

y

x=30t v =30

y=40-4,9t v =-9,8t

Una vez, conocidas las ecuaciones paramétricas y las de las componentes de

la velocidad, procedemos determinar lo que se nos pide. Se procede entonces de la siguiente manera:

EJECUTAR la resolución del problema: a) Para determinar a qué distancia horizontal de la base del edificio cae la

botella, se obtiene previamente el tiempo de vuelo de la botella. El tiempo de vuelo se tiene cuando y = 0, esto es:

2v v

40y = 0 = 40 - 4,9 t , t = t = 2,86 s4,9

!

Ahora se sustituye este valor del tiempo de vuelo en la ecuación de x(t), obteniéndose que

v vx(t ) = 30(2,86) x(t ) = 85,8 m!

b) Para determinar la velocidad de la botella, cuando llega al suelo, se sustituye el tv en las ecuaciones de las componentes de la velocidad, obteniéndose entonces,

c)

x

y

mv =30 ms ˆ ˆ v=(30i-28j)

m sv =-9,8(2,86)=-28 s

!""

#$""%

!

su magnitud está dado por:

2 2 mv= 30 +28 v=41.04 s

!

y su dirección es:

-1 o-28φ=tan φ=-43,0330

⎛ ⎞ ⇒⎜ ⎟⎝ ⎠

López Frontado, R. (2011). Física general I, Texto UNA. 2da edición. Caracas: UNA

Page 72: Fisica General i Una

79

c) Para determinar la velocidad de la botella, cuando está a 2 m de altura, se tiene:

2 2 38y(t)=2=40-4,9t t = t=2,78 s

4,9→ ⇒

Se sustituye luego, este tiempo en las ecuaciones de las componentes de la velocidad:

x

y

mv =30 ms ˆ ˆ v=(30i-27,2j)

m sv =-9,8(2,78)=-27,2 s

⎧⎪⎪ ⇒⎨⎪⎪⎩

!

La magnitud de la velocidad y su dirección son:

2 2

-1 o

mv= 30 +(-27,2) v=40,53 s

-27.2φ=tan φ=-42,230

!

" # !$ %& '

EVALUAR el resultado del problema: los resultados demuestran que las

velocidades en cualquier punto de la trayectoria descrita por el cuerpo tiene dos componentes, una en la dirección del eje x, y la otra en la del eje y, la cual es negativa por estar dirigida hacia abajo, o sea en sentido contrario al eje y.

Ejercicio propuesto 2.4.-

En un juego de béisbol se da un batazo de vuelta entera, de modo que la pelota apenas pasa por encima de una pared de 21 m de altura, localizada a 130 m de la base de bateo. La pelota sale formando un ángulo de 35º con la horizontal, y la resistencia del aire es despreciable. Suponiendo que la pelota se golpea a una altura de 1 m del piso, calcule:

La rapidez inicial de la pelota. El tiempo que tarda en llegar a la pared. La velocidad y su rapidez al pasar sobre la pared.

López Frontado, R. (2011). Física general I, Texto UNA. 2da edición. Caracas: UNA

Page 73: Fisica General i Una

80

Ejercicio propuesto 2.5.-

Un automóvil está estacionado en un plano inclinado muy pronunciado que domina el océano en un lugar, en el que tal plano forma un ángulo de 37º con la horizontal. El negligente conductor deja el automóvil en neutro y los frenos están dañados. El automóvil empieza a rodar desde el reposo hacia abajo del plano inclinado, con una aceleración constante de 4 m/s2, y recorre 50 m hasta el borde del acantilado. El acantilado tiene 30 m de altura por arriba del océano. Calcule:

a) La rapidez del automóvil cuando llega al borde del acantilado y el

tiempo que tarda en llegar a él.

b) La velocidad del automóvil al caer al mar.

c) El tiempo total que dura su movimiento.

d) ¿A qué distancia horizontal de la base del acantilado cae el automóvil en el agua?

Ejemplo 2.5.- (MOVIMIENTO DE ENCUENTRO EN 2D)

Un cañón situado en un acantilado a una altura de 110 m puede disparar

proyectiles a una velocidad de 200 m/s, bajo un ángulo de 37º con la horizontal. El artillero que acciona el cañón, observa por medio del radar, que un barco desconocido se acerca a la costa con una velocidad constante de 7,2 m/s. Se pregunta: ¿a qué distancia se encontraba el barco de la costa, tal que el artillero hace un disparo y logra hacer impacto? DATOS: Ho = 110 m, VAo = 200 m/s, VB = 7,2 m/s, θ = 37°.

SOLUCIÓN: IDENTIFICAR los conceptos pertinentes: En este problema se consideran el movimiento de los cuerpos bajo el efecto

de una aceleración constante, tomando en consideración que para producirse el encuentro es necesario que la posición de los cuerpos, sea la misma.

López Frontado, R. (2011). Física general I, Texto UNA. 2da edición. Caracas: UNA

Page 74: Fisica General i Una

81

PLANTEAR el problema: Se establecen la ecuaciones de movimiento de cada cuerpo por separado,

considerando que sus movimientos se producen por efecto de aceleración de gravedad, la cual supondremos que su valor se mantiene constantes, para pequeñas variaciones de alturas. Sus movimientos los podemos esquematizar en la (figura 2.7).

Figura 2.7

Se escoge un sistema de coordenadas, representado por los ejes x,y, común para los dos cuerpos, tanto para el proyectil como para el barco.

La aceleración que actúa sobre el proyectil es la aceleración de gravedad, que

se pueda sea constante. La ecuación de movimiento de los cuerpos, se puede escribe entonces, como

2o o

1r(t) = r + v t+ at2

r r rr

Tomando en cuenta las condiciones iniciales de los cuerpos con respecto al

sistema de coordenadas escogido, se tiene: Cuerpo A (Proyectil) Cuerpo B (Barco)

Ao Ao Bo

o oAo Ao Ao Bo B

A Bx

ˆr = H j r = 0ˆ ˆ ˆv = v cos 37 i+ v sen 37 j v = -v i

ˆa = -gj a = 0

! !

! !

! !

Sustituyendo estas condiciones iniciales en la ecuación de movimiento del

cuerpo A, y del cuerpo B, se tienen,

o o 2A o Ao A Ao A A

B B B

1ˆ ˆ ˆ ˆr (t) =H j+ v t cos37 i+ v t sen37 j- gt j2

ˆr (t) = (-v t +D) i

!

!

Las ecuaciones paramétricas serán:

Cuerpo A Cuerpo B o

A Ao A B B B

o 2A o Ao A A B

x (t) = v t cos37 x (t) = -v t +D1y (t) = H + v t sen37 - gt y (t) = 02

López Frontado, R. (2011). Física general I, Texto UNA. 2da edición. Caracas: UNA

Page 75: Fisica General i Una

82

EJECUTAR la resolución del problema:

De acuerdo al enunciado del problema, consideremos que el movimiento de ambos cuerpos son simultáneos, o sea que el tiempo que tarda cuerpo A en llegar al punto de encuentro es igual al tiempo que tarda el cuerpo B, esto es tA = tB. Las condiciones del encuentro son:

A B A Bx = x cond (1), y y = y cond (2)

Entonces se tiene

oAo B

o 2o Ao

v tcos37 = -v t+D cond. (1)1H + v tsen37 - gt = 0 cond. (2)2

Al sustituir los datos del problema en la ecuación de la condición (2), se

determina el tiempo, esto es,

2 o 24,9 t - 200sen37 t-110 = 0 4,9 t -120,4 t-110 = 0 t = 25,4 s!

Luego se sustituye este tiempo en la ecuación de la condición (1), así se tiene

la distancia D, a la cual se encontraba el barco en el momento del disparo, esto es:

oD=200cos37 (25,4)+7,2(25,4)D=4057,1+182,9 D=4240,0 m⇒

EVALUAR el resultado del problema: La distancia a la que se encuentran el proyectil y el barco es de 4240,0 m con

respecto al artillero.

SUGERENCIA: Resuelva el problema considerando que el barco, en lugar de acercarse se está alejando de la costa.

MOVIMIENTO CURVILÍNEO

Cuando una partícula describe una trayectoria curvilínea cualquiera, la

velocidad v!

de la partícula es tangente a la trayectoria en cualquier punto de la curva; mientras que la aceleración a

! está dirigida hacia el lado cóncavo de la

trayectoria, de tal forma que la podemos descomponer en dos componentes perpendiculares entre si, como se muestra en la figura (2.8), esto es: una en la

López Frontado, R. (2011). Física general I, Texto UNA. 2da edición. Caracas: UNA

Page 76: Fisica General i Una

83

dirección tangente, así tendremos la componente tangente de la aceleración, llamada como aceleración tangencial Ta

!; y otra componente en la dirección

perpendicular, llamada aceleración normal Na!

.

Figura 2.8

T N

T T N N

a=a +aˆ ˆa=a u +a u (2.15)

! ! !

!

T

T

TT

ˆv=vu , entoncesdv d ˆa= = (vu )dt dt

ˆdudv ˆa= u +v (2.16)dt dt

!

!!

!

Si la trayectoria fuese una recta, uT sería constante en magnitud y dirección,

por lo que Tˆdu =0.dt

Cuando la trayectoria es curva, la dirección de Tu cambia, dando un valor

diferente de cero para Nˆdu .dt

Figura 2.9

En la figura (2.9) consideremos dos puntos sucesivos, muy cercanos, en la trayectoria de la partícula A y A´, separados una distancia ds. Las normales (líneas perpendiculares) a la trayectoria, en los puntos A y A´, se intersectan en el punto C, llamado centro de curvatura. El radio de curvatura ρ = CA.

La aceleración está dada por

2

T Ndv vˆ ˆa= u + u (2.17)dt ρ

r

Comparando la ecuación (2.14) con la ecuación (2.16), se tiene

2

T Ndv va = y a = (2.18)dt !

López Frontado, R. (2011). Física general I, Texto UNA. 2da edición. Caracas: UNA

Page 77: Fisica General i Una

84

La magnitud de la aceleración en el punto A es:

22 22 2T N

dv va= a +a = + (2.19)dt !

! "! "# $# $

% & % &

Si el movimiento curvilíneo es uniforme, la rapidez v = constante, y por lo tanto la aceleración tangencial Ta

! = 0.

Si el movimiento curvilíneo es rectilíneo, el radio de curvatura

ρ

=

, entonces la aceleración normal

Na!

= 0.

Así se puede observar que las componentes de la aceleración, tienen un significado físico bien definido. Aceleración tangencial Ta

! esta relacionado con el cambio de magnitud de

la velocidad. Aceleración normal Na

! esta relacionado con el cambio de dirección de la

velocidad. Un caso muy especial del movimiento curvilíneo es el movimiento circular,

donde la trayectoria descrita por la partícula es un círculo; por eso este tipo de movimiento se llama movimiento circular.

MOVIMIENTO CIRCULAR En este tipo de movimiento, el radio de curvatura es el radio de la

circunferencia (ρ = R = CA). La velocidad de la partícula v!

es tangente al círculo, y por lo tanto es perpendicular al radio R, como se muestra en la figura (2.10), esto es:

Figura 2.10

El arco de circunferencias=R! . La rapidez ds dθv= =R =Rω (2.20)dy dt

donde dθω=dt

es la velocidad angular, representa la

variación del ángulo en la unidad de tiempo, y se expresa en rad/s o s-1.

López Frontado, R. (2011). Física general I, Texto UNA. 2da edición. Caracas: UNA

Page 78: Fisica General i Una

85

La posición angular de una partícula que se mueve en un movimiento circular está dado por (t)! . También se puede definir el desplazamiento angular como el cambio de la posición angular, esto es:

f i= - (2.21)! ! !"

Tanto la posición angular como el desplazamiento angular se expresan en rad

en el sistema de unidades SI. La velocidad angular está dada por:

d= (2.22)dt!

"

Cuando la velocidad angular de la partícula no es constante, o sea que

cambia a medida que transcurre el tiempo, la aceleración angular

α

está dado por:

2

2

d d= = (2.23)dt dt! "

#

La aceleración angular α, se expresa en rad/s2 o s-2 Si la aceleración angular α es constante (o sea cuando el movimiento

circular es uniformemente acelerado), la velocidad angular para cualquier tiempo es:

o= + t (2.24)ω ω α

La posición angular θ, para cualquier tiempo está dada por

2

o ot= + t+ (2.25)2αθ θ ω

Un caso de interés especial es el de movimiento circular uniforme, donde ω

es constante. En este caso, el movimiento es periódico, esto es que la partícula pasa por cada punto del círculo en intervalo de tiempos iguales.

Período es el tiempo que tarda una partícula en realizar una vuelta completa,

o revolución; se expresa en segundos (s). Frecuencia es el número de revoluciones por unidad de tiempo; se expresa en rev/s o s-1 o en Hertz (Hz).

López Frontado, R. (2011). Física general I, Texto UNA. 2da edición. Caracas: UNA

Page 79: Fisica General i Una

86

Así, si en un tiempo t, la partícula realiza n revoluciones, el período T es:

tiempo tPeríodo= T= (2.26)No. de vueltas n

No. de vueltas nla frecuencia= f= (2.27)tiempo t

El período y la frecuencia están relacionadas por 1T=f

La velocidad angular se puede expresar Δθω=Δt

Cuando la partícula da una vuelta completa Δθ = 2 π, y el intervalo de tiempo Δt = T, entonces

2= =2 f (2.28)Tπω π

De la definición de las aceleraciones tangencial y normal, se tiene

Tdv da = =R =R (2.29)dt dt

ω α

2 2 2

2N

v Ra = = =R (2.30)R R

ω ω

En un movimiento circular, tanto la velocidad angular como la aceleración

angular, se pueden expresar en forma vectorial. Así se tiene:

v= ×r ydv dra= = × = ×v= ×( ×r) (2.31)dt dt

!

! ! ! !

!! !!!

! !!! ! ! !

La figura (2.11) muestra la relación vectorial entre la velocidad angular, la

velocidad lineal y el vector posición en el movimiento circular. La figura (2.12)

López Frontado, R. (2011). Física general I, Texto UNA. 2da edición. Caracas: UNA

Page 80: Fisica General i Una

87

muestra la relación vectorial entre la velocidad lineal, aceleración tangencial, aceleración normal, velocidad angular, aceleración angular y el vector posición.

Figura 2.11

Figura 2.12

Es importante notar que la relación vectorial mostrada en la figura (2.12) se

refiere para un Movimiento Circular Uniformemente Acelerado. Cuando el Movimiento de la Partícula es Circular Uniformemente Desacelerado, tanto la aceleración tangencial Ta

! como la aceleración angular !! , están dirigidas en sentido

contrario.

Para darnos una idea del movimiento circular de un cuerpo, veamos los siguientes ejemplos

Ejemplo 2.6 Se tiene un disco que gira con movimiento circular uniforme 13,2 rad cada 6

segundos.

a) Calcule la velocidad angular del disco.

b) Calcule el período y la frecuencia de rotación.

c) ¿Qué tiempo le tomará al disco en girar un ángulo de 780º?

d) ¿Qué tiempo le tomará al disco en dar 12 revoluciones?

DATOS: =13,2 rad,θΔ t=6 sΔ , =780ºθ , n = 12 rev.

López Frontado, R. (2011). Física general I, Texto UNA. 2da edición. Caracas: UNA

Page 81: Fisica General i Una

88

SOLUCIÓN: IDENTIFICAR los conceptos pertinentes al problema: En este problemas

consideremos las condiciones necesarias para que el cuerpo describa un movimiento circular, tomando en cuenta el efecto de la aceleración que actúa sobre el disco.

PLANEAR Y EJECUTAR la solución del problema: considerando que el

movimiento circular es uniforme, lo cual nos indica que la aceleración angular es cero. Así se tiene que:

a) Para determinar la velocidad angular, se tiene que

Δθ 13,2 rad radω= = ω =2,2 Δt 6 s s

!

b) El período T de rotación, es

2π 2πT= = T=2,86 sω 2,2

!

La frecuencia f es

1 1f = = f = 0,35 HzT 2,86

c) Para determinar el tiempo que tarda en describir un ángulo de 780º , se

tiene que

o

o

o

o

= t, pero se conoce que 2 rad 360 x rad 780

2 780x rad = x = 4,33 rad360

4,33Pero = t t = = t = 6,2,2

! " #

##

! #! "

"

$

$

%

$ % 18 s

d) Para determinar el tiempo que tarda en dar 12 revoluciones, se tiene

que

24= 2 12 = 24 rad, de donde t = t = 34,3 s2,2

!" ! ! #

EVALUAR el resultado del problema: los resultados y sus unidades

concuerdan con el sistema de unidades escogidos previamente.

López Frontado, R. (2011). Física general I, Texto UNA. 2da edición. Caracas: UNA

Page 82: Fisica General i Una

89

Ejemplo 2.7

Una partícula se está moviendo en un círculo, cuyo radio R = 50 cm, de

acuerdo a la ley θ = 3t2 + 2t, donde θ se expresa en radianes y t en segundos. Calcule:

a) La velocidad angular y la aceleración angular después de 4 s.

b) Las aceleraciones tangencial y normal en ese mismo instante.

DATOS: R = 0,5 m. SOLUCIÓN: IDENTIFICAR los conceptos pertinentes: En este problema se considera que la aceleración no es constante, por esa

razón las incógnitas pedidas corresponden a un tiempo determinado. PLANTEAR Y EJECUTAR el problema: En este problema consideramos como determinar la velocidad y la aceleración

angular a partir de la ecuación que expresa el movimiento de la partícula. Así se tiene que:

a) Para determinar la velocidad angular y la aceleración angular, se tiene

2

dθω= =6t+2, entoncesdt

radω(4 s)=24+2 ω=26 s

dω rad α= =6 dt s

!

!

b) La aceleración tangencial y la normal, están dadas por:

T T 2

2 2N N 2

ma = R = 0,5×6 a = 3 s

ma = R = 0,5(26) a = 3,38 s

α

ω

EVALUAR el resultado del problema: estos valores corresponden a las incógnitas pedidas en este movimiento circular, para un tiempo de 4 s.

López Frontado, R. (2011). Física general I, Texto UNA. 2da edición. Caracas: UNA

Page 83: Fisica General i Una

90

Ejercicio propuesto 2.6.- Un volante de 30 cm de radio, tiene una velocidad angular de 1200 rpm y al

cabo de 10 s su velocidad es de 400 rpm. Calcule:

a) La aceleración angular del volante.

b) Número de vueltas que ha dado el volante en ese tiempo.

c) El tiempo que tarda en detenerse.

d) La rapidez de un punto situado en el borde del volante 2 s antes de detenerse.

Ejercicio propuesto 2.7.-

Un disco de radio R = 50 cm, se acelera uniformemente desde una velocidad angular ω1 = 8π rad/s hasta una velocidad angular ω2 = 10π rad/s, y da tres vueltas completas durante esta aceleración. Calcular:

a) El tiempo que duró la aceleración.

b) La aceleración angular.

c) La velocidad y las aceleraciones normal y tangencial de un punto de la

periferia del disco al cabo de tres vueltas.

López Frontado, R. (2011). Física general I, Texto UNA. 2da edición. Caracas: UNA

Page 84: Fisica General i Una

91

RESUMEN

Las ecuaciones usadas, para estudiar el movimiento de una partícula, son:

r(t), Posición de la partícula.dr(t)v(t)= , Velocidad de la partícula.dt

dv(t)a(t)= , Aceleración de la partícula.dt

r

rr

rr

Si la aceleración ar de la partícula en movimiento es constante, y tiene una

velocidad ovr y posición or

r en el momento inicial, esto es cuando t = 0, entonces sus vectores posición y velocidad para cualquier tiempo posterior están dados por:

2o o

o

1r(t)=r +v t+ at 2

v(t)=v +at

! ! !!

!! !

Si el movimiento de la partícula se realiza a lo largo de una dirección, ya sea

en la dirección horizontal o vertical, se utilizan las correspondientes componentes de los vectores posición y de la velocidad, esto es, las componentes en x si el movimiento es en dirección horizontal, y las componentes en y si el movimiento es en la dirección vertical.

Si el movimiento de la partícula es en caída libre, o bajo una aceleración

constante ay = -g (aceleración de gravedad). El movimiento de proyectiles es un tipo de movimiento bidimensional bajo

aceleración constante, donde ax = 0, ay = - g. Una partícula que se mueve en un circulo de radio r con rapidez constante v

está en movimiento circular uniforme. La magnitud de ra es la aceleración centrípeta ac. Esto es,

2

cva =r

, y su dirección es hacia el centro del círculo.

López Frontado, R. (2011). Física general I, Texto UNA. 2da edición. Caracas: UNA

Page 85: Fisica General i Una

92

EJERCICIOS DE AUTOEVALUACIÓN:

Instrucciones: en su libreta de apuntes resuelva los siguientes ejercicios, si tiene dificultad en alguno de ellos, consulte a sus compañeros o al asesor de su centro local.

2.1.- Si la ecuación de la coordenada de un objeto en función del tiempo

es 3 3x(t)=(2,2 m/s )t -(18 m/s)t, determine: a) ¿Cuál es la componente de la velocidad media del objeto entre ti = 1,0 s y tf = 6,0 s? b) ¿Cuál es la componente de la velocidad del objeto en t = 5,0 s? c) ¿Cuál es el desplazamiento del objeto entre ti = 1,o s y tf = 6,0 s? d) ¿Cuáles la distancia recorrida por el objeto entre ti = 1,0 s y tf = 6,0 s?

2.2.- Una jardinera inicialmente en reposo, cae desde el borde de una

ventana se mantiene situado a 6,2 m sobre el suelo. a) ¿Cuál es la rapidez de la jardinera cuando choca con el suelo? b) ¿Cuánto tarda en caer? c) ¿Cuál es la altura de la jardinera a los 0,5 s de haber iniciado su movimiento? d) ¿Cuál es la aceleración de la jardinera a los 0,5 s?

2.3.- Un balón de fútbol se desplaza horizontalmente una distancia de 17 m

antes de chocar con el suelo. El punto desde el que fue lanzado está 1,5 m por encima del suelo y el ángulo de lanzamiento fue de 16°. ¿Cuál fue el módulo de la velocidad inicial de la pelota?

2.4.- Dos autos viajan a la misma rapidez sobre una carretera recta. El

primero lleva una delantera de 100 m al segundo. El segundo automóvil desarrolla una aceleración constante de 2,4 m/s2, mientras que la aceleración constante del primero es de 1,8 m/s2. a) Determine el tiempo necesario para que el segundo alcance al primero. b) Calcule la diferencia en rapidez del segundo con respecto al primero , al efectuar el rebase.

2.5.- Un niño hace girar a una piedra en un círculo horizontal situado a 1,9 m

sobre el suelo por medio de una cuerda de 1,4 m de longitud. La cuerda se rompe y la piedra sale disparada horizontalmente, golpeando el suelo a 11 m de distancia horizontal. ¿Cuál fue la aceleración centrípeta de la piedra mientras estaba en movimiento circular? RESPUESTA DE EJERCICIOS PROPUESTOS:

Estas respuestas se corresponden con los problemas ubicados en el desarrollo de la unidad.

López Frontado, R. (2011). Física general I, Texto UNA. 2da edición. Caracas: UNA

Page 86: Fisica General i Una

93

Respuesta al ejercicio propuesto 2.1.- DATOS: , o o2

m mˆ ˆv 30 i , a 2 i r 0s s

= = − =! !!

SOLUCIÓN:

Figura 2.13

a) Tomando en cuenta las condiciones iniciales del problema, las ecuaciones de posición y de la velocidad de la partícula son:

2 2o

o

1x(t) x v t at , x(t) 30t t2

v(t) v at v(t) 30 2t

o= + + ⇒ = −

= − ⇒ = −

b) Para determinar la distancia recorrida por el auto antes de detenerse,

se necesita conocer el tiempo que tarda en detenerse, esto es el tiempo que tarda el auto en tener una rapidez igual a cero, así se tiene

30v 0 30 2t, t t 15s2

= = − = → =

Ahora, sustituyendo este tiempo en la expresión de x (t), obtenemos la

distancia recorrida por el auto antes de detenerse, esto es,

2x(15)=10(15)-(15) x=225 m⇒

Respuesta al ejercicio propuesto 2.2.-

DATOS: Ho = 50 m, tv = 15 s, g = 9,8 m/s2 SOLUCIÓN:

Figura 2.14

Como la aceleración que tiene la partícula, después de haber sido disparada hacia arriba, es la aceleración de la gravedad, la cual es constante, las ecuaciones que proporcionan el movimiento y la velocidad de la partícula en cualquier instante, son: se tiene que 2

o o o1r r v t at (1), y v v at (2)2

= + + = +r r r rr r r

Las condiciones iniciales del problemas son: o o o o

ˆ ˆ ˆr H j, v v j, a gj= = = −! !!

López Frontado, R. (2011). Física general I, Texto UNA. 2da edición. Caracas: UNA

Page 87: Fisica General i Una

94

Aplicando las condiciones iniciales, las ecuaciones (1) y (2) son:

2o o o

1ˆ ˆ ˆ ˆ ˆr H j v tj gt j y v v j gtj2

= + − = −! !

Las ecuaciones paramétricas, que indican la posición de la partícula, y la

velocidad de la misma, en cualquier instante posterior al lanzamiento de la partícula son:

2o o o

1y H v t gt (3) y v v gt (4)2

= + − = −

a) Para determinar la velocidad inicial, aplicamos la ec. (3) en el punto C.

Así se tiene que,

2C o o

my 0 50 v (15) 4,9(15) v 70,2s

= = + − ⇒ =

b) Para determinar esta velocidad, sustituimos en la ec. (4) el tiempo de

t = 13 s, esto es:

mv(13s) 70,2 9,8(13) v(13 s) 57,2s

= − ⇒ = −

c) Para determinar el punto más alto de su trayectoria, se debe obtener

previamente el tiempo que tarda en llegar a este punto. Esto se obtiene considerando que la velocidad en ese punto vale cero, esto es:

B B

70,2v 0 70,2 9,8 t t 7,16 s9,8

= = − → = =

sustituyendo ahora este valor en la ec. (3), se tiene:

yB = 50 + 70,2 (7,16) - 4,9 (7,16)2 ⇒ yB = 301,43 m

Respuesta al ejercicio propuesto 2.3.-

SOLUCIÓN:

Figura 2.15

a) Para darnos una idea de cómo es el movimiento de los vehículos es conveniente hacer un diagrama de sus velocidades en función del tiempo, como se muestra en la figura (2.15). De donde se tiene que la velocidad alcanzada por el auto en 6 s es

1 1 1 1mv =a t =2 6 v =12 s

× →

López Frontado, R. (2011). Física general I, Texto UNA. 2da edición. Caracas: UNA

Page 88: Fisica General i Una

95

Y la distancia recorrida por el auto en ese tiempo es:

22

1 1 1 11 2(6)x = a t = x =36 m2 2

En el gráfico de la figura (2.16) se observa que la velocidad del auto aumenta uniformemente hasta alcanzar su valor de 12 m/s, como lo muestra la recta desde A hasta B, después de lo cual la velocidad se mantiene constante. También se observa que la distancia recorrida por el auto es de 36 m, mientras que la distancia recorrida por el camión es xc=vct1=5(6)= 30 m; lo que quiere decir que el auto a los 6 s ya ha pasado al camión. La posición y el tiempo de encuentro se obtiene cuando las posiciones de los vehículos son iguales. Esto es que a cx =x , La posición del auto y del camión están dados por:

2a 1 c c

1x = a t y x =v t2

La posición y el tiempo de encuentro se obtienen cuando las posiciones de los vehículos son iguales. Esto es que a cx =x , así se tiene que el tiempo de encuentro tE está dado por:

2 c1 E c E E E

1

2v1 2 5a t =v t , donde t = = t =5 s2 a 2

× ⇒

Y la posición de encuentro xE se obtiene sustituyendo el tE en cualquiera de las ecuaciones xa o xc, así se tiene que

2 2

1 EE E

a t 2(5)x = = x =25 m2 2

!

b) Cuando la velocidad del camión es de 10 m/s, se tiene que la distancia recorrida por el camión en 6 s es de 60 m, lo cual nos indica que el camión está muy adelante del auto, por lo tanto, la posición del auto en función del tiempo y la del camión en función del tiempo están dadas por:

2

a 1 1 1 c c 11x = a t +v t, x =v (t +t)2

López Frontado, R. (2011). Física general I, Texto UNA. 2da edición. Caracas: UNA

Page 89: Fisica General i Una

96

Aplicando la condición de encuentro para determinar el tiempo de encuentro, se tiene

2 2

1 1 1 E c 1 E E E

E E E E

1 1a t +v t =v (t +t ), 2(6) +12t =10(6+t )2 236+12t =60+10t 12t -10t =60-36

E E2t =24 t =12 s⇒

Y la posición de encuentro está dada por:

E Ex =60+10 12=60+120 x =180 m× ⇒

c) Cuando la velocidad del camión es de 15 m/s, el auto no lo alcanza debido a que la máxima velocidad del auto es de 12 m/s.

Respuesta al ejercicio propuesto 2.4.-

DATOS: HB = 21,0 m, xB = 130 m, θ = 35º, yo = 1,0 m. SOLUCIÓN:

Figura 2.16

a) La aceleración de la partícula es la aceleración de gravedad gr , y la consideramos constante, entonces las ecuaciones del movimiento y de la velocidad de la partícula, las podemos escribir de la siguiente manera:

2o o o

1r(t) r v t at (1), v(t) v at (2)2

= + + = +r r r rr r r

Las condiciones iniciales son:

o

o o o

ˆr 1.0 j mˆ ˆv v cos35º i v sen35º j

ˆa gj

=

= +

= −

!

!

!

López Frontado, R. (2011). Física general I, Texto UNA. 2da edición. Caracas: UNA

Page 90: Fisica General i Una

97

Sustituyendo las condiciones iniciales en las ecuaciones (1) y (2) se tiene:

2

o o

o o

gtˆ ˆ ˆ ˆr(t) 1,0 j (v cos35º i v sen35º j) t j2

ˆ ˆ ˆv(t) v cos35º i v sen35º j gtj

= + + −

= + −

r

r

Las ecuaciones paramétricas y las componentes de las velocidades, son las siguientes:

ox o2

y oo

x(t) v cos35º t (3) v v cos35º (5) gt v v sen35º gt (6)y(t) 1,0 v sen35º t (4)

2

=⎧ =⎧⎪ ⎪⎨ ⎨ = −= + − ⎪⎩⎪⎩

Para determinar la rapidez inicial, despejamos el tiempo de la ecuación (3), y lo sustituimos en la ecuación (4), así se tiene

2

2 2o o

x 9,8xt y 1,0 xtan35ºv cos35º 2v cos 35º

= → = + −

Tomando en cuenta los valores de xB = 130 m, y de yB = 21 m, en el punto B, se tiene:

2

o2 2o

9,8(130) m21 1,0 (130)tan35º v 36,2s2v cos 35º

= + − ⇒ =

b) Para determinar el tiempo que tarda la pelota en llegar a la pared, se sustituye la vo en

o

x 130t t 4,38sv cos35º 36,2cos35º

= = ⇒ =

c) La velocidad de la pelota cuando pasa por el punto B está dada por:

B

B

ˆ ˆv 36,2cos35º i (36,2sen35º 9,8 4,38) jmˆ ˆv (22,68 i 22,16 j)s

= + − ×

⇒ = −

!

!

Respuesta al ejercicio propuesto 2.5.-

DATOS: d = 50 m Ho = 30 m a1 = 4 m/s2 θ = 37º

López Frontado, R. (2011). Física general I, Texto UNA. 2da edición. Caracas: UNA

Page 91: Fisica General i Una

98

SOLUCIÓN:

Figura 2.17

a) Cuando el auto se mueve desde A hasta

B, se tiene

2 2B A 1 A

2B B

v v 2a d, donde v 0,mv 2 4 50 v 20s

= + =

= × × ⇒ =

El tiempo que tarda en moverse el auto, desde A hasta B, está dado por:

= BB 1 1 1 1

1

v 20v a t t t 5 sa 5

= → = ⇒ =

b) Cuando el auto se mueve desde B hasta C, la aceleración que actúa

es la aceleración de gravedad, la cual podemos considerar constante. Entonces, las ecuaciones de las coordenadas y de las componentes de la velocidad en función del tiempo son:

,

B x B2

B y B

x(t) v cos t (1) v v cos (2)

gty(t) H v sen t (3), v v sen gt (4)2o

θ θ

θ θ

= =

= − − = − −

Para determinar el tiempo de vuelo, o sea el tiempo que tarda el auto desde el punto B hasta el punto C, se observa que yC = 0. De la ecuación (3), se tiene que

22

C o B

2

v v

gty 0 H v sen t , 4,9 t 20sen37º t 30 02

12,04 (12.04) 4 4,9 30t t 1,53 s

9,8

θ= = − − → + − =

− ± + × ×= ⇒ =

La velocidad del auto cuando cae al agua es,

xC xC

yC yC

mv 20cos37º v 15,97smv 20sen37º 9,8(1,53) v 27,04s

= ⇒ =

= − − ⇒ = −

Cmˆ ˆv (15,97 i 27,04 j)s

⇒ = −!

López Frontado, R. (2011). Física general I, Texto UNA. 2da edición. Caracas: UNA

Page 92: Fisica General i Una

99

c) El tiempo total de movimiento del auto, tTotal = t1 + tv,

TTotal = 5 + 1,54 ⇒ tTotal = 6,54 s

d) Para determinar la distancia horizontal a la que cae el automóvil del

acantilado, se sustituye el tiempo de vuelo en la ecuación (1). Esto es:

xC = 20 (cos 37º ) 1,54 ⇒ xC = 24,60 m

Respuesta al ejercicio propuesto 2.6.-

DATOS:

1 ,

,

1

2 2

2 rad1200 rpm 125,6660 s

t 10 s2 rad400 rpm 41,8960 s

πω ω

πω ω

⎫⎛ ⎞= =⎜ ⎟ ⎪⎝ ⎠ ⎪ Δ =⎬⎛ ⎞ ⎪= =⎜ ⎟ ⎪⎝ ⎠ ⎭

SOLUCIÓN: a) La aceleración del sistema está dada por

2

(41,89 125.66) rad8,38t 10 sωα αΔ −= = ⇒ = −Δ

b) Para determinar el número de vueltas que gira el volante, primero se

determina el desplazamiento angular del mismo. Así se tiene que

2

o ot(t) t2

αθ θ ω= + +

Tomando en cuenta las condiciones iniciales, se tiene que θo = 0, y ωo= ω1. Entonces:

28,37(10)(10) 125,66(10) 837,60 rad

2θ θ= − → =

837,602 n n n 133,3 vueltas.2 2θθ ππ π

= → = = ⇒ =

c) Para determinar el tiempo que tarda en detenerse, se necesita obtener

previamente la velocidad angular en función del tiempo. Así se tiene que:

ω (t) = ωo + α t. O que ω (t) = 41,89 - 8,38 t

López Frontado, R. (2011). Física general I, Texto UNA. 2da edición. Caracas: UNA

Page 93: Fisica General i Una

100

El tiempo que tarde en detenerse es cuando ω = 0, esto es:

0 = 41,89 - 8,38 t, → t = 41,89 / 8,38 ⇒ t = 4,99 s

d) La rapidez del punto situado en el borde del volante, está dado por v = ω R. Pero antes debemos determinar la velocidad angular del volante en el instante pedido. Así tenemos:

[ ]

v R, (2,99) 41,89 8,38(2,99)mv(2,99) 41,89 8,38(2,99) 0,3 v(2,99) 5,05s

ω ω= = −

= − × ⇒ =

Respuesta al ejercicio propuesto 2.7.-

DATOS: ω1 = 8π rad/s N = 3 vueltas R = 0,5 m

ω2 = 10π rad/s

SOLUCIÓN: De acuerdo a las condiciones del problema, el disco se acelera

uniformemente, lo cual quiere decir que la aceleración angular es constante, por lo tanto las expresiones para la posición angular y la velocidad angular en función del tiempo, la podemos escribir de la siguiente manera:

2

1 2 1t(t) t (1), y t (2)2

αϕ ω ω ω α= + = +

a) Para obtener el tiempo que dura la aceleración de la rueda, se despeja

la aceleración angular α de la ecuación (2), y se sustituye en la ecuación (1). Se tiene que,

22 1 2 1 1 2

1

1 2

( ) tt, tt t 2 2

2 2(2 n) 2(2 3)t t 0,67 s(10 8) 18

ω ω ω ω ω ωα ϕ ω

ϕ π πω ω π π

− − += = + =

= = = ⇒ =+ +

b) Para determinar la aceleración angular, se tiene:

2 12

10 8 rad9,38t 0,67 s

ω ω π πα α− −= = ⇒ =

c) La velocidad de cualquier punto situado en la periferia, está dado por

mv R 0,5 10 v 15.7s

ω π= = × ⇒ =

López Frontado, R. (2011). Física general I, Texto UNA. 2da edición. Caracas: UNA

Page 94: Fisica General i Una

101

La aceleración normal

2 2n n 2

ma R 0,5(10 ) a 493,5s

ω π= = ⇒ =

La aceleración tangencial

T T 2

ma R 0,5 9,38 a 4,7s

α= = × ⇒ =

RESPUESTAS DE EJERCICIOS DE AUTOEVALUACIÓN

Respuesta al Ejercicio de Autoevaluación 2.1 a) 76,6 m/s; b) 162,00 m/s; c) 383,0 m; d) 401,8 m

Respuesta al Ejercicio de Autoevaluación 2.2 a) - 11,02 m/s b) 1,14 s; c) 4,98 m; d) - 9,8 m/s2

Respuesta al Ejercicio de Autoevaluación 2.3 4,9 m/s

Respuesta al Ejercicio de Autoevaluación 2.4 a) 18,26 s; b) 10,95 m/s

Respuesta al Ejercicio de Autoevaluación 2.5 - 223 m/s2

López Frontado, R. (2011). Física general I, Texto UNA. 2da edición. Caracas: UNA

Page 95: Fisica General i Una

103

MÓDULO II

DINÁMICA DE LA PARTÍCULA

En el módulo anterior se estudiaron diferentes tipos de movimientos de la partícula, sean unidimensionales, bidimensionales o en el espacio, sin considerar las causas que producían estos movimientos. En este módulo se estudiarán movimientos de partículas, pero tomando en cuenta las causas que los producen. Para ello se dará una idea sobre los conceptos de fuerzas (o interacciones entre las partículas), y de masa de las partículas.

El movimiento de las partículas se estudia usando las leyes enunciadas por

Newton. Estas leyes son principios, formulados empíricamente, basados en observaciones experimentales que se han realizado en experimentos de sistemas macroscópicos de la naturaleza, que se mueven a velocidad pequeña con respecto a la velocidad de la luz.

Estas leyes son muy importantes, constituyen la base de la mecánica clásica

(o mecánica newtoniana). Por lo tanto, la comprensión de estos principios fundamentales permite entender la mayor parte de los movimientos comunes de los cuerpos.

Considerando que algunos problemas de Dinámica son más difíciles de

resolver usando las leyes de Newton, se introducirá un método que usa los conceptos de Trabajo y Energía. La aplicación de este método va mucho más allá de la mecánica.

OBJETIVO

Aplicar las leyes básicas del movimiento en el comportamiento de una

partícula, en la resolución de problemas.

ESTRUCTURA DEL MÓDULO UNIDAD 3.- LEYES DE NEWTON Y APLICACIONES. UNIDAD 4.- TRABAJO Y ENERGÍA.

López Frontado, R. (2011). Física general I, Texto UNA. 2da edición. Caracas: UNA

Page 96: Fisica General i Una

105

UNIDAD 3

LEYES DE NEWTON Y APLICACIONES

En esta unidad se estudia el movimiento de las partículas, usando las leyes

enunciadas por Newton. Estas leyes son principios formulados empíricamente, basados en observaciones que se han realizado en experimentos de sistemas macroscópicos de la naturaleza, que se mueven a velocidades pequeñas con respecto a la velocidad de la luz.

Las leyes de Newton son: Primera Ley o principio de inercia. Segunda Ley o principio de la proporcionalidad de la fuerza y la aceleración. Tercera Ley o principio de acción y reacción. Estas leyes son muy importantes, por ser fundamentales para la comprensión

del movimiento de los cuerpos. Esto se debe a que no pueden deducirse o demostrarse a partir de otros principios, además permiten entender la mayor parte de los movimientos comunes de los cuerpos. Estos principios constituyen la base de la mecánica clásica (o mecánica newtoniana). Sin embargo, es necesario resaltar que para describir movimientos, cuando las velocidades son altas (cercanas a la de la luz) y para cuerpos de pequeñas dimensiones (dentro del átomo), es necesario hacerle algunas modificaciones.

Se explicará lo que se entiende por Diagrama de Cuerpos Libres. Se

describen también las fuerzas de contacto que actúan sobre un cuerpo que desliza y descansa sobre una superficie.

Se aplicará el concepto de fuerza en un movimiento circular.

CONOCIMIENTOS PREVIOS:

El estudiante, para la correcta comprensión de esta unidad, deber tener los siguientes conocimientos: 1.- Unidades de longitud, masa y tiempo (Unidad 1). 2.- Magnitudes vectoriales (Unidad 1). 3.- Conceptos de velocidad y aceleración (Unidad 1).

López Frontado, R. (2011). Física general I, Texto UNA. 2da edición. Caracas: UNA

Page 97: Fisica General i Una

106

3.1 OBJETIVO

Aplicar las leyes de Newton, en la resolución de problemas de dinámica de una partícula. 3.2 RECOMENDACIONES PARA EL ESTUDIO DEL CONTENIDO

En esta unidad, el estudiante debe comprender claramente los conceptos básicos siguientes:

Fuerzas (o Interacción), Masas, Equilibrio de un cuerpo, Peso de un cuerpo, Diagrama de Cuerpo Libre.

CONCEPTOS BÁSICOS: Fuerza (o Interacción), Masa, Equilibrio de un cuerpo, Peso de un cuerpo, Diagrama de Cuerpo Libre (DCL).

Para una mejor comprensión del contenido de la unidad, realice la lectura de

los conceptos relacionados con el movimiento de los cuerpos aplicando las leyes de Newton y los ejemplos presentados para ello en los capítulos señalados en el Plan de Curso; utilice las técnicas de lectura que le facilite la comprensión del tema.

3.3 CONTENIDO

3.3.1 LEYES DE NEWTON. 3.3.2 TIPOS DE FUERZAS. 3.3.3 FUERZA DE ROCE. 3.3.4 FUERZA ELÁSTICA. 3.3.5 DINÁMICA EN EL MOVIMIENTO CIRCULAR.

3.3.1 LEYES DE NEWTON

El término Fuerza cotidianamente lo relacionamos con el cambio que se produce en el estado de movimiento o en las deformaciones de los cuerpos. Por ejemplo: si un cuerpo está en reposo sobre una superficie cualquiera y de pronto se

López Frontado, R. (2011). Física general I, Texto UNA. 2da edición. Caracas: UNA

Page 98: Fisica General i Una

107

mueve, decimos que su movimiento se debe al efecto que una fuerza ejerce sobre él; si ahora tenemos un cuerpo que desliza sobre una superficie y de pronto se detiene, entonces decimos que una fuerza lo ha detenido.

El concepto de Fuerza nos proporciona una descripción cualitativa de la interacción entre dos cuerpos o entre un cuerpo y su entorno. Cuando la interacción se produce por contacto directo entre dos cuerpos, se dice que la fuerza es de contacto, mientras que si la interacción se produce entre dos cuerpos que están separados, se dice que la fuerza es de largo alcance (de acción a distancia).

Si un cuerpo interactúa con otros cuerpos, como se muestra en la figura (3.1),

la fuerza neta que se ejerce sobre este cuerpo, es la suma vectorial de las fuerzas que ejercen los otros cuerpos sobre él.

Figura 3.1

Esto es,

n

R1 2 3 i

i

F =F +F +F +...= F (3.1)∑! ! ! ! !

Si se descompone cada fuerza de acuerdo al sistema de coordenadas elegido, se tiene

n nR Rx ix y iy

1 i

F = F , y F = F (3.2)∑ ∑

La magnitud de la fuerza neta está dada por:

R R2 R2x y

R R2 R2 R2x y z

F = F +F en dos dimensiones (3.3)

F = F +F +F en tres dimensiones (3.4)

Es bueno recordar que experimentalmente se ha comprobado que las fuerzas se comportan como vectores, se deben usar las reglas de la suma vectorial para obtener la fuerza neta que actúan sobre un cuerpo.

La fuerza neta, algunas veces se denomina fuerza total o fuerza resultante. Si la fuerza neta ejercida sobre un cuerpo es cero, la aceleración de éste también es cero, y por lo tanto su velocidad permanece constante. Esto indica que si la fuerza neta que actúa sobre el cuerpo es cero, éste permanece en reposo o continúa su movimiento a velocidad constante. Se dice, además, que el cuerpo está en equilibrio cuando su velocidad es constante (o cuando está en reposo).

Diagrama de cuerpo libre El paso más importante en la resolución de un problema usando las leyes de Newton, es trazar un dibujo apropiado: denominado el diagrama de cuerpo libre, que consiste en trazar todas las fuerzas que actúan sobre el cuerpo, incluyendo cualquier fuerzas de campo, como la gravitacional.

López Frontado, R. (2011). Física general I, Texto UNA. 2da edición. Caracas: UNA

Page 99: Fisica General i Una

108

Para ilustrar estos conceptos, veamos el siguiente ejemplo.

Ejemplo 3.1.-

Consideremos un bloque que está en reposo sobre la superficie de una mesa,

al cual se le aplica una fuerzaF!, que forma un ángulo θ con la horizontal, halándolo

sobre la superficie. Hacer un análisis de la situación, mostrada en la figura (3.2a), tomando en cuenta las fuerzas que actúan sobre el cuerpo, indicando a cuál tipo de fuerza pertenece, hasta obtener una expresión que nos dé la fuerza resultante (o neta) que actúa sobre el mismo.

La figura (3.2b) muestra todas las fuerzas que actúan sobre el cuerpo. Éste es

un dibujo muy importante llamado diagrama de cuerpo libre (DCL), donde se indican las fuerzas que actúan sobre el bloque. Así se tiene:

Figura 3.2

F!

, es la fuerza aplicada sobre el cuerpo a través de la cuerda. N!

, es la fuerza que ejerce la superficie sobre el bloque para que no se caiga.

Fµr

, es la fuerza de roce que ejerce la superficie sobre el bloque oponiéndose a su deslizamiento.

mg

! , es la fuerza gravitatoria que ejerce la Tierra sobre el bloque. Las fuerzas F, N y Fµ

r r r son fuerzas de contacto, mientras que mg! es una fuerza

de acción a distancia.

La fuerza resultante es RµF =mg+N+F +F

r r r rr

López Frontado, R. (2011). Física general I, Texto UNA. 2da edición. Caracas: UNA

Page 100: Fisica General i Una

109

Las componentes de las fuerzas en las direcciones de los ejes Ox y Oy, son:

Rx µ

Ry

F =Fcosθ -F

F =Fsenθ +N-mg

La fuerza resultante, escrita en forma vectorial es,

Rµˆ ˆF =(Fcosθ -F )i+(Fsenθ +N-mg)j

!

La magnitud de la fuerza resultante es,

2 2µF= (Fcosθ -F ) +(Fsenθ +N-mg)

PRIMERA LEY DE NEWTON

Iniciamos el estudio de fuerzas en la siguiente situación, tomada como ejemplo. Supongamos que se coloca un libro sobre la superficie de una mesa perfectamente nivelada, como se muestra en la figura (3.3). Imaginemos que la mesa está sobre la plataforma de un camión que está estacionado; el libro permanecerá sobre la mesa.

Si el camión se mueve con velocidad constante, el libro permanece donde está colocado. Si ahora el camión acelera, el libro empezará a moverse a lo largo de la mesa, en sentido contrario al movimiento del camión.

Figura 3.3

Del ejemplo presentado, el movimiento de un cuerpo se puede observar

desde cualquier número de marcos de referencia. La primera ley de Newton del movimiento, también llamada ley de inercia, define un conjunto especial de marcos de referencia llamados marcos inerciales. Así se tiene que si un cuerpo no interactúa con otros cuerpos, es posible identificar un marco de referencia, en el cual el cuerpo tiene aceleración cero. Este marco se llama marco de referencia inercial.

Un marco de referencia que se mueve con velocidad constante con respecto a

estrellas distantes, es la mejor aproximación de un marco inercial. Por esa razón se puede considerar que nuestro planeta es la mejor aproximación a ese marco inercial.

López Frontado, R. (2011). Física general I, Texto UNA. 2da edición. Caracas: UNA

Page 101: Fisica General i Una

110

La Primera Ley de Newton establece que: “Todo cuerpo tiende a mantener su estado inicial, sea de reposo o de

movimiento uniforme, si no existe una fuerza externa que lo modifique”.

La tendencia de un cuerpo para resistir cualquier intento de cambiar su velocidad se llama inercia.

Si sobre un cuerpo no actúa fuerza alguna (o su fuerza neta es cero), la

aceleración del mismo es cero. Esto es una expresión de la condición de equilibrio, que se puede escribir en la siguiente forma:

RF =0 (3.5)!

!

Esto se logra, cuando el cuerpo está aislado (caso ideal), o cuando la suma

de todas las fuerzas que actúan sobre él sea cero. La Primera Ley de Newton también se conoce como ley de Inercia.

MASA Es una propiedad de un cuerpo que nos indica cuánta resistencia presenta un

cuerpo a cambios en su velocidad. o sea que nos dice cuánta inercia tiene el cuerpo. La masa es una propiedad inherente al cuerpo, independiente del entorno y del método que se usa para medirlo. La unidad de la masa en el sistema SI es el kilogramo (kg).

La masa no debe confundirse con el peso. Esto es que, mientras el peso es la fuerza gravitacional ejercida sobre el cuerpo, que depende de la posición a la que se encuentra el cuerpo; la masa es una propiedad inherente del cuerpo, y es igual en cualquier posición del cuerpo.

Masa y peso son dos cantidades

diferentes

MOMENTO LINEAL

Al describir el movimiento de un cuerpo, tomando en cuenta las causas que lo

produce, se incluye el concepto de masa, por lo tanto es conveniente definir el concepto denominado momento lineal, el cual establece una relación entre la masa y la velocidad del cuerpo. El momento lineal se representa por la letra p

r, es una

cantidad vectorial proporcional a la velocidad con que se mueve el cuerpo, y tiene la

López Frontado, R. (2011). Física general I, Texto UNA. 2da edición. Caracas: UNA

Page 102: Fisica General i Una

111

misma dirección de ella. La expresión matemática que nos proporciona esta relación está dada por:

p=mv (3.6)! !

La Primera Ley de Newton establece que el cuerpo se mueve con velocidad constante, entonces indica que el momento lineal del cuerpo también es constante, de donde podemos concluir que la Primera Ley de Newton expresa el Principio de conservación del momento lineal.

Basándonos en los conceptos emitidos hasta los momentos, responda la

siguiente pregunta: Pregunta 3.1.-

¿Es posible que actúen fuerzas sobre un cuerpo en reposo? SEGUNDA LEY DE NEWTON

La Segunda Ley de Newton proporciona una relación entre la aceleración que adquiere un cuerpo y la fuerza neta que actúa sobre él. La Segunda Ley de Newton, matemáticamente está expresada por la relación:

RF =ma (3.7)!

! !

La fuerza, usando el Sistema Internacional de Unidades (SI), se expresa en

Newton (N), donde N = kg.m/s2. Otra forma de expresar la Segunda Ley de Newton es la que resulta de aplicar

el concepto de momento lineal, así se tiene:

R dp dv dmF = =m +v (3.8)dt dt dt!! !! !

Si en la ecuación (3.8) se considera que la masa se mantiene constante, se

obtiene la ecuación (3.7).

López Frontado, R. (2011). Física general I, Texto UNA. 2da edición. Caracas: UNA

Page 103: Fisica General i Una

112

Tomando en cuenta los conceptos emitidos hasta los momentos, responda la

siguiente pregunta:

Pregunta 3.2-

¿Si sobre un cuerpo actúa una fuerza neta, adquiriendo así una aceleración, entonces se puede afirmar que el cuerpo siempre se mueve en la misma dirección de la fuerza? TERCERA LEY DE NEWTON

La Tercera Ley de Newton indica que cuando dos cuerpos interactúan entre sí (sean los cuerpos 1 y 2), la fuerza que el primer cuerpo ejerce sobre el segundo es igual y de sentido contrario a la fuerza que el segundo cuerpo ejerce sobre el primero. Esto es, que la fuerza que el cuerpo 1 ejerce sobre el cuerpo 2 es igual y de sentido opuesto a la que el cuerpo 2 ejerce sobre el cuerpo 1.

Figura 3.4

12 21F =-F (3.9)

! !

Si denominamos como fuerza de acción a 12F!

, y como fuerza de reacción a

21F!

, entonces se puede decir que la fuerza de acción es igual y opuesta a la fuerza de reacción, como se muestra en la Figura (3.4). Las fuerzas de acción y de reacción actúan en cuerpos diferentes.

Con el fin de aclarar estas leyes, veamos los siguientes ejemplos:

Ejemplo 3.2.-

Se tiene un bloque de masa m colocado en el piso de un ascensor, que sube con una aceleración A

!.

a) Haga un DCL de las fuerzas que actúan sobre el bloque.

López Frontado, R. (2011). Física general I, Texto UNA. 2da edición. Caracas: UNA

Page 104: Fisica General i Una

113

b) Indique las fuerzas de acción y de reacción.

c) Escriba las ecuaciones de movimiento del bloque al aplicarle la 2ª ley de Newton.

d)

Figura 3.5 SOLUCIÓN: a) DCL del bloque.

La fuerzaN

r, llamada normal, la ejerce el piso del

ascensor sobre el bloque. La fuerza gravitatoria gF

r, llamada peso, es la fuerza atractiva

ejercida por la Tierra sobre el bloque.

b) Para indicar las fuerzas de acción y de reacción, se deben considerar

los cuerpos que intervienen en cada interacción, así se tiene,

La fuerza Nr

se debe a la interacción bloque-piso. La fuerza de acción es la que el piso ejerce sobre el bloque, o seaN

r; la de reacción es la que el bloque

ejerce sobre le piso, llamémosla 'Nr

.

En la fuerza peso gF!

interactúan bloque y Tierra, la fuerza de acción es gF

! que

ejerce la Tierra sobre el bloque, y la fuerza de reacción es la fuerza que el bloque ejerce sobre la Tierra, llamémosla gF'

r.

c) Al aplicar al bloque la 2ª ley de Newton, se tiene:

gF = mA, N-F = mA∑ r r r r r,

El bloque tiene la misma aceleración del ascensor.

López Frontado, R. (2011). Física general I, Texto UNA. 2da edición. Caracas: UNA

Page 105: Fisica General i Una

114

Ejercicio propuesto 3.1.-

A un bloque de masa m, que está sobre la superficie de un plano inclinado liso

(sin fricción), se le aplica una fuerza aFr

paralela al plano. a) Haga un DCL de las fuerzas que actúan sobre el bloque. b) Indique las fuerzas de acción y de reacción que intervienen.

c) Determine la aceleración que adquiere el bloque.

Figura 3.6

3.3.2 TIPOS DE FUERZAS

Como se estudió en la unidad anterior, existen dos tipos de fuerzas: una de acción directa entre dos cuerpos de contacto, la cual se llama fuerza de contacto; y la otra, la cual se produce entre cuerpos que están separados, llamada fuerza de largo alcance (o de acción a distancia). Estos tipos de fuerzas son válidos desde el punto de vista macroscópico, ya que desde el punto de vista microscópico, todas las fuerzas están comprendidas en las de largo alcance.

Entre las fuerzas de contacto, se pueden mencionar: la tensión que ejerce

una cuerda sobre un cuerpo; la fuerza de fricción que ejerce una superficie sobre un cuerpo que deslice sobre él; la fuerza que un resorte ejerce sobre un cuerpo, etc,. Entre las fuerzas de acción a distancia, se puede mencionar: la fuerza de gravedad que ejerce la Tierra sobre un cuerpo; la fuerza de interacción que se ejerce entre cuerpos cargados eléctricamente; la atracción gravitatoria entre los cuerpos.

FUERZA DE GRAVEDAD Y EL PESO

Todos los cuerpos son atraídos por la Tierra. La fuerza ejercida por la Tierra sobre un cuerpo, se denomina fuerza de gravedad gF

r. Esta fuerza está dirigida

López Frontado, R. (2011). Física general I, Texto UNA. 2da edición. Caracas: UNA

Page 106: Fisica General i Una

115

hacia el centro de la Tierra y su magnitud se denomina peso del cuerpo. La relación matemática que nos expresa el peso de un cuerpo es:

gF =mg (3.10)! !

El peso depende de la gravedad gr

, y por lo tanto varía de acuerdo a la ubicación del cuerpo.

Es importante destacar las diferencias entre la masa y el peso, las cuales

son:

a) La masa es una cantidad escalar; mientras que el peso es una cantidad vectorial (dirigida hacia el centro de la Tierra).

b) La masa, es una cantidad inherente del cuerpo, independiente de la

ubicación del cuerpo; mientras que el peso es una cantidad que depende de la ubicación del cuerpo.

d) En el Sistema de Unidades Internacionales (SI), la unidad de la masa

es el kilogramo (kg); mientras que del peso es el Newton (N).

Basándonos en los conceptos emitidos hasta los momentos, responda la

siguiente pregunta:

Pregunta 3.3.-

Del siguiente enunciado, ¿cuándo Ud., se sienta en una silla, cuál entre las siguientes, es la fuerza de reacción a la fuerza normal ejercida por la silla sobre Ud?: a) la fuerza gravitacional que actúa sobre Ud, b) la fuerza ejercida por Ud sobre el asiento de la silla, c) ninguna de estas fuerzas.

López Frontado, R. (2011). Física general I, Texto UNA. 2da edición. Caracas: UNA

Page 107: Fisica General i Una

116

RECOMENDACIÓN PARA LA RESOLUCIÓN DE PROBLEMAS Para que pueda resolver los problemas relacionados con las leyes de Newton

y los diferentes tipos de movimiento, se recomienda:

IDENTIFICAR los conceptos pertinentes:

a) Cuando el problema implique fuerza que actúen sobre un cuerpo en equilibrio, es conveniente usar la 1ª ley de Newton. Recuerde que un cuerpo está en equilibrio, si está en reposo o se mueve con velocidad constante.

b) Si en el problema intervienen dos o más cuerpos, es necesario usar

también la 3ª ley de Newton, la cual nos permite relacionar la fuerza entre cada par de cuerpos.

c) Cuando intervienen fuerzas que actúan sobre cuerpos con aceleración,

es necesario usar la 2ª ley de Newton PLANTEAR el problema con los pasos siguientes:

d) Haga dibujo sencillo de la situación física, con dimensiones y ángulos.

e) Dibuje un Diagrama de Cuerpo Libre (DCL) para cada cuerpo escogido. Incluya en cada cuerpo, las fuerzas que actúan sobre él, y no incluya las fuerzas que el cuerpo ejerce sobre otros. En el DCL incluya solo los vectores de fuerzas; no incluya otros vectores, comomar , ni velocidad ni aceleración.

f) Muestre los ejes de coordenadas explícitamente en el diagrama, y

determine las componentes de las fuerzas con referencia a esos ejes. Si conoce la dirección de la aceleración, es conveniente tomarla como uno de los ejes. Si hay 2 o más cuerpos, puede usar un sistema de ejes distinto para cada uno.

g) Escriba la Primera ley de Newton (en problemas con aceleración cero)

o la Segunda ley de Newton (en problemas donde si hay aceleración) en forma de componentes, usando el sistema de coordenadas definido en el paso (f).

López Frontado, R. (2011). Física general I, Texto UNA. 2da edición. Caracas: UNA

Page 108: Fisica General i Una

117

h) Compruebe siempre la coherencia de unidades, siempre es adecuado. Cuando sea cantidades, y compare los resultados con lo que esperaba intuitivamente.

EJECUTAR la solución como sigue: Resuelva las ecuaciones para obtener los valores de las incógnitas del

problema. Si en el problema están implicados más de un cuerpo, repita los pasos anteriores, para cada cuerpo.

Exprese las relaciones en forma algebraica como relaciones entre las

aceleraciones de los distintos cuerpos. Es conveniente observar que en las ecuaciones para cada cuerpo, los signos de las componentes, deben ser consecuentes con los ejes escogidos para ese cuerpo.

EVALUAR los resultados:

i) Observe si sus resultados son lógicos. Si el resultado es una expresión

simbólica o fórmula, trate de encontrar casos especiales, para los que pueda estimar los resultados. Compruebe que su fórmula funciona en tales casos.

j) Si tiene dudas respecto a su resultado, consulte al asesor de su Centro

Local.

El manejo eficiente del contenido de esta unidad es muy importante

debido a que las Leyes de Newton constituyen los principios fundamentales para la comprensión del movimiento de los cuerpos.

Ahora con el fin de aclarar la aplicación de las leyes de Newton, veamos los siguientes ejemplos:

Ejemplo 3.3.- (Aplicación de la 2ª ley de Newton)

Figura 3.7

Los cuerpos mostrados en la figura (3.7) están unidos con una cuerda ideal, inetensible y sin peso que pasa a través de la polea sin fricción. Suponiendo que los cuerpos deslizan sin fricción, determine:

a) La aceleración con que se mueven los cuerpos (algebraicamente).

López Frontado, R. (2011). Física general I, Texto UNA. 2da edición. Caracas: UNA

Page 109: Fisica General i Una

118

b) Las tensiones en las cuerdas (algebraicamente).

c) Los valores, tanto de las aceleraciones como de las tensiones obtenidas, cuando m1 = 200 g, m2 = 180 g, θ = 37º.

DATOS: m1 = 200 g, m2 = 180 g, θ = 37º.

SOLUCIÓN: IDENTIFICAR los conceptos pertinentes:

En este problema se aplican las leyes de Newton, tomando en cuenta que los dos cuerpos están unidos por medio de una cuerda ideal e inextensible, lo cual nos indica que se puede considerar que la fuerza producida en un punto de la misma, se trasmite con igual valor a lo largo de toda la cuerda, y que la polea es ideal, o sea que no produce fuerza de fricción alguna sobre la cuerda.

PLANTEAR el problema: Empezaremos por dibujar un diagrama de cuerpo libre (DCL), indicando todas

las fuerzas que actúan sobre cada uno de los cuerpos. Se selecciona un sistema de coordenadas, es importante tener la siguiente precaución: escoger el sistema de coordenadas haciendo coincidir la dirección del eje x con la dirección del movimiento de los cuerpos, esto se hace para evitar confusión al establecer las ecuaciones de movimiento en cada cuerpo, al aplicar la 2ª ley de Newton.

EJECUTAR la resolución del problema: a) Para resolver este problema, conviene iniciarlo con el DCL de cada

cuerpo, esto es:

Aplicando la 2ª ley de Newton, se tiene

Cuerpo m1 Cuerpo m2 x 1 1 2 2

y 1 1

F : T-m gsen θ =m a m g-T=m a

F : N -m gcos θ =0

!!

López Frontado, R. (2011). Física general I, Texto UNA. 2da edición. Caracas: UNA

Page 110: Fisica General i Una

119

a) Para determinar la aceleración con que se mueven los cuerpos, considerando que ambos cuerpos se mueven con la misma aceleración, por ser la cuerda inextensible, se despeja T de la ecuación del segundo cuerpo, así se tiene que:

2 2T =m g-m a

Luego se sustituye T en la ecuación del primer cuerpo, y se despeja la aceleración, obteniéndose que

( )2 12 2 1 1

1 2

m - m sen gm g- m a- m gsen = m a a =

m + mθ

θ ⇒

b) La tensión T ejercida por la cuerda, se obtiene al sustituir la aceleración obtenida en el inciso anterior, de donde se tiene

( )

( )

2 22 1 1 2 2 2 1 2

2 21 2 1 2

1 2

1 2

m - m sen g m m g+ m g- m g+ m m gsenT = m g- m =

m + m m + m

m m g 1+ sen T =

m + m

θ θ

θ

⎡ ⎤⎢ ⎥⎣ ⎦

b) Los valores de la aceleración y de la tensión, se obtienen al sustituir los valores numéricos dados como datos del problema, así se tiene:

( )

( )

o

2

o

0,18 - 0,2sen 37 9,8 ma = a = 1,538 0,2 + 0,18 s

0,2×0,18×9,8 1+ sen 37T = T = 1,49 N

0,38

EVALUAR los resultados del problema: es bueno recordar que los valores obtenidos de la aceleración y de la tensión de la cuerda, corresponden a la consideración ideal del problema, donde la tensión se supone que es la misma en todos los puntos de la cuerda, y que la polea no produce ningún efecto sobre la misma.

López Frontado, R. (2011). Física general I, Texto UNA. 2da edición. Caracas: UNA

Page 111: Fisica General i Una

120

Ejemplo 3.4.- Los cuerpos, mostrados en la figura (3.8) están unidos por una cuerda inextensible,

Figura 3.8

Suponiendo que no hay fricción en las poleas, calcule: a) Las aceleraciones de los cuerpos

(algebraicamente). b) Las tensiones en las cuerdas (algebraicamente). c) Obtener los valores de los incisos (a) y (b) cuando

m1 = 8 kg y m2 = 2 kg.

DATOS: m1 = 8 kg y m2 = 2 kg. SOLUCIÓN:

IDENTIFICAR los conceptos pertinentes: En este problema se aplican las ecuaciones de Newton tomando en cuenta

que las cuerdas ideales, esto es que son inextensibles; y las poleas no producen efecto de fricción sobre las cuerdas.

PLANTEAR el problema: Como se observa en el enunciado del problema, los cuerpos m1 y m2, tienen

diferentes aceleraciones, por lo que se debe buscar una relación entre ellas (relación llamada de vínculo). Para obtener esa relación de vínculo, se asigna un sistema de coordenadas a los cuerpos, asignándosele una posición x a cada uno de los cuerpos y de las poleas, así se tiene,

Ahora se designa como l1 y l2 las longitudes constantes de las cuerdas, las cuales son:

( ) ( )1 1 p1

2 p1 p1 p2 2 p2

2 p1 p2 2

l = x - x

l = x + x - x + x - x

l = 2x - 2x + x

Las longitudes de las cuerdas, se expresan en función de las posiciones de los diferentes cuerpos.

López Frontado, R. (2011). Física general I, Texto UNA. 2da edición. Caracas: UNA

Page 112: Fisica General i Una

121

Al derivar estas ecuaciones por dos veces consecutivas, se obtienen las aceleraciones de cada cuerpo. Así se tiene,

1 p1 1 p1

p1 2 p1 2

2 1

0 = a - a a = a

0 = 2a + a 2a = -a

la ecuación de vínculo : a = -2a

Conocida la relación de vínculo entre las aceleraciones, se dibujan los DCL de

los cuerpos, incluyendo las poleas, así se tiene

Aplicando la 2ª ley de Newton a cada uno de los cuerpos, se tiene: Cuerpo m1 Polea 1 Polea 2 Cuerpo m2 1 1 1 1 1 2 2 s 2 2 2 2m g- T = m a T - 2T = 0 2T - T = 0 m g- T = m a

EJECUTAR la resolución del problema: a) Para determinar las aceleraciones de los cuerpos, se tiene:

1 1 1 1 1 2 2 2 2 2T = m g-m a T = 2T T = m g-m a

De donde se tiene:

1 1 1 2 2 2

1 1 1 2 2 1

m g-m a =2(m g-m a )m g-m a =2m g-2m (-2a )

Así se obtienen:

( ) ( )1 2 1 21 2

1 2 1 2

m -2m g 2 m -2m g a = y a =-

m +4m m +4m⇒

b) Las tensiones de las cuerdas T1 y T2, son:

( )1 1 2 1 21 1 1

1 2 1 2

1 1 22 2

1 2

m m -2m g 6m m gT =m g- T =

m +4m m +4mT 3m m g

T = T =2 m +4m

López Frontado, R. (2011). Física general I, Texto UNA. 2da edición. Caracas: UNA

Page 113: Fisica General i Una

122

c) Los valores de las aceleraciones y de las tensiones son:

( )1 1 2

2 2 2

1 1

2 2

8 - 2×2 9,8 ma = a = 2,45 8 + 4×2 s

ma = -2×2,45 a = -4,90 s

6×8×2×9,8T = T = 58,8 N16

58,8T = T = 29,4 N2

EVALUAR los resultados del problema: los valores de las aceleraciones de los cuerpos y de las tensiones ejercidas por cada cuerda corresponden a lo esperado por el enunciado del problema, los cuales son diferentes y están de acuerdo a las relaciones de vínculo de las aceleraciones.

3.3.3 FUERZA DE FRICCIÓN (o de ROCE)

Fuerza de fricción es la fuerza que se opone al movimiento de un cuerpo, cuando desliza sobre una superficie o a través de un medio viscoso.

La magnitud de la fuerza de fricción es proporcional a la fuerza normal que

ejerce la superficie sobre el cuerpo. Esa proporcionalidad está dada por una constante, denominada coeficiente de fricción, y se representa por la letra

µ

. Esto se expresa matemáticamente por F

µ = µN. Experimentalmente se ha comprobado que existen dos tipos de coeficientes

de fricción: el estático y el dinámico (o cinético). El coeficiente de fricción que al multiplicarse por la normal, nos proporciona la fuerza necesaria para poner en movimiento un cuerpo que estaba inicialmente en reposo, se denomina coeficiente de fricción estática (

µ

e). Entonces Fµ = µe N. Ahora, si el cuerpo está en movimiento sobre una superficie cualquiera, el

coeficiente de fricción que al multiplicarse con la normal, proporciona la fuerza de fricción que actúa sobre el cuerpo, se denomina coeficiente de fricción dinámica (

µ

d). Entonces Fµ = µd N. Los valores de µe y µd dependen del tipo de materia en contacto. Por lo

general los valores de µe > µd. Los valores de µ están entre 0,03 y 1,0.

López Frontado, R. (2011). Física general I, Texto UNA. 2da edición. Caracas: UNA

Page 114: Fisica General i Una

123

Para aclarar estos conceptos, veamos los siguientes ejemplos:

Ejemplo 3.5.- Como se muestra en la figura (3.9), sobre un bloque de 100 kg de masa, que inicialmente está

Figura 3.9

en reposo sobre una superficie horizontal, se aplica una fuerza en dirección horizontal Fa, hacia la derecha. El coeficiente de fricción estático es 0,4, y el coeficiente de fricción dinámico es 0,2. Con base en los datos proporcionados, determine la aceleración del cuerpo, si: a) La fuerza aplicada Fa = 350 N. b) La fuerza aplicada Fa = 450 N.

DATOS: m = 100 kg, µe = 0,4, µd = 0,2.

SOLUCIÓN: IDENTIFICAR los conceptos pertinentes: Para la resolución del problema se

aplica las leyes de Newton, tomando en cuenta las fuerzas que actúan sobre el cuerpo, incluyendo la fuerza de fricción.

PLANTEAR la solución del problema: Con el fin de resolver el problema se

dibuja un DCL, donde se indican las fuerzas que actúan sobre el bloque, estableciendo luego las ecuaciones de movimiento del mismo, al aplicar la 2ª ley de Newton.

Aplicando la 2ª ley de Newton, se tiene

x a µ

y

F : F -F =ma

F : N-mg=0

!!

EJECUTANDO la solución del problema: a) Considerando que se han dado los coeficientes de fricción estático y

dinámico, se debe comprobar si el bloque se mueve al aplicarle la fuerza Fa. Para que el bloque se mueva, es necesario que la fuerza aplicada Fa sea mayor que la fuerza de fricción máxima, esto es Fa > Fµmax. Donde Fµmax = µe N = µe mg.

,max ,maxF = 0,4×100×9,8 F = 392 Nµ µ→

López Frontado, R. (2011). Física general I, Texto UNA. 2da edición. Caracas: UNA

Page 115: Fisica General i Una

124

Si Fa = 350 N, se observa que Fa < Fµmax , por lo tanto el bloque no se mueve.

b) Si Fa = 450 N, se observa que Fa > Fµmax , por lo tanto el bloque se mueve, entonces la aceleración es,

a

2

F - mg 450 - 0,2×100×9,8a = =m 100

m a = 2,54 s

El bloque se mueve hacia la derecha con una aceleración de 2,54 m/s2.

EVALUAR los resultados del problema: los resultados nos muestran la

diferencia que existe entre el coeficiente de fricción estático y el dinámico, razón por la cual, en aquellos problemas donde se den los dos coeficientes de fricción, es necesario comprobar si el sistema se mueve o no.

Ejemplo 3.6.-

Si ahora se considera que en el ejemplo 3.5, el bloque inicialmente se estaba

moviendo hacia la izquierda con una velocidad de 7 m/s, sobre la misma superficie. Describa el movimiento subsiguiente del bloque, si se aplica hacia la derecha una fuerza:

a) Fa = 350 N. b) Fa = 450 N.

DATOS: vo = 7,0 m/s. SOLUCIÓN: IDENTIFICAR los conceptos pertinentes: Al igual que en el ejemplo 3.5, se aplican las leyes de Newton, para describir

el movimiento del bloque. PLANTEAR el problema: Para este problema se dibuja un esquema del posible movimiento del bloque,

tomando en cuenta las condiciones iniciales del mismo, dibujando un DCL y aplicando la 2ª ley de Newton. Así se tiene que:

López Frontado, R. (2011). Física general I, Texto UNA. 2da edición. Caracas: UNA

Page 116: Fisica General i Una

125

Figura 3.10

Aplicando la 2ª ley de Newton, se tiene

x a d

y

F : F +F = ma, donde F = mg

F : N- mg = 0

µ µ µ∑∑

EJECUTANDO la solución del problema:

a) Cuando Fa = 350 N, la aceleración que adquiere el bloque es: a d

a d

2

F + mgF + mg = ma a =

m350 + 0,2×100×9,8 ma = a = 5,46

100 s

µµ

La aceleración actúa en sentido contrario al movimiento del bloque, o sea que se produce un movimiento retardatriz del bloque, el cual recorrerá una distancia hacia la izquierda hasta detenerse. Esta distancia será:

2 22 2 o

o

2

v - vv = v + 2ax x =

2a0 - 7x = x = -4,49 m

2×5,46!

Este resultado indica que el bloque se desplaza 4,49 m hacia la izquierda, donde se detiene. Como la fuerza Fa = 350 N es incapaz de mover al bloque, entonces el bloque permanecerá en reposo en ese punto.

b) Cuando Fa = 450 N, el bloque adquiere una aceleración hacia la

derecha, esto es, c)

2

450+192 ma= a=6,46 100 s

!

La distancia recorrida hacia la izquierda será,

2-7x= x=-3,79 m

2×6,46!

López Frontado, R. (2011). Física general I, Texto UNA. 2da edición. Caracas: UNA

Page 117: Fisica General i Una

126

Lo que indica que el bloque se mueve hacia la izquierda una distancia de 3,79 m, donde se detiene momentáneamente, pero la Fa = 450 N si mueve al bloque sobre la superficie, por lo que el bloque se desplazará luego hacia la derecha con una aceleración af = 2,54 m/s2, como se obtuvo en la parte (b) del ejemplo 3.5.

EVALUANDO la solución del problema: Este problema muestra cómo es el

comportamiento del cuerpo cuando se aplican las fuerzas de acuerdo a las condiciones dadas inicialmente. Por eso es importante analizar cada problema, para aplicar los conceptos pertinentes con su s condiciones, antes de proceder a resolverlo.

FUERZA DE FRICCIÓN EN FLUIDOS Cuando un cuerpo se mueve en un medio viscoso, se produce una fuerza de

fricción entre el medio y la superficie del cuerpo, la cual se opone a su desplazamiento y es proporcional a la rapidez del mismo. Esto es, Ff = -λv, donde λ es un coeficiente de fricción debido al medio viscoso, y depende de la naturaleza de la materia, viscosidad y forma del cuerpo.

Ejemplo 3.7.- Consideremos el movimiento de un cuerpo de masa m que desciende en el interior de un medio viscoso. SOLUCIÓN: IDENTIFICAR los conceptos pertinentes: En este problema se aplican las leyes de Newton, tomando en cuenta las

fuerzas que actúan sobre e cuerpo. PLANTEAMIENTO Y EJECUCIÓN del problema: Se hace un dibujo del DCL, donde se indican todas las fuerzas que actúan

sobre el mismo.

López Frontado, R. (2011). Física general I, Texto UNA. 2da edición. Caracas: UNA

Page 118: Fisica General i Una

127

Figura 3.11

Aplicando la 2ª ley de Newton, se tiene

g fF -F =ma

dvmg- v =mdt

!

De esta ecuación se puede determinar la rapidez límite con que desciende el cuerpo en el medio viscoso. dvdt

, Indica la variación de la velocidad en función

del tiempo.

La velocidad límite se obtiene cuando dv = 0dt

. Esto es,

Lmgmg- v = 0 v =!!

"

De la primera ecuación de movimiento aplicando la 2ª ley de Newton, también

se puede determinar cómo varía la rapidez en función del tiempo a medida que desciende, esto es,

v t

0 0

- tm

mg m dv dv- v = , = - dtmgdt mv-

mgv- mg ln = - t v = 1- emg m-

!

!! !

!

!!!

!

"

# $% & '

( )

* *

Analizando esta ecuación, se tiene que en t = 0, v = 0, y para un tiempo t lo

suficientemente grande, se tiene mgv=!

, lo cual coincide con la velocidad límite de los

cuerpos. EVALUANDO la solución del problema: este problema nos demuestra que el

movimiento de los cuerpos también depende de las condiciones del medio en el cual se mueve, esto es si se mueve en el aire, o en un fluido cualquiera, y como afectan estos factores al movimiento del cuerpo.

Ejercicio propuesto 3.2.-

Dos bloques de masas m1 = 10 kg, y m2 = 15 kg, apoyados uno contra el otro, descansan sobre una superficie horizontal lisa. Se aplica al bloque m1 una fuerza Fa = 80 N. Calcular:

a) La aceleración con que se mueve el sistema. b) Las fuerzas de interacción entre los dos bloques.

López Frontado, R. (2011). Física general I, Texto UNA. 2da edición. Caracas: UNA

Page 119: Fisica General i Una

128

c) Las mismas preguntas anteriores, para el caso en que el coeficiente de rozamiento de los bloques con el suelo sea µd = 0,3.

Ejercicio propuesto 3.3.-

Un bloque de 3 kg parte del reposo desde la parte superior de un plano inclinado 30º con la horizontal, y desliza una distancia de 2 m hacia abajo del plano en 1,5 s. Calcule:

a) La aceleración del bloque.

b) La fuerza de rozamiento que actúa sobre el bloque.

c) El coeficiente de rozamiento dinámico entre el bloque y la superficie del

plano.

d) La rapidez del bloque después que ha deslizado 2 m.

Ejercicio propuesto 3.4.- La cuña que se muestra en la figura (3.14) se mueve sobre una superficie

horizontal. Un bloque de 5 kg se coloca sobre el plano inclinado de la cuña. Si el coeficiente de fricción estático entre el bloque y la superficie del plano inclinado de la cuña es µe = 0,4, calcule cuáles son los valores máximo y mínimo de la aceleración con que debe moverse la cuña para que el bloque no deslice sobre su superficie.

Ejercicio propuesto 3.5.-

Figura 3.12

Sea el sistema mostrado en la figura (3.12), conformado por los bloques de masas m1 = 0,5 kg, y m2 = 0,1 kg. El coeficiente de fricción estática y dinámica entre el bloque m1 y la superficie de la mesa son: µe = 0,3, y µd = 0,2, respectivamente. Los bloques inicialmente están en reposo. Las poleas son pequeñas y de masas despreciables, y las cuerdas inextensibles. a) ¿Puede el sistema ponerse en movimiento? Justifique su repuesta.

López Frontado, R. (2011). Física general I, Texto UNA. 2da edición. Caracas: UNA

Page 120: Fisica General i Una

129

b) Suponga ahora que el sistema estaba inicialmente moviéndose hacia la

derecha, calcule la aceleración de cada bloque y las tensiones de cada cuerda.

3.3.4 FUERZA ELÁSTICA

Existen fuerzas que varían con la posición, entre este tipo de fuerzas está la

elástica, que ejerce un resorte cuando se quiere deformar, sea alargándolo o comprimiéndolo. De acuerdo a la ley de Hooke, cuando se ejerce una fuerza para deformar un resorte, éste ejerce una fuerza que se opone a esa deformación.

Matemáticamente se puede expresar por eF =-k! !

donde k representa una constante de proporcionalidad llamada constante

elástica del resorte, la cual es característica de cada resorte, dependiendo de su naturaleza. Δl es la deformación del resorte, donde Δl = l – lo, siendo lo la longitud del resorte sin deformación, y l es la longitud del resorte deformado.

En la figura (3.13) se tiene un resorte, cuya longitud sin deformar es =xol ; cuando está deformado su longitud es =x+ x!l , por lo tanto la deformación del resorte es = xΔ Δ! . La fuerza que el resorte opone a su deformación

es eF .

Figura 3.13

Δl es la deformación del resorte, donde Δl = l – lo, siendo lo la longitud del

resorte sin deformación, y l es la longitud del resorte deformado.

3.3.5 MOVIMIENTO CIRCULAR

Ahora consideraremos el movimiento curvilíneo de un cuerpo.

Figura 3.14

Supongamos que una partícula se mueve a lo largo de una curva cualquiera, bajo el efecto de una fuerza, como se muestra en la figura (3.14). Aplicando la 2ª ley de Newton, se tiene F=ma

r r

t t

La componente tangencialdv F =ma =mdt

López Frontado, R. (2011). Física general I, Texto UNA. 2da edición. Caracas: UNA

Page 121: Fisica General i Una

130

2

n nvLa componente normal F =ma =m!

Donde ρ es el radio de curvatura de la trayectoria. Como un caso particular

del movimiento curvilíneo se tiene al movimiento circular, donde el radio de curvatura es el radio R de la circunferencia.

Entonces,

t

22

n

dv dωF=m =mR =mRdt dtv=Rω vF =m =mRωR

!"##

$ %##&

Para este tipo de movimiento es conveniente escoger un sistema de

coordenadas que gira a medida que gira el cuerpo.

Supongamos una partícula de masa m que gira alrededor de un eje vertical, describiendo una trayectoria circular de radio R con una rapidez constante. El sistema de coordenadas que describe el movimiento de esta partícula está conformado por los ejes: radial ( r ) y el eje vertical (z), esto se muestra en la figura (3.15).

Figura 3.15

En estos tipos de problemas, es conveniente utilizar un sistema de

coordenadas llamado de coordenadas polares, dados por :

( ) ˆˆ ˆr,φ,z , donde r,φ,k son los vectores unitarios en esas direcciones, respectivamente.

Así, las componentes radiales y transversales de la fuerza se expresan por

medio de las ecuaciones siguientes:

2r φˆ ˆF =-mRω r y F =mRα φr r

López Frontado, R. (2011). Física general I, Texto UNA. 2da edición. Caracas: UNA

Page 122: Fisica General i Una

131

Para aclarar estos conceptos, veamos los siguientes ejemplos:

Ejemplo 3.8.-

Una cuenta de 100 g se desliza libremente en una cuerda de 0,8 m de longitud. Los extremos de la cuerda están atados a una varilla vertical en los puntos A y B, los cuales están separados una distancia de 0,4 m, como se muestra en la figura (3.16). Cuando la varilla gira, BC es horizontal e igual a 0,3 m.

a) ¿Cuál es la tensión en la cuerda?

b) ¿Cuál es la rapidez de la cuenta en C?

DATOS: m = 0,1 kg, L = 0,8 m, AB = 0,4 m, BC = 0,3 m SOLUCIÓN: IDENTIFICAR los conceptos pertinentes: En el problema las leyes de Newton, se aplican a los cuerpos que describen

trayectorias circulares alrededor de un eje. PLANTEAR el problema: Para ello es necesario hacer un esquema del movimiento del cuerpo, con su

respectivo diagrama de cuerpo libre (DCL). Se usa luego la 2ª ley de Newton, para escribir las ecuaciones que describen el movimiento del mismo. Así se tiene,

Figura 3.16

Aplicando la 2ª ley de Newton, F ma=∑! ! , se tiene:

2

r

z

vF : Tsen mr

F : Tcos mg 0

θ

θ

− = −

− =

∑∑

López Frontado, R. (2011). Física general I, Texto UNA. 2da edición. Caracas: UNA

Page 123: Fisica General i Una

132

EJECUTAR el problema: a) Para determinar la tensión de la cuerda, se despeja T de la segunda

ecuación. Tenemos

mg 0,1 9,8T T 1,23N0,4cos0,5

θ×= = ⇒ =

b) La velocidad de la cuenta está dada por:

0,31,23 0,3Trsen m0,5v v 1,49m 0,1 s

θ⎛ ⎞× × ⎜ ⎟⎝ ⎠= = ⇒ =

EVALUAR el resultado: el resultado está en correspondencia con lo

establecido en el enunciado del problema.

Ejercicio propuesto 3.6.- Un cuerpo de masa m = 5 kg, se encuentra sobre una superficie cónica lisa

ABC, sujeta por una cuerda de longitud L = 70 cm, y está girando alrededor del eje EE’ con una velocidad angular de 2 rad/s. La pared del cono forma una ángulo de 37º con la vertical. Calcule:

a) La tensión que el hilo soporta.

b) La reacción de la superficie sobre el cuerpo.

c) La velocidad angular necesaria para reducir la reacción del plano a

cero.

Ejercicio propuesto 3.7.-

Un pequeñísimo cubo de masa m se halla en el interior de un embudo, que

gira alrededor de un eje vertical a una velocidad angular constante. La pared del embudo forma un ángulo θ = 53º con la horizontal. El coeficiente de fricción estática

López Frontado, R. (2011). Física general I, Texto UNA. 2da edición. Caracas: UNA

Page 124: Fisica General i Una

133

entre el cubo y el embudo es µe = 0,3, y el centro del cubo está a una distancia r del eje de rotación. Calcule los valores máximo y mínimo de la velocidad angular para los cuales el cubo no se moverá con respecto al embudo.

Ejercicio propuesto 3.8.-

El manual del conductor afirma que un conductor que viaje a 48 km/h y quiera detenerse tan rápidamente como sea posible, viajará 10 m antes de que ponga el pie sobre el freno. El auto recorre 21 m más antes de llegar al reposo.

¿Qué coeficiente de fricción debe existir entre las ruedas del automóvil y el piso? ¿Cuál es el radio mínimo para tomar una curva a 48 km/h sin patinar?

Ejercicio propuesto 3.9.-

Un carrusel completa una revolución en 12,0 s. Si un niño de 45,0 kg está sentado sobre el piso horizontal del carrusel a 3,0 m del eje de rotación, encuentre:

a) La aceleración del niño.

b) La fuerza horizontal de rozamiento que actúa sobre él.

c) ¿Cuál es el coeficiente mínimo de rozamiento estático necesario para

evitar que el niño deslice?

Ejercicio propuesto 3.10.-

Un juego de un parque de diversiones se compone de un gran cilindro vertical que gira en torno a su eje, lo suficientemente rápido para que cualquier persona en su interior se mantenga contra la pared cuando se quita el piso. El coeficiente de fricción estática entre la persona y la pared es µe. Con base en lo expuesto,

López Frontado, R. (2011). Física general I, Texto UNA. 2da edición. Caracas: UNA

Page 125: Fisica General i Una

134

a) Obtenga una expresión que muestre el período máximo necesario para

evitar que la persona caiga.

b) Si R = 4,0 m y µe = 0,4, ¿cuántas revoluciones por minuto efectúa el cilindro?

Pregunta 3.4.-

Usted está a bordo de una rueda de la fortuna que está girando en un círculo verticalmente con rapidez constante. El carro en el que usted está sentado siempre mantiene su orientación correcta hacia arriba. ¿Cuál es la dirección de la aceleración centrípeta de los pasajeros cuando están en la posición más alta de la rueda?

RESUMEN Un marco de referencia inercial es aquel en el que un cuerpo que no interactúa con otros cuerpos experimenta aceleración nula. La primera ley de Newton establece que un cuerpo, visto desde un marco inercial, en reposo permanece en reposo, y si se mueve en movimiento uniforme en línea recta mantiene ese movimiento. La segunda ley de Newton expresa que la aceleración experimentada por un cuerpo, es directamente proporcional a la fuerza neta que actúa sobre el, e inversamente proporcional a su masa. F=ma!

! ! La fuerza gravitacional que se ejerce sobre un objeto es igual al producto de su masa y la aceleración en caída libre bajo la acción de la gravedad g! : gF =mg

! ! . El peso de un cuerpo es la magnitud de la fuerza gravitacional. La tercera ley de Newton indica que cuando dos cuerpos interactúan entre si, la fuerza del primer cuerpo sobre el segundo es igual y opuesta en dirección a la fuerza que el segundo cuerpo ejerce sobre el primero.

López Frontado, R. (2011). Física general I, Texto UNA. 2da edición. Caracas: UNA

Page 126: Fisica General i Una

135

EJERCICIOS DE AUTOEVALUACIÓN:

En su libreta de apunte desarrolle los siguientes problemas. Luego de resolver los ejercicios, compare sus resultados con los presentados en la última página de la unidad. En cado de tener dudas, consulte con el asesor de su centro local.

3.1.- Sobre un objeto se ejercen las dos fuerzas siguientes:

1ˆ ˆF =(-2,4i+6,4j) N

! y 2

ˆ ˆF =(8,5i-9,7j) N!

. a) ¿Cuál es el módulo de cada una de estas fuerzas? b) ¿Cuál es ángulo entre cada una de estas fuerzas y el eje x? c) Determine el módulo y la dirección de la fuerza resultante sobre el objeto.

3.2.- Una caja de 24 kg está en reposo sobre el suelo y tiene una soga atada

a la parte de arriba. La máxima tensión que la soga puede resistir sin romperse en 310 N. ¿Cuál es el tiempo mínimo en que la caja puede ser levantada verticalmente a una distancia de 4,6 m tirando de la soga?

3.3.- Para comenzar a deslizar un bloque de 2,6 kg sobre una superficie

horizontal, se necesita una fuerza horizontal de 28 N. a) Cuánto vale el coeficiente de fricción estático µe entre el bloque y la superficie? b) Si una fuerza de horizontal 19 N mantiene al bloque deslizándose a velocidad constante, ¿cuánto vale el coeficiente de fricción dinámico µd entre el bloque y la superficie?

3.4.- Un bloque de 3,4 kg desliza hacia abajo por una pendiente, que forma un ángulo de 37° con la horizontal. El coeficiente de fricción dinámico entre el bloque y la superficie es 0,37. Determine el módulo de: a) la fuerza normal que actúa sobre el bloque; b) la fuerza de rozamiento que actúa sobre el bloque; y c) la aceleración del bloque.

3.5.- Como se muestra en la figura, un bloque B tiene una masa m, el carro

C tiene una masa M, y el coeficiente de rozamiento estático entre el bloque y el carro es µe. Determine

Figura 3. 17

el valor mínimo de F para que el bloque no se deslice hacia abajo. No tome en cuenta los efectos de rozamiento que tiendan a frenar al carro, así como tampoco el efecto de rotación de las ruedas.

López Frontado, R. (2011). Física general I, Texto UNA. 2da edición. Caracas: UNA

Page 127: Fisica General i Una

136

3.6.- Un automóvil de 875 kg da una vuelta en la esquina con un radio R = 15 m. El módulo de la velocidad del automóvil es v = 7,5 m/s, y la calle es horizontal. Determine el módulo de:

a) la aceleración del automóvil; b) la fuerza de rozamiento que actúa sobre el

automóvil; c) Determinar el ángulo entre la fuerza resultante que actúa sobre el automóvil y la vertical.

3.7.- Un rotor es una atracción de un parque de diversión que consiste de un

habitáculo cilíndrico que gira alrededor de un eje vertical. Los pasajeros entran y sitúan la espalda contra la pared, entonces el habitáculo comienza a girar. La velocidad de giro aumenta y cuando alcanza un valor mínimo, el suelo se abre, exponiendo a los pasajeros al azar caerse en un foso. a) Determine la expresión de la velocidad mínima de los pasajeros, en función del coeficiente de fricción entre el pasajero y la pared, de forma que los pasajeros no deslicen hacia debajo de foso; b) La expresión del periodo máximo del movimiento que corresponde.

RESPUESTAS A LAS PREGUNTAS: Estas respuestas se corresponden con las preguntas presentadas en el

desarrollo de la unidad. Respuesta a la pregunta 3.1.- Si, por ejemplo cuando un cuerpo está en reposo sobre una superficie, las

fuerzas que actúan sobre el cuerpo son su peso y la fuerza normal que la superficie ejerce sobre el cuerpo.

Respuesta a la pregunta 3.2.- No necesariamente, por ejemplo cuando la fuerza neta que actúa sobre el

cuerpo va en sentido contrario al movimiento, produciendo así un efecto retardatriz sobre el mismo.

Respuesta a la pregunta 3.3.- La opción (b) nos proporciona la repuesta correcta, por ser la fuerza de

reacción la que la persona ejerce sobre la silla. Respuesta a la pregunta 3.4.- La dirección de la aceleración centrípeta siempre es hacia el centro del

círculo, por lo tanto en la parte más alta de su trayectoria la aceleración centrípeta está dirigida hacia abajo.

López Frontado, R. (2011). Física general I, Texto UNA. 2da edición. Caracas: UNA

Page 128: Fisica General i Una

137

RESPUESTAS A LOS EJERCICIOS PROPUESTOS: Estas respuestas se corresponden con los ejercicios propuestos presentados

en el desarrollo de la unidad. Respuesta al ejercicio propuesto 3.1.-

SOLUCIÓN:

a) DCL de las fuerzas.

Como el cuerpo se mueve sobre una superficie lisa, las fuerzas que actúan sobre el bloque son: La normalN

r, que ejerce la superficie sobre el

cuerpo; la fuerza aplicada aFr

; y la fuerza gravitatoria (o peso) que ejerce la Tierra sobre el cuerpo.

b) Fuerzas de acción y de reacción. La fuerza normal N

r se debe a la interacción

bloque-plano. La fuerza de acción es la fuerza Nr

ejercida por el plano sobre el bloque, mientras que la fuerza de reacción 'N

! es la

fuerza que el bloque ejerce sobre el plano.

La fuerza peso se debe a la interacción bloque-Tierra. La fuerza peso mg! es la fuerza de acción que ejerce la Tierra sobre el bloque, mientras que la fuerza de reacción mg'r la ejerce el bloque sobre la Tierra.

La fuerza aFr

es la fuerza aplicada por una persona sobre el bloque. La fuerza de acción es la fuerza aF

r, mientras que la fuerza de

reacción es aF 'r

la ejerce el bloque sobre la persona.

López Frontado, R. (2011). Física general I, Texto UNA. 2da edición. Caracas: UNA

Page 129: Fisica General i Una

138

c) Aplicando la 2ª Ley de Newton, tomando en cuenta el DCL, se tiene:

x a

y

F : F -mg senθ =ma

F : N-mg cosθ =0

F a= -g senθm

!

""

Respuesta al ejercicio propuesto 3.2.- DATOS: m1 =10 kg, m2 = 15 kg, Fa = 80 N, µd = 0,3.

SOLUCIÓN:

Figura 3.18 (a) (b)

a) En primer lugar consideramos que no existe fuerza de rozamientos entre los bloques y la superficie lisa. Del DCL se escriben las ecuaciones de movimiento para cada bloque. Así se tiene que:

Bloque m1 Bloque m2

x a 1 1 2 2

y 1 1 2 2

F : F F' m a (1) F' m a (2)

F :N m g 0 (3) N m g 0 (4)

− = =

− = − =∑∑

Como los cuerpos se mueven unidos, entonces a1 = a2 = a. Al sustituir F’ en la ecuación (1), se tiene

aa 1 1 2

1 2

2

FF F' m a m a m a a

m m80 ma a 3,2

10 15 s

= + = + → =+

= ⇒ =+

b) La fuerza entre los bloques F’ = m2 a = 15 (3,2) ⇒ F´ = 48 N

López Frontado, R. (2011). Física general I, Texto UNA. 2da edición. Caracas: UNA

Page 130: Fisica General i Una

139

c) Ahora tomaremos en cuenta que existe rozamiento entre los bloques y la superficie del plano, las ecuaciones de movimiento correspondiente son:

x a 1 1 2 2

y 1 1 2 2

F : F F' F m a (5), F' F m a (6)

F : N m g 0 (7), N m g 0 (8)µ µ− − = − =

− = − =∑∑

donde Fµ1 = µd N1 = µd1 m1g y Fµ2 = µd m2g Al despejar a de la ecuación (5), se tiene

a 2 2 1 1 a 1 2 1 2

a 1 22

1 2

F F m a F m a, F F F (m m )a

F (m m )g 80 0,3(10 15)9,8 ma a 0,26m m 10 15 s

µ µ µ µ

µ− − − = − − = +

− + − += = ⇒ =+ +

La fuerza entre los bloques está dada por:

2 2F' m a F 15(0,26 0,3 9,8) F' 48Nµ= + = + × ⇒ =

Respuesta al ejercicio propuesto 3.3.- DATOS: m = 3,0 kg, θ = 30º , d = 2,0 m, t = 1,5 s.

SOLUCIÓN:

Figura 3.19

DCL

López Frontado, R. (2011). Física general I, Texto UNA. 2da edición. Caracas: UNA

Page 131: Fisica General i Una

140

a) Para determinar la aceleración, tenemos que 2

o oatx(t) x v t2

= + +

pero xo = 0, y vo = 0 (por partir desde el reposo). Así se tiene que

2

2 2 2

at 2x 2(2) mx(t) a a 1,782 t (1,5) s

= → = = ⇒ =

b) Para determinar las fuerzas de rozamiento que actúan sobre el bloque, se hace el diagrama de cuerpo libre (DCL), y luego se aplica la 2ª Ley de Newton, esto es F =ma∑

r r . Así se tiene que

x

y

F : mgsen F ma (1)

F : N mgcos 0 (2)µθ

θ

− =

− =∑∑

De la ecuación (1), se despeja la fuerza de fricción Fµ, así se tiene F m(gsen a) 3,0(9,8sen30º 1,78) F 9,36Nµ µθ= − = − ⇒ =

c) De la ecuación (2) obtenemos que N = mg cos θ, además se tiene que

FF N mgcos ,

mgcos9,36 0,37

3 9,8cos30º

µµ µ µ θ µ

θ

µ µ

= = =

= ⇒ =×

d) La velocidad de la partícula después de haber deslizado 2 m, es

B Bmv at 1,78(1,5) v 2,67s

= = ⇒ =

Respuesta al ejercicio propuesto 3.4.- DATOS: m = 5 kg µe = 0.4 θ = 37º

Figura 3.20

López Frontado, R. (2011). Física general I, Texto UNA. 2da edición. Caracas: UNA

Page 132: Fisica General i Una

141

SOLUCIÓN:

Para calcular la aceleración máxima, usamos el DCL (a). Al aplicar la 2ª ley de Newton, o sea F ma=∑

! ! , obteniéndose que

x max

y

F : Nsen37º F cos37º ma (1)

F : Ncos37º F sen37º mg 0 (2)µ

µ

+ =

− − =∑∑

Pero Fµ = µe N. Al sustituir Fµ en las ecuaciones (1) y (2), y al despejar la aceleración amax, se tiene

emax

e

max 2

(sen37º cos37º )g (0,6 0,4 0,8)9,8acos37º sen37º (0,8 0,4 0,6)

ma 16,1s

µµ

+ + ×= =− − ×

⇒ =

Para calcular la aceleración mínima, se usa el DCL (b). Al aplicar la 2ª ley de Newton, se tiene que

x min

y

F : Nsen37º F cos37º ma (3)

F : Ncos37º F sen37º mg 0 (4)µ

µ

− =

+ − =∑∑

Nuevamente, al sustituir Fµ en las ecuaciones (3) y (4), luego despejar la aceleración, se obtiene.

emin

e

min 2

(sen37º cos37º )g (0,6 0,4 0,8)9,8a(cos37º sen37º ) (0,8 0,4 0,6)

ma 2,45s

µµ

− − ×= =+ + ×

⇒ =

Por lo tanto, el rango de valores de la aceleración para que no se mueva el bloque sobre la superficie de la cuña es,

2 2

m m2,45 a 16,1s s

! ! .

Respuesta al ejercicio propuesto 3.5.- DATOS: m1 = 0,5 kg, m2 = 0,1 kg, µe = 0,3, µd = 0,2

López Frontado, R. (2011). Física general I, Texto UNA. 2da edición. Caracas: UNA

Page 133: Fisica General i Una

142

SOLUCIÓN:

(a) (b) (c)

Aplicando la 2ª ley de Newton a cada uno de los cuerpos, se tiene:

x 1 1 1 2 1 2 2 2 2

y 1 1

F : T -F =m a 2T -T =0 m g-T =m a

F : N -m g=0µ∑

En este tipo de problema existe una polea móvil, la polea p1; por lo tanto, las aceleraciones de los bloques son diferentes. Para determinar la relación entre las aceleraciones es necesario obtener una ecuación que relacione a las aceleraciones de los bloques, llamada ecuación de vínculo. Para obtener esta ecuación, se toma un eje de referencia, con respecto al cual se le asigna una posición de cada uno de los cuerpos. Esto es:

Sean 1 2yl l , las longitudes de las cuerdas que unen los diferentes cuerpos, las cuales son constantes. Así tenemos,

1 p1 1

2 2 p1 p p1

x x

(x x ) (x x )

= −

= − + −

!

!

Al derivarla dos veces, obtenemos las aceleraciones correspondientes, esto

es,

0 = ap1 - a1 → a1 = ap1 0 = a2 - 2 ap1 + 0 → a2 = 2 ap1 ⇒ 2 a1 = a2

a) Para conocer si el sistema se mueve o no, se escriben las ecuaciones de movimiento considerando al sistema inicialmente en reposo, esto es,

T1 - Fµ1e = 0, y m2g - T2 = 0, ⇒ Fµ1e = 2 m2g

López Frontado, R. (2011). Física general I, Texto UNA. 2da edición. Caracas: UNA

Page 134: Fisica General i Una

143

Para que el sistema se mueva es necesario que Fµ1e > Fµ1max, esto es,

Fµ1e = 2x 0,1x 9,8 = 1,96 N, y Fµ1max = µe N = µe m1g = 0,3x0,5x9,8 = 1,47 N,

Como observamos, Fµ1e > Fµ1max , razón por la cual el sistema se mueve.

b) Conociendo que el sistema se mueve, entonces, a partir de los DCL de los cuerpos del sistema, tenemos que,

Para el bloque m1, y para el bloque m2

T1 - µd m1g = m1 a1 m2g - T2 = m2 a2

T1 = m1a1 + µdm1a1 T2 = m2g - m2a2

T1 = 2T2 → m1a1 + µdm1g = 2 m2g - 2 m2 (2 a1)

2 d 11 1 2

1 2

2 1 2 2

(2m m )g (2 0,5 0,2 0,1)9,8 ma a 1,089(m 4m ) 0,1 4 0.5 s

my a 2a 2 1,089 a 2,178s

µ− × − ×= = ⇒ =+ + ×

= = × ⇒ =

Las tensiones serán:

1 1 1 d 1

12 2

T m (a g) 0,5(1,089 0,2 9,8) T 1,525NT 1,525y T T 0,762N2 2

µ= + = + × ⇒ =

= = ⇒ =

Respuesta al ejercicio propuesto 3.6.- DATOS: m = 5 kg, L = 70 cm, ω = 2 rad/s, θ = 37o

López Frontado, R. (2011). Física general I, Texto UNA. 2da edición. Caracas: UNA

Page 135: Fisica General i Una

144

SOLUCIÓN:

Figura 3.21

Aplicando la 2ª ley de Newton, se tiene:

2

r

z

F : Ncos θ -Tsen θ =-mrω

F : Nsen θ +Ncos θ -mg=0∑∑

a) Para calcular la tensión del hilo, despejamos N en la primera ecuación,

y luego lo sustituimos en la segunda ecuación, esto es:

2

2

Tsen - mr N = donde r = Lsencos

Tsen - mr sen + Tcos = mgcos

θ ω θθ

θ ω θ θθ

2 2 2

2 2 2

Tsen - mr sen + Tcos = mgcosT = mgcos +m r = mgcos +m Lsen

! " ! ! !

! " ! " !

Sustituyendo los valores, se tiene:

( )22T = 5×9,8×0,8 + 5×2 ×0,7× 0,6 T = 44,24 N⇒

b) La reacción que ejerce la superficie sobre el cuerpo es:

2Tsen - m Lsen 44,24×0,6 - 8,4N = =

cos 0,8 N = 22,67 N

θ ω θθ

c) El valor de la velocidad angular necesario para que la reacción N sea

cero se obtiene considerando N = 0. Aplicando la 2ª ley de Newton, se tiene:

López Frontado, R. (2011). Física general I, Texto UNA. 2da edición. Caracas: UNA

Page 136: Fisica General i Una

145

2r

2z

F : - Tsen = -m Lsen

g 9,8F : Tcos - mg = 0, donde = =Lcos 0,7×0,8

rad = 4,18 s

! " !

! "!

"#

$$

Respuesta al ejercicio propuesto 3.7.- DATOS: θ = 53°, µe = 0,3

SOLUCIÓN:

Figura 3.22

Cuando se va a determinar la velocidad angular máxima, el DCL y las ecuaciones de movimiento, son:

Aplicando la 2ª ley de Newton, se tiene:

2

r

z

F : -Nsen -F cos = -m r (1)

F : Ncos -F sen - mg = 0 (2)

µ

µ

θ θ ω

θ θ

∑∑

Dividiendo la ec. (1) entre la ec. (2), se tiene:

( )( )

( )( )

22N sen + cos g sen + cosmr = , =

N cos - sen mg r cos - senθ µ θ θ µ θω ωθ µ θ θ µ θ

El valor de la velocidad angular máxima es

( )( )

2max

9,8 0,8 + 0,3×0,6 rad= = 8,42 0,8 0,6 - 0,3×0,8 s

ω ω⇒

López Frontado, R. (2011). Física general I, Texto UNA. 2da edición. Caracas: UNA

Page 137: Fisica General i Una

146

Para determinar la velocidad angular mínima, el DCL y las ecuaciones de movimiento son:

Aplicando la 2ª ley de Newton, se tiene

2r µ

z µ

F : -Nsenθ +F cosθ =-mω r (3)

F : Ncosθ +F senθ-mg=0 (4)

∑∑

Dividiendo la ec. (3) entre la ec. (4), se tiene

( )( )

2N sen - cos m r=N cos + sen mg

θ µ θ ωθ µ θ

De donde se obtiene la velocidad angular mínima, esto es

( )( )

( )( )

2min

g senθ-µ cosθ 9,8 0,8-0,3×0,6 radω = = , ω =4,38 r cosθ +µ senθ 0,38 0,6+0,3×0,8 s

Respuesta al ejercicio propuesto 3.8.- DATOS: vo = 48 km/h = 13,33 m/s, d1 = 10 m, d2 = 21 m.

SOLUCIÓN:

Figura 3.23

a) Del diagrama de cuerpo libre DCL, aplicando la 2ª ley de Newton, se tiene:

x

y

F : -F =ma (1)

F : N-mg=0 (2), F = N= mg

µ

µ µ µ

∑∑

Despejando la aceleración a de la ec. (1), se tiene mga=- =- g (3)

mµ µ

López Frontado, R. (2011). Física general I, Texto UNA. 2da edición. Caracas: UNA

Page 138: Fisica General i Una

147

La ecuación de movimiento y de su velocidad,

2

o oatx=v t- (4), y v=v -at (5)2

De la ec. (5) , y considerando que vc = 0, se tiene que ovt=

a

Sustituyendo el tiempo t en la ec. (4),

2 2 2o o o

2

v av vx= - a= (6)

a 2x2a→

Igualando la ec. (6) con la ec (3), se tiene

2 2 2 2o o o

2

v v v (13,33)g= , = = = =0,4322x 2gx 2gd 2 9,8 21

µ µ µ⇒× ×

b) Para determinar el radio mínimo.

2

r

z

mvF : -F =-R

F : N-mg=0

µ∑∑

2 2 2mv v (13,33)mg= , R= = R=41,97 m

R g 0,432 9,8µ

µ→ ⇒

×

Respuesta al ejercicio propuesto 3.9.- DATOS: n = 1 rev, t = 12 s, m = 45,0 kg, r = 3,0 m SOLUCIÓN:

Figura 3.24

a) De acuerdo al enunciado del problema, el movimiento circular es uniforme, esto es que la velocidad angular se mantiene constante, y por lo tanto, la aceleración del niño es la aceleración centrípeta. Así se tiene:

2

2cv 2 2 radˆ ˆa = r=ω rr, pero = = =0,524 r T 12 s

π πω ωr

Entonces, 2

2 2c c 2

v mˆ ˆ ˆ ˆa = r=ω rr=(0,524) 3r a =0,823r r s

⇒r r

López Frontado, R. (2011). Física general I, Texto UNA. 2da edición. Caracas: UNA

Page 139: Fisica General i Una

148

b) Considerando el DCL, se tiene

r µ r µF : -F =-ma =-(45,0)(0.823) F =37,01 N!"

c) Del DCL, se conoce que

zF : N-mg=0, además se conoce que F = Nµ µ∑

µµ

F 37,01µ mg=F µ = = µ=0,084mg 45(9,8)

! "

Respuesta al ejercicio propuesto 3.10.- DATOS: R = 4,0 m, µe = 0,4 Diagrama del problema.

Figura 3.25

SOLUCIÓN: a) Aplicando la 2ª ley de Newton, para describir el movimiento circular de

la persona, se tiene,

2r

z µmax µmax

2 2µmax e e

e

F : -N=-mRω

F : F -mg=0 donde F =mg

gademás F =µ N=mg, µ mRω =mg ω =µ R

!

""

El período máximo Tmax necesario es:

emax max

e

µ R2πT = T =2π gg

µ R

!

López Frontado, R. (2011). Física general I, Texto UNA. 2da edición. Caracas: UNA

Page 140: Fisica General i Una

149

b) Para determinar las revoluciones por minuto que efectúa el cilindro, se tiene,

e

ω 1 g 1 9,8 revf= 60= 60= ×60 f=23,6 2 2 R 2 0,4×4 minπ π µ π

RESPUESTA A LOS EJERCICIOS DE AUTOEVALUACIÓN

Respuesta al ejercicio de autoevaluación 3.1 a) F1 = 6,84 N, F2 = 12,90 N; b) α = 249,44°, β = 311,23°;

c) R = 6,94 N, θ = 315,52°

Respuesta al ejercicio de autoevaluación 3.2

tmin = 0,84 s

Respuesta al ejercicio de autoevaluación 3.3 a) µe = 1,1; b) µd = 0,75

Respuesta al ejercicio de autoevaluación 3.4 a) N = 26,61 N; b) Fµ = 9,85 N; c) a = 3,0 m/s2

Respuesta al ejercicio de autoevaluación 3.5 ,min

e

mgF =µ µ

Respuesta al ejercicio de autoevaluación 3.6 a) a = 3,75 m/s2; b) Fµ = 3281 N; c) θ = 20,9°

Respuesta al ejercicio de autoevaluación 3.7

emin max

e

RgRa) v = ; b) T =2g

µπµ

López Frontado, R. (2011). Física general I, Texto UNA. 2da edición. Caracas: UNA

Page 141: Fisica General i Una

151

UNIDAD 4

TRABAJO Y ENERGÍA

En esta unidad se estudia el movimiento de la partícula, usando los conceptos de Trabajo y Energía. Se inicia explicando el concepto de Trabajo, tanto el producido por una fuerza constante como el producido por una fuerza variable. Luego se estudia el concepto de Energía. La importancia del estudio de este último concepto, proviene del Principio de Conservación de Energía, el cual nos dice que la energía se puede convertir de una forma a otra pero no puede crearse ni destruirse. Aunque el concepto de energía está presente en muchos de los procesos que se producen cotidianamente en la naturaleza, en el resto de este texto se usará para una amplísima gama de fenómenos físicos. En esta unidad sólo se usará en la mecánica.

Conoceremos una importante forma de energía, la energía cinética o

energía de movimiento, y su relación con el concepto de trabajo, a través del teorema de trabajo y energía cinética. También se estudiará la rapidez con que se realiza trabajo, a través del concepto de Potencia.

Otra forma de energía a estudiar es la Energía Potencial, que es la energía asociada con la posición de un sistema, no con su movimiento. El concepto de energía potencial está asociado a la presencia de fuerzas conservativas, así se logrará conocer la existencia de fuerzas conservativas y de la fuerzas no conservativas.

Se demostrará que la suma de la energía cinética y potencial, llamada

energía mecánica, es constante durante el movimiento. Esto constituye el enunciado general de la ley de conservación de la energía en general, conocido como uno de los principios más fundamentales y trascendentales de las ciencias.

CONOCIMIENTOS PREVIOS:

El estudiante, para la correcta comprensión de esta unidad, debe tener los siguientes conocimientos: 1.- Producto escalar de dos vectores (Unidad 1). 2.- Vector de posición y su variación (Unidad 1 y 2). 3.- Ecuación fundamental de la Dinámica (Unidad 3). 4.- Concepto de fuerza (Unidad 3).

López Frontado, R. (2011). Física general I, Texto UNA. 2da edición. Caracas: UNA

Page 142: Fisica General i Una

152

4.1 OBJETIVO

Aplicar los conceptos de Trabajo, Energía y Potencia en la resolución de problemas de dinámica de una partícula. 4.2 RECOMENDACIONES PARA EL ESTUDIO DEL CONTENIDO

En esta unidad, el estudiante debe comprender claramente los conceptos básicos siguientes: Concepto de Trabajo. Relación entre Trabajo y Energía Cinética. Concepto de Potencia. Relación entre Trabajo y Energía Potencial. Fuerzas Conservativas.

CONCEPTOS BÁSICOS: Trabajo. Energía cinética. Potencia. Energía potencial. Relación trabajo-energía. Fuerzas conservativas.

Para una mejor comprensión del contenido de la unidad, realice la lectura de los conceptos relacionados con el movimiento de los cuerpos aplicando las idea del Teorema de Trabajo y Energía, y los principios de Conservación de la energía en general, y los ejemplos presentados para ello en el texto; utilice las técnicas de lectura que le faciliten la comprensión del tema. 4.3 CONTENIDO

4.3.1 DEFINICIÓN DE TRABAJO. 4.3.2 ENERGÍA CINÉTICA Y POTENCIAL. 4.3.3 POTENCIA. 4.3.4 TEOREMA TABAJO Y ENERGÍA. 4.3.5 PRINCIPIO DE CONSERVACIÓN DE ENERGÍA.

4.3.1 DEFINICIÓN DE TRABAJO.

Para definir el concepto de trabajo consideremos una partícula, que se mueve a lo largo de una trayectoria cualquiera C, bajo la acción de una fuerza F

r. Debido a

la acción de la fuerza, en un tiempo muy corto, la partícula se desplaza ligeramente, siendo este desplazamiento AA'=dr

r.

López Frontado, R. (2011). Física general I, Texto UNA. 2da edición. Caracas: UNA

Page 143: Fisica General i Una

153

Figura 4.1

El trabajo efectuado por la fuerza Fr

sobre la partícula se define como el producto escalar de la fuerza por el desplazamiento. Matemáticamente se expresa por: dW=F•dr (4.1)

r r

Aplicando la definición del producto escalar, se puede expresar también por

dW=Fdscosθ (4.2)

Donde F y ds son las magnitudes de la fuerza aplicada y del desplazamiento

de la partícula, respectivamente, y θ es el ángulo formado por la fuerza y el desplazamiento.

De la ecuación (4.2), se tiene que Fcosθ es la componente tangencial FT de la

fuerza aplicada. Entonces, dicha ecuación también se puede escribir

TdW=F ds (4.3)

En la ecuación (4.2) también se observa que si la fuerza aplicada es

perpendicular al desplazamiento, entonces se tiene que cos 90º = 0; por lo tanto el trabajo realizado por dicha fuerza es cero.

Cuando la partícula se desplaza desde una posición A hasta una posición B,

el trabajo realizado será:

B B B

TA A A

W= F•dr= Fdscosθ= F ds (4.4)∫ ∫ ∫r r

A veces es conveniente representar gráficamente FT vs s.

De acuerdo a la gráfica,

!T

B

TA

dW=F ds

W= F ds=Área sombreada

bajo la curva

Figura 4.2

López Frontado, R. (2011). Física general I, Texto UNA. 2da edición. Caracas: UNA

Page 144: Fisica General i Una

154

Otra situación se presenta cuando la fuerza aplicada es constante en magnitud y dirección; en este caso, el cuerpo se mueve rectilíneamente en la dirección de la fuerza. Así se tiene:

B B

A AW= Fds=F ds=Fs (4.5)! !

Si ahora los vectores de la fuerza aplicada y el desplazamiento de la partícula

se expresan en función de sus componentes, se tiene:

( )

x y z

B B

x y zA A

ˆ ˆ ˆ ˆ ˆ ˆF=F i+F j+F k, y dr=dxi+dyj+dzk

El trabajo es

W= F•dr= F dx+F dy+F dz (4.6)∫ ∫

! !

! !

Figura 4.3

Otro caso es aquél donde varias fuerzas

1 2 3 n(F, F , F ,..., F )! ! ! !

actúan sobre una partícula, como se muestra en la figura (4.3). En este caso, se tiene que el trabajo ejercido sobre la partícula está dado por 31 2 4FF F FW=W +W +W +W

rr r r

O sea, que

( ) ( )B B

1 2 3 1 2 3A AB

R

A

W= F •dr+F •dr+F •dr+ = F +F +F + •dr

W= F •dr (4.7)

∫ ∫∫

r r r r r rr r r r

r r

En este caso, el trabajo es igual al producto de la fuerza resultante RF

r

multiplicada por el desplazamiento de la partícula drr.

La unidad internacional (SI) usada para expresar el trabajo es el Joule (J) =

N.m = kg.m2/s2. Otra unidad usada para expresar el trabajo en el sistema de unidades cgs es el erg = dina.cm = gr.cm2/s2. La equivalencia entre el Joule y el erg es:

1 J = (105 dinas)(102 cm) = 107 erg.

Existen otras unidades para el trabajo, las cuales demostraremos más

adelante.

López Frontado, R. (2011). Física general I, Texto UNA. 2da edición. Caracas: UNA

Page 145: Fisica General i Una

155

Basado en los conceptos emitidos hasta los momentos, responda la siguiente

pregunta:

Pregunta 4.1

¿Realiza algún trabajo cuando una persona con una caja de 20 kg de masa, colocada sobre sus hombros, camina luego una distancia horizontal de 20 m? Razone su repuesta.

A continuación se presentan unos ejemplos de problemas, donde Ud. puede

visualizar las aplicaciones de los conceptos tratados en esta unidad. Pero antes vea las Sugerencias y las Recomendaciones presentadas para resolver estos problemas.

RECOMENDACIONES PARA RESOLVER PROBLEMAS DE DINÁMICA DE

PARTÍCULA USANDO TRABAJO Y ENERGÍA. IDENTIFICAR los conceptos pertinentes: el teorema de trabajo-energía es

muy conveniente en situaciones en las que se desea relacionar la rapidez de un cuerpo en un punto de su movimiento, con su rapidez en otro punto. También es muy útil si el problema implica movimiento con fuerzas variables, en una trayectoria curva o ambas cosas. Su uso no es muy conveniente, si el problema implica tiempo transcurrido.

PLANTEAR el problema con los siguientes pasos: a) Escoja las posiciones iniciales y finales del cuerpo, use el subíndice 1

para el estado inicial, y el subíndice 2 para el estado final. y dibuje un Diagrama de Cuerpo Libre (DCL) con todas las fuerzas que actúan sobre él.

b) Indique las fuerzas que actúen sobre el cuerpo. Escoja un sistema de

coordenadas.

c) Haga una lista de las cantidades conocidas y desconocidas, y decida cuáles son las incógnitas.

López Frontado, R. (2011). Física general I, Texto UNA. 2da edición. Caracas: UNA

Page 146: Fisica General i Una

156

EJECUTAR la solución:

d) Escriba expresiones para las energías cinéticas y potenciales iniciales y finales (K1, K2, U1 y U2).

e) Use la relación WTotal = K2 – K1, donde K representa la energía cinética

del cuerpo en las posiciones inicial o final.

f) Recuerde que el trabajo total está dado por

WTotal = WC + WNC.

Donde WC es el trabajo realizado por fuerzas conservativas, WNC es el trabajo realizado por fuerzas no conservativas. Además WC = - ΔU = U1 – U2, donde U es la energía potencial del cuerpo en la situación inicial o final.

g) Despeje las cantidades desconocidas.

EVALUAR sus resultados.

h) Verifique si el resultado es lógico físicamente. Tenga presente, aquí y más adelante, que el trabajo efectuado por cada fuerza debe estar representado en U1 – U2 = - ΔU o bien en WNC, pero nunca en ambos. El trabajo gravitacional está incluido en ΔU; y procure no incluirlo también en WNC.

i) Las dudas que se presenten despéjelas con su asesor del Centro Local.

Recuerde que el manejo eficiente del contenido de esta unidad es

importante, por su repercusión en el contenido de las subsiguientes unidades del curso de Física General I.

Ejemplo 4.1

A un bloque de 60 kg de masa se le aplica una fuerza Fr

paralela a la superficie de una rampa de 10º de inclinación, para subirla con una velocidad constante de 10 m/s. Considerando que la superficie de la rampa es lisa, determine el trabajo realizado sobre el bloque en 2 minutos: a) por cada una de las fuerzas que actúan sobre él; b) por todas las fuerzas.

DATOS: m = 60 kg, θ = 10º, v = 10 m/s, t = 2x60= 120 s.

López Frontado, R. (2011). Física general I, Texto UNA. 2da edición. Caracas: UNA

Page 147: Fisica General i Una

157

SOLUCIÓN: IDENTIFICAR los conceptos pertinentes: De acuerdo al enunciado del problema, se pide determinar el trabajo

producido por las fuerzas conservativas que actúan sobre el cuerpo, ya que la fuerza no conservativa es cero debido a que no hay fricción entre el cuerpo y la superficie del plano.

PLANTEAR el problema: 1.- En primer lugar se elabora un diagrama de cuerpo libre (DCL), donde

se indican todas las fuerzas que actúan sobre el mismo, como se dibuja en la figura (4.4). Así se tiene que el trabajo realizado cuando el cuerpo asciende por la rampa inclinada está dado por:

Total mg N FW =W +W +Wr rr

En esta figura se establece el sistema de coordenadas x e y, orientado

en la dirección del movimiento del cuerpo.

Figura 4.4

En esta figura se establece el sistema de coordenadas x e y, orientado en la dirección del movimiento del cuerpo. Aplicando la 2ª ley de Newton, se tiene

x

y

F : F-mgsen =0F : N-mgcos =0

!

!""

EJECUTAR la solución del problema:

Previo al cálculo del trabajo realizado por cada fuerza, es necesario determinar la distancia recorrida por el cuerpo sobre la superficie del plano inclinado en 2,0 s. Así se tiene que

mx=vt=10 ×120ss

x=1200 m→

Y la fuerza F aplicada está dada por:

oF = mgsen = 60×9,8×sen10

F = 102,11 Nθ

López Frontado, R. (2011). Física general I, Texto UNA. 2da edición. Caracas: UNA

Page 148: Fisica General i Una

158

a) Para determinar el trabajo realizado por cada una de las fuerzas, se tiene que: el trabajo efectuado por la fuerza F

r aplicada es:

F

F 5

W = F• x = 102,11×1200 W = 1,23×10 J!

El trabajo realizado por la fuerza normalN

!es:

WN = 0, debido a que la fuerza N

r es perpendicular al

desplazamiento del bloque.

El trabajo realizado por la fuerza pesomgres:

mg o

mg 5

W =-mgsenθ•x=-60×9,8×sen10 ×1200 W =-1,23×10 J⇒

b) El trabajo total es, Total F N mg 5 5

Total

W =W +W +W =1,23×10 +0-1,23×10 W =0⇒

EVALUAR el resultado del problema: El resultado es lógico, ya que el enunciado del problema nos indica que el

movimiento es a velocidad constante, y cuando esto sucede, no se produce trabajo neto sobre el cuerpo, como lo veremos más adelante aplicando el teorema de trabajo y energía cinética.

Ejemplo 4.2

Calcule el trabajo realizado por un resorte cuando se le extiende una distancia

de 2,0 cm sin aceleración. Se conoce que al colgar del resorte una masa de 6,0 kg, la longitud del resorte aumenta en 1,5 cm.

DATOS: m1 = 6,0 kg, 1x =1,5 cm, x2=2,0 cm.

SOLUCIÓN: IDENTIFICAR los conceptos pertinentes: en este problema se pide determinar

el trabajo realizado por el resorte, el cual puede resolverse aplicando la definición de trabajo o aplicando la condición de que la fuerza del resorte es una fuerza conservativa.

López Frontado, R. (2011). Física general I, Texto UNA. 2da edición. Caracas: UNA

Page 149: Fisica General i Una

159

PLANTEAR el problema: Para determinar el trabajo realizado por el resorte, tomando en cuenta la figura (4.5), se distinguen la posición del resorte representada en A (donde el resorte tiene su longitud original sin deformar) y la posición del resorte en C (donde el resorte está estirado). Además se calcula previamente el valor de la constante elástica, aplicando las condiciones de equilibrio cuando el resorte está completamente estirado usando la masa m.

EJECUTAR la resolución del problema:

Para ello, consideremos que el resorte cuelga desde la posición O. Cuando el otro extremo del resorte está en A, no existe algún cuerpo que cuelgue del resorte. En la posición B, se cuelga el cuerpo de masa m = 6,0 kg; asi se observa que en esta posición el cuerpo está en equilibrio. Se tiene entonces,

1

1

3-2

mgkx - mg = 0, k =x

6,0×9,8 Nk = k = 3,92×10 m1,5×10

!

Figura 4.5

Para alargar al resorte una distancia x sin aceleración, es necesario aplicar al

mismo una fuerza Fa que sea en magnitud igual a la fuerza hace el resorte oponiéndose a ese alargamiento, esto es Fa = Fr = kx2. Esta fuerza aumenta constantemente al aumentar x2. El trabajo realizado por el resorte está dado por

a2

a

F 2xF 2

0

kxW = -kxdx W = -

2→∫

Al sustituir sus valores, se obtiene

a a

3 -2 2F F3,92×10 (2,0×10 )W = - W = 0,784 J

2!

El valor obtenido para el trabajo realizado por la fuerza aplicada Fa es lógico.

EVALUAR el resultado del problema: El valor obtenido para el trabajo realizado por la fuerza aplicada Fa es lógico. Además este resultado obtenido nos indica que el trabajo realizado por el resorte depende solo de sus posiciones inicial y final.

López Frontado, R. (2011). Física general I, Texto UNA. 2da edición. Caracas: UNA

Page 150: Fisica General i Una

160

Ejemplo 4.3.- Una fuerza F = 12 t N actúa sobre una partícula de 3 kg de masa, que está

limitada a moverse a lo largo del eje x. Si la partícula parte del reposo, determinar el trabajo efectuado por la fuerza durante los primeros 3 s.

DATOS: m = 3,0 kg, t = 3,0 s.

SOLUCIÓN: IDENTIFICAR los conceptos pertinentes: Para determinar el trabajo realizado se tiene que W = ∫Fdx, donde la fuerza F

no es constante en cuanto al tiempo se refiere, depende del tiempo transcurrido. PLANTEAR Y EJECUTAR la solución del problema: Para determinar el trabajo producido por la fuerza es necesario determinar el

desplazamiento dx del cuerpo en función del tiempo, aplicando la 2ª ley de Newton, así se tiene que

2

2v t t2

0 0 0

F 12t m dvF = ma , donde a = = = 4 t , pero a = , m 3 dts

4 t mdv = adt = 4 t dt v = v = 2t 2 s

→∫ ∫ ∫

2dxv = dx = vdt = 2t dt

dt→

Entonces se tiene que

4t t

2 3

0 04

24tW = (12t)(2t dt) = 24t dt = J4

En t = 3,0 s W = 6(3) = 486 J!

" "

EVALUAR la solución del problema: en el problemas se observa que el trabajo depende del tiempo, y el resultado del trabajo final obtenido corresponde cuando t = 3,0 s.

López Frontado, R. (2011). Física general I, Texto UNA. 2da edición. Caracas: UNA

Page 151: Fisica General i Una

161

4.3.2 ENERGÍA CINÉTICA Y POTENCIAL

De la ecuación (4.4) se tiene que

Tdv dsF ds=m ds=mdv mvdvdt dt

Entonces el trabajo realiza sobre la partícula cuando se mueve desde A hasta

B, será:

2 2BB A

A

mv mvW= mvdv= - (4.8)2 2!

La ecuación (4.8) nos indica que independientemente de la forma funcional de la fuerza y de la trayectoria descrita por la partícula, el valor del trabajo efectuado por

la fuerza es siempre igual a la diferencia entre las magnitudes 21( mv )2

evaluada al

final y al comienzo de la trayectoria. Esta cantidad 21( mv )2

, que depende de la

rapidez de la partícula en los puntos A y B, se denomina Energía cinética y se designa por K.

La ecuación (4.8) también puede expresarse por

B AW=K -K =ΔK (4.9)

O sea, que el trabajo efectuado sobre una partícula es igual al cambio de Energía cinética. La unidad usada para expresar la Energía cinética es igual a la de Trabajo, o sea el Joule (J).

Cuando la fuerza aplicada es constante en magnitud y dirección, se tiene

B B

B AA A

B A

W= F•dr=F• dr=F•(r -r )

W=F•r -F•r (4.10)

∫ ∫r r rr r r r

r rr r

La ecuación (4.10) nos indica que el trabajo es independiente de la trayectoria descrita por la partícula, sólo depende de la cantidad F r

r ri evaluada en la posición

inicial y final. Cuando el trabajo realizado por la partícula sólo depende de la cantidad F rr ri evaluada en la posición inicial y final, se dice que la fuerza es conservativa.

López Frontado, R. (2011). Física general I, Texto UNA. 2da edición. Caracas: UNA

Page 152: Fisica General i Una

162

La cantidad F rr ri es designada por la letra

U, y se denomina Energía potencial. Así se tiene que cuando la fuerza es conservativa, el trabajo se puede expresar por

B

A BA

W= F•dr=U -U =- U (4.11)Δ∫! !

ES IMPORTANTE RESALTAR QUE: El trabajo realizado por fuerzas conservativas es independiente de su trayectoria, sólo depende de su posición inicial y final de la partícula.

Así se tiene que el trabajo realizado por una fuerza conservativa a lo largo de

una trayectoria cerrada es cero, esto es:

OW = F•dr=0

donde significa que la trayectoria es cerrada.

!!

! !""

Con el fin de demostrar la relación del trabajo realizado por fuerzas

conservativas, lo cual se puede considerar igual a menos la diferencia de energía potencial, consideremos el siguiente ejemplo:

Determinar el trabajo realizado sobre una partícula que se mueve a los largo

de una trayectoria curvilínea C, desde el punto A hasta el punto B, bajo el efecto de la fuerza de gravedad g(F =mg)

! !.

Figura 4.6

La fuerza de gravedad para pequeñas diferencias de altura la podemos considerar constante, así se tiene

g

A A A B B B

ˆF =mg=-mgj yˆ ˆ ˆ ˆr =x i+y j , r =x i+y j

! !

! !

El trabajo realizado sobre la partícula, mientras se mueve desde el punto A

hasta el punto B, es

( ) ( ) ( ) ( )( )

AB B A B A B A

AB B A AB A B

ˆ ˆ ˆW = F• r - r = -mgj • x - x i+ y - y j

W = -mg y - y W = mgy - mgy

⎡ ⎤⎣ ⎦⇒

r r r

El resultado de este problema, nos indica que el trabajo realizado por la

Fuerza de gravedad no depende de la trayectoria descrita por la partícula, sino tan sólo de la diferencia yA-yB entre las alturas de los extremos; y por lo tanto, la fuerza de gravedad es una fuerza conservativa. Así se puede concluir que la energía potencial de la fuerza de gravedad es Ug = mgy.

López Frontado, R. (2011). Física general I, Texto UNA. 2da edición. Caracas: UNA

Page 153: Fisica General i Una

163

Otras fuerzas conservativas, y sus respectivas energías potenciales a usarse frecuentemente son:

Fuerza elástica 2

k kk xF =k x U = (4.12)

2ΔΔ →

Donde Δx es la deformación del resorte.

Fuerza gravitatoria 1 2 1 2G G2

Gm m Gm mˆF =- r U =- (4.13)r r

→!

Fuerza eléctrica e 1 2 e 1 2e e2

K q q K q qˆF =± r U =± (4.14)r r

!!

Tomando en cuenta los conceptos emitidos hasta los momentos, responda la

siguiente pregunta: Pregunta 4.2 Una persona viaja en su automóvil por una autopista a 65 km/h. Su coche

tiene energía cinética. De pronto la persona aplica los frenos para detenerse debido a un congestionamiento de tránsito, razón por la cual el coche patina debido a la fuerza de fricción hasta detenerse. ¿En qué está la energía cinética que su coche tenía? 4.3.3 POTENCIA

Desde el punto de vista práctico en la ingeniería, es importante no solo conocer el trabajo realizado por una fuerza sobre un objeto, sino también la rapidez con la que se realiza ese trabajo. Para medir la rapidez del trabajo realizado se considera la proporción entre el trabajo y el tiempo empleado en realizarlo. La relación del trabajo con el tiempo, se conoce conceptualmente como potencia. La potencia nos proporciona la eficiencia con que se realiza un trabajo. La expresión matemática que nos da la potencia media es:

m mTrabajo realizado WP = , P = (4.15)

Intervalo de tiempo empleado tΔ

López Frontado, R. (2011). Física general I, Texto UNA. 2da edición. Caracas: UNA

Page 154: Fisica General i Una

164

La potencia instantánea se puede definir como

t 0

W dWP= lim = (4.16)t dtΔ → Δ

También se conoce que dW=F•dr

r r Por lo tanto, se tiene que

drP=F• =F•v (4.17) dt

!! ! !

La unidad usada en el sistema Internacional (SI) para denotar la potencia es el

watt (W). 1 W = 1 J/s = 1 kg.m2/s2. Otra unidad usada es 1 hp (horsepower) = 746 W.

Ejemplo 4.4

Un automóvil cuya masa es de 900 kg, sube por una colina de 10º de

inclinación, con velocidad constante de 45 km/hr. Calcule el trabajo efectuado por el motor en 6 min, y la potencia media desarrollada por él.

DATOS: m = 900 kg, θ = 10º, v = 45 km/h = 12,5 m/s, Δt = 360 s.

SOLUCIÓN: IDENTIFICAR los conceptos pertinentes: En este problema se pide

determinar el trabajo realizado por el motor, tomando en cuenta que el movimiento del carro es uniforme, o sea que no tiene aceleración. Se pide también la potencia media desarrollada por él.

PLANTEAR Y EJECUTAR la solución del problema:

Figura 4.7

Sea F! la fuerza aplicada por el motor del

automóvil. Aplicando la 2ª ley de Newton, se tiene F – mgsen θ = 0 Debido a que la aceleración a = 0, porque el movimiento del carro es a velocidad constante. Así se tiene que, F = mg sen θ

López Frontado, R. (2011). Física general I, Texto UNA. 2da edición. Caracas: UNA

Page 155: Fisica General i Una

165

El trabajo efectuado por el motor es,

o

6

W=F.x, donde x=vtW=mgsenθ .vt=900×9,8sen10 ×12,5×6×60 W=6,892×10 J!

La potencia media desarrollada por el motor, aplicando la definición de

potencia, está dada por:

6

4m m

W 6,892×10P = = P = 1,915×10 Wt 360

⇒Δ

EVALUAR la solución del problema: los resultados obtenidos para el trabajo y

la potencia desarrollada por el motor, presentan una correspondencia lógica. 4.3.4 TEOREMA TRABAJO Y ENERGÍA

El trabajo total realizado sobre un cuerpo, cuando se mueve bajo la acción de

fuerzas conservativas y no conservativas, es

TotalC NC

C

NC

W =W +W (4.18)donde W =trabajo realizado por fuerzas conservativasy W =trabajo realizado por fuerzas no conservativas

Usando las ecuaciones (4.9) y (4.11), se tiene

NC

B A A B NC

ΔK=-ΔU+W (4.19)K -K =U -U +W (4.20)

Las ecuaciones (4,19) o (4,20) representan el Teorema Trabajo- Energía.

Ejemplo 4.5

Un plano inclinado tiene 13 m de largo y su base 7 m. Un cuerpo de 2,0 kg de

masa resbala desde arriba con una velocidad inicial de 100 cm/s. Si el coeficiente de fricción dinámico es 0,2, determine: ¿cuáles son su energía cinética y su velocidad al llegar al final del plano?

DATOS: b = 7,0 m, d = 13,0 m, m = 2,0 kg, vo = 100 cm/s.

López Frontado, R. (2011). Física general I, Texto UNA. 2da edición. Caracas: UNA

Page 156: Fisica General i Una

166

SOLUCIÓN:

IDENTIFICAR los conceptos pertinentes: En este problema se aplica el teorema de trabajo-energía cinética mientras el

cuerpo se desplaza sobre una superficie inclinada. PLANTEAR la solución del problema: Se considera el movimiento del cuerpo desde un punto situado en la parte alta

del plano inclinado (punto inicial) hasta el punto más bajo del plano (punto final), tomando en cuenta que el trabajo realizado es igual a la diferencia de energía cinética del cuerpo cuando pasa por el punto final menos la energía cinética en el punto inicial de su movimiento.

EJECUTAR la solución del problema: Se hace un esquema como el mostrado

en la figura (4.8), al cual se le hace un DCL.

De la figura se tiene,

-1 o

oA

7θ=cos θ=57,413

h =13sen57,4

⎛ ⎞ →⎜ ⎟⎝ ⎠

y el trabajo realizado por la fuerza de fricción es, FW = -F (13) = - mg(13)cosµ

µ µ θ

Figura 4.8

Aplicando el Teorema Trabajo-Energía, se tiene:

NC

FB A A B

K = - U+ WEsto es,

K -K = U -U + W µ

Δ Δ

La energía cinética del cuerpo, cuando pasa por la parte más baja del plano, es:

2A

B A

2A

B

mvK = +mgh - mgcos .(13)

2mv

K = +mg(13)sen - mg(13)cos2

µ θ

θ µ θ

2o o

B

B

2× (1)K = + 254,8sen57,4 - 50,96cos57,42

K = 188,20 J!

López Frontado, R. (2011). Física general I, Texto UNA. 2da edición. Caracas: UNA

Page 157: Fisica General i Una

167

La velocidad del cuerpo cuando pasa por la parte más baja es:

22B B

B B

2B B

mv 2KK = v =

2 m2×188,20 mv = v = 13,72

2 s

EVALUAR el resultado del problema: los resultados obtenidos se corresponden lógicamente con las condiciones del problema. 4.3.5 PRINCIPIO DE CONSERVACIÓN DE ENERGÍA

Cuando en el movimiento de un cuerpo sólo actúan fuerzas conservativas, o las fuerzas no conservativas son de valor despreciable, se tiene que el trabajo de fuerzas no conservativas es cero. Entonces se tiene que el Teorema de Trabajo y energía expresado en la ecuación (4.18), se puede escribir como:

c

B A A BK=W ó K -K =U -U (4.21)Δ De donde se tiene

A A B BK +U =K +U =constante (4.22)

La cantidad K + U = E se llama la energía mecánica de la partícula, de donde se obtiene que:

EA = EB = cte (4.23)

En otras palabras, la energía mecánica total de la partícula se mantiene

constante. Esto constituye el Principio de conservación de la Energía mecánica de una partícula.

Considerando los conceptos emitidos hasta los momentos, responda las siguientes preguntas:

Pregunta 4.3

Cuando un trineo avanza por un campo nivelado y cubierto de nieve a velocidad constante, ¿se realiza algún trabajo? Razone su repuesta.

López Frontado, R. (2011). Física general I, Texto UNA. 2da edición. Caracas: UNA

Page 158: Fisica General i Una

168

Pregunta 4.4

En un sistema aislado, ¿cuál de las siguientes afirmaciones es un enunciado

correcto de la cantidad que se conserva? a) energía cinética; b) energía potencial; c) energía cinética más energía potencial; y d) energía cinética y energía potencial.

Pregunta 4.5

Un bloque se conecta a un resorte ligero suspendido verticalmente del techo. Cuando se desplaza hacia abajo desde su posición de equilibrio y se suelta, el bloque oscila hacia arriba y abajo. En el sistema bloque, resorte y la Tierra, ¿qué forma de energía hay durante el movimiento?

Ejemplo 4.6

Un cuerpo de 200 g de masa se deja caer desde una altura de 5,0 m, sobre

un montón de arena. Si el cuerpo penetra en la arena 10 cm antes de detenerse, ¿qué fuerza constante ejerció la arena sobre él?

SOLUCIÓN: IDENTIFICAR los conceptos pertinentes:

Analizando el enunciado del problema, se observa que el movimiento del cuerpo se realiza en dos etapas. La primera etapa, mientras se mueve desde A hasta el punto B (su movimiento es de caída libre), considerándose despreciable el efecto de la fuerza de fricción, donde el trabajo realizado por las fuerzas no conservativas es cero, razón por la cual se tiene que la energía mecánica en el punto A es igual a la energía mecánica en el punto B.

Figura 4.9

López Frontado, R. (2011). Física general I, Texto UNA. 2da edición. Caracas: UNA

Page 159: Fisica General i Una

169

La segunda etapa, es cuando el cuerpo penetra en la arena, o sea cuando se

mueve desde el punto B hasta el punto C; en esta etapa la fuerza ejercida por la arena es una fuerza de roce ejercida por la arena sobre el cuerpo, esta fuerza no es conservativa. PLANTEAR el problema:

Consideremos el nivel a la altura de B, como nivel de referencia, donde yB = 0. Así se tiene que, desde A → B,

2B

A B A B

2B

mvE = E , donde E = mgy, y E =

2Al sustituir, se tiene

mvmgy = (1)

2

Luego, tomando en cuenta el movimiento desde B → C, se tiene

nc

2B

r

K = - U+ W

mv0 - = 0 ( mg y) -F y (2)

2

Δ Δ

− − Δ Δ

EJECUTAR la solución del problema: se sustituye la ecuación (2) en la ecuación (1). Así se obtiene que

r r

r

r

-mgy=mgΔy-FΔy, FΔy=mgΔy+mgyy 5F =mg(1+ )=0,2×9,8 1+Δy 0,1

F =99.86 N

! "# $% &

'

EVALUAR la solución: se observa que el valor de F es lógico por estar dentro de los parámetros previstos.

López Frontado, R. (2011). Física general I, Texto UNA. 2da edición. Caracas: UNA

Page 160: Fisica General i Una

170

Ejercicio propuesto 4.1.- Como se muestra en la figura (4.10), un cuerpo de masa m = 2,0 kg,

está sujeto por:

Figura 4.10

medio de una cuerda, de longitud L = 1,5 m, a un punto O del techo. Inicialmente el cuerpo está en la posición A. Se suelta el cuerpo, describiendo una trayectoria circular de radio L, alrededor del punto O; cuando pasa por el punto más bajo de su trayectoria (en el punto B) se encuentra con la superficie horizontal rugosa; en esa posición una hojilla corta la cuerda que sostiene al cuerpo.

Si el coeficiente entre la superficie rugosa y el cuerpo es µ = 0,2, determine la

distancia D que el cuerpo recorre antes de detenerse.

Ejercicio propuesto 4.2.-

Una fuerza neta de magnitud F = 7,0 x2, que forma un ángulo constante de 31º con el eje +x, actúa sobre un objeto de 200 g, mientras éste se mueve paralelamente al eje x.

a) ¿Qué rapidez tiene el objeto en x2 = 1,5 m, si su rapidez era de 4,0 m/s en x1 = 1,0 m?

b) ¿Qué trabajo total se realiza sobre el objeto, cuando se desplaza desde

x1 hasta x2?

Ejercicio propuesto 4.3.- En la figura se muestran dos bloques, uno de masa m1 = 50,0 kg, y el otro de masa

López Frontado, R. (2011). Física general I, Texto UNA. 2da edición. Caracas: UNA

Page 161: Fisica General i Una

171

Figura 4.11

m2 = 100 kg, que se conectan entre sí por medio de una cuerda ligera e inextensible. La polea no representa fricción y su masa es despreciable. El coeficiente de fricción dinámico entre el bloque m1 y la pendiente es µd = 0,25. Determine el cambio de energía cinética del bloque m1 cuando se mueve de A a B, una distancia de 20,0 m.

Ejercicio propuesto 4.4.-

En la figura se muestra un bloque de 10,0 kg que se suelta desde el punto A.

La pista no ofrece fricción excepto en la superficie horizontal a partir del punto B. La distancia BC tiene una longitud de 6,0 m. El bloque se mueve hacia abajo por la pista, golpea un resorte de constante de fuerza k = 2250 n/m y lo comprime 30,0 cm a partir de su posición de equilibrio antes de quedar momentáneamente en reposo.

Determine el coeficiente de fricción cinético entre la superficie BC y el bloque.

Figura 4.12

Ejercicio propuesto 4.5.-

Una partícula de 0,4 kg desliza sobre un carril circular horizontal, que tiene 1,5 m de radio. A la partícula se le imprime una rapidez inicial de 8 m/s. Después de una revolución, su rapidez cae hasta 6 m/s debido a la fricción.

a) Calcule el trabajo realizado por la fuerza de rozamiento en una

revolución.

b) Calcule el coeficiente de fricción dinámico entre la partícula y la superficie del carril.

c) ¿Cuál es el número total de revoluciones que efectuará la partícula

antes de llegar al reposo.

López Frontado, R. (2011). Física general I, Texto UNA. 2da edición. Caracas: UNA

Page 162: Fisica General i Una

172

RESUMEN:

El trabajo realizado por una fuerza aplicada sobre una partícula, cuando

ésta se desplaza desde un punto A hasta un punto B, a lo largo de una trayectoria C, está dada por:

B B

ABA A

W = F dr= Fdscosθ∫ ∫r ri

Cuando sobre la partícula actúan varias fuerzas, y por efecto de ellas, ésta

se desplaza desde A hasta B, a lo largo de una trayectoria, se tiene que el trabajo realizado está dado por:

i

n nBF R R

AB i iAi=1 i=1

W = W = F dr, donde F = F∑ ∑∫r r r rr

i

La energía de la partícula que depende de su rapidez, denominada Energía Cinética, representa por la letra K, está dada por 21K= mv

2. El trabajo realizado en

estas condiciones, está dado por: 2 2AB B A B A

1 1W = K=K -K = mv - mv2 2

!

Cuando el trabajo realizado sobre una partícula, depende de su posición

inicial y de su posición final, siendo independiente de la trayectoria recorrida por la partícula, se dice que la fuerza que produce este tipo de trabajo es conservativa. Tomando en cuenta que existen fuerzas conservativas y no conservativas, se tiene en que el trabajo realizado se puede expresar por C NC

AB AB ABW =W +W .

Donde el trabajo realizado por las fuerzas conservativas está dado por C

AB A BW =- U=U -U! . Siendo UA y UB la energía potencial de la partícula, la cual depende de la posición de ella.

Para determinar la eficiencia de un trabajo realizado, se determina la

potencia desarrollado por una fuerza, así se tiene que potencia media está dada por m

WP =tΔ.

La potencia instantánea está dada por

t 0

W dWP= lim =t dt! " !

.

López Frontado, R. (2011). Física general I, Texto UNA. 2da edición. Caracas: UNA

Page 163: Fisica General i Una

173

Cuando el trabajo realizado solo depende de fuerzas conservativas, se tiene

que,

B A A B

A B

K=- U ó K -K =U -U E =E =constante, donde E=K+U

! !

"

Esto constituye el Principio de conservación de energía mecánica de la

partícula.

EJERCICIOS DE AUTOEVALUACIÓN

Desarrolle en su libreta de apuntes los siguientes ejercicios. Luego de resolver los ejercicios, compare sus respuestas con los presentados al final de la unidad. Si tiene alguna duda consulte al asesor de su centro local.

4.1.- Un bloque de 20 kg de masa es impulsada con una velocidad inicial cuyo módulo es de 5,2 m/s para que suba por una rampa de 25°. El coeficiente de rozamiento cinético entre las superficies es de 0,34. Determine el trabajo realizado sobre el bloque cuando alcanza su punto más alto en la rampa, por: a) la fuerza resultante que actúa sobre el bloque; b) el peso del bloque; c) la fuerza normal; y d) la fuerza de rozamiento.

4.2.- El tirador de bola de un juego llamado “pinball” está equipado con un

resorte que tiene una constante elástica de 1,20 N/cm. La superficie sobre la cual se mueve la bola está inclinada 10° con respecto a la horizontal. Si el resorte se comprime inicialmente 5,0 cm, encuentre la rapidez de lanzamiento de una bola de 100 g cuando se suelta del tirador. La fricción y la masa del tirador son despreciables.

4.3.- Un elevador de 650 kg inicia desde el reposo, sube durante 3,0 s con

aceleración constante hasta que alcanza una rapidez de crucero de 1,75 m/s. ¿Cuál es la potencia promedio del motor de ese elevador durante este periodo?

4.4.- Un paracaidista de 80,0 kg salta de un globo a una altitud de 1000 m y

abre el paracaídas a una altitud de 200 m. A) Si se supone que la fuerza retardadora total sobre el paracaidista es constante a 50,0 N con el paracaídas cerrado y constante a 3600 N con el paracaídas abierto, ¿cuál es la rapidez del paracaidista cuando llega al suelo?; b) ¿A qué altura debe abrirse el paracaídas para que la rapidez final del paracaidista sea de 5,0 m/s?

López Frontado, R. (2011). Física general I, Texto UNA. 2da edición. Caracas: UNA

Page 164: Fisica General i Una

174

4.5.- Un objeto de 1,0 kg se desliza a la derecha sobre una superficie que

tiene un coeficiente de fricción dinámico de 0,25. El objeto tiene una rapidez de vi = 3,0 m/s cuando hace contacto con un resorte ligero que tiene una constante elástica de 50,0 N/m. El objeto se detiene después que el resorte se ha comprimido una distancia d. Luego el objeto es impulsado hacia la izquierda por el resorte , y continúa moviéndose en esa dirección más allá de la posición no estirada del resorte, deteniéndose a una distancia D a la izquierda del resorte no estirado. Encuentre: a) la distancia de compresión d; b) la rapidez con el objeto abandona al resorte; c) la distancia D donde el objeto no se detiene. RESPUESTA A LAS PREGUNTAS:

Estas respuestas se corresponden con las preguntas presentados en el desarrollo de la unidad.

Respuesta a la pregunta 4.1 La persona no realiza ningún trabajo, debido a que, de acuerdo con la

definición de Trabajo, la fuerza perpendicular al desplazamiento no produce trabajo, y en este caso la fuerza ejercida por la persona para sostener la caja sobre sus hombros está dirigido verticalmente hacia arriba, y por lo tanto es perpendicular al desplazamiento horizontal.

Respuesta a la pregunta 4.2 Debido a la fricción entre las ruedas y el pavimento, la energía cinética que

traía el automóvil se transforma en energía interna y parte se transfiere por ondas mecánicas, como es el sonido producido por el frenazo.

Respuesta a la pregunta 4.3 Como este trineo se desplaza por el camino novelado a una velocidad

constante, no se produce ningún trabajo, debido a que no ninguna variación de energía cinética y además todas las fuerzas que actúan sobre el trineo son perpendiculares al desplazamiento.

Respuesta a la pregunta 4.4 Se conserva la energía mecánica total, que es igual a la suma de la energía

cinética más la energía potencial. Respuesta a la pregunta 4.5 Este sistema presenta cambio en la energía cinética, así cómo en las energías

potenciales de gravitación y elástica.

López Frontado, R. (2011). Física general I, Texto UNA. 2da edición. Caracas: UNA

Page 165: Fisica General i Una

175

RESPUESTAS DE LOS EJERCICIOS PROPUESTOS: Estas respuestas corresponden con los ejercicios propuestos que se

presentan en el desarrollo de la unidad.

Respuesta al ejercicio propuesto 4.1 DATOS: m=2,0 kg, L = 1,5 m, µd = 0,2. SOLUCIÓN: Al comprender el enunciado del problema, se observa que el movimiento del

cuerpo se realiza en dos etapas, la primera donde no existe roce; y la segunda donde existe roce entre el cuerpo y la superficie, así se tiene que:

Desde A hasta B, se tiene µ NC A B

2B

F =0 W =0, E =E ,

Tomemos nivel de B como referencia.mv

mgL= (1)2

→!

Desde B hasta C, se tiene

C NC

2B

µ

F 0 ΔK=W +W

mv0- =-F D (2)

2

µ ≠ →!

Sustituyendo la ecuación (1) en la ecuación (2), se tiene mgL = F D, donde F = mg

mgL = mgD,L 1,5D = = D = 7,5 m

0,2

µ µ µµ

µ

Respuesta al ejercicio propuesto 4.2 DATOS: F = 7,0 x2, m = 200 g = 0,2 kg, θ = 31º , vA = 4,0 m/s, x1 = 1,0 m, x2 = 1,5 m

López Frontado, R. (2011). Física general I, Texto UNA. 2da edición. Caracas: UNA

Page 166: Fisica General i Una

176

SOLUCIÓN:

Figura 4.13 a) Para determinar la velocidad del cuerpo cuando pasa por la posición B,

se usa el teorema de trabajo y energía, esto es:

2 2

1 1

x xF F 2

x xK=W , pero W = Fcos dx= 7,0x cos dxθ θΔ ∫ ∫

2

1

2 2 3x 2 2 2 2B A

B 2 1 Ax

mv mv 7,0x 2 7,0- = cos , v = (x -x )cos31°+v2 2 3 3m

θ⎛ ⎞ ×⎜ ⎟⎝ ⎠

2 2 2B B

2 7,0 mv = (1,5) -(1,0) 0,86+16 v =7,97 3 0,2 s× ⎡ ⎤ × ⇒⎣ ⎦×

b) El trabajo realizado sobre el cuerpo al moverse desde A hasta B, es:

T TW =23,3 2,375 0,86 W =47,6 J× × ⇒

Respuesta al ejercicio propuesto 4.3 DATOS: m1 = 50 kg, m2 = 100 kg, µd = 0,25, AB = 20 m SOLUCIÓN: Aplicando el teorema de Trabajo-Energía, se tiene µ1 2Fm g m gΔK=W +W +W

o o1 d 1 2

o o

ΔK=0-m g(ABsen37 )-µ m g(ABcos37 )+m g(AB)

ΔK= -50×20sen37 -0,25×50×20cos37 +100×20 ×9,8

ΔK=11746 J

⎡ ⎤⎣ ⎦⇒

Respuesta al ejercicio propuesto 4.4 DATOS: m = 10 kg, BC = 6,0 m, k = 2250 N/m, ΔxD = 30,0 cm,

hA = 3 m

López Frontado, R. (2011). Física general I, Texto UNA. 2da edición. Caracas: UNA

Page 167: Fisica General i Una

177

SOLUCIÓN: Para determinar el coeficiente de fricción entre el bloque y la superficie en el

tramo BC, aplicamos el teorema trabajo y energía en su movimiento desde A hasta D. Así se tiene que:

e

2FFmg

AD A

2

2 A

A

k xK =W +W +W , 0-0=mgh -0+0- -F (BC+ x)2

k xmgh -k x 20=mgh - - mg(BC+ x), =2 mg(BC+ x)

µµ

µ µ

ΔΔ Δ

ΔΔ Δ →

Δ

22250(0,3)10 9,8 3-2= =0,3122

10 9,8(6+0,3)µ µ

× ×⇒

×

Respuesta al ejercicio propuesto 4.5 DATOS: m = 0,4 kg, R = 1,5 m, vo = 8 m/s, vf = 6 m/s SOLUCIÓN:

Figura 4.14

a) Aplicando el teorema de trabajo y energía, se tiene TotalK WΔ = . Pero FTotalW =W µ , debido a que la fuerza peso y la normal son perpendiculares al desplazamiento y por lo tanto no producen trabajo. Así se tiene:

22Fof

F F

mvmvK W

2 20,4W (36 64) W 5,6 J2

µ

µ µ

Δ = − =

= − ⇒ = −

b) Para determinar el coeficiente de fricción dinámico entre la partícula y la

superficie del carril, se tiene que:

FF

d d

d d

WW F d mg(2 R)mg2 R

( 5,6) 0,1520,4 9,8 2 (1,5)

µµ

µ µ π µπ

µ µπ

= − = − → =

−= ⇒ =× ×

c) Para determinar el número de vueltas (o revoluciones) N, es necesario

conocer la distancia recorrida por la partícula antes de detenerse. Así se tiene que

2F o

d

2 2o

d

mvK W 0 mg(2 Rn)

2v (8)n n 2,28 rev.g4 R 4 0,152 9,8 1,5

µ µ π

µ π π

Δ = → − = −

= = ⇒ =× × × ×

López Frontado, R. (2011). Física general I, Texto UNA. 2da edición. Caracas: UNA

Page 168: Fisica General i Una

178

RESPUESTA DE LOS EJERCICIOS DE AUTOEVALUACIÓN Respuesta al ejercicio de autoevaluación 4.1 a) WT = - 270 J; b) Wmg = - 156,6 J; c) WN = 0; d) WFµ = - 114,2 J Respuesta al ejercicio de autoevaluación 4.2 vB = 1,68 m/s Respuesta al ejercicio de autoevaluación 4.3 P = 327,99 W Respuesta al ejercicio de autoevaluación 4.4 a) v = 97,98 m/s; b) H = 206,48 m Respuesta al ejercicio de autoevaluación 4.5 a) d = 0,422 m; b) vf = 2,62 m/s; c) D = 1,40 m

López Frontado, R. (2011). Física general I, Texto UNA. 2da edición. Caracas: UNA

Page 169: Fisica General i Una

179

MÓDULO III

DINÁMICA DE SISTEMAS DE PARTÍCULAS Hasta los momentos se ha estudiado el movimiento de los cuerpos, como si fueran cuerpos puntuales o partículas que tienen masa pero no tamaño. En realidad esta restricción no es muy grave, por cuanto todos los puntos de un cuerpo en movimiento de traslación se comportan como si fueran una partícula. Sin embargo, esta restricción no es válida para muchos objetos en movimiento, sobre todo si el cuerpo gira o vibra durante su movimiento. Consideremos además que todo cuerpo (u objeto) está constituido por un sistema de partículas. Existe un punto del sistema de partículas cuyo movimiento, bajo la influencia de fuerzas externas, se puede estudiar como el de una partícula simple. Este punto se llama centro de masa. En este módulo se describirá cómo hallar el centro de masa del sistema de partículas, y se demostrará qué reglas para el movimiento del centro de masa conducen a las grandes leyes de conservación que encontraremos: la conservación del momento lineal y del momento angular. Estos conceptos y principios estudiados en este módulo nos permitirán describir el movimiento general de los cuerpos, el cual comprende los movimientos de traslación y los movimientos de rotación. Estudiaremos, además, las interacciones de las partículas que constituyen el sistema entre si y con el entorno. También se aplicarán estos conceptos en los movimientos de otros sistemas de partículas, tales como: cuerpos rígidos; cuerpos en el universo; cuerpos oscilatorios; y fluidos.

OBJETIVO

Aplicar los conceptos y principios que definen el movimiento de un sistema de partículas, bajo una situación dada. ESTRUCTURA DEL MÓDULO: UNIDAD 5.- MOMENTO LINEAL Y MOMENTO ANGULAR. COLISIÓN. UNIDAD 6.- TRABAJO Y ENERGÍA PARA UN SISTEMA DE PARTÍCULAS.

COLISIÓN. UNIDAD 7.- CUERPOS RÍGIDOS. UNIDAD 8.- GRAVITACIÓN UNIVERSAL. UNIDAD 9.- MOVIMIENTO OSCILATORIO. UNIDAD 10.- FLUIDOS.

López Frontado, R. (2011). Física general I, Texto UNA. 2da edición. Caracas: UNA

Page 170: Fisica General i Una

181

UNIDAD 5

MOMENTOS LINEAL y ANGULAR

En esta unidad se extiende la aplicación de las leyes de la mecánica clásica

para describir los movimientos de sistemas de partículas. Aunque se conoce que en un sistema de partículas, las partículas que forman parte del mismo, interactúan entre si y con partículas del entorno, se considerará una partícula representativa localizada en un punto perteneciente al sistema, con movimiento bajo la influencia de fuerzas externas, que puede ser estudiada como el de una partícula simple. Esta partícula se denomina centro de masa.

Se describirá cómo hallar la posición del centro de masa del sistema de

partículas. Se estudiará además el concepto de momento lineal (o cantidad de movimiento lineal). Este concepto se relaciona con el movimiento de traslación del sistema, razón por la cual se demostrarán las reglas simples para el movimiento de su centro de masa, conduciendo así a la segunda ley de conservación que conocemos: la ley de conservación del momento lineal.

También se estudiarán los conceptos de Momento Angular (o Cantidad de

Movimiento Angular), Momento de Inercia y del Momento de Torsión (o Torca). Estos conceptos caracterizan las rotaciones del sistema de partículas. También se estudiará la relación que existe entre el momento angular y el momento de torsión, llegando finalmente a la situación, en la cual se anula el momento de torsión debido a las fuerzas externas sobre un sistema de partículas, conduciendo así a la llamada Ley de Conservación del Momento Angular.

CONOCIMIENTOS PREVIOS Para que el estudiante comprenda el estudio de esta unidad, es necesario

poseer los siguientes conocimientos previos: 1.- Punto material (Unidad 1). 2.- Cinemática del punto (Unidad 2). 3.- Ecuación fundamental de la Dinámica (Unidad 3).

López Frontado, R. (2011). Física general I, Texto UNA. 2da edición. Caracas: UNA

Page 171: Fisica General i Una

182

5.1 OBJETIVO Aplicar los conceptos de Momento Lineal, la 1ª Ley Fundamental de la

Mecánica, Momento Angular y la 2ª Ley Fundamental de la Mecánica en la resolución de problemas de dinámica de sistemas de partículas.

5.2 RECOMENDACIONES PARA EL ESTUDIO DEL CONTENIDO

En esta unidad, el estudiante debe

comprender claramente los conceptos básicos siguientes:

Momento Lineal, Centro de masa, el

principio de Conservación del Momento Lineal, Momento Angular, Momento de Inercia, Momento de Torsión y el principio de Conservación del Momento Angular.

CONCEPTOS BÁSICOS: Momento Lineal, Centro de masa, el principio de Conservación del Momento Lineal, Momento Angular, Momento de Inercia, Momento de Torsión, el principio de Conservación del Momento Angular.

Para una mejor comprensión del contenido de la unidad, realice la lectura de

los conceptos relacionados con el movimiento de un sistema de partículas aplicando los conceptos de Momento Lineal, de Momento Angular y los ejemplos presentados; utilice las técnicas de lectura que le facilite la comprensión del tema.

5.3 CONTENIDO

5.3.1 DEFINICIÓN DE MOMENTO LINEAL. 5.3.2 IMPULSO 5.3.3 CENTRO DE MASA. 5.3.4 MOMENTO LINEAL PARA UN SISTEMA DE PARTÍCULAS. 5.3.5 1ª LEY FUNDAMENTAL DE LA MECÁNICA. 5.3.6 DEFINICIÓN DE MOMENTO ANGULAR. 5.3.7 MOMENTO ANGULAR PARA UN SISTEMA DE PARTÍCULAS. 5.3.8 MOMENTO DE INERCIA.

López Frontado, R. (2011). Física general I, Texto UNA. 2da edición. Caracas: UNA

Page 172: Fisica General i Una

183

5.3.9 MOMENTO DE TORSIÓN. 5.3.10 2ª LEY FUNDAMENTAL DE LA MECÁNICA. 5.3.11 PRINCIPIO DE CONSERVACIÓN DE MOMENTO ANGULAR.

5.3.1 DEFINICIÓN DEL MOMENTO LINEAL Existen situaciones donde es conveniente conocer una expresión que

relacione la masa de un objeto con su movimiento. Como se estudió en la unidad 3, el momento lineal de una partícula es un concepto que establece una relación entre la masa y la velocidad del cuerpo.

El momento lineal de una partícula de masa, se define como el producto de

su masa m por la velocidad v!

de su movimiento, y se representa por la letra p!

. Esto es,

p=mv (5.1)r r

El momento lineal es una cantidad vectorial, cuya magnitud es (mv) y

dirección igual a la de la velocidad v!

. Sus dimensiones son MLT-1, y en el sistema de unidades SI se expresa por kg.m.s-1.

Se puede relacionar el momento lineal de una partícula con la fuerza

resultante que actúa sobre ella, a través de la 2ª ley de Newton. Así se tiene que:

dp d(mv)F= = (5.2)dt dt∑! !!

De esta ecuación se observa que si

F=0!!

, se tiene que dp =0dt

r, por lo tanto el

momento lineal p!

se mantiene constante. Lo mismo podemos decir si la partícula está aislada. Esto constituye el Principio de Conservación del Momento Lineal.

Para una partícula: el PRINCIPIO DE CONSERVACIÓN DEL MOMENTO LINEAL, nos indica que el momento lineal de la misma se mantiene constante, esto es

i fp =pr r

López Frontado, R. (2011). Física general I, Texto UNA. 2da edición. Caracas: UNA

Page 173: Fisica General i Una

184

Basados en los conceptos emitidos hasta los momentos, responda la

siguiente pregunta: Pregunta 5.1 Si dos cuerpos tienen igual energía cinética, ¿cómo se comparan las

magnitudes de sus momentos lineales?

5.3.2 IMPULSO

Como se ha visto, el momento lineal de la partícula cambia si una fuerza neta

actúa sobre ella. De acuerdo a la 2ª ley de Newton, se tiene:

nR R

ii=1

dpF = F= dp=F dtdt

→∑rr r rr

Para determinar el cambio en el momento lineal de una partícula, se integra,

obteniéndose

f

i

t Rf i t

Δp=p -p = F dt (5.3)∫!! ! !

La cantidad de la derecha de la ecuación (5.3) recibe el nombre de impulso

proporcionado por la fuerza F!

sobre la partícula durante el intervalo de tiempo Δt = tf – ti. El impulso es un vector definido por

f

i

t

tI= Fdt=Δp (5.4)∫! ! !

Expresión conocida como Teorema impulso-momento.

En base a los conceptos emitidos hasta los momentos, responda la siguiente

pregunta: Pregunta 5.2 A medida que un cuerpo cae hacia la Tierra, el momento de la pelota aumenta

debido al aumento de su rapidez, ¿esto significa que el momento lineal no se conserva en esta situación?

López Frontado, R. (2011). Física general I, Texto UNA. 2da edición. Caracas: UNA

Page 174: Fisica General i Una

185

Ejemplo 5.1.- Se suelta un balón, de masa m = 2,0 kg, desde una altura HA = 2 m. Una vez

que el balón choca con el suelo, rebota hasta una altura HC = 1,7 m. Determine: a) El impulso debido a la gravedad al caer el balón.

b) El impulso al chocar con el suelo.

DATOS: m = 2,0 kg, HA = 2,0 m, HC = 1,7 m SOLUCIÓN: IDENTIFICAR los conceptos pertinentes: Para determinar el impulso debido a la fuerza de la gravedad, se calcula el

momento lineal del balón justo antes de chocar con el suelo. Para esto, es necesario conocer la velocidad del balón en ese instante.

PLANTEAR el problema: Suponiendo despreciable el roce producido por el aire, mientras cae el balón,

desde el punto A hasta el punto B, como se muestra en la figura (5.1) se tiene que la energía mecánica del balón se mantiene constante.

Figura 5.1

Esto es, 2B

A B A

2Ba A Ba

mvE = E mgH =

2mv = 2gH = 2×9,8×2 v = 6,26 s

!

!

EJECUTAR la solución: a) Para determinar el impulso debido a la fuerza de la gravedad, se tiene

Ba A BaˆI = p - p = mv - 0 = 0,2(-6,26 j)

kg.mˆ I = -1,252 j s

! ! ! !

!

b) Para determinar el impulso durante el choque del balón con el suelo, es

necesario conocer el momento del balón justo después de choque.

López Frontado, R. (2011). Física general I, Texto UNA. 2da edición. Caracas: UNA

Page 175: Fisica General i Una

186

Para esto, consideremos ahora el movimiento del balón, desde el instante que rebota hasta alcanzar su máxima altura, esto es desde B hasta el punto C. Nuevamente, se puede considerar que la energía mecánica se conserva, y se tiene que justo después de rebotar:

2Bd

Bd C C

2Bd C Bd

mvE = E = mgH

2mv = 2gH = 2×9,8×1,7 v = 5,77 s

!

!

El impulso durante el choque es:

[ ]Bd Ba Bd Ba

ˆI = p - p = m(v - v ) = 0,2 5,77 - (-6,26) jkg.m I = 2,41 j

s⇒

! ! ! ! !

! !

EVALUAR la repuesta Si pudiéramos medir el tiempo que tardó el balón en caer y el tiempo del

choque, podríamos comparar las fuerzas media de impulso en cada uno de los casos.

5.3.3 CENTRO DE MASA Para describir el movimiento de un sistema de partículas es bastante útil hacer

uso del concepto llamado centro de masa. Supongamos que se tiene un sistema de partículas con masas m1, m2, m3,.., cuyas posiciones son 1 2 3r , r , r ,...,

! ! !

respectivamente. Se puede entonces definir el vector posición del centro masa CMrr

de las partículas en función de los vectores posiciones 1 2 3r , r , r ,...,

! ! ! de las partículas

por: n

i i1 1 2 2 3 3 n n i=1

CM1 2 3 n

n

1 2 3 n ii=1

m rm r +m r +m r +.....+m rr = = (5.5)m +m +m +......+m M

donde M=m +m +m +...+m = m

!

!

rr r r r

r

Cuando en lugar de un sistema de partículas, se tiene un cuerpo cuya masa

se presenta como una distribución continua de masa, tal como los llamados cuerpo rígido, la posición del centro de masa de ese cuerpo está dado por:

CM1r = rdm (5.6)M ∫

! !

donde r

r es la distancia del elemento diferencial de masa dm, hasta el origen del

sistema de coordenadas de referencia.

López Frontado, R. (2011). Física general I, Texto UNA. 2da edición. Caracas: UNA

Page 176: Fisica General i Una

187

5.3.4 MOMENTO LINEAL DE UN SISTEMA DE PARTÍCULAS Derivando CMr

r de la ecuación (5.5) con respecto al tiempo, se tiene la

velocidad del Centro de Masa del sistema, esto es:

nCM i

CM ii=1

dr dr1v = = m (5.7)dt M dt∑r r

r

n

i i1 1 2 2 3 3 n n i=1

CM1 2 3 n

mvm v +m v +m v +......+m vv = = (5.8)m +m +m +......+m M

∑r

r r r rr

De la definición de momento lineal, se tiene i i ip =mv

! !, entonces podemos

escribir: n

ii=1

CM CM

p Pv = = ó P=Mv (5.9)M M

!r r

rr r

donde iP= p∑

r r, es el Momento Lineal Total del Sistema de partículas.

Este resultado es muy importante, por

cuanto sugiere que el Momento Total del Sistema es el mismo que correspondería al caso en que toda la masa del sistema estuviese concentrada en el Centro de Masa, moviéndose con velocidad CMv

!. Por esta

razón, CMv!

se llama algunas veces la Velocidad del Sistema.

EL MOMENTO LINEAL TOTAL DEL SISTEMA:

CMP=Mvr r

Sugiere que el Momento Total del sistema es el mismo que correspondería al caso, en que toda la masa del sistema estuviese concentrada en Centro de Masa.

Donde iP= p∑

r r, es el Momento Lineal Total del Sistema de partículas.

5.3.5 PRIMERA LEY FUNDAMENTAL DE LA MECÁNICA

Aplicando la 2ª ley de Newton a un sistema de partículas, se tiene

ext CMCM

dvdPF = =M =Ma (5.10)dt dt∑r rr r

Esta ecuación constituye la 1a ley

fundamental de la mecánica, la cual nos dice que la suma de las fuerzas externas que actúan sobre un sistema de partículas es igual al cambio del momento lineal del sistema de partículas respecto al tiempo.

LA 1ª LEY FUNDAMENTAL DE LA MECÁNICAS, dice

que: ext dPF =

dt!r

r

López Frontado, R. (2011). Física general I, Texto UNA. 2da edición. Caracas: UNA

Page 177: Fisica General i Una

188

¿Porqué se toma en la ecuación (5.19) solo la suma de las fuerzas externas del sistema? Para responder esta pregunta, supongamos un sistema de partículas S1, formado por dos partículas de masas m1 y m2, que interactúan entre sí, pero a su vez también interactúan con partículas del sistema externo que lo rodea, S2, como se muestra en la figura (5.2), donde 12 21F y F

! ! son las fuerzas de

interacción mutua, de acuerdo con el principio de acción-reacción de Newton, y 1 2F y F

r rson las

fuerzas resultantes de la interacción de cada partícula con las partículas del sistema externo S2.

Aplicando la 2ª ley de Newton para cada

partícula, se tiene

1 21 12 2 21

1 21 12 2 21

dp dp=F +F y =F +Fdt dt

dp dpdP = + =F +F +F +Fdt dt dt

r rr r r r

r r r r r r r

Pero, por la ley de acción-reacción, se tiene

que

12 21F = -F! !

, por lo tanto

ext1 2

dP dP=F +F = Fdt dt

! "! !

! ! !

Lo que se quería demostrar.

Figura 5.2

Si el sistema está aislado, o la extF =0!

!, se

tiene que dP =0dt

!

, y por lo tanto el momento lineal

del sistema permanece constante, esto es, P!

es constante. Esto constituye el Principio de Conservación del Momento Lineal de un sistema de partículas.

POR EL PRINCIPIO DEL MOMENTO LINEAL DE UN SISTEMA DE PARTÍCULAS,

CM

CM

P=Mv =constante v =constante→

! !

!

Tomando en cuenta las consideraciones anteriores, el Centro de Masa del

sistema se mueve con velocidad constante con relación a un sistema de referencia inercial. También puede fijarse un sistema de referencia en el centro de masa de un

López Frontado, R. (2011). Física general I, Texto UNA. 2da edición. Caracas: UNA

Page 178: Fisica General i Una

189

sistema aislado, y en relación con este sistema inercial, el centro de masa estará en reposo ( CMv =0

!). Este es el llamado sistema de referencia del centro de masa, o

sistema C de referencia. Por esta razón, se tiene que CM i,CMP = p =0∑r r

. Si el sistema de partículas está aislado y en reposo, se tiene que

CMCM

drv = =0,dt

rr

Se concluye entonces que CMr

r es constante.

A continuación se presentan unos ejemplos de problemas, donde Ud. puede

visualizar las aplicaciones de los conceptos tratados en esta unidad. Pero antes vea las Sugerencias y las Recomendaciones presentadas para resolver estos problemas.

RECOMENDACIONES PARA RESOLVER PROBLEMAS DE DINÁMICA DE

SISTEMAS DE PARTÍCULAS USANDO LOS CONCEPTOS DE MOMENTO LINEAL

IDENTIFICAR los conceptos pertinentes. Decida si se aplica o no el principio

de conservación de momento lineal. Acuérdese que esto sólo es cierto si la resultante de las fuerzas externa que actúan sobre el sistema de partículas es cero. De no ser así, no se puede usar la conservación del momento lineal.

Decida también si se aplica el principio de conservación del momento

angular. Recuerde que esto sólo es posible si el momento de torsión resultante producido por las fuerzas externas que actúan sobre el sistema de partículas es cero. De no ser así, no es posible aplicar el principio de conservación del momento angular.

PLANTEAR el problema siguiendo estos pasos: a) Defina un sistema de coordenadas. Dibuje los ejes, indicando la

dirección positiva de cada uno. Es conveniente escoger un marco de referencia inercial. Casi todos los problemas de la unidad tratan casos bidimensionales, donde los vectores sólo tienen componentes (x, y); lo cual puede ampliarse para incluir componentes z, si es necesario.

López Frontado, R. (2011). Física general I, Texto UNA. 2da edición. Caracas: UNA

Page 179: Fisica General i Una

190

b) Trate cada cuerpo como una partícula. Haga dibujos de “antes” y “después”, incluyendo vectores para representar las velocidades conocidas. Rotule los vectores con magnitudes, ángulos, componentes y demás informaciones dadas, asignando signos algebraicos a las incógnitas. Use letras (no números) para rotular las partículas. Es conveniente usar subíndices para identificar las velocidades para antes y después de la interacción, respectivamente.

c) Identifique las incógnitas de las variables desconocidas. EJECUTAR la solución como sigue: d) Escriba las ecuaciones en término de símbolos, igualando las

componentes x total inicial del momento lineal con las componentes x final, usando px = mvx para cada partícula. Escriba otras ecuaciones para las componentes y, usando py = mvy para cada partícula.

f) Resuelva estas ecuaciones para determinar los resultados requeridos.

Recuerde que en algunos problemas tendrá que pasar de las componentes de una velocidad a su magnitud y dirección, o viceversa.

EVALUAR la repuesta. f) ¿Es lógica esta respuesta? Si tiene alguna duda en las repuestas,

consúltelas a su asesor del Centro Local.

Recuerde que el manejo eficiente del contenido de esta unidad es

importante, por su repercusión en el contenido de las subsiguientes unidades del curso de Física General I.

Ejemplo 5.2.- En un determinado instante, tres partículas se mueven como se muestra en la

figura (5.3). Las masas y velocidades de las partículas son: m1 = 2 kg, m2 = 0,5 kg, m3 = 1 kg, v1 = 1 m/s, v2 = 2 m/s, v3 = 4 m/s, v1f = 3 m/s. Las partículas están sujetas a sus interacciones mutuas, no existen fuerzas externas sobre el sistema. Después de un cierto tiempo, son observadas de nuevo y se encuentra que m1 se mueve en la forma mostrada, mientras que m2 está en reposo. Para esta situación descrita:

a) Determine la velocidad final de m3.

López Frontado, R. (2011). Física general I, Texto UNA. 2da edición. Caracas: UNA

Page 180: Fisica General i Una

191

b) Determine la velocidad del CM del sistema en los dos instantes

mencionados.

c) Si en cierto momento, las posiciones de las masas son: m1(-0,8 m,-1.1 m), m2(0,8 m, -1,1 m), y m3(1,4 m, 0,8 m), dibuje una línea que muestre la trayectoria del CM del sistema.

DATOS: m1 = 2,0 kg, m2 = 0,5 kg, m3 = 1,0 kg, v1 = 1 m/s, v2 = 2 m/s, v3 = 4 m/s, v1f = 3 m/s

SOLUCIÓN: IDENTIFICAR los conceptos pertinentes: de acuerdo al enunciado del

problema, se observa que no existen fuerzas externas que actúen sobre el sistema, por lo tanto se conserva el momento lineal del sistema.

PLANTEAR el problema: Las posiciones antes y después de la partículas se representan en la figura (5.3).

Figura 5.3

EJECUTAR la solución del problema: a) Considerando que no existen fuerzas externas que actúen sobre el sistema, en la dirección del movimiento, se tiene que

ext

o f

o oo 1 1 2 2 3 3 3 3

f 1 1f 3 3 f

F = 0, P = constante, P = Pˆ ˆ ˆ ˆP = m v i+m v j- m v cos30 i- m v sen30 j

ˆP = -m v i+m v

→∑r r r r

r

r r

Al igualar el momento lineal antes y el momento lineal después, se tiene

o o1 1 2 2 3 3 3 3 1 1f 3 3fˆ ˆ ˆ ˆ ˆm v i+m v j-m v cos30 i-m v sen30 j = -m v i+m v

!

López Frontado, R. (2011). Física general I, Texto UNA. 2da edición. Caracas: UNA

Page 181: Fisica General i Una

192

De esta ecuación se despeja la velocidad de la partícula m3, en el momento final, así se tiene:

o o3 3 f 1 1 3 3 1 1f 2 2 3 3

3 f

3 f

ˆ ˆm v = (m v - m v cos30 + m v ) i+ (m v - m v sen30 ) j(2×1-1×4×0.87 + 2×3) (0,5×2 -1×4×0,5)ˆ ˆv = i+ j

1 1mˆ ˆ v = 4,54 i-1j s

! "# $ %

!

!

!

b) Para determinar la CMv

! del sistema de partículas, aplicando la ecuación (5.6) se tiene:

CM CM

o o1 1 3 3 2 2 3 3

CM

CM

P = MV , pero P = cte V = cte.ˆ ˆ(m v - m v cos30 ) i (m v - m v sen30 ) j

V = +M M

ˆ ˆ(2×1-1×4×0,87) i (0,5×2 -1×4×0,5) jV = +3,5 3,5

!! ! ! !

!

!

CMmˆ ˆ V = -0,417 i- 0,286 j s

! "# $ %!

La dirección de la CMV

! está dada por:

-1 o-0,286= tan = 34,42-0,417

φ φ⎛ ⎞ ⇒⎜ ⎟⎝ ⎠

c) Para dibujar la línea que representa la trayectoria descrita por el centro

de masa del sistema, es necesario determinar previamente la posición del Centro de Masa en un instante dado. Supongamos que para un instante inicial (t = 0), la posición del centro de masa sea:

o

1 1 2 2 3 3CM

ˆ ˆ ˆ ˆ ˆm r +m r +m r 2(-0,8 i-1,1j)+0,5(0,8 i-1,1j)+1(1,4 i+0,8 j)r = =M 3,5

! ! !!

o

o

CM

CM

ˆ ˆ ˆ ˆ ˆ ˆ-1,6 i- 2,2 j+ 0,4 i- 0,55 j+1,4 i+ 0,8 jr = 3,5

ˆ ˆ r = (0,057 i- 0,56 j) m!

r

r

La ecuación de la posición de la partícula en función del tiempo está dada por:

o oCM CM CM CM

ˆ ˆ ˆ ˆr =r +v t r =(0,0578i-0,56j)+(-0,417i-0,286j)t⇒r r rr

López Frontado, R. (2011). Física general I, Texto UNA. 2da edición. Caracas: UNA

Page 182: Fisica General i Una

193

Así se obtiene la trayectoria del CM del sistema, como la mostrada en la figura (5.4):

Figura 5.4

EVALUAR la respuesta: Los resultados obtenidos en los incisos (a), (b) y (c)

son lo correspondientes, lo cual se demuestra en la trayectoria del CM aproximada, representada en la figura (5.4)

Ejemplo 5.3.- Un cuerpo de masa m = 12 kg, originalmente en reposo sobre una superficie

horizontal, explota emitiendo una partícula de masa m1 = 4 kg con una velocidad v1 = 12 m/s, y en ángulo recto a la dirección de la primera partícula emite otra de masa m2 = 2 kg con velocidad v2 = 15 m/s.

a) ¿Cuál es la velocidad de la masa residual?

b) ¿En qué dirección se mueve la masa residual?

DATOS: m = 12 kg, m1 = 4 kg, v1 = 12 m/s, m2 = 2 kg,

v2 = 15 m/s SOLUCIÓN: IDENTIFICAR los conceptos pertinentes: Una explosión es producida por la acción de fuerzas internas, por lo tanto se

puede considerar que extF =0!r

, y por lo tanto el momento lineal del sistema se mantiene constante, lo cual nos indica que se aplica la conservación del momento lineal del sistema.

López Frontado, R. (2011). Física general I, Texto UNA. 2da edición. Caracas: UNA

Page 183: Fisica General i Una

194

PLANTEO del problema:

Figura 5.5

La figura (5.5) representa la situación antes (cuando el cuerpo está ubicado en el origen de coordenadas escogidos); y la situación después (cuando las partículas m1 y m2) salen con velocidades 1f 2fv y v

! ! . Las fuerzas que intervienen en dicha

explosión son fuerzas internas, las fuerzas externas, como los pesos y las normales, son perpendiculares al movimiento de las partículas, y por lo tanto no producen efectos. Por esa razón, se tiene que

extxy xy

xyi xyf

F = 0, P = constante

Asi se tiene que P = P (1)

!"! !

! !

Como el cuerpo está inicialmente en reposo, se tiene:

xyi xy f 1 1 2 2 r rˆ ˆP = 0, y P = m v i+m v j+m v

! ! ! Sustituyendo en ecuación (12), se tiene 1 1 2 2 r r

ˆ ˆ0 =m v i+m v j+m v!

EJECUTAR la solución del problema:

a) Para determinar la velocidad de la masa residual, se despeja la velocidad, así se tiene:

1 1 2 2r r

r

ˆ ˆ ˆ ˆm v i+ m v j 4×12 i+ 2×15 j mˆ ˆv = - = - v = (-8,0 i- 5,0 j) m 6 s

!! !

b) La dirección en la que se mueve la masa residual es igual a la dirección

de la velocidad rvr . Así se tiene:

-1 o-5,0=tan =-32

-8,0φ φ⎛ ⎞ ⇒⎜ ⎟⎝ ⎠

Como la partícula se mueve en el tercer cuadrante, de acuerdo a los signos

de las componentes de la velocidad, la dirección es:

Dirección = 32º + 180º = 212º

EVALUAR la solución: Los resultados son lógicos debido a que el momento de la masa residual debe ser igual pero de sentido contrario al producidos por las

López Frontado, R. (2011). Física general I, Texto UNA. 2da edición. Caracas: UNA

Page 184: Fisica General i Una

195

masas m1 y m2, para que pueda conservarse el momento lineal del sistema antes y después de la explosión.

Ejemplo 5.4.-

Figura 5.6

Un niño de 40 kg está parado en un extremo de un listón de madera, cuya masa es de 60 kg, distribuida uniformemente, que tiene una longitud de 4 m, como se muestra en la figura 5.4. El listón con el niño descansan sobre una superficie horizontal lisa. El niño ve un juguete que está pegado al otro extremo del listón y se dirige hacia él con el fin de recogerlo.

a) Describa el movimiento subsiguiente del sistema niño-listón.

b) ¿Podrá recoger el niño el juguete sin bajarse del listón? (El niño sólo

puede alcanzar una distancia de 1 m fuera del listón).

DATOS: mn = 40 kg, mL = 60 kg, L = 4 m. SOLUCIÓN: IDENTIFICAR los conceptos pertinentes: Como no existe roce entre el listón

y la superficie sobre la que se mueve, se tiene que las únicas fuerzas que actúan sobre el sistema niño-listón son internas, por lo tanto es aplicable el principio de conservación de momento lineal.

PLANTEAR el problema: a) Para describir el movimiento subsiguiente del sistema niño-listón, es

necesario tomar como posición antes la presentada en la figura (5.6a), la cual corresponde cuando el niño está en su posición inicial. La posición después la presentada en la figura 5.4b,la cual corresponde cuando el niño se ha desplazado al otro extremo del listón con el fin de recoger el juguete. Así se tiene

López Frontado, R. (2011). Física general I, Texto UNA. 2da edición. Caracas: UNA

Page 185: Fisica General i Una

196

Cuando el niño se dirige hacia el otro extremo del listón, como se muestra en la figura (5.6b), con el fin de recoger el juguete, el listón se desliza hacia atrás, tal que la posición del CM del sistema niño-listón se mantiene constante. Por eso, la posición del niño respecto al CM cuando llega al otro extremo del listón (x’n) debe ser la misma que tenía cuando estaba en el primer extremo (xn); y la posición del centro de masa del listón con respecto al CM del sistema (x’L), debe ser igual a la que tenía antes (xL). Por lo tanto, el listón se desplazará hacia atrás una distancia d = xL + x’L.

EJECUTAR la solución:

b) Para saber si el niño logra recoger el juguete sin tener que bajarse del listón, se debe comprobar si la distancia d es menor que 1,0 m, que es la distancia máxima que puede estirarse el niño para tomar el juguete. Además se tiene que las fuerzas que actúan durante el movimiento de los cuerpos son fuerzas internas; las fuerzas externas, como los pesos y las normales son perpendiculares al movimiento, y por lo tanto no producen efecto sobre los mismos. Así se tiene que:

ext

x

xi CMi CM

F = 0, por lo tanto P = constanteP = 0, V = 0, x = constante.→ ∴∑

!

De donde se tiene que:

n ni L Li n nf L Lf

CMi CMfn L n L

n nf ni L Lf Li

m x +m x m x +m xx = = =x

m +m m +m m (x -x )=-m (x -x )→

López Frontado, R. (2011). Física general I, Texto UNA. 2da edición. Caracas: UNA

Page 186: Fisica General i Una

197

Pero n nf ni

L Lf Li

n/L

x = x - x , es el desplazamiento del niño.x = x - x , es el desplazamiento del listón.x es el desplazamiento del niño respecto al listón.

ΔΔΔ

Se tiene además, que n n/L LΔx =Δx +Δx , sustituyendo en la ecuación (1), se obtiene:

n n/L L L Lm (Δx +Δx )=-m Δx Al despejar ΔxL, se tiene:

n n/LL L

n L

m Δx 40×4 40×4Δx =- =- =- Δx =-1,6 mm +m 40+60 100

Así vemos que el listón se aleja 1,6 m del juguete, y por lo tanto, el niño no puede recoger el juguete sin bajarse del listón.

EVALUAR el resultado: Sí es lógico, ya que al moverse el niño desde un

extremo al otro del listón, el listón se mueve hacia la izquierda, para mantener constante la posición del centro de masa del sistema niño-listón.

Ejercicio propuesto 5.1.- Un niño, que está sobre una patineta en reposo, lanza horizontalmente hacia

delante una pelota de 0,5 kg de masa. La masa combinada del sistema niño + patineta = 36 kg. Calcule la velocidad inicial de retroceso del niño + patineta, si la velocidad de la pelota es tal que hubiera podido alcanzar una altura de 15 m si se hubiese lanzado verticalmente.

Ejercicio propuesto 5.2.- Ricardo, quien tiene una masa de 78,4 kg, y Judith, quien pesa menos, se

divierten al anochecer en un lago dentro de una canoa de 31,6 kg. Cuando la canoa está en reposo en aguas tranquilas, intercambian asientos, los cuales se hallan separados a una distancia de 2,93 m y simétricamente situados con respecto al centro de la canoa. Ricardo observa que la canoa se movió 41,2 cm con relación a un tronco sumergido y calcula la masa de Judith. Determine el valor de la masa de Judith calculada por Ricardo.

López Frontado, R. (2011). Física general I, Texto UNA. 2da edición. Caracas: UNA

Page 187: Fisica General i Una

198

Ejercicio propuesto 5.3.- Se tienen dos masas, inicialmente en reposo sobre una superficie horizontal

lisa, comprimiendo un resorte que se encuentre entre ellas y que no está sujeto a ninguna de las masas. Cuando las masas se liberan, el resorte las acelera, proporcionando a la masa m1 una velocidad de 5 m/s hacia la izquierda y a la masa m2 una velocidad de 15 m/s hacia la derecha.

a) ¿Cuál es el momento lineal del sistema después que se dejan libres las

masas?

b) ¿Cuál es el cociente entre m1 y m2?

c) Si m1 = 12 kg, ¿cuál es la velocidad del centro de masa del sistema?

d) ¿Cuánta energía cede el resorte a las masas?

e) Si k = 100 N/cm, ¿cuál era la compresión que tenía el resorte?

Ejemplo 5.5.-

Figura 5.7

Las masas m1 = 2,0 kg y m2 = 4,0 kg, unidas por un resorte de masa despreciable y constante elástica k = 100 N/m, se colocan sobre una superficie lisa de una pendiente, que forma un ángulo θ = 37º con la horizontal, como se muestra en la figura (5.5). Se sueltan desde el reposo, desde las posiciones mostradas en la figura, donde x1o = 2,0 m y x2o = 3,0 m. Mientras deslizan los bloques sobre la superficie de la pendiente, determine la ecuación de movimiento del centro de masa del sistema.

DATOS: m1 = 2,0 kg, m2 = 4,0 kg, k = 100 N/m, θ = 37º, x1o =2,0 m,

x2o = 3,0 m.

López Frontado, R. (2011). Física general I, Texto UNA. 2da edición. Caracas: UNA

Page 188: Fisica General i Una

199

SOLUCIÓN: IDENTIFICAR los conceptos pertinentes: En este problema se trabaja con el sistema de partículas como si estuviese

concentrado en el centro de masa (CM), para ello es necesario aplicar la 1ª ley fundamental de la mecánica (esto es la aplicación de la 2ª ley de Newton para describir el movimiento del sistema de partícula).

PLANTEAR Y EJECUTAR el problema: En primer lugar se aplica la ecuación de movimiento del Centro de Masa del

sistema está dado por: 2

CM CMo CMo CM1r = r +V t+ a t2

!! ! !

Para aplicar la 1ª ley fundamental de la mecánica, se hace previamente un

DCL de cada una de las masas, así se tiene que:

Al aplicar la 1ª ley fundamental de la mecánica, se tiene:

extCM

1 1 2 2 1 2 CM

dPF = =Madt

m g+N +m g+N = (m +m )a

!r

r r

r rr r r

No se toma en cuenta la fuerza elástica eF

!producida por el resorte, debido a

que ella es una fuerza interna del sistema. Así se tiene

extx 1 2 1 2 CMxexty 1 1 2 2

F : m gsen + m gsen = (m +m )a

F : N - m gcos +N - m gcos = 0

! !

! !""

oCMx CMx CMy2

mDonde a = gsen = 9,8sen37 a = 5,9 , a = 0s

θ →

También se tiene que el sistema parte desde el reposo, esto es, CMx CMyv =v =0. La posición inicial del Centro de Masa del sistema está dada por :

1 1o 2 2oCMo

1 2

CMo CMo

m x +m x 2×2+ 4×3x = =m +m 2+ 4

x = 2,67 m y y = 0

La ecuación de movimiento del Centro de Masa del Sistema de partículas

queda expresada por: 2

CMˆ r = (2,67+2,95t ) i m!

!

López Frontado, R. (2011). Física general I, Texto UNA. 2da edición. Caracas: UNA

Page 189: Fisica General i Una

200

EVALUAR el resultado del problema: este resultado nos indica el movimiento del sistema de partícula como si estuviese concentrado en el centro de masa (CM) del mismo.

5.3.6 DEFINICIÓN DE MOMENTO ANGULAR El Momento Angular L

r con respecto al origen O, de una partícula de masa m

que se mueve con una velocidad vr

, se define como el producto cruz (ó vectorial) del vector posición instantáneo r

! de la partícula y su momento linealp=mv

! !. Esto es,

L=r×p,

o L=mr×v (5.11)

! ! !

! ! !

Las unidades SI del momento angular son kg.m2.s-1.

Figura 5.8

El momento angular es un vector

perpendicular al plano formado por el vector posición y el momento lineal, cuya dirección está dado por la regla de la mano derecha.

La magnitud del momento angular está dado por:

L=mrvsenθ (5.12) Donde θ es el ángulo entre r y p

r r.

Debe tomarse en cuenta que si el vector posición r

r y el momento lineal p

r son

paralelos, el momento angular es cero; y si el vector posición rr

y el momento lineal p!

son perpendiculares, la magnitud del momento angular es L = mrv. En este último caso, la partícula describe una trayectoria circular de radio r, por lo que el momento angular se puede expresar por:

2L=mr ω, (5.13)

donde v=rω, y ω es la velocidad angular de rotación.

López Frontado, R. (2011). Física general I, Texto UNA. 2da edición. Caracas: UNA

Page 190: Fisica General i Una

201

Basado en los conceptos emitidos hasta los momentos, responda la siguiente

pregunta: Pregunta 5.3 Cuando una rueda de radio R gira alrededor de un eje fijo que pase por su

centro, ¿todos los puntos sobre la rueda tienen: a) la misma rapidez angular, y b) la misma rapidez lineal?

5.3.7 MOMENTO ANGULAR PARA UN SISTEMA DE PARTÍCULAS El Momento Angular Total de un sistema de partículas alrededor de un punto,

se define como la suma vectorial de los momentos angulares de cada una de las partículas, esto es:

n

1 2 3 n ii=1

n

i i ii=1

L=L +L +L +.....+L = L

L= m r×v (5.14)

!

!

! ! ! ! ! !

! ! !

Si el sistema de partículas está girando alrededor de un eje, con una

velocidad angular ω, el momento angular se puede expresar usando la siguiente ecuación,

n

2i i

i=1L= mr ω L=Iω (5.15)! " #$ %

& '(

La cantidad n

2i i

i=1I= mr! es una cantidad escalar, denominada momento de

inercia. Vectorialmente, el momento angular se puede expresar por

L=Iω (5.16)! !

Como se puede observar, el momento angular es un vector cuya dirección es

paralela a la velocidad angular.

López Frontado, R. (2011). Física general I, Texto UNA. 2da edición. Caracas: UNA

Page 191: Fisica General i Una

202

El momento angular para un sistema de partículas también se puede expresar de la siguiente forma:

CM CM CML=L +Mr ×v (5.17)r r r r

Donde CML

r es el momento angular del sistema en relación con su centro de

masa, M es la masa total del sistema, y CM CMr y v! !

son la posición y velocidad del Centro de Masa en relación a un origen inercial.

El momento angular del sistema de partículas con relación al centro de masa,

lo puede obtener usando la siguiente ecuación:

n

i i, CM i,CMi=1

CMn

i i,CM ii=1

m r ×v

L = o (5.18)

m r ×v

⎧⎪⎪⎨⎪⎪⎩

! !

!

! !

Los vectores CM CMr y v

! ! representan las posiciones y las velocidades de cada

partícula con respecto al Centro de Masa. A continuación se presentarán unos ejemplos de problemas, donde Ud. puede

visualizar las aplicaciones de los conceptos tratados en esta unidad. Recuerde que el manejo eficiente del contenido de esta unidad es

importante, por su repercusión en el contenido de las subsiguientes unidades del curso de Física General I.

Basado en los conceptos emitidos hasta los momentos, responda la siguiente pregunta:

Pregunta 5.4 Si una patinadora se mueve hacia un poste, ¿cuál sería el momento angular

de ella respecto al poste?

López Frontado, R. (2011). Física general I, Texto UNA. 2da edición. Caracas: UNA

Page 192: Fisica General i Una

203

Ejemplo 5.6.- Dos partículas de masas m1 = 4,0 kg, y m2 = 5,0 kg, que se encuentran en los

puntos 1 2r = (0;1;1) y r = (-1;0;2)! ! , se mueven, con relación a un observador O, con

velocidades -11

ˆv =10 i m.s! y 2

ˆ ˆv = (12cos120º i+12sen120º j)r m.s-1. Con base en las

condiciones expuestas, determine: a) La posición del centro de masa.

b) El momento angular del sistema con relación al observador.

c) El momento angular del sistema con relación con su Centro de Masa.

DATOS: m1 = 4,0 kg, m2 = 5,0 kg. SOLUCIÓN: IDENTIFICA los conceptos pertinentes: En este problema se aplican los conceptos de momento angular del sistema

de partículas, refiriéndolo a un sistema inercial y a un sistema ubicado en el centro de masa (CM) del mismo sistema.

PLANTEAR Y EJECUTAR la solución del problema: Se aplican las expresiones que permiten determinar el momento angular del

sistema de partículas con respecto al centro de masan

CM i i,CM ii=1

L = m r ×v!! ! ! , y a un sistema

inercialn

o i i, ii=1

L = m r ×v!r r r .

Antes de resolver el ejercicio, expresemos las posiciones y las velocidades de

las partículas en forma vectorial, esto es,

1 1 1

2 2 2

mˆ ˆ ˆm = 4,0 kg, r = (j+ k) m, v 10,0 i s

mˆ ˆ ˆ ˆm = 5,0 kg, r = (- i+ 2k) m, v = (-6 i+10,4 j) s

r r

r r

López Frontado, R. (2011). Física general I, Texto UNA. 2da edición. Caracas: UNA

Page 193: Fisica General i Una

204

a) La posición del centro de masa CMrr

es,

1 1 2 2CM

1 2

CM

ˆ ˆ ˆ ˆ ˆ ˆ ˆ ˆm r + m r 4(j+ k) + 5(- i+ 2 j) 4 j+ 4k- 5 i+10 jr = = =m + m 4 + 5 9

ˆ ˆ ˆ r = (-0,56 i+ 0,44 j+1,56k) m⇒

r rr

r

b) El momento angular del sistema en relación con el observador O está

dado por:

O 1 1 1 2 2 2

O2

O

ˆ ˆ ˆ ˆ ˆ ˆ ˆL = m r× v +m r × v = 4(j+k)× (10 i) + 5(- i+ 2k)× (-6 i+10,4 j)ˆ ˆ ˆ ˆ ˆL = -40k+ 40 j- 60 j- 52k-104 i

kg.mˆ ˆ ˆ L = (-104 i- 20 j- 52k) s

r r rr r

r

r

c) El momento angular del sistema en relación con su Centro de Masa, aplicando la ecuación (7.8) se tiene

CM 1 1,CM 1 2 2,CM 2

1,CM 1 CM

1,CM

2,CM 2 CM

2,CM

L = m r × v + m r × vˆ ˆ ˆ ˆ ˆr = r - r = j+ k+ 0,56 i- 0,44 j-1,56k

ˆ ˆ ˆ r = (0,56 i- 0,56 j- 0,56k) mˆ ˆ ˆ ˆ ˆr = r - r = - i+ 2k+ 0,56 i- 0,44 j-1,56k

ˆ ˆ ˆ r = (-0,44 i- 0,44 j+ 0,44k)

! ! !! !

! ! !

!

! ! !

! m

Por lo tanto, el momento angular con respecto al centro de masa, está dado por:

CM

CM2

CM

ˆ ˆ ˆ ˆL = 4(0,56 i- 0,56 j- 0,56k)× (10 i)ˆ ˆ ˆ ˆ ˆ + 5(-0,44 i- 0,44 j+ 0,44k)× (-6 i+10,4 j)

ˆ ˆ ˆ ˆ ˆ ˆL = 22,4k- 22,4 j- -13,2k-13,2 j- 22,9k- 22,9 i

kg.mˆ ˆ ˆ L = (-22,9 i- 35,6 j-13,2k) s

r

r

r

EVALUAR el resultado del problema: Estos resultados nos indica que el momento angular de un sistema de

partículas depende del sistema de referencia con respecto al cual se ha calculado.

López Frontado, R. (2011). Física general I, Texto UNA. 2da edición. Caracas: UNA

Page 194: Fisica General i Una

205

Ejercicio propuesto 5.4

Figura 5.9

En la figura se muestra dos partículas de masas, m1 = 4 kg y m2 = 6 kg, que se mueven con velocidades, 1

ˆv =2i m/sr

y 2ˆv =3j m/s

r . Determine: a) El momento angular total del sistema relativo a O. b) El momento angular total del sistema relativo al centro de masa (CM) del sistema.

Ejercicio propuesto 5.5

Figura 5.10

En la figura se muestra una piedra de 0,3 kg que tiene una velocidad horizontal de 12,0 m/s, cuando está en el punto P. ¿Qué momento angular tiene la piedra con respecto al punto O en ese instante?

5.3.8 MOMENTO DE INERCIA El momento de Inercia (o llamado también Inercia Rotacional) es la

resistencia de un objeto a los cambios en su estado de movimiento rotacional. Los objetos en rotación tienden a permanecer en rotación, mientras que los cuerpos que no giran tienden a permanecer sin girar. Del mismo modo que se requiere una fuerza para cambiar el estado de movimiento de un objeto, un momento de torsión es necesario para cambiar su estado de movimiento rotacional.

Así como la inercia en el sentido lineal, la inercia rotacional depende de la

distribución de la masa. Cuanto mayor sea la distancia entre el grueso de la masa de un objeto y el eje alrededor del cual se efectúa la rotación, mayor será la inercia rotacional. Cuando una partícula gira alrededor de un eje, su momento de inercia (o inercia rotacional) se expresa por:

2I=mr (5.19)

López Frontado, R. (2011). Física general I, Texto UNA. 2da edición. Caracas: UNA

Page 195: Fisica General i Una

206

Donde m es la masa de la partícula y r es su distancia al eje de rotación. El momento de inercia I es una cantidad escalar, y sus unidades en el sistema internacional (SI) son Kg.m2.

Cuando las partículas de un sistema giran alrededor de un eje, el momento de

inercia del sistema está dado por

2 2 2 21 1 2 2 3 3 n n

n2

i ii=1

I=m r +m r +m r +....+m r

I= mr (5.20)∑

Cuando se tiene una distribución continua de masa, el momento de inercia

respecto a un eje, está dado por 2I= r dm (5.21)∫ Donde r es la distancia del elemento diferencial de masa al eje de rotación. En el cálculo del momento de inercia, también es importante aplicar el

Teorema de Steiner de los ejes paralelos, el cual nos dice que: “El momento de inercia de un sistema de partículas, con respecto a un

eje arbitrario, es igual al momento de inercia del sistema alrededor de un eje paralelo que pase por el centro de masa, más la masa total del sistema por la distancia entre los dos eje elevada al cuadrado”

Figura 5.11

La figura (5.11) es una representación de un sistema de partículas, cuyo centro de masa es CM, al cual le aplicamos el Teorema de ejes paralelos para determinar el momento de inercia respecto a un sistema de coordenadas cuyo origen es O, y sus ejes son paralelos al sistema de coordenadas con origen en el Centro de Masa del sistema de partículas.

Matemáticamente, el Teorema de Steiner de los ejes paralelos, se puede

expresar por la fórmula siguiente:

2CMI=I +Md (5.22)

Donde ICM es el momento de inercia del sistema con respecto al centro de

masa (CM); M es la masa total del sistema (M = Σi mi); y d es la distancia perpendicular entre los dos ejes paralelos.

López Frontado, R. (2011). Física general I, Texto UNA. 2da edición. Caracas: UNA

Page 196: Fisica General i Una

207

Ejemplo 5.7.- Una varilla delgada de 1 m de longitud tiene una masa despreciable. Se

colocan 5 cuerpos a lo largo de ella, cada uno con una masa de 1 kg, y situados a 0 cm, 25 cm, 50 cm, 75 cm, y 100 cm de uno de sus extremos. Calcule el momento de inercia del sistema con respecto a un eje perpendicular a la varilla, el cual pasa a través de: a) un extremo; b) la segunda masa; c) verifique además que se cumple el teorema de Steiner de los ejes paralelos.

DATOS: m = 1,0 kg. SOLUCIÓN: IDENTIFICAR los conceptos pertinentes: se aplica el concepto de momento

de inercia de la varilla con respecto a diferentes puntos, incluyendo también el teorema de Steiner.

PLANTEAR Y EJECUTAR la solución del problema: Se aplica las expresiones que nos permiten determinar el momento de inercia

de un sistema de partículas respecto a un eje que pase por un punto cualquiera, esto es,

n2

i ii=1

I= mr! ; y el momento de inercia respecto a un eje que pase por un punto

cualquiera y sea paralelo al eje que pase por el centro de masa, aplicando el teorema de Steiner 2

1 CMI = I +Md . Para cumplir con esto, usemos el esquema del sistema presentado en la

figura (5.12)

Figura 5.12

a) El momento de inercia respecto al eje que pase por el extremo 1 está dado por:

2 2 2 2

1 1 2 3 4 5

12

1

I =m (0)+m (0,25) +m (0,5) +m (0,75) +m (1,0)I =0+1×0,0625+1×0,25+1×0,5625+1×1,0

I =1,875 kg.m!

López Frontado, R. (2011). Física general I, Texto UNA. 2da edición. Caracas: UNA

Page 197: Fisica General i Una

208

c) El momento de inercia respecto al eje que pase por el punto 2 es,

2 2 2 22 1 2 3 4 5

22

2

I = m (0,25) +m (0) +m (0,25) +m (0,5) +m (0.75)I = 1×0,0625 + 0 +1×0,0625 +1×0,25 +1×0,5625

I = 0,9375 kg.m!

d) Para comprobar el teorema de Steiner, como ejemplo, apliquémoslo a

un eje que pase por el punto 1, así se tiene

21 CMI = I +Md

El centro de masa está en el punto 3.

2 2 2 2CM 1 2 3 4 5

2CM CM

I = m (0,5) +m (0,25) +m (0)+m (0,25) +m (0,5)

I = 2×0,25 + 2×0,0625 I = 0,625 kg.m→

2 2 2 2

21 1

Md =5×(0,5) Md =1,25 kg.m I =0,625+1,25 I =1,875 kg.m

!

" #

Este resultado comprueba la validez del teorema de Steiner.

EVALUAR los resultados del problema: Estos resultados presentan una

correspondencia lógica con lo expresado en el enunciado del problema. Además se puede observar que el momento de inercia se puede obtener de varias formas diferentes, como se demuestra en los resultados obtenidos en el inciso (a) y en el inciso (c), los cuales coinciden.

Considerando los conceptos emitidos hasta los momentos, responda la

siguiente pregunta: Pregunta 5.5 Para un aro que yace sobre el plano xy, ¿cuál de las siguientes situaciones

requiere que se realice mayor trabajo por un agente externo para acelerar el aro desde el reposo hasta alcanzar una rapidez angular ω, si: a) rotación alrededor del eje z que pasa a través del centro del aro, o b) rotación alrededor de un eje paralelo al eje z, que pasa por el punto P situado sobre el borde del aro?

5.3.9 MOMENTO DE TORSIÓN (O MOMENTO DE FUERZA) Supongamos que a una partícula de masa m, situada en un punto A, se le

aplica una fuerza F!

, como se muestra en la figura (5.13).

López Frontado, R. (2011). Física general I, Texto UNA. 2da edición. Caracas: UNA

Page 198: Fisica General i Una

209

Figura 5.13

El momento de torsión producido por la fuerza F!

, está dado por =Fb.τ

Donde b es el brazo de fuerza. De la figura se tiene b=rsenθ. Entonces =rF senτ θ

En forma vectorial, el momento de torsión es expresado por

=r×F (5.23)!rrr

El momento de torsión es un vector perpendicular al plano formado por los

vectores de posición y de fuerza. En la ecuación (5.23) se observa que si la fuerza aplicada es perpendicular al vector posición, el momento de torsión es =rF! . En este caso, el tipo de movimiento proporcionado por la fuerza es circular.

Si la fuerza aplicada es paralela a la posición, el momento de torsión es nulo.

Vemos así que, si la fuerza aplicada pasa por el eje de rotación, ella no produce momento de torsión; este tipo de fuerza cuya línea de acción pasa por el eje de rotación se denomina fuerza central.

Cuando se tiene un sistema de partículas, el momento de torsión está dado

por:

Total1 1 2 2 3 3 n nn

Totali i

i=1

=r×F +r ×F +r ×F +....+r ×F

= r×F (5.24)

τ

τ ∑

r r r rr r r rr

rrr

5.3.10 SEGUNDA LEY FUNDAMENTAL DE LA MECÁNICA Como se ha visto, el momento angular de una partícula está dado por L=r×p

! ! !

Al derivar el momento angular con respecto al tiempo, se tiene dL dr dp dr= ×p+r× , pero p=mv y =v, v es paralelo a p, dt dt dt dt

dr dpy por lo tanto ×p=0. También se tiene que =F dt dt

! ! ! !! ! ! !! ! !

! ! !!

López Frontado, R. (2011). Física general I, Texto UNA. 2da edición. Caracas: UNA

Page 199: Fisica General i Una

210

Así se obtiene que

dL dL=r×F= = (5.25)dt dt

! !" #r r

rr r r

Lo cual nos indica que el momento de torsión neto que actúa sobre una

partícula es igual a la razón de cambio respecto al tiempo del momento angular.

Cuando se tiene un sistema de partículas, la ecuación (5.25) puede escribirse

τ∑r

rext dL= (5.26)dt

Esta expresión constituye la 2a ley

Fundamental de la Mecánica, y la cual nos dice que “el momento de torsión externo que actúa sobre un sistema de partículas es igual a la razón de cambio respecto al tiempo del momento angular total del sistema”.

LA 2ª LEY FUNDAMENTAL DE LA MECÁNICA, está dada por:

ext dL=dt

!"!

!

El momento de torsión y el momento angular deben evaluarse respecto

al mismo origen de un sistema de referencia inercial.

Tomando en cuenta los conceptos emitidos hasta el momento, responda la

siguiente pregunta: Pregunta 5.6 Si una partícula se mueve en una línea recta, y a Ud. se le dice que el

momento de torsión neto que actúa sobre ella es cero alrededor de algún punto no especificado. Indique si alguna de las siguientes opciones son falsas o verdaderas:

a) La fuerza neta sobre la partícula debe ser cero; b) La velocidad de la

partícula debe ser constante.

López Frontado, R. (2011). Física general I, Texto UNA. 2da edición. Caracas: UNA

Page 200: Fisica General i Una

211

Ejemplo 5.8

El momento angular de un volante que tiene una inercia de rotación de 0,142 kg.m2 disminuye de 3,07 a 0,788 kg.m2/s en 1,53 s.

a) Halle el momento de torsión promedio que actúa sobre el volante

durante este período. b) Suponiendo una aceleración angular uniforme, ¿cuál es el ángulo

girado por el volante en ese lapso de tiempo? c) ¿Cuánto trabajo se efectuó sobre el volante? d) ¿Cuánta potencia promedio fue suministrada por el volante?

DATOS: I = 0,142 kg.m2, Lo = 3,07 kg.m2/s, Lf = 0,788 kg.m2/s, Δt = 1,53 s SOLUCIÓN: IDENTIFICAR los conceptos pertinentes: En este problema se aplica la 2ª ley fundamental de la mecánica, esto es

ext dL=dt

!"!

! , para determinar luego el trabajo realizado sobre el volante.

PLANTEAR Y EJECUTAR la solución del problema: Partiendo de la 2ª ley fundamental se procede a resolver el problema, así se tiene que, a) El momento de torsión está dado por:

2

m m 2

L 0,788-3,07 kg.m= = =-1,492 t 1,53 s

! !"

#"

b) Para determinar el ángulo girado en ese lapso de tiempo, se tiene que

τm = Iα, de donde

m2

-1,492 rad= = =- 10,5 I 0,142 s

!" "#

López Frontado, R. (2011). Física general I, Texto UNA. 2da edición. Caracas: UNA

Page 201: Fisica General i Una

212

También se tiene que Lo = I ωo, de donde

oo o

L 3,07 rad= = =21,62 I 0,142 s

! !"

El ángulo 2

ot= t+2

!" #$

210,5(1,53)=21,62 1,53- =20,79 rad=1191,2°2

θ θΔ × ⇒ Δ

c) Para determinar el trabajo realizado en ese lapso de tiempo, se tiene

que

22of f

f fII L 0,788 radW= K= - , donde = = =5,55

2 2 I 0,142 sωω ω ωΔ →

2 20,142W= (5,55) -(21,62) W=-31,0 J2

! " #$ %

d) La potencia promedio

m mW -31,0P = = P =-2026 W

t 1,53!

"

EVALUAR el resultado del problema: El resultado obtenido corresponde lógicamente a los valores de las incógnitas

pedidas en el enunciado del problema.

5.3.11 PRINCIPIO DE CONSERVACIÓN DEL MOMENTO ANGULAR En la ecuación (5.25) encontramos que si el momento de torsión producido

por las fuerzas externas es cero, o si el sistema de partículas está aislado, se tiene que

ext dL=0, =0, por lo tanto L=Constantedt

!"!

!!

Los cual nos indica que si “el momento de torsión neto sobre un sistema

de partículas es cero, el momento angular se mantiene constante”. Esto constituye el Principio de Conservación del Momento Angular.

El momento de torsión neto es cero, cuando su suma es cero, o cuando el

sistema está aislado.

López Frontado, R. (2011). Física general I, Texto UNA. 2da edición. Caracas: UNA

Page 202: Fisica General i Una

213

Ejemplo 5.9.- Dos niños con patines de hielo, cada uno con una masa de 25 kg, están

unidos por medio de una cuerda resistente y muy liviana de 2,6 m de largo y de masa despreciable, están rotando a 5 rpm sobre una pista de hielo lisa, con respecto a un eje que pasa por su centro. Si los niños acortan cada uno 60 cm de la longitud de la cuerda, determine:

a) La nueva velocidad angular con qué giran ahora los niños.

b) El cambio en la energía cinética de rotación del sistema.

DATOS: m = 25 kg, ω1 = 2π5/60 = 0,5236 rad/s L1 = 2,60 m,

r1= 1,30 m L2 = 1,40 m, r2 = 0,70 m

SOLUCIÓN: IDENTIFICAR los conceptos pertinentes: En este problema se aplica el principio de conservación del momento angular

del sistema, debido a la carencia de momento de torsión ejercido, ya que las fuerzas que acortan la distancia entre los niños son fuerzas paralelas al vector posición.

PLANTEAR Y EJECUTAR la solución del problema: Cuando los niños acortan la longitud de la cuerda, ejercen fuerzas que son

internas, y por lo tanto ext

i fdLτ =0, =0, L=constante. L =Ldt

→ ∴∑r

r r rr

C,iC,i i C,f f f i

C,f

II ω =I ω ω = ω

I→

Figura 5.14

a) Al acortarse la longitud de la cuerda, 60 cm cada uno, los momentos de inercia, antes y después de acortarse, son:

2 2 2

C,i i C,i

2 2 2C,f f C,f

I = 2mr = 2×25×(1,3) I = 84,5 kg.m

I = 2mr = 2×25×(0,7) I = 24,5 kg.m

!

!

López Frontado, R. (2011). Física general I, Texto UNA. 2da edición. Caracas: UNA

Page 203: Fisica General i Una

214

La velocidad angular final, con que giran los niños, son:

f f84,5×p rad = = 1,8059 24,5×6 s

! !" #

b) El cambio de la energía cinética rotacional ΔK está dado por:

( )

2 2C,f f C,i i

2 2

1 1K = I - I2 21K = 24,5× (1,8059) - 84,5× (0,5236)21K = 79,90 - 23,17 K = 28,37 J2

ω ωΔ

⎡ ⎤Δ ⎣ ⎦

Δ ⇒ Δ

EVALUAR el resultado del problema: El resultado obtenido corresponde

lógicamente con lo pedido en el enunciado del problema.

Ejercicio propuesto 5.6 Una masa m está unida a una cuerda que pasa por un pequeño hoyo en una

superficie horizontal sin fricción. La masa inicialmente gira con una velocidad v1 en un círculo de radio r1. La cuerda se jala después lentamente desde abajo, disminuyendo el radio del círculo a r2

a) ¿Cuál es la velocidad de la masa cuando el radio es r?

b) Encuentre la tensión en la cuerda como una función de r.

c) ¿Cuánto trabajo W se efectúa al mover m de r1 a r?

d) Obtenga valores numérico para v, T y W cuando r = 0,1 m, conociendo

que m = 50,0 g, r1 = 0,3 m, y v1 = 1,5 m/s.

López Frontado, R. (2011). Física general I, Texto UNA. 2da edición. Caracas: UNA

Page 204: Fisica General i Una

215

RESUMEN

El momento lineal de una partícula está dado por:

p=mv! !

Impulso f

i

t

tI= Fdt=Δp∫! ! !

La posición del Centro de Masa está dado por:

n n

CM i i ii=1 i=1

1r = m r, donde M= mM ! !

! !

La velocidad del Centro de Masa, está dado por:

n n

CM i i ii=1 i=1

1v = m v , donde M= mM ! !

! !

El momento lineal total de un sistema de partículas está dado por:

n n

CM i i ii=1 i=1

P=Mv , donde P= p = mv! !! ! !! !

La 2ª de Newton aplicada a un sistema de partículas es,

ext CMCM

dvdPF = =M =Madt dt∑! !! !

Casos particulares.-

a.- Si el sistema de partículas está aislado o la suma de la fuerza externa que actúa sobre el sistema es nula, esto es que

extF =0∑r

, se tiene que dP =0 P=constantedt

→!

!.

Ley de conservación del momento lineal.

López Frontado, R. (2011). Física general I, Texto UNA. 2da edición. Caracas: UNA

Page 205: Fisica General i Una

216

b.- Si el sistema, además de estar aislado se encuentra

inicialmente en reposo, se tiene que

CM CM CMP=Mv =0 v =0, r =constante! "! !! ! .

El momento angular de una partícula está dado por

L=r×p, o L=mr×vr rr r r r

El momento angular para un sistema de partículas está dado por

n

1 2 3 n ii=1

n

i i ii=1

L=L +L +L +.....+L = L

L= m r×v

!

!

! ! ! ! ! !

! ! !

Si el sistema de partículas está girando con una velocidad angular !

alrededor de un eje determinado se tiene que

L=I!! !

Donde n

2i i

i=1I= mr! , es el momento de inercia del sistema respecto al eje de

rotación. Tomando en consideración el Centro de Masa del sistema de partículas, se

tiene CM CM CML=L +Mr ×v

r r r r El momento de torsión de una partícula esta dado por: El momento de torsión total de un sistema de partículas está dado por:

Total1 1 2 2 3 3 n nn

Totali i

i=1

=r×F+r ×F +r ×F +....+r ×F

= r×F

!

! "

! ! ! !! ! ! !!

!!!

La 2ª ley fundamental de la mecánica (relacionada con la rotación del

sistema) está dada por ext dL=

dtτ∑

!!

López Frontado, R. (2011). Física general I, Texto UNA. 2da edición. Caracas: UNA

Page 206: Fisica General i Una

217

EJERCICIOS DE AUTOEVALUACIÓN En su libreta de apuntes desarrolle los siguientes ejercicios. Luego de resolver

los ejercicios, comprare sus respuestas con las presentadas al final de la unidad. En caso de no coincidir sus repuestas, consulte con el asesor de su centro local.

5.1.- Un niño, cuya masa es de 26,0 kg, en un bote de masa 55,0 kg, lanza

horizontalmente un paquete de 5,4 kg con una rapidez de ˆ10,0i m/s . Calcule la velocidad del bote inmediatamente después, suponiendo que estaba en reposo.

5.2.- Un proyectil de 200 kg, que es disparado con una rapidez de 100 m/s y

con un ángulo de 60º con la horizontal, explota, rompiéndose en tres partes de igual masa en el punto más alto de su trayectoria. Si dos de los fragmentos se mueven con la misma rapidez, justo después de la explosión que el proyectil completo tenía justo antes de la explosión; pero uno de los fragmentos se mueve verticalmente hacia abajo y el otro fragmento lo hace horizontalmente, determine: a) la velocidad del tercer fragmento justo después de la explosión; b) la energía liberada en la explosión.

5.3.- La pelota de tenis puede dejar la raqueta de un jugador profesional

durante un servicio con una rapidez de 65,0 m/s. Si la masa de la pelota es de 6,0x10-2 kg y está en contacto con la raqueta durante 0,03 s, ¿cuál es la fuerza promedio sobre la pelota?

5.4.- Una partícula está en la posición ˆ ˆ ˆr=(1,0i+2,0j+3,0k) m

! , y viaja con una velocidad ˆ ˆ ˆv=(-5,0i-4,5j-3,1k) m/s

! . Si su masa es de 7,6 kg, ¿Cuál es su momento angular respecto al origen?

5.5.- Una persona está de pie con las manos al lado sobre una plataforma

que está girando a una frecuencia angular de 1,3 rev/s. Si la persona levanta ahora los brazos hasta una posición horizontal, la frecuencia angular disminuye a 0,8 rev/s. a) ¿A qué se debe esto? b) ¿Qué tanto cambió el momento de inercia de la persona?

López Frontado, R. (2011). Física general I, Texto UNA. 2da edición. Caracas: UNA

Page 207: Fisica General i Una

218

RESPUESTA A LAS PREGUNTAS:

Estas respuestas se corresponden con las preguntas presentadas en el

desarrollo de la unidad. Respuesta a la pregunta 5.1 La relación entre las magnitudes de los momentos lineales de los cuerpos son

dependen de cómo estén relacionados las masas de los mismos, y esto se debe a

que la energía cinética está dada por 2

21 pK= mv =2 2m

. Así se tiene que 2

2 2 2 21 1 1 11 1 1 2 2 1 1 22

2 1 22

p 2m K mp =2m K y p =2m K = p = p

2m K mp! "

Respuesta a la pregunta 5.2 Cuando el cuerpo cae su rapidez varía a medida que cae, por lo tanto su

momento lineal también varía, pero el momento lineal del sistema cuerpo-Tierra se mantiene constante debido a que la caída del cuerpo a Tierra se debe a la fuerza de atracción que ejerce la Tierra sobre cuerpo, lo cual es una fuerza interna del sistema,

y como se tiene que para un Sistema se cumple que ext dPF =dt∑r

r; en este caso se tiene

que extF =0,∑r

se tiene que el momento lineal del sistema P=constante.!

Respuesta a la pregunta 5.3 Cuando una rueda gira con una rapidez angular, todos los puntos de la rueda

tienen la misma rapidez angular, pero la rapidez lineal de cada varía debido a que para cada punto la rapidez lineal varía con la distancia r con respecto al centro de la rueda, esto es v=r! .

Respuesta a la pregunta 5.4 El momento angular de la niña con respecto al poste es cero, debido a que el

momento angular está dado por L=mr v, L=mrvsen ,!" #r r r donde el ángulo θ=180º

formado entre r y v, siendo que sen θ=0.

López Frontado, R. (2011). Física general I, Texto UNA. 2da edición. Caracas: UNA

Page 208: Fisica General i Una

219

Respuesta a la pregunta 5.5 Requiere más trabajo cuando el eje pasa por el borde del aro, debido a que el

trabajo que está dado por 2z

1W= I -02

ω , el cual depende del momento de inercia.

Teniéndose que el momento de inercia cuando el eje pasa por el borde del aro I’z’ =2MR2 es mayor que el momento de inercia cuando el eje pase por el centro del aro Iz = MR2.

Respuesta a la pregunta 5.6 La opción (a) es verdadera debido a que =r F=0, si F=0! "

! !!! ; la opción (b) es falsa

por cuanto dL d= = (mr v)=0dt dt

! "

!! !! , solo si la velocidad v! y el vector posición r! son

paralelos.

RESPUESTA A LOS EJERCICIOS PROPUESTOS:

Estas respuestas se corresponden con los ejercicios propuestos presentados

en el desarrollo de la unidad. Respuesta al ejercicio propuesto 5.1 DATOS: mn + p = M = 36 kg, mp = 0,5 kg, h = 15 m. SOLUCIÓN:

Figura 5.15 Las fuerzas que intervienen en el lanzamiento de la pelota son fuerza internas,

por lo tanto el momento lineal en la dirección horizontal se mantiene constante, esto es,

o f o f p p

p p

ˆ ˆP =P , P 0, y P MVi m v iˆ ˆ 0 MVi m v i (1)

→ = = +

→ = +

r r r r

López Frontado, R. (2011). Física general I, Texto UNA. 2da edición. Caracas: UNA

Page 209: Fisica General i Una

220

Para conocer la velocidad con que fue lanzada la pelota, necesitamos calcular

la velocidad necesaria para que la pelota logre llegar hasta una altura de 15 m. Se aplica el teorema de trabajo y energía, considerando despreciable el efecto del aire, y que no existe fricción sobre la pelota; se tiene que la energía en el punto de inicio debe ser igual a la energía en la parte más alta, esto es,

2

p pi f p p

m vE E , m gh v 2gh

2= = → =

Ahora se sustituye en la ecuación (1), así se tiene,

p p pm v m 2gh 0,5 2 9,8 15 mV V 0,24M M 36 s

× ×= − = − = − ⇒ = −

Siendo V= - 0,24 m/s la velocidad con que se mueve el sistema niño-patineta

después de haber lanzado la pelota, el signo negativo significa que sistema niño-patineta se mueve en sentido contrario al de la pelota.

Respuesta al ejercicio propuesto 5.2 DATOS: mR = 78,4 kg, mc = 31,6 kg, L = 2,93 m, d = 0,412 m SOLUCIÓN: En el movimiento de este sistema, no existen fuerzas externas que actúen en

la dirección del movimiento, esto es ∑Fx = 0, entonces, Px = constante. Además, como Px = MVCMx = constante, se tiene que VCMx = constante, pero VCmxo = 0 debido a que el sistema inicialmente está en reposo. Esto nos indica que la posición del centro de masa XCMx = constante.

Figura 5.16

López Frontado, R. (2011). Física general I, Texto UNA. 2da edición. Caracas: UNA

Page 210: Fisica General i Una

221

Se tiene que d = 2xc = 0,412 m, de donde xc = 0,206 m. Como XCM = constante, se tiene que XCMi = XCMf,

R R J J C C R R J J C C

CMi CMfR J C R J C

m x +m x +m x m x' +m x' +m x'x = = = x

m +m +m m +m +m

( ) ( )( )

R R C C J J J J C C R R

J C C C R C

C C R CJ

C

-m x +m x +m x = -m x' -m x' +m x'm 1,465 + x = -m x +m 1,465 - x

-m x +m 1,465 - xm =

1,465 + x

J J-31,6×0,206 + 78.4×1,239m = m = 55,23 kg

1,668!

Siendo la masa de Judith mJ = 55,23 kg. Respuesta al ejercicio propuesto 5.3 DATOS: 1 2

m mˆ ˆv 5 i , v 15 is s

= − =r r , m1 = 12 kg, k = 100 N/cm

Figura 5.17

SOLUCIÓN a) El sistema inicialmente está en reposo, por lo tanto oxP 0= . La fuerza

que impulsa las masas, es la fuerza del resorte, y es una fuerza interna del sistema, por lo tanto xP = constante, entonces se tiene que,

ox fx fxP P P 0= ⇒ =

Este resultado nos indica que el momento lineal del sistema después de dejarse libre las masas sigue siendo cero.

b) Para determinar el cociente entre m1 y m2, se tiene

fx 1 1 2 2 1 1 2 2

1 2 1

2 1 2

ˆ ˆP 0 m v i m v i m v m vm v m15 3m v 5 m

= = − + → =

= = ⇒ =

Resultado que nos indica que el cociente entre las masas es igual a 3.

López Frontado, R. (2011). Física general I, Texto UNA. 2da edición. Caracas: UNA

Page 211: Fisica General i Una

222

c) La velocidad del centro de masa sel sistema, se obtiene, tomando en cuenta que

ox fx CM CMP P 0 Mv v 0= = = ⇒ =

d) Para determinar la energía cedida por el resorte a las partículas, es

necesario calcular el valor de la masa m2. Esto se obtiene a partir de la relación de las masas; así se tiene que,

12 2

m 12m m 4kg3 3

= = → =

Entonces la energía cinética cedida por el resorte a las partículas es,

2 2 2 2

1 1 2 2m v m v 12 ( 5) 4 (15)K K 600J2 2 2 2

× − ×= + = + ⇒ =

e) Aplicando el teorema de trabajo y energía, se obtiene

2F

2

k xK W K 0 02

2K 2 600x x 0,35mk 10000

eΔΔ = → − = −

×Δ = = ⇒ Δ =

La compresión del resorte es Δ= 0,35 m. Respuesta al ejercicio propuesto 5.4

Figura 5.18

DATOS: m1= 4kg, m2= 6kg, 1 2

ˆ ˆv =2i m/s, v =3j m/s.! !

SOLUCIÓN: a) El momento angular total del sistema respecto a O es

o i i i i i 1 1 1 2 2 2i i

L = r p = mr v =m r v +m r v! ! ! !" "r r r r r rr r r

Pero 1 2

ˆ ˆr =3j m y r =4i mr r

o

2

o

ˆ ˆ ˆ ˆ ˆ ˆL =4(3j) (2i)+6(4i) (3j)=-24k+72k

kg.mˆ L =48k s

! !

"

!

!

López Frontado, R. (2011). Física general I, Texto UNA. 2da edición. Caracas: UNA

Page 212: Fisica General i Una

223

b) El momento angular con respecto al centro de masa CM, está dado por

C i i 1 1 1 2 2 2i

L = r' p =m r' v +m r' v! ! !"r r r r rr r donde

1 1 2 2C C

1 2

ˆ ˆm r +m r 4(3j)+6(4i) ˆ ˆr = = r =(1,2j+2,4i) mm +m 4+6

!! !

! !

1 1 C 1

2 2 C 2

ˆ ˆ ˆ ˆ ˆr' =r -r =3j-1,2j-2,4i r' =(-2,4i+1,8j) mˆ ˆ ˆ ˆ ˆr' =r -r =4i-1,2j-2,4i r' =(1,6i-1,2j) m

!

!

r r r r

r r r r

Entonces

C2

C

ˆ ˆ ˆ ˆ ˆ ˆL =4(-2,4i+1,8j) (2i)+6(1,6i-1,2j) (3j)

kg.mˆ L =14,4k s

× ×

r

r

Siendo CL!

el momento angular del sistema respecto al centro de masa CM. Respuesta al ejercicio propuesto 5.5 DATOS: m = 0,3 kg, v = 12,0 m/s, r = 8,0 m. SOLUCIÓN:

Figura 5.19

Expresando la posición y la velocidad en forma vectorial, se tiene,

o oˆ ˆr=-8,0cos 37 i+8,0sen 37 j

ˆ ˆr=(-6,39i+4,81j) m mˆv=12,0i s

!

!

!

El momento angular L

! de la piedra respecto al punto O, está dado por

2kg.mˆ ˆ ˆL=r×p=mr×v=0,3(-6,39i+4,81j)×(12,0 i) L=-17,32 k

s!

! ! !! ! ! !

López Frontado, R. (2011). Física general I, Texto UNA. 2da edición. Caracas: UNA

Page 213: Fisica General i Una

224

Respuesta al ejercicio propuesto 5.6 DATOS: m = 50,0 g, r = 0,1 m, r1 = 0,3 m , v1 = 1,5 m/s SOLUCIÓN:

Figura 5.20

a) La fuerza T se trasmite a través de la cuerda y es paralela al radio r. Por definición

r F 0τ = × =!!!

La 2ª ley fundamental de la

mecánica establece que dLdt

τ =!

!

Si dL0 0,dt

τ = → =r

r

L constante,se tiene=!

que Li = Lf → mr1 v1 = m r v. De donde se obtiene que la expresión de la velocidad final de la partícula en función de r, esto es

1 1v rvr

⇒ = .

b) Para determinar la tensión de la cuerda, se aplica la 2ª ley de Newton,

la cual establece que 2

rvF mr

=∑ . Así se tiene que

22

1 13

m(r v )vT m Tr r

= ⇒ =

c) El trabajo realizado al disminuir el radio desde r1 hasta r, se obtiene

conociendo que

2 221 1

212

mv mvmvW K W2 2 r2 1

r

= Δ = − ⇒ =⎡ ⎤

−⎢ ⎥⎣ ⎦

d) Los valores de la velocidad de la partícula, de la tensión de cuerda y del

trabajo realizado, al sustituirse los valores numéricos dados en el inciso d) son:

1 1

2 21 13 3

2

2

r v 0,3 1,5 mv v 4,5r 0,1 sm(r v ) 0,05(0,3 1,5)T T 10,1Nr (0,1)

0,05 (1,5)W W 0,45 J

0,32 10,1

×= = ⇒ =

×= = ⇒ =

⎡ ⎤×⎣ ⎦= ⇒ =⎡ ⎤⎛ ⎞ −⎢ ⎥⎜ ⎟⎝ ⎠⎢ ⎥⎣ ⎦

López Frontado, R. (2011). Física general I, Texto UNA. 2da edición. Caracas: UNA

Page 214: Fisica General i Una

225

RESPUESTA A LOS EJERCICIOS DE AUTOEVALUACIÓN: R 5.1 ˆ-0,667i m/s R 5.2 a) ˆ ˆ(100,0i+50,0j) m/s; b) 3,3x105 J R 5.3 130 N. R 5.4 2ˆ ˆ ˆ(55i-90j+42k) kg.m /s R 5.5 a) La frecuencia angular decrece debido a que el momento de

inercia aumenta; b) 1,6.

López Frontado, R. (2011). Física general I, Texto UNA. 2da edición. Caracas: UNA

Page 215: Fisica General i Una

227

UNIDAD 6

TRABAJO Y ENERGÍA PARA SISTEMAS DE PARTÍCULAS. COLISIÓN

Luego de realizar el estudio de los conceptos de Trabajo y Energía,

aplicándolos a sistemas de partículas, Se tomarán en cuenta las interacciones de las partículas que conforman el sistema, entre sí y con el entorno. También se abordará el estudio de las leyes de conservación de energía.

Se estudia además las colisiones (o choques) entre objetos, donde se

aplican las leyes de conservación de la energía y del momento lineal.

CONOCIMIENTOS PREVIOS: El estudiante para la correcta comprensión de esta unidad, debe tener los

siguientes conocimientos: 1.- Conceptos de Centro de masa (Unidad 5) 2.- Conceptos de Momento Lineal y Angular para un sistema de partículas

(Unidad 5). 3.- Principios de Conservación de los Momentos lineales y angulares para

un sistema de partículas (Unidad 5). 4.- Conceptos de trabajo y energía para una partícula (Unidad 4). 5.- Principios de Conservación de Energía para una partícula (Unidad 4).

6.1 OBJETIVO Aplicar los conceptos de Energía cinética de rotación para sistema de

partículas; Trabajo, Energía y Potencia, colisión en la resolución de problemas de dinámica de sistemas de partículas.

López Frontado, R. (2011). Física general I, Texto UNA. 2da edición. Caracas: UNA

Page 216: Fisica General i Una

228

6.2 RECOMENDACIONES PARA EL ESTUDIO DEL CONTENIDO

En esta unidad, el estudiante debe

comprender claramente los siguientes conceptos básicos:

Energía para un sistema de

partículas. Teorema Trabajo-Energía para un sistema de partículas. Colisión (o choques).

CONCEPTOS BÁSICOS: Energía cinética para un sistema de partículas. Trabajo y energía para un sistema de partículas. Principio de Conservación de energía para un sistema de partículas. Colisión (o choques)

Para una mejor comprensión del contenido de la unidad, realice la lectura de

los conceptos relacionados con el movimiento de un sistema de partículas, aplicando los conceptos de Energía cinética de rotación, de trabajo y energía, los principios de conservación de la energía, y de colisión (o choques) en los ejemplos relacionados para ello, en los capítulos señalados en el Plan de Curso, utilice las técnicas de lectura que le facilite la comprensión del contenido.

6.3 CONTENIDO

6.3.1 ENERGÍA CINÉTICA PARA UN SISTEMA DE PARTÍCULAS. 6.3.2 RELACIÓN TRABAJO Y ENERGÍA PARA UN SISTEMA DE

PARTÍCULAS. 6.3.3 COLISIÓN (CHOQUE).

6.3.1 ENERGÍA CINÉTICA PARA UN SISTEMA DE PARTÍCULAS Consideremos un sistema de partículas, donde cada partícula se mueve con

una velocidad dada. La energía cinética de cada partícula está dada por

2i i i1K = mv2

La energía cinética del sistema de partículas está dada por

n n2

i i ii=1 i=1

1K= K = m v (6.1)2! !

López Frontado, R. (2011). Física general I, Texto UNA. 2da edición. Caracas: UNA

Page 217: Fisica General i Una

229

Figura 6.1

Si ahora consideramos un sistema de partículas, donde cada una se mueve describiendo una trayectoria circular alrededor de un eje zz’ cualquiera con una velocidad angular ω; se tiene entonces que la rapidez de cada partícula está dada por

i iv =r!

Por lo tanto, la energía cinética del sistema de partículas está dado por

n2 2

i ii=1

1K= mr ω 2

! "# $% &'

La cantidad I = Σmi ri2, se denomina Momento de Inercia del sistema de

partículas. El momento de inercia es una cantidad escalar, dada por el producto de la masa de cada partícula y la distancia de esa partícula al eje de rotación del sistema. Sus unidades en el Sistema Internacional (SI) son kg.m2. La Energía cinética del sistema de partículas, para este sistema en rotación, se puede escribir como

21K= Iω (6.2)

2

Esta expresión de la energía cinética está relacionada con el movimiento de

rotación del sistema, por lo que se denomina Energía Cinética de Rotación, o Energía Cinética Rotacional.

Si ahora consideramos que el sistema de partículas se traslada con una

velocidad, dada por la velocidad del Centro de Masa ( CMV!

); y además está girando alrededor de un eje que pasa por el Centro de Masa del mismo con una velocidad angular ω, la energía cinética de este sistema será:

Traslación Rotación

2 2CM C

n n2

i C i ii=1 i=1

K=K +K1 1K= MV + I ω (6.3)2 2

donde M= m , I = mr! !

López Frontado, R. (2011). Física general I, Texto UNA. 2da edición. Caracas: UNA

Page 218: Fisica General i Una

230

A continuación se presentan unos ejemplos de problemas, donde Ud. puede

visualizar las aplicaciones de los conceptos tratados en esta unidad. Pero antes vea las Sugerencias y las Recomendaciones presentadas para resolver estos problemas.

RECOMENDACIONES PARA RESOLVER PROBLEMAS DE DINÁMICA

DE UN SISTEMA DE PARTÍCULAS USANDO LOS CONCEPTOS DE TRABAJO Y ENERGÍA.

IDENTIFICAR los conceptos pertinentes: a) Se pueden usar las relaciones del trabajo y la energía, y la

conservación de la energía para obtener las relaciones entre la posición y el movimiento de un sistema de partículas. Las recomendaciones presentadas para resolver problemas de energía para una partícula son igualmente válidas para sistemas de partículas, aunque la única diferencia es que la energía cinética de un sistema de partículas puede involucrar tanto el movimiento de rotación como el movimiento de traslación del mismo, lo cual implica que la energía cinética de rotación se pueda expresar en términos del momento de inercia I y de la velocidad angular ω del sistema (K= Iω2 /2).

b) Cuando trabaje con problemas que impliquen colisión (o choques)

entre dos objetos, las condiciones de conservación lineal y las condiciones del cambio de energía producido en una colisión.

PLANTEAR el problema:

c) Establezca un sistema de coordenadas y defina sus velocidades

respecto de ese sistema.

d) Escriba expresiones para el momento lineal total en el momento justo antes de la colisión y justo después de ella. Recuerde que el momento del sistema es el que es constante, no los momentos lineales de los objetos individuales.

e) Si la colisión es elástica, la energía cinética se conserva y Ud. puede

igualar la energía cinética total del sistema antes del choque con la energía cinética total después del choque, para obtener una relación adicional entre las velocidades.

f) Si la colisión es inelástica, la energía cinética no se conserva y es

probable que se requiera información adicional. Si la colisión es

López Frontado, R. (2011). Física general I, Texto UNA. 2da edición. Caracas: UNA

Page 219: Fisica General i Una

231

perfectamente inelástica, las velocidades finales de los dos objetos son iguales. Proceda a resolver las ecuaciones del momento lineal respecto de las cantidades desconocidas.

EJECUTAR la solución del problema:

Una vez planteadas las ecuaciones o sistemas de ecuaciones necesarias para obtener las incógnitas a determinar, verifique que el número de ecuaciones coincide con el número de incógnitas a determinar.

Resuelva las ecuaciones o los sistemas de ecuaciones pertinentes para

obtener los valores o fórmulas de las incógnitas a determinar. EVALUAR los resultados: compruebe que los resultados obtenidos son

lógicos. Si es posible verifique los resultados probándolos para casos especiales o valores extremos, comparándolos con lo que espera intuitivamente.

Recuerde que el manejo eficiente del contenido de esta unidad es

importante, por su repercusión en el contenido de las subsiguientes unidades del curso de Física General I.

Ejemplo 6.1 Dos partículas de masas 2 kg y 3 kg, se mueven con relación a un observador

inercial, con velocidades v1 = 10 m/s a lo largo del eje x, y v2 = 8 m/s en un ángulo de 120º con el eje x. Para el sistema descrito, determina:

a) La velocidad del centro de masa.

b) La energía cinética total de las partículas.

DATOS: m1 2 kg, m2 = 3 kg, v1 = 10 m/s, v2 = 8 m/s, SOLUCIÓN: IDENTIFICAR los conceptos pertinentes al problema: En este problema se aplicarán los conceptos de velocidad del centro de masa

CMv! de un sistema de partículas; y la energía cinética total de un sistema de partículas.

López Frontado, R. (2011). Física general I, Texto UNA. 2da edición. Caracas: UNA

Page 220: Fisica General i Una

232

PLANTEAR y EJECUTAR solución del problema: a) Para determinar la velocidad del centro de masa de las partículas, se

expresan previamente las velocidades de cada una en forma vectorial, esto es

o o

1 2

2

m mˆ ˆ ˆv = 10 i y v = 8cos120 i+ 8sen120 j s s

mˆ ˆ v = -4 i+ 6,93 j s

r r

r

La velocidad del centro de masa está dada por:

1 1 2 2CM

1 2

CM CM

ˆ ˆ ˆm v +m v 2×10 i+ 3(-4 i+ 6,93 j)V = =m +m 2 + 3

ˆ ˆ(20 -12) i+ 20,8 j mˆ ˆV = V = 1,6 i+ 4,2 j 5 s

!

! !!

! !

La magnitud de la velocidad del centro de masa es

2 2CM CM

mV = (1,6) + (4,2) V = 4,49 s

!

Su dirección está dada por: -1 4,2=tan =69,15°1,6

φ φ⎛ ⎞ ⇒⎜ ⎟⎝ ⎠

b) La energía cinética total del sistema está dado por:

2 2CMMV 5×(4,49)K = =

2 2 K = 50,4 J!

EVALUAR el resultado: El resultado de la energía cinética del sistema de

partículas está dada tomando en cuenta el movimiento del centro de masa del mismo.

6.3.2 RELACIÓN TRABAJO ENERGÍA PARA UN SISTEMA DE PARTÍCULAS Consideremos un sistema de partículas S1, constituido por dos partículas

(para simplificar el tratamiento) de masas m1 y m2; el cual está rodeado por un sistemas de partículas S2. Las partículas del sistema S1 interactúan entre si y con las partículas del sistema S2. Sean 1F

r y 2F

!, las fuerzas resultantes que actúan sobre las

López Frontado, R. (2011). Física general I, Texto UNA. 2da edición. Caracas: UNA

Page 221: Fisica General i Una

233

partículas m1 y m2, respectivamente, debido a las interacciones de éstas con las partículas del sistema S2; 12F

r, la fuerza de interacción entre las dos partículas 1 y 2.

Supongamos que las partículas se muevan, en un instante dado, con

velocidades 1ov!

y 2ov!

, respectivamente; y en un momento posterior, sus velocidades son 1f 2fv y v

r r, debidas al efecto que ejercen las fuerzas de interacción sobre ellas.

Figura 6.3

Aplicando la Relación Trabajo-Energía para el sistema de partículas S1, se tiene

ext intΔK=W +W (6.4) donde

ΔK = Kf – Ko , Wext es el trabajo realizado por las fuerzas externas al sistema. Wint es el trabajo realizado por las fuerzas internas del sistema, entre cada par de partículas.

De acuerdo a la ecuación (6.4), se tiene que el cambio de energía cinética

de un sistema de partículas es igual al trabajo realizado sobre el sistema por las fuerzas externas e internas.

Si las fuerzas internas son conservativas, se tiene:

int int intij,o ij,f

int int extf o ij,o ij,f

W =-ΔU=U -U

la ecuación (6.4) se puede escribir K -K =U -U +W

int Int ext

f ij,f o ij,o

extp,f p,o

(K +U )-(K +U )=W

E -E =W (6.5)

Donde int

P ijE =K+U se denomina la Energía propia del Sistema. De la ecuación (6.20) se tiene que, si un sistema de partículas está

aislado, o la suma de las fuerzas externas es cero, la Energía propia del sistema se mantiene constante, como se manifiesta en la ecuación (6.21).

López Frontado, R. (2011). Física general I, Texto UNA. 2da edición. Caracas: UNA

Page 222: Fisica General i Una

234

Si el sistema está aislado, o extF =0!!

, se tiene que

ext

p,f p,o p,f p,o

int intf ij,f o ij,o

W =0 E -E =0 E =E ,

la energia propia se mantiene constante, esto es K +U =K +U (6.6)

∴ →

Si ahora se tiene que las fuerzas externas son conservativas, entonces

ext ext ext exto f

ext extp,f p,o o f

T To f

W =- U =U -U , entonces se tiene

E -E =U -U ,

E =E (6.7)

Δ

donde T int ext

ijE =K+U +U , se denomina Energía Total del Sistema. De la ecuación (6.7), se tiene que la Energía Total del Sistema es

constante. Esto se cumple siempre y cuando las fuerzas, tanto las internas como las externas, sean conservativas.

Ejemplo 6.2

Figura 6.2

Un regulador de velocidad, como se muestra en la figura (6.2), la cual consiste de dos pequeñas esferas, cada una de masa 0,9 kg, unidas a un eje vertical por dos varillas ligeras. Cuando dicho eje vertical gira a 800 rpm, se encuentra que las esferas describen una trayectoria circular horizontal de 0,18 m de radio. Determine el trabajo efectuado sobre el regulador para llevarlo desde el reposo hasta su velocidad final de rotación. Desprecie la fricción y considere a las esferas pequeñas como masas puntuales. DATOS: m = 0.9 kg, fi = 0, ff = 800 rpm, r = 0,18 m.

López Frontado, R. (2011). Física general I, Texto UNA. 2da edición. Caracas: UNA

Page 223: Fisica General i Una

235

SOLUCIÓN: IDENTIFICA los conceptos pertinentes: Se aplica la relación de trabajo y energía cinética a un sistema de partículas. PLANTEAR y EJECUTAR la solución del problema: En este problema se aplica la relación que existe entre trabajo y energía

cinética para un sistema de partículas, f iW= K=K -K! . Considerando que cuando el sistema está en reposo, la energía cinética es cero. También se tiene que las esferas tienen igual masa, por lo tanto m1 = m2 = m. Así se tiene que el trabajo realizado el regulador está dado por:

2 2 2

eje f f

22

1W=2 I -0, W=mr (2 f )2

2 800W=0,9(0,18) W=204,66 J60

! "

"

# $% &' (

# $ )% &' (

EVALUAR el resultado: El valor obtenido para el trabajo corresponde a la incógnita pedida en el

enunciado y se corresponde de una forma lógica con él.

Ejemplo 6.3

Figura 6.4

El sistema mostrado en la figura, está constituido por: dos cuerpos de masas m1 =4 kg y m2 = 12 kg, unidos por una cuerda de masa despreciable que pasa a través de una polea ideal, sin masa y sin fricción, que gira alrededor de su eje. Se sueltan desde el reposo cuando la masa m2 está a una altura de 2 m sobre el piso. Cuando la masa m2 llega al piso, calcule:

a) las velocidades de los cuerpos. b) La altura máxima alcanzada por m1. DATOS: m1 =4 kg, m2 = 12 kg, H = 2 m.

López Frontado, R. (2011). Física general I, Texto UNA. 2da edición. Caracas: UNA

Page 224: Fisica General i Una

236

SOLUCIÓN: IDENTIFICAR los conceptos pertinentes: En este problema, el sistema se considera que está formado por las masas m1

y m2, ya que la polea se considera ideal, y las cuerda con masa despreciable sin fricción. Se aplica entonces el teorema de trabajo-energía dado como la variación de energía cinética y el trabajo realizado por la fuerza peso, por lo que se tiene que la energía mecánica (E = K + U) se mantiene constante.

PLANTEAMIENTO y EJECUCIÓN del problema: a) Para determinar la velocidad de los cuerpos cuando llegan a la posición

B (o sea cuando m2 llega al piso), aplicando el principio de conservación de la energía mecánica, se tiene

A B A A B B

2 21 1B 2 2B

2 1

E =E K +U =K +U ,

m v m v0+m gH= + +m gH

2 2

!

Pero 1B 2Bv =v , debido a que la cuerda se considera ideal e inextensible, y por lo tanto la velocidad de cualquier punto de ella va a tener la misma velocidad. Así se tiene que

2

1 2 1B 2 11(m +m )v = (m -m )gH2

Despejando de esta ecuación la velocidad en la posición B y sustituyendo sus valores, se obtiene el valor de la velocidad de los cuerpos en la posición B. Esto es,

2 2 11B

1 2

1B 2B

2(m - m )gH 2(12 - 4)9,8×2,0v = =(m + m ) (12 + 4)

m v = v = 4,43 s

!

b) Para calcular la altura máxima alcanzada por el cuerpo m1, después

que el cuerpo m2 ha llegado al suelo, se estudia el movimiento del cuerpo m1 desde la posición B hasta la posición C, posición a la cual llega cuando su velocidad es cero; así se tiene

2 2 2C B BC BC

2

BC BC

v = v - 2gH , 0 = (4,43) - 2×9,8H

(4,43)H = H = 1,0013 m2×9,8

!

López Frontado, R. (2011). Física general I, Texto UNA. 2da edición. Caracas: UNA

Page 225: Fisica General i Una

237

Así se tiene que la altura máxima ymax alcanzada es Hmax = H + HBC → Hmax = 3,0013 m EVALUACIÓN del resultado: Es lógica la respuesta por estar de acuerdo con los valores previstos.

Ejercicio propuesto 6.1.- Una persona de 55 kg de masa está parada en el centro de una plataforma

giratoria de radio 2,5 m y momento de inercia igual a 670 kg.m2. La plataforma gira sin fricción con velocidad angular de 2,0 rad/s. La persona camina radialmente hacia el borde de la plataforma. Tomando en consideración lo antes planteado, para el sistema plataforma más persona:

a) Calcule la velocidad angular cuando la persona llega al borde.

b) Compare las energías cinéticas rotatorias antes y después de la

caminata de la persona.

6.3.3 COLISIÓN (Ó CHOQUES) El término de Colisión (o Choque) se usa

para representar el evento en el cual, dos partículas (o sistemas) que se aproximan entre si como se muestra en la figura (6.5), interactúan intensamente durante un brevísimo intervalo de tiempo, produciendo un intercambio de momento lineal y de energía. Durante una colisión las fuerzas externas al sistema son tan insignificantes frente a las fuerzas internas debido a la interacción de una sobre la otra.

Figura 6.5

Por lo tanto, se tiene que

extF =0, P=constante, o sea→∑ ! ! que se cumple el principio de

conservación del momento lineal total. Esto es:

a dP =P (6.8)! !

López Frontado, R. (2011). Física general I, Texto UNA. 2da edición. Caracas: UNA

Page 226: Fisica General i Una

238

donde aP!

es el momento lineal del sistema antes de la colisión, y dPr

es el momento

lineal del sistema después de la colisión. El momento lineal del sistema 1 2P=p +p! ! !

La ecuación (6.8), se puede escribir

1a 2a 1d 2d 1d 1a 2d 2a

1 2

p +p =p +p , p -p =-(p -p ) Δp =-Δp!

! ! ! ! ! ! ! !

! !

Vemos así que se produce un intercambio de momento lineal entre las

partícula, esto es, que la cantidad de movimiento lineal que gana una partícula es igual a la cantidad de movimiento lineal que pierde la otra partícula.

Observemos que sucede con la energía del sistema de partículas durante una

colisión. Para ello se tiene que la energía propia del sistema de partículas está dada por: int

pE =K+U . Aunque la energía propia del sistema se conserva durante una colisión, la

energía cinética original no siempre se conserva. Esto es:

int inta a d d

int intd a a d

K +U =K +U (6.9)

K -K =U -U

Usualmente, parte de la energía cinética es transformada por las fuerzas

internas en otras formas de energía, tales como térmica, potencial, o de algún otro tipo. En general, las colisiones se clasifican de acuerdo al cambio que se produzca en la energía cinética. Para ello, denominemos por Q el cambio de energía cinética durante la colisión, la cual está dada por

int int

d a a dQ=K -K =U -U (6.10) Cuando Q = 0, la energía cinética del sistema se conserva, o sea que no se

produce un intercambio de energía entre las partículas y el medio que lo rodea; se dice entonces que la colisión es elástica. Si Q ≠ 0, se produce un intercambio de energía entre las partículas y el medio que lo rodea; se dice entonces que la colisión es inelástica. Esta colisión inelástica puede ser: inelástica endoérgica, cuando Q < 0; esto quiere decir que existe una pérdida de la energía cinética con un correspondiente aumento de la energía potencial interna, es decir (Kd < Ka, y Ua

int < Ud

int), lo que nos indica que el sistema absorbe energía del medio que lo rodea; inelástica exoérgica (o explosiva) cuando Q > 0, o sea que se produce un aumento de la energía cinética a expensa de la energía potencial interna, esto es (Kd > Ka, y Ua

int > Udint ).

López Frontado, R. (2011). Física general I, Texto UNA. 2da edición. Caracas: UNA

Page 227: Fisica General i Una

239

Cuando los cuerpos quedan unidos después de la colisión, se dice que la colisión es completamente inelástica.

Cuando la colisión entre dos cuerpos se produce de forma frontal (o central),

las velocidades de los cuerpos después de la colisión están relacionados con las velocidades antes de la colisión por la expresión:

1f 2f

1i 2i

v -ve= - (6.11)v -v

Donde e es denominado coeficiente de restitución, el cual toma valores

entre 1 y 0, dependiendo del tipo de colisión que se tenga. Cuando la colisión es elástica, se tiene que e = 1; mientras que cuando la colisión es completamente inelástica, e = 0; para cualquier colisión inelástica, e toma valores entre 0 y 1. Esta expresión fue propuesta por Newton y tiene validez solamente aproximada.

En base a los conceptos emitidos hasta los momentos, responda las siguientes preguntas.

Pregunta 6.1 ¿En una colisión elástica, donde además de tener las fuerzas entre los

cuerpos deben ser conservativas, la energía cinética de cada partícula debe ser la misma antes y después de la colisión?

Pregunta 6.2 ¿En una colisión inelástica, además de cambiar la energía cinética del sistema

también debe cambiar el momento lineal del sistema?

Ejemplo 6.4 Una partícula de masa m1 = 0,5 kg moviéndose a 0,60 m/s choca con otra

partícula de masa m2 = 0,3 kg, que está en reposo. Después del choque, la primera

López Frontado, R. (2011). Física general I, Texto UNA. 2da edición. Caracas: UNA

Page 228: Fisica General i Una

240

partícula se mueve a 0,3 m/s en una dirección que hace un ángulo de 53º con la dirección original. Para esta situación, determine:

a) La velocidad de la segunda partícula después de la colisión.

b) Diga el tipo de colisión producida.

DATOS: m1 = 0,5 kg, m2 = 0,3 kg, v1o = 0,6 m/s, v1f = 0,3 m/s. SOLUCIÓN: IDENTIFICAR los conceptos pertinentes del problema: Este problema se refiere a la colisión entre dos partículas, donde se aplica el

principio de conservación de momento lineal durante la colisión y se puede producir un cambio en la energía cinética del sistema de partículas durante la colisión, indicándonos así que tipo de colisión se produjo entre las partículas.

PLANTEAR y EJECUTAR la solución del problema: En la figura (6.5) se hace un dibujo donde se esboza la situación de las

partículas antes de la colisión y lo que sucede después de la colisión.

Figura 6.5

a) En una colisión, generalmente sólo actúan fuerzas internas, por lo tanto,

extF = 0, P = constante→∑r r

a d 1a 1d 2dP =P , p +0=p +pr r r r r

O también o o

1 1a 1 1d 1 1d 2 2dˆ ˆ ˆm v i+0 =m v cos53 i+m v sen53 j+m v

! Al despejar 2dv

! , se tiene

o o1 1a 1 1d 1 1d

2d2 2

2d

m v -m v cos53 m v sen53ˆ ˆv = i- jm m

0,5×0,6 - 0,5×0,3×0,6 0,5×0,3×0,8ˆ ˆv = i- j0,3 0,3

r

r

2d 2dmˆ ˆ ˆ ˆv = (1- 0,6) i- 0,4 j v = 0,7 i- 0,4 j s

⇒r r

La magnitud de 2dv

! es,

2 22d 2d

mv = (0,7) + (0,4) v = 0,809 s

!

López Frontado, R. (2011). Física general I, Texto UNA. 2da edición. Caracas: UNA

Page 229: Fisica General i Una

241

b) Para conocer el tipo de colisión, es necesario calcular el valor de Q,

esto es:

2 2 2d a 1 1d 2 2d 1 1a

2 2 2

1 1 1Q=K -K = m v + m v - m v2 2 2

0,5(0,3) 0,3(0,806) 0,5(0,6)Q= + -2 2 2

Q=0,0225+0,0975-0,09 Q=0,3 J>0!

EVALUAR el resultado: Este resultado nos indica que la variación de la energía cinética Q es diferente

de cero y positiva, de donde podemos concluir que la colisión es inelástica explosiva, o sea que durante la colisión el sistema de partículas emite energía al medio que le rodea.

Ejercicio propuesto 6.2.- Un proyectil de 30 g se dispara contra un bloque de 3,5 kg, inicialmente en

reposo sobre una superficie horizontal rugosa. El coeficiente de fricción dinámico entre el bloque y la superficie es igual a 0,2. Si el proyectil atraviesa el bloque y sale de él con una rapidez que es igual a la mitad de la que traía antes del choque; mientras que el bloque, después del impacto, desliza sobre la superficie una distancia d = 80 cm, antes de detenerse, determine la rapidez inicial del proyectil.

Ejemplo 6.5.-

Figura 6.6

La figura muestra un proyectil de masa m y rapidez vo, que es disparado contra un bloque de un péndulo de masa M, el cual está suspendido por una varilla rígida de longitud ! y masa despreciable El proyectil se incrusta en el bloque. ¿Cuál es el valor mínimo de vo tal que el bloque del péndulo oscile todo un círculo vertical completo?

López Frontado, R. (2011). Física general I, Texto UNA. 2da edición. Caracas: UNA

Page 230: Fisica General i Una

242

SOLUCIÓN: IDENTIFICAR el problema: Al analizar el enunciado del problema, se debe tomar en cuenta la colisión

producida entre el proyectil y el bloque en la posición A; luego la relación de trabajo-energía que se produce sobre el bloque con el proyectil, cuando de desplaza desde la posición A hasta la posición B; y por último la aplicación de la 2ª ley de Newton en la posición B, para obtener así el valor mínimo del bloque-proyectil cuando pasa por B.

PLANTEAR el problema: Tomando en cuenta la posición en A, donde se produce la colisión, se aplica

el principio de conservación del momento lineal, lo cual nos dice que el momento lineal del sistema se mantiene constante, esto es

a d o AP =P , mv =(m+M)V (1)r r

Mientras el bloque-proyectil de mueve desde A hasta B: se aplica el principio

de conservación de energía mecánica (E=K+U), el cual nos dice que la energía mecánica se mantiene constante. Así se tiene que

2 2

A B A B

2 2A B

1 1E =E , (m+M)V = (m+M)V +(m+M)g(2 )2 2

V =V +4g (2)→

l

l

Pero en el punto B, al aplicar la 2ª ley de Newton, se tiene que

2BVT+(m+M)g=(m+M)l

La velocidad mínima en VB, se obtiene cuando T=0, 2

B V =g! ! EJECUTAR la solución del problema: Al sustituir este valor en la ecuación (2), se tiene que 2

A AV =g +4g , V = 5g!! ! !

López Frontado, R. (2011). Física general I, Texto UNA. 2da edición. Caracas: UNA

Page 231: Fisica General i Una

243

De donde se obtiene la velocidad inicial mínima del proyectil, para que el bloque logre dar la vuelta completa alrededor del punto O, esto es,

o A o

m+M m+Mv = V v = 5gm m

⇒ !

EVALUAR la solución del problema: Este resultado corresponde a la fórmula que nos proporciona la velocidad que

debía traer el proyectil, de masa m, antes de hacer impacto con el bloque, de masa M, para que el sistema bloque-proyectil en su movimiento, logre dar la vuelta completa alrededor del centro O de la circunferencia.

Ejercicio propuesto 6.3.-

Figura 6.7

Como se muestra en la figura, un proyectil de masa m = 5,0 g, es disparado a un bloque de masa M = 0,8 kg, inicialmente en reposo sobre la superficie de una mesa de superficie rugosa (µc = 0,15). El proyectil permanece en el bloque después del impacto, y después desliza una distancia d1 = 0,5 m, hasta el borde de la mesa, después de la cual cae.

Si la altura de la mesa es Hm = 1,2 m, determine la distancia D de la parte baja

de la mesa, a la cual cae.

Ejercicio propuesto 6.4.-

Figura 6.8

Considere el camino ABC sin fricción, como se muestra en la figura. Un bloque de masa m1 = 5 kg se suelta desde A. Al llegar a la parte más baja de su trayectoria (o sea, en el punto B) se produce una colisión elástica frontal contra un bloque de masa m2 = 10 kg, inicialmente en reposo. Calcule la máxima altura a la cual regresará m1 después de la colisión.

López Frontado, R. (2011). Física general I, Texto UNA. 2da edición. Caracas: UNA

Page 232: Fisica General i Una

244

Ejercicio propuesto 6.5.- Una piedra de 0,100 kg descansa sobre una superficie horizontal lisa. Si una

bala de 4,0 g, que viaja horizontalmente a 450 m/s, golpea la piedra y rebota horizontalmente a 90º de su dirección original, con una rapidez de 300 m/s, determine:

a) La velocidad de la piedra después del golpe.

b) ¿Es perfectamente elástico el choque?

Ejercicio propuesto 6.6.- Un bloque de masa m1 = 1,88 kg se desliza a lo largo de una mesa sin fricción

a una velocidad de 10,3 m/s. Directamente enfrente de él, y moviéndose en la misma dirección, está un bloque de masa m2 = 4,92 kg que se mueve a razón de 3,27 m/s. Un resorte carente de masa con una fuerza constante de k = 11,2 N/cm está unido a la parte posterior de m2. Cuando los bloques chocan, ¿cuál es la máxima compresión del resorte?

Ejercicio propuesto 6.7.- Un bombero de 75 kg se desliza hacia abajo por un poste con una fuerza de

fricción constante de 300 N, que retarda su movimiento. Una plataforma horizontal de 20 kg es sostenida por un resorte en el pie del poste para amortiguar la caída. El bombero inicia su movimiento desde el reposo a 4,0 m sobre la plataforma, y la constante del resorte es de 4000 N/m. Determine:

a) La rapidez del bombero justo antes de que choque con la plataforma.

b) La compresión máxima que sufre el resorte.

(Suponga que la fuerza friccionante actúa durante todo el movimiento).

López Frontado, R. (2011). Física general I, Texto UNA. 2da edición. Caracas: UNA

Page 233: Fisica General i Una

245

RESUMEN: La energía cinética de un sistema de partículas está dado por:

2 2CM C

1 1K= MV + I ω2 2

El teorema trabajo-energía para un sistema de partículas está

dado por. T ext intK=W =W +W!

Donde Wext es el trabajo realizado por las fuerzas externas al

sistema, y Wint es el trabajo realizado por las fuerzas internas del sistema de partículas.

La energía propia del sistema de partículas está dado por:

intP ijE =K+U

La energía total del sistema está dado por:

T int extijE =K+U +U

Cuando se produce la colisión (o choque) entre dos partículas (o

sistema de partículas) el momento lineal antes de la colisión es igual al momento lineal después de la colisión, independientemente de la naturaleza de éste. Una colisión elástica es aquella en la que la energía cinética total se conserva. Una colisión inelástica es aquella en la que se produce un cambio en la energía cinética total del sistema. Una colisión perfectamente inelástica es aquella donde los cuerpos permanecen unidos después de la colisión.

EJERCICIOS DE AUTOEVALUACIÓN

En su libreta de apuntes el desarrolle los siguientes ejercicios. Luego de

resolver los ejercicios, compare sus respuestas con las presentadas en la última página de la unidad. En caso de no acertar o tener dudas al respecto, consulte con otros compañeros con el al asesor de su centro local.

López Frontado, R. (2011). Física general I, Texto UNA. 2da edición. Caracas: UNA

Page 234: Fisica General i Una

246

6.1.- Imagine que un auto deportivo de 1100 kg, estacionado en una colina, sin freno de mano aplicado, rodó hasta la base de la colina y se mueve a 20,0 m/s por un camino horizontal hacia el este. El conductor de un camión de 6500 kg, que viaja en sentido contrario, ve cómo el auto se aproxima y decide pararlo chocando de frente con él. Los dos vehículos quedan pegados después del choque. Si el camión se movía a 10,0 m/s cuando choca con el auto: a) ¿Qué velocidad (magnitud y dirección) tendrán los dos vehículos después de la colisión? b) ¿Cuál sería el cambio de energía del sistema de los dos vehículos durante la colisión?

6.2.- Una bala de 8,0 g se dispara horizontalmente a un bloque de madera

de 1,2 kg que descansa en una superficie horizontal. El coeficiente de fricción dinámica entre el bloque y la superficie es de 0,2. Si la bala queda incrustada en el bloque, que se desliza 0,30 m por la superficie antes de detenerse, ¿Qué rapidez tenía inicialmente la bala?

6.3.- Un bloque de plomo de 20,0 kg cuelga de un gancho, atado a un

alambre delgado de 3,5 m de longitud, y puede oscilar en un círculo completo. De repente, un dardo de acero de 5,0 kg lo golpea horizontalmente, incrustándose en él, ¿qué rapidez inicial mínima debe tener el dardo para que la combinación dardo-bloque describa un rizo completo después del impacto?

6.4.- Una bala de 6,0 g viaja horizontalmente con rapidez de 400 m/s y

choca con un bloque de madera de 1,2 kg que estaba en reposo en una superficie plana. La bala atraviesa el bloque y sale con una rapidez de 120 m/s. Si el bloque después del impacto desliza una distancia de 0,50 m sobre la superficie, a) ¿cuál es el coeficiente de fricción dinámico entre el bloque y la superficie? b) ¿qué tipo de colisión se produjo entre la bala y el bloque?

RESPUESTA A LAS PREGUNTAS:

Estas respuestas se corresponden con las preguntas presentadas en el

desarrollo de la unidad. Respuesta a la pregunta 6.1 No se cumplen las condiciones esbozada en la pregunta; lo que si cumple es

que además de que las fuerzas entre los cuerpos sean conservativas, la energía cinética del sistema es el mismo antes y después del choque.

Respuesta a la pregunta 6.2 En una colisión inelástica se conserva el momento lineal del sistema antes y

después de la colisión, pero la energía cinética del sistema antes y después son diferentes.

López Frontado, R. (2011). Física general I, Texto UNA. 2da edición. Caracas: UNA

Page 235: Fisica General i Una

247

RESPUESTA A LOS EJERCICIOS PROPUESTOS: Estos resultados se corresponden con los ejercicios propuestos presentados

en el desarrollo de la unidad. Respuesta al ejercicio propuesto 6.1 DATOS: mp = 55,0 kg, R = 2,5 m, Ii = 670,0 kg.m2, ωI = 2,0 rad/s. SOLUCIÓN: a) Para obtener la velocidad angular del sistema formado por la

plataforma y la persona, debemos considerar que la fuerza ejercida entre la persona y el piso de la plataforma está en dirección radial, por lo tanto, esta fuerza no ejerce momento de torsión, teniendo así que el momento angular del sistema con respecto al eje de rotación se mantiene constante, esto es,

i f i i f f

i plat f plat pers

2 2i f

2f

L =L Iω =Iω , dondeI =I e I =I +I

I =670,0 kg.m , I =670+55,0(2,5)

I =1013,8 kg.m

!r r

Entonces se tiene,

i if f

f

Iω 670,0×2,0 radω = = ω =1,32 I 1013,8 s

!

b) Al comparar las energías cinéticas rotatorias antes y después de la

caminata de la persona, se tiene

2f f

2rf rf

rf ri2 2ri ri i i

I ωE K 1013,8×(1,32)2= = = E =0,661EE K Iω 670×(2,0)

2

!

López Frontado, R. (2011). Física general I, Texto UNA. 2da edición. Caracas: UNA

Page 236: Fisica General i Una

248

Respuesta al ejercicio propuesto 6.2 DATOS: m = 30 g, M = 3,5 kg, µd = 0,2, d = 80 cm. SOLUCIÓN:

Figura 6.9 Para estudiar el movimiento de este sistema, veamos que sucede durante la

colisión en A. En la colisión sólo actúan fuerzas internas, por lo tanto el momento lineal del sistema se mantiene constante, esto es:

a d

o oo A A

P = Pv mv

mv = m +MV , = MV2 2

!

De A hasta el punto B, se tiene

mF 2A

2 222oAo 2

1K = W 0 - MV = - Mgd, 2

mv 8 gM dV 1d = = v =2 g 2 g 2M m

d

d

d d

µ

µµ µ

Δ

⎛ ⎞ →⎜ ⎟⎝ ⎠

22o o-3 2

8×0,2×9,8×0,8(3,5) mv = v =413,2 s(30×10 )

Respuesta al ejercicio propuesto 6.3 DATOS: m = 5,0 g, M = 0,8 kg, µd = 0,15, d1 = 0,5 m, Hm = 1,2 m SOLUCIÓN: Para resolver este problema, es necesario considerar lo que sucede en el

punto de impacto A; luego, el deslizamiento del bloque desde la posición en A hasta la posición en B; y por último el movimiento del bloque, como proyectil desde el punto B hasta el punto C, así se tiene que:

En A, se produce la colisión del proyectil con el bloque, manteniéndose

constante el momento lineal, así se tiene

a d o A

oA A

P =P , esto es mv =(m+M)Vmv 0,005 500 mV = = V =3,11 m+M 0,805 s

!"

r r

López Frontado, R. (2011). Física general I, Texto UNA. 2da edición. Caracas: UNA

Page 237: Fisica General i Una

249

En el movimiento de A hasta B, se tiene que FK=W µΔ .

2 2B A 1

2 2 2B A 1 B

1 1(m+M)V - (m+M)V =- (m+M)gd2 2

mV =V -2 gd =(3,11) -2 0,15 9,8 0,5 V =2,86 s

µ

µ × × × →

En el movimiento de B hasta C, el movimiento del bloque que podemos

considerar como un proyectil) está dado por:

B

2m

x=V t1y=H - gt2

!"#"$

El tiempo de vuelo se obtiene conociendo que yC = 0, así se tiene

2 mC m v

2H1y =H - gt , t =2 g

La distancia D se obtiene conociendo que xC=D, esto es,

mC B

2H 2 1,2x =D=V =2,86 D=1,41 mg 9,8

× ⇒

Respuesta al ejercicio propuesto 6.4 DATOS: m1 = 5 kg, m2 = 10 kg, HA = 5 m SOLUCIÓN: Para resolver el problema, en primer lugar analizamos el movimiento de m1

desde A hasta B. Como no existe roce, la energía mecánica se mantiene constante, esto es, EmA = EmB,

21 1o

1 A 1o A

1o

m vm gH v 2gH 2 9,8 5

2mv 9,9s

= → = = × ×

⇒ =

En B, se produce la colisión, así tenemos que

ox fx 1 1o 1 1f 2 2fP =P , m v =m v +m v (1)∴ Como el choque es frontal y elástico, el coeficiente de restitución

1o 2f 1fe=1, v =v -v (2)!

López Frontado, R. (2011). Física general I, Texto UNA. 2da edición. Caracas: UNA

Page 238: Fisica General i Una

250

Al sustituir los valores en las ecuaciones (1) y (2), se obtiene:

1f 2f 1f 2f

1f 2f 1f 2f

5 9,9=5v +10v 9,9=v +2v9,9=-v +v 9,9=-v +v× →

Resolviendo el sistema de ecuaciones, se obtiene:

1f 2fm mv 3,3 y v 6,6s s

= − =

En el movimiento de m1 desde B hasta D, se tiene: EmB = EmD

2 21 1f 1f

1 D D

2

D D

m v vm gH H

2 2g( 3,3)H H 0,56m2 9,8

= → =

−= ⇒ =×

Respuesta al ejercicio propuesto 6.5 DATOS: vbo = 450 m/s, vbf = 300 m/s, mb = 4 g = 4x10-3 kg,

mp = 0,1 kg SOLUCIÓN:

Figura 6.10

a) Durante una colisión, se mantiene constante el momento lineal del sistema, esto es, i fP=P

! !.

b bo b bf p pf

b bo b bfpf

p

ˆ ˆm v i m v j m vˆ ˆm v i m v j

vm

= +

−=

!

!

pf pf0.004 mˆ ˆ ˆ ˆv (450 i 300 j) v (18,1i 12,0 j)0.1 s

= − ⇒ = −r r

b) Para determinar si la colisión es elástica o no, es necesario obtener la

diferencia de Energía Cinética, que se produce.

22 2p pfb bf b bo

f o

2 2 2

m vm v m vK K K

2 2 2

1K 0,004(300) 0,1(21,53) 0,004(450)2

K 201,6 J

⎡ ⎤Δ = − = + −⎢ ⎥

⎢ ⎥⎣ ⎦

⎡ ⎤Δ = + −⎣ ⎦

⇒ Δ = −

Lo que nos indica que la colisión es inelástica.

López Frontado, R. (2011). Física general I, Texto UNA. 2da edición. Caracas: UNA

Page 239: Fisica General i Una

251

Respuesta al ejercicio propuesto 6.6 DATOS: m1 = 1,88 kg, v10 = 10,3 m/s, m2 = 4,92 kg, v20 = 3,27 m/s,

k = 11,2 N/cm SOLUCIÓN:

Figura 6.11

En una colisión entre dos cuerpos el momento lineal se conserva, esto se debe a que las fuerzas que intervienen en una colisión son fuerzas internas del sistema, esto es,

e

extx ox fx 1 1o 2 2o 1 2 f

F 2 2 2 21 2 f 1 1o 2 2o

F 0, P P , m v m v (m m )v (1)1 1 1 1K W (m m )v m v m v 0 k x (2)2 2 2 2

= → = + = +

Δ = + − − = − Δ

Al despejar vf de la ecuación (1), se tiene

1 1o 2 2o1 1o 2 2o 1 2 f f

1 2

f f

m v m vm v m v (m m )v , v

m m1,88 10,3 4,92 3,27 mv v 5,21

6,8 s

++ = + =

+× + ×= ⇒ =

La máxima compresión se obtiene a partir de la ecuación (2), así se tiene que

2 2 2 21 2 f 1 1o 2 2o

2 2 22 1 1o 2 2 1 2 f

k x (m m )v m v m v

m v m v (m m )vx

ko

Δ = − + + +

+ − +Δ =

2 2 22

2

1,88(10,3) 4,92(3,27) 6,8(5,21)x1120

199,5 52,61 184,58x x 0,246m1120

+ −Δ =

+ −Δ = ⇒ Δ =

Respuesta al ejercicio propuesto 6.7 DATOS: mb = 75 kg, Fµ = 300 N, mp = 20 kg, Ho = 4,0 m,

k = 4000 N/m

López Frontado, R. (2011). Física general I, Texto UNA. 2da edición. Caracas: UNA

Page 240: Fisica General i Una

252

SOLUCIÓN:

Figura 6.12

a) Para determinar la rapidez del bombero justo cuando llega a la plataforma, se analiza el movimiento del bombero desde A hasta B, esto es,

F

FB A

ΔE=W

E -E =W

µ

µ

2b bB b o o

o2bB o

b

bB

1m v -m gH =-F H2

2F H 300v =2gH - =2 9,8- 4,0m 75

m v =6,81 s

µ

µ ⎛ ⎞⎜ ⎟⎝ ⎠

b) Para determinar la compresión máxima del resorte, es necesario ver

que sucede, primero, durante la colisión del bombero con la plataforma en el punto B. Se conoce que durante la colisión se conserva el momento lineal del sistema, esto es,

( )i f b bB b p Bf

b bBBf Bf

b p

P=P m v = m +m v

m v 75×6,81 mv = = v =5,376 m +m 75+20 s

Luego se estudia el movimiento del sistema desde el punto B (justo después de la colisión) hasta su máxima compresión en el punto C. Esto es,

( ) ( )

( ) ( )

FC Bf

2 2b p b p Bf

22

2

E -E =W1 1kΔy - m +m gΔy- m +m v =-F Δy2 2

95 5,3764000 Δy - 95×9,8-300 Δy- =02 2

2000Δy -631Δy-1372,8=0 Δy=1,001 m

µ

µ

López Frontado, R. (2011). Física general I, Texto UNA. 2da edición. Caracas: UNA

Page 241: Fisica General i Una

253

RESPUESTA A LOS EJERCICIOS DE AUTOEVALUACIÓN Respuesta al ejercicio de autoevaluación 6.1 a) 5

fmˆv =-5,66i ; b) K=-4,235 10 Js

! "!

Respuesta al ejercicio de autoevaluación 6.2

omv =163,8 s

Respuesta al ejercicio de autoevaluación 6.3

omv =65,5 s

Respuesta al ejercicio de autoevaluación 6.4

da) =0,2; b) K=-478,4 J Inelástico endoérgico.µ Δ

López Frontado, R. (2011). Física general I, Texto UNA. 2da edición. Caracas: UNA

Page 242: Fisica General i Una

255

UNIDAD 7

CUERPOS RÍGIDOS

Hasta estos momentos se ha estudiado el comportamiento de partículas y sistemas de partículas, aplicando los conceptos y principios fundamentales que constituyen la mecánica clásica.

Como un ejemplo de sistema de partículas, donde se aplicarán estas leyes y

principios fundamentales que constituyen la mecánica clásica, se estudiará el sistema de cuerpos rígidos. Los cuerpos rígidos son sistemas de partículas en los cuales la distancia, entre dos puntos cualesquiera, se mantiene constante durante el movimiento.

El movimiento de un cuerpo rígido, considerado como un sistema de

partículas, se aborda aplicando el principio de la cinemática que permite considerar el movimiento del rígido en tres formas especialmente simples: traslación, rotación y roto-traslación. Además se considerarán las causas de estas traslaciones y rotaciones, estudiándolas separadamente. Para ello es necesario la aplicación de los conceptos de centro de masa y de momento de inercia.

También se estudiarán las situaciones en las que un cuerpo rígido se

encuentre aislado o donde la fuerza neta que actúe sobre él sea nula, condición conocida como de equilibrio.

CONOCIMIENTOS PREVIOS: Para la correcta comprensión de esta unidad, el estudiante debe tener los

siguientes conocimientos: 1.- Concepto de Centro de Masas (Unidad 5). 2.- Ecuaciones del movimiento (Unidad 5). 3.- Momento lineal y momento angular de sistemas de partículas (Unidad

5). 4.- Trabajo y energía de sistemas de partículas (Unidad 6).

López Frontado, R. (2011). Física general I, Texto UNA. 2da edición. Caracas: UNA

Page 243: Fisica General i Una

256

7.1 OBJETIVO Aplicar los conceptos de la dinámica de sistemas de partículas al

estudio de cuerpos rígidos.

7.2 RECOMENDACIONES PARA EL ESTUDIO DEL CONTENIDO En esta unidad, el estudiante debe comprender claramente cómo se aplican

los principios y leyes fundamentales en el estudio del comportamiento de los sistemas de partículas denominados Cuerpos Rígidos.

Para una mejor comprensión del contenido de la unidad, realice la lectura de

los conceptos relacionados con el movimiento de los cuerpos, aplicando las ideas estudiadas en el movimiento de sistemas de partículas y las leyes fundamentales de la mecánica en el estudio de los sistemas de esta unidad, y los ejemplos presentados, y utilice las técnicas de lectura que le faciliten la comprensión del tema.

7.3 CONTENIDO 7.2.1 DINÁMICA DE CUERPOS RÍGIDOS. 7.2.2 TIPOS DE MOVIMIENTOS DE CUERPOS RÍGIDOS: TRASLACIÓN,

ROTACIÓN Y ROTO-TRASLACIÓN. 7.2.3 TRABAJO Y ENERGÍA PARA CUERPOS RÍGIDOS.

7.2.4 EQUILIBRIO DE CUERPOS RÍGIDOS.

7.3.1 DINÁMICA DE CUERPOS RÍGIDOS Un Cuerpo Rígido es un sistema de partículas, donde la distancia entre cada

par de partículas se mantiene constante durante su movimiento. El movimiento de un cuerpo cualquiera (denominado Cuerpo Rígido) se

estudia aplicando los principios cinemáticos, los cuales permiten considerarlo como la combinación de dos movimientos especialmente simples: el de traslación y el de rotación.

Para iniciar el estudio del movimiento de traslación, consideremos el

concepto de centro de masa o centro de inercia de un cuerpo rígido, el cual es aquel punto cuya masa es igual a la masa del sistema, y su vector posición está dado por:

López Frontado, R. (2011). Física general I, Texto UNA. 2da edición. Caracas: UNA

Page 244: Fisica General i Una

257

CM1r = rdm (7.1)M !

! !

donde dm= dm= dVdV

! !"

El momento lineal de un cuerpo rígido está dado por:

CMP=Mv (7.2)r r

Lo que indica que el centro de masa del cuerpo rígido se mueve de igual

forma que se mueve una partícula material, cuya masa sea igual a la masa del cuerpo rígido.

La suma de las fuerzas externas que actúan sobre el cuerpo rígido está dado

por:

extCM

dPF = =Ma (7.3)dt∑r

r r

Esta ecuación es la 2ª ley de Newton para el movimiento de traslación de un

cuerpo rígido, llamada también 1ª ley fundamental de la mecánica. Para describir el movimiento de rotación de un cuerpo rígido, es necesario

adaptar el concepto de momento de inercia de un sistema de partículas, dado por la

ecuación n

2i i

i=1I= mr! , para determinar el de un cuerpo rígido, el cual queda expresado

matemáticamente por:

2I= r dm (7.4)! Entonces el momento angular que nos describe el movimiento de rotación de

un cuerpo rígido se puede expresar por:

L=I (7.5)ω! !

Al derivar el momento angular con respecto al tiempo, se tiene el momento de

todas las fuerzas que actúan sobre el cuerpo, esto es,

ext dL= =I (7.6)dt

! "#!

!!

López Frontado, R. (2011). Física general I, Texto UNA. 2da edición. Caracas: UNA

Page 245: Fisica General i Una

258

Esta ecuación es la 2ª ley de Newton aplicada al movimiento de rotación de un cuerpo rígido, llamada también 2ª ley fundamental de la mecánica.

Es importante aclarar que para considerar la ecuación (7.6) como

válida, es necesario que tanto el momento de torsión como el momento angular se evalúen con respecto al mismo sistema de referencia, esto es:

O CO C

dL dL= ó =dt dt

! !" "r r

r r

Donde el subíndice O significa que los momentos de fuerza y angular

se refieren al sistema inercial O, y el subíndice C significa que ambos se refieren a un sistema centro de masa CM.

7.3.2 TIPOS DE MOVIMIENTOS DE CUERPOS RÍGIDOS MOVIMIENTO DE TRASLACIÓN Un cuerpo rígido tiene un movimiento de traslación cuando se traslada de un

punto a otro. Consideremos el movimiento del cuerpo desde (a) hasta (b), los puntos del cuerpo describen trayectorias paralelas en la dirección de su movimiento, como se puede observar en la figura (7.1), o sea que las trayectorias AA’, BB’, y CC’ descritos por los puntos A, B y C del cuerpo en su movimiento son paralelas.

Figura 7.1

Las condiciones que se deben cumplir en este tipo de movimiento son:

CM

CMdVdPF= =M =Ma

dt dt=0 (7.7)!

"

"

rr r

r

MOVIMIENTO DE ROTACIÓN Un cuerpo rígido tiene movimiento de rotación cuando gira alrededor de un

eje, así se tiene que todos los puntos del cuerpo describen trayectorias circulares alrededor del eje, como se muestra en la figura (7.2), donde el cuerpo gira alrededor del punto C, y los puntos A y B describen trayectorias circulares alrededor de C, esto es:

López Frontado, R. (2011). Física general I, Texto UNA. 2da edición. Caracas: UNA

Page 246: Fisica General i Una

259

Figura 7.2

Las condiciones que deben cumplirse para este tipo de movimiento son las siguientes:

C C

F=0

dL dω= =I =I α (7.8)dt dt

!

"

"

r

r rrr

MOVIMIENTO DE ROTO-TRASLACIÓN Un cuerpo rígido tiene movimiento de roto-traslación cuando su movimiento

está compuesto de una traslación del cuerpo y una rotación alrededor de un eje; esto es que los puntos del cuerpo, además de describir trayectorias paralelas, también describen trayectorias circulares alrededor de un eje, como se muestra en la figura (7.3), en el caso de una rueda de radio R que rueda sobre una superficie horizontal, de tal forma que su centro C se desplaza horizontalmente hasta C’, y el punto A del borde de la rueda describe una trayectoria circular alrededor de su centro C.

Figura 7.3

Las condiciones que debe cumplir el cuerpo (en este caso, la rueda) para

realizar este tipo de movimiento son:

CM

C

F=Ma

=I α (7.9)τ∑∑

r r

rr

Un caso importante en el movimiento de traslación y rotación combinado es el

de rodar sin resbalar. En el movimiento de esta rueda, el punto (1) está en reposo instantemente para que no resbale. Si el radio de la rueda es R y su rapidez angular alrededor del centro de masa es ω , la magnitud de cmv =Rω , por tanto:

cmv =R (condición para rodar sin resbalar) (7.10)!

López Frontado, R. (2011). Física general I, Texto UNA. 2da edición. Caracas: UNA

Page 247: Fisica General i Una

260

Como se muestra en la figura (7.4),

Figura 7.4 En la figura (7.4a), la rueda entera se traslada con velocidad cmv

!; en la figura

(7.4b) la rueda gira en torno al centro de masa, rapidez en el borde = vcm; en la figura (7.4c) se tiene el rodamiento sin deslizamiento.

Considerando los conceptos presentados hasta los momentos, responda la

siguiente pregunta: Pregunta 7.1 Si un cuerpo rígido está girando con una rapidez angular ω < 0, siendo el

vector de velocidad angular y el de aceleración angular antiparalelos, entonces, ¿cómo es el comportamiento del movimiento angular del cuerpo?

Tomando en cuenta los conceptos emitidos hasta los momentos, responda la

siguiente pregunta. Pregunta 7.2 Pedro y Ana se divierten en un tiovivo (volantín). Pedro sube a un caballo que

está en el borde exterior de la plataforma circular, el doble de alejado del centro de la plataforma que Ana, quien pasea en un caballo que está más hacia el centro del aparato. Si el tiovivo está girando a una rapidez constante, ¿qué relación existe entre la rapidez tangencial de Pedro con respecto a la de Ana?

López Frontado, R. (2011). Física general I, Texto UNA. 2da edición. Caracas: UNA

Page 248: Fisica General i Una

261

7.3.3 TRABAJO Y ENERGÍA PARA CUERPOS RÍGIDOS Si el cuerpo rígido se está trasladando, su energía cinética se expresa por

2trasl CM

1K = MV (7.11)2

Si el cuerpo rígido está rotando, su energía cinética se expresa por

2rot C

1K = I ω (7.12)2

Si el cuerpo rígido está trasladándose y rotando, entonces su energía cinética

está expresada por

trasl rot

2 2CM C

K=K +K1 1K= MV + I ω (7.13)2 2

Aplicando el Teorema de Trabajo y Energía para un Cuerpo Rígido, se tiene

e m

nc nc

ii=1

F Fmg

ΔK= W = W + W

ΔK=W +W +W +.... (7.14)

! ! !

Si el cuerpo rígido cambia de altura al moverse, también se debe considerar la

energía potencial gravitacional, así se tiene que U=Mgycm (7.15) Suponiendo que una fuerza tangencial tanF

r actúa en el borde de un disco

pivoteado, el trabajo dW efectuado por tanFr

mientras un punto del borde se mueve una distancia ds es tan tandW=F ds=F Rd , donde ds=Rdθ θ , y conociendo que

tan zF R es el momento de torsión τ , se tiene que el trabajo efectuado por un momento de torsión está dado por:

2

1zW= d , (7.16)

!

!" !#

López Frontado, R. (2011). Física general I, Texto UNA. 2da edición. Caracas: UNA

Page 249: Fisica General i Una

262

Si el momento de torsión es constante y el cambio de ángulo 2 1= -! ! !" , se tiene el trabajo efectuado por un momento de torsión constante, el cual está dado por:

2 1W= ( - )= (7.17)! " " ! "#

Si ahora tomamos la rapidez con que se realiza trabajo o sea la potencia P,

se tiene

z z zdW dP= = , P= (7.18)dt dt

θτ τ ω

Considerando los conceptos descritos hasta los momentos, responda las

siguientes preguntas. Pregunta 7.3 Una sección de tubo hueco y un cilindro sólido tienen radio, masa y longitud

iguales. Si ambos cuerpos giran alrededor de sus largos ejes centrales con la misma rapidez angular, ¿cuál de los cuerpos tiene mayor energía cinética de rotación?

Pregunta 7.4 Dos esferas ruedan hacia abajo por un plano inclinado, iniciando desde el

reposo. Si la esfera A, la cual tiene la misma masa y radio que la esfera B, pero la esfera A es sólida mientras que la B es hueca, ¿cuál llega primero a la parte más baja del plano inclinado?

7.3.4 EQUILIBRIO DE CUERPOS RÍGIDOS Las condiciones que deben satisfacer un cuerpo rígido para que esté en

equilibrio, son: F=0

=0 (7.19)!""

r

r

Es bueno aclarar la diferencia que existe entre equilibrio y reposo. Un

cuerpo está en equilibrio cuando se cumplen las condiciones de las ecuaciones (7.19), pero además puede tener un movimiento uniforme; mientras que un cuerpo está en reposo cuando cumple las condiciones de las ecuaciones (7.19), pero no está en movimiento.

López Frontado, R. (2011). Física general I, Texto UNA. 2da edición. Caracas: UNA

Page 250: Fisica General i Una

263

Tomando en cuenta los conceptos emitidos hasta los momentos, responda la

siguiente pregunta. Pregunta 7.5 Si un cuerpo está sometido a dos fuerzas iguales y opuestas aplicada en

puntos simétricos respecto al centro de masa, se puede afirmar que el cuerpo está: a) en equilibrio de fuerzas y de par de torsión; b) en equilibrio de fuerzas pero no en equilibrio de par de torsión; c) en equilibrio de par de torsión pero no en equilibrio de fuerzas; d) no está en equilibrio de fuerzas ni de par de torsión:

A continuación se presentan unos ejemplos de problemas, donde Ud. podrá

visualizar las aplicaciones de los conceptos tratados en esta unidad. Pero antes vea las Sugerencias y las Recomendaciones presentadas para resolver estos problemas.

RECOMENDACIONES PARA RESOLVER PROBLEMAS DE DINÁMICA

DEL SISTEMA DE PARTÍCULAS DENOMINADO CUERPOS RÍGIDOS. La estrategia para resolver problemas del movimiento de cuerpos rígidos es

muy similar a la presentada para resolver problemas del movimiento de partículas.

IDENTIFICAR los conceptos relevantes: Las ecuaciones extCM

dPF = =Mvdt!!

! ! y

zz

dL= =Idtzτ α∑r

rr , son útiles en todos los tipos de problemas que describan los

movimientos de cuerpos rígidos, sea por ejemplo el movimiento de traslación pura, de rotación pura, de roto-traslación y en el estado de equilibrio, aplicando por supuesto las condiciones pertinentes para cada uno de esto movimientos.

PLANTEAR el problema usando estos pasos: a) Haga un dibujo de la situación y escoja el cuerpo o cuerpos que

estudiará. b) Dibuje un diagrama del cuerpo libre para cada cuerpo, aislando el

cuerpo e incluyendo todas las fuerzas que actúan sobre él. Una nueva

López Frontado, R. (2011). Física general I, Texto UNA. 2da edición. Caracas: UNA

Page 251: Fisica General i Una

264

consideración es que debe mostrar con exactitud la forma del cuerpo, incluyendo todas las dimensiones y ángulos que necesitará para los cálculos del momento de torsión.

c) Escoja ejes de coordenadas para cada cuerpo indicando un sentido positivo de rotación positivo para cada cuerpo que gire. Si hay una aceleración lineal, lo más sencillo es escoger un eje positivo en esa dirección. Si se conoce el sentido de zα , se simplifican los cálculos al escogerse ése sentido como sentido de rotación positivo. Si se representa una fuerza en función de sus componentes, es conveniente tachar la fuerza original para no incluirla dos veces.

EJECUTAR la solución como sigue:

1.- Para cada cuerpo del problema, decida si su movimiento es

traslacional, rotacional o ambos. Dependiendo del comportamiento del

cuerpo aplique extCM

dPF = =Mvdt!!

! ! y zz

dL= =Idtzτ α∑r

rr . Escriba ecuaciones

apartes para cada cuerpo. 2.- Si existen relaciones geométricas entre los movimientos de dos o más

cuerpos, como una cuerda que se enrolla en una polea girándola, exprésela en forma algebraica, como por ejemplo las relaciones entre dos aceleraciones lineales o una aceleración lineal y una angular.

3.- Verifique que el número de ecuaciones coincida con el número de

incógnitas. Resuelva el sistema de ecuaciones para obtener la incógnita o las incógnitas.

EVALUAR la respuesta: compruebe que los signos algebraicos de sus

resultados sean lógicos. Siempre que pueda, verifique los resultados para casos especiales o valores extremos y compárelos con lo que espera intuitivamente.

Recuerde que el manejo eficiente del contenido de esta unidad es

importante, por su repercusión en el contenido de las subsiguientes asignaturas del Plan de Estudios en Ingeniería y otras ciencias.

Ejemplo 7.1.- (Movimiento de traslación de un cuerpo rígido) Un bloque (paca) de paja de 25 kg que mide 0,75 m de largo, se mueve sobre

una superficie horizontal con una rapidez constante proporcionada por una fuerzaF!.

Si el coeficiente de fricción dinámico entre el bloque y la superficie es 0,25:

López Frontado, R. (2011). Física general I, Texto UNA. 2da edición. Caracas: UNA

Page 252: Fisica General i Una

265

a) Calcule la magnitud deF

r.

b) Determine el valor de h con el cual la paca apenas comenzará a

volcarse.

DATOS: M = 25 kg, largo = 0,75 m, ancho = 0,25 m, alto = 0,50 m, µd = 0,25

SOLUCIÓN: IDENTIFICAR: se usan las condiciones de traslación de un cuerpo rígido,

tomando en cuenta que la suma resultante de las fuerzas que actúan sobre el mismo es igual al producto de la masa del cuerpo por la aceleración del centro de masa del mismo; y la suma del momento de torsión es igual a cero. Como el bloque se mueve en la superficie horizontal con una rapidez constante, se tiene que la aceleración del centro de masa es cero.

PLANTEAR y EJECUTAR: tomando en cuenta los DCL del cuerpo mostrado

en la figura (7.5).

Figura 7.5

DCL

a) Para determinar la fuerza F, usemos la figura (7.5a), de donde tenemos

que

x cm cm cm

y

F : F-F = Ma , pero a =0 debido a que la v =constante

F : N-Mg = 0µ∑

De donde se obtiene que

dF=F = Mg=0,2×25×9,8µ µ F=61,25 N!

López Frontado, R. (2011). Física general I, Texto UNA. 2da edición. Caracas: UNA

Page 253: Fisica General i Una

266

b) Para determinar la altura H, usamos la figura (7.5b), de donde se tiene,

x

y

A

F : F- F = 0

F : N- Mg = 0

a= 0 - FH+Mg = 0, de donde se tiene 2

Mga 25×9,8×0,25que H = = H = 0,5 m2F 2×61,25

µ

τ

∑∑∑

EVALUAR el resultado: la fuerza necesaria para mover la paca de paja es de

61,25 N. La altura a la cual se debe aplicar la fuerza para que se voltee es de 0,5 m.

Ejemplo 7.2.- (Movimiento de rotación de un cuerpo rígido) Un cuerpo, cuyo peso es de 50,0 N, se une al extremo libre de una cuerda

ligera enrollada alrededor de una polea de 0,25 m de radio y 3,0 kg de masa. La polea es un disco sólido, puede girar libremente en un plano vertical en torno del eje horizontal que pasa por su centro. El cuerpo se libera 6,0 m sobre el piso. Con base en lo expuesto, determine:

a) La tensión en la cuerda, la aceleración de la masa.

b) La energía cinética del cuerpo cuando golpea el piso.

El momento de inercia de la polea es: 2MRI

2=

DATOS: mg = 50,0 N, R = 0,25 m, M = 3,0 kg, Ho = 6,0 m SOLUCIÓN: IDENTIFICAR: en este problema se aplican los conceptos de momento de

inercia y de momento de torsión para estudiar el movimiento de rotación de un cuerpo. También se usa la relación que existe entre una aceleración angular y una aceleración tangencial.

PLANTEAR el problema: se hace un gráfico del movimiento del cuerpo,

representado en la figura (7.5) y su respectivo diagrama de Cuerpo Libre. Luego se establecen las establecen las leyes de Newton que describen cómo se produce el movimiento de cada cuerpo, determinando primero la tensión de la cuerda y la aceleración angular con que gira la cuerda, tomando en cuenta la relación entre la aceleración angular del disco con la aceleración lineal del cuerpo mientras desciende

López Frontado, R. (2011). Física general I, Texto UNA. 2da edición. Caracas: UNA

Page 254: Fisica General i Una

267

en su movimiento. Posteriormente se obtiene la energía cinética del cuerpo cuando golpea el piso aplicando el teorema de trabajo y energía.

Figura 7.6

EJECUTAR la solución del problema: Se aplican y se resuelven las leyes de

Newton a cada cuerpo, en la forma siguiente: a) Aplicando las leyes de Newton a la polea y al cuerpo, se tiene:

Polea Cuerpo o

2

o

TR = I mg- T = ma

MRPero I = y a = R2

!

!

Entonces, la tensión T de la cuerda es:

2T 2T = , mg- T = mR = mRMR MR

mg 50,0T = = T = 11,36 N2m 2×50,01+ 1+M 9,8×3,0

! !

"

La aceleración a de la masa es:

2

2T 2×11,36 ma = R = R = a = 7,57 MR 3,0 s

α ⇒

b) Para determinar la energía del cuerpo cuando llega al suelo, se obtiene

previamente la velocidad del cuerpo usando el teorema de trabajo energía, esto es:

mgf i o

222o f of

o f o2

K = W , K - K = mgH

I w Imv 1+ - 0 = mgH , m+ v = mgH2 2 2 R

!

" #$ %& '

López Frontado, R. (2011). Física general I, Texto UNA. 2da edición. Caracas: UNA

Page 255: Fisica General i Una

268

2 o2mgH 2×50,0×6 mv = = v = 9,53 M 3 sm+ 5,1+2 2

!" # " #$ % $ %& ' & '

La energía cinética del cuerpo cuando llega al suelo, es

( )22

ff f

5,1× 9,53mvK = = K = 231,8 J

2 2⇒

EVALUAR la solución: la tensión de la cuerda es de 11,36 N, la aceleración

con la cual desciende el cuerpo es de 7,57 m/s2. La energía cinética con que llega el cuerpo al suelo es de 231,8 J.

Ejemplo 7.3.- (Movimiento de roto-traslación de un cuerpo rígido) Si un automóvil que viaja a 78,3 km/h tiene ruedas de 77,0 cm de diámetro,

determine: a) ¿Cuál es la velocidad angular de las ruedas con respecto al eje?

b) Si el automóvil se detiene uniformemente en 28,6 vueltas de las ruedas

(sin patinar), ¿cuál será la aceleración angular de las ruedas?

c) ¿Cuánto avanza el automóvil durante este período de frenado? DATOS: R = 77,0/2 cm, v = 78,3 km/h = 21,75 m/s, n = 28,6 vueltas.

SOLUCIÓN: IDENTIFICAR: en este problema se usan los conceptos relacionados con el

movimiento de rotación de un cuerpo, aplicando los conceptos de velocidad angular y desplazamiento angular del mismo.

PLANTEAR: para la resolución del problema es necesario considerar la

relación entre la velocidad angular y la velocidad lineal, como también el desplazamiento angular del cuerpo.

EJECUTAR la resolución del problema: a) La velocidad angular v 21,75×2 rad= = = 56,49

R 0,77 s! !"

López Frontado, R. (2011). Física general I, Texto UNA. 2da edición. Caracas: UNA

Page 256: Fisica General i Una

269

b) Para determinar la aceleración angular de las ruedas, es necesario determinar el ángulo total girado por las ruedas, considerando que no patina. Esto es,

= 2 n = 2 ×28,8 = 179.7 radθ π π θ→

También se conoce que

of o f= - t, = 0, entonces t =

!! ! " !

"

22o

0

2 2o

2

t= t- , sustituyendo t, se tiene =2 2(56,49) rad= = = 8,88

2 2×179,7 s

ωαθ ω θα

ωα αθ

c) La distancia que avanza la rueda es

0,77×179,7D = R = D = 69,19 m

2! "

EVALUAR los resultados del problema: La velocidad angular de rotación de la rueda es de 56,59 rad/s; la aceleración

angular de las ruedas, cuando la misma ha dado 28,8 vueltas es de 8,88 rad/s2; y la distancia que avanza la rueda es de 69,19 m.

Ejercicio propuesto 7.1.- Se ejerce un momento de torsión neto constante de 20,0 N.m sobre una rueda

pivotante durante 8,0 s, aumentando la velocidad angular de la rueda de 0 a 100 rpm (revoluciones por minutos). Luego se deja de aplicar el par externo y la fricción de los cojinetes de la rueda la detienen en 70,0 s. Calcule:

a) El momento de inercia de la rueda alrededor del eje de rotación.

b) El momento de torsión de fricción.

c) El número de revoluciones que da la rueda en ese lapso de 70,0 s.

López Frontado, R. (2011). Física general I, Texto UNA. 2da edición. Caracas: UNA

Page 257: Fisica General i Una

270

Ejercicio propuesto 7.2.- Un volante en la forma de un cilindro sólido de radio R = 0,6 m y masa M = 15

kg, puede llevarse hasta una velocidad angular de 12 rad/s en 0,6 s, por medio de un motor que ejerce un momento de torsión constante. Después de que el motor se apaga, el volante efectúa 20 revoluciones antes de detenerse por causa de la fricción (supuesta constante durante la rotación).

¿Qué porcentaje de la potencia generada por el motor se emplea para vencer

la fricción?

Ejercicio propuesto 7.3.- Un cilindro macizo, de masa 2,0 kg y radio 5 cm, rueda sin deslizamiento por

un plano inclinado 30º. El momento de inercia IC = MR2/2. Suponiendo que el cilindro partió del reposo y que no hay rozamiento, determine:

a) El momento de inercia respecto a la generatriz de contacto con el piso.

b) La aceleración lineal del CM en el descenso.

c) Su velocidad después de recorrer una distancia de 3 m en el plano..

Ejercicio propuesto 7.4 En una máquina Atwood un bloque tiene una masa de 512 g y el otro una

masa de 463 g. La polea que está montada en chumaceras horizontales sin fricción, tiene un radio de 4,9 cm. Cuando es liberada a partir del reposo, se observa que el bloque más pesado cae 76,5 cm en 5,11 s. Calcule la inercia de rotación de la polea.

López Frontado, R. (2011). Física general I, Texto UNA. 2da edición. Caracas: UNA

Page 258: Fisica General i Una

271

Ejercicio propuesto 7.5 Dos bloques, de masas m1 = kg y m2 = 20 kg, están conectados, como se

muestra en la figura, por una cuerda de masa despreciable que pasa a través de una polea de radio 0,25 m, y momento de inercia I. El bloque sobre el plano inclinado liso, cuyo ángulo θ = 37º, se mueve hacia arriba con aceleración constante de 2 m/s2.

a) Determine las tensiones T1 y T2 en las dos partes de la cuerda.

b) Encuentre el momento de inercia I de la polea.

Ejercicio propuesto 7.6 En la figura se muestra,

Figura 7.7

Una pequeña esfera de masa m y de radio ro = 1,5 cm, que rueda sin resbalar sobre la pista mostrada cuyo radio Ro = 26,0 cm. La esfera empieza a rodar a una altura Ro arriba del fondo de la pista, y abandona la pista cuando pasa por el punto B, donde θ = 35º.

En el momento que la esfera abandona la pista, obtenga: a) Su rapidez.

b) La distancia D, desde la base de la pista, a la que toca el suelo.

López Frontado, R. (2011). Física general I, Texto UNA. 2da edición. Caracas: UNA

Page 259: Fisica General i Una

272

Ejemplo 7.4.- (Equilibrio de cuerpo rígido) Un limpiador de ventanas de 74,6 kg usa una escalera de mano de 10,3 kg

que tiene 5,12 m de largo. Sitúa un extremo a 2,45 m de una pared y descansa la parte superior contra una ventana, cuyos vidrios están quebrados y sube por la escalera. Cuando llega a 3,10 m se rompe la ventana. Despreciando la fricción entre la escalera y la ventana, y suponiendo que la base de la escalera no desliza, calcule justo antes de que se rompa la ventana:

a) La fuerza ejercida sobre la ventana por la escalera.

b) La magnitud y dirección de la fuerza ejercida sobre la escalera por el

suelo. DATOS: mL = 74,6 kg, M = 10,3 kg, L = 5,12 m, D = 2,45 m,

LL = 3,10 m. SOLUCIÓN: IDENTIFICAR: en este problema el sistema limpiador escalera es estacionario

(en reposo) por lo que se usan las condiciones de equilibrio Se usan también los conceptos de fuerza normal, coeficiente de roce estático. La fuerza de contacto en la parte inferior es la suma vectorial de la fuerza normal y la fuerza de fricción (fuerza horizontal) que evita el deslizamiento de la escalera sobre la superficie..

PLANTEAR: Se elabora un diagrama de cuerpo libre (DCL), expresando las

ecuaciones correspondientes a las condiciones de equilibrio, resolviendo dichas ecuaciones.

Figura 7.8

Aplicando las condiciones de equilibrio para un cuerpo rígido, se tiene,

x A x

y y L

F =F -F =0

F =F -Mg-m g=0

!!

B L ALτ =0, -mgL cos -Mg cos +F Lsen =02

! ! !"

donde

( )-1 -1

o

2,45 =cos =cos 0,479 5,12

=61,41

θ

θ

⎛ ⎞⎜ ⎟⎝ ⎠

López Frontado, R. (2011). Física general I, Texto UNA. 2da edición. Caracas: UNA

Page 260: Fisica General i Una

273

EJECUTAR la solución del problema: a) Para determinar FA, ejercida por la ventana sobre la escalera,

despejamos de la ecuación del momento de torsión, así se tiene,

( )

L L

A

o

A o

A

Lm L +M gcosθ2F =

Lsenθ74,6×3,10+10,3×2,56 9,8cos61,41 1208,1F = =

4,49595,12sen61,41 F =268,71 N

⎛ ⎞⎜ ⎟⎝ ⎠

b) Para determinar la fuerza que el suelo ejerce sobre la escalera, se

obtiene previamente Fx y Fy, esto es,

( ) ( )x A x

y L y

B x y B

F -F =0, F =268,71 NF = M+m g= 10,3+74,6 9,8 F =832,02 N

ˆ ˆ ˆ ˆF =F i+F j F =(268,71i+832,02j) N

→→

⇒r r

La magnitud y la dirección de FB son:

( ) ( )2 22 2

B x y B

-1 o

F = F +F = 268.71 + 832,02 F =874,34 N

832,02φ=tan φ=72,1268,71

⎛ ⎞ ⇒⎜ ⎟⎝ ⎠

EVALUAR los resultados del problema: La fuerza aplicada por la ventana sobre la escalera es la fuerza normal que

ejerce la ventana sobre la escalera FA= 268,71 N. La fuerza ejercida por el piso sobre la escalera está dada por B

ˆ ˆF =(268,71i+832,02j) N!

, donde su magnitud y su dirección son 874,34 N y 72,1º, respectivamente.

Ejercicio propuesto 7.7 Un pescante uniforme AB de 1200 N se sostiene por un cable, como se

muestra en la figura. El pescante se sujeta con un perno en la parte inferior, y en la parte superior se cuelga un cuerpo de 2000 N. Encuentre:

a) La tensión en el cable.

López Frontado, R. (2011). Física general I, Texto UNA. 2da edición. Caracas: UNA

Page 261: Fisica General i Una

274

b) Las componentes de la fuerza de reacción en el perno.

Ejercicio propuesto 7.8 Una varilla uniforme de 4 m de largo y peso W se sostiene por una cuerda en

un extremo. El otro extremo descansa en una pared vertical donde se mantiene debido a la fricción. El coeficiente de fricción estático entre la pared y la varilla es µe = 0,5.

Encuentre la distancia más cercana al punto A, donde se puede colgar un peso extra W, sin que la varilla resbale de A.

Ejercicio propuesto 7.9 En el sistema mostrado, el puntal AB es uniforme y tiene una longitud L y

pesa 200 N. El objeto suspendido por el pivote también pesa 200 N. Calcule: a) La tensión T en cada cable.

b) La fuerza (magnitud y dirección) ejercida sobre el puntal por el pivote.

Ejercicio propuesto 7.10

Figura 7.9

En la figura se muestra una viga uniforme AB, de masa mv = 20 kg y longitud L = 4 m, soporta bloques de masas m1 = 30 kg y m2 = 70 kg en dos posiciones, como muestra la figura. La viga descansa en dos puntos A y C. Como en el punto C, situado a una distancia de 2,5 m desde A, existe un eje suave la viga puede girar alrededor del punto C. Con base en lo expuesto, obtenga el valor de la máxima distancia x, en la cual se puede colocar el bloque m2 sin romper el equilibrio de la viga.

López Frontado, R. (2011). Física general I, Texto UNA. 2da edición. Caracas: UNA

Page 262: Fisica General i Una

275

Ejercicio propuesto 7.11

Figura 7.10

Como se muestra en la figura, de cada extremo de una cuerda que pasa por la garganta de una polea de 20 cm de radio, cuelgan dos cuerpos de 3,0 kg y de 7 kg de masa, respectivamente. La polea puede girar alrededor de un eje horizontal. Si la masa de la polea es de 2 kg, calcule: a) La aceleración lineal del movimiento del sistema. b) El momento angular total del sistema respecto a su eje cuando la velocidad de los cuerpos es de 5 m/s.

RESUMEN El movimiento de los cuerpos

rígidos puede ser de traslación, o de rotación, o la combinación de ambos. La relación dinámica para el movimiento traslacional está dada por la suma resultante de las fuerzas que debe ser igual al cambio de momento lineal del cuerpo. La relación dinámica para el movimiento rotacional está dada por la suma de los momentos de torsión de un cuerpo el cual debe ser igual al cambio del momento angular del mismo.

Para que un cuerpo rígido esté en

equilibrio es necesario que se cumplan dos condiciones: que la suma resultante de las fuerzas debe ser cero; y que la suma de los momentos de torsión respecto a cualquier punto debe ser cero.

Ecuaciones de movimiento de un cuerpo rígido:

dPF= (1),dtdL= (2)dt

!

"

"

!!

!!

Condiciones de equilibrio del cuerpo rígido:

F=0, =0 (3)!" "! !

López Frontado, R. (2011). Física general I, Texto UNA. 2da edición. Caracas: UNA

Page 263: Fisica General i Una

276

Como el movimiento del cuerpo

puede ser la combinación del traslacional y del rotacional, la energía cinética se puede expresar como la suma de la energía cinética de traslación más la de rotación. También se puede aplicar a la dinámica la segunda ley de Newton que describe el movimiento del centro de masa y el equivalente rotacional de esa ley que describe la rotación en torno al centro de masa. Si el cuerpo rueda sin resbalar, la relación especial entre el movimiento del centro de masa y el movimiento rotacional está dada por la ecuación (7).

2 2cm cm

cm

z cm z

cm

1 1K= Mv + I (4)2 2F=Ma (5)

=I (6)v =R (7)(rodamiento sin deslizamiento)

ω

τ αω

∑∑

! !

!!

EJERCICIOS DE AUTOEVALUACIÓN En su libreta desarrolle los siguientes ejercicios. Luego de resolver los

ejercicios, compare sus respuestas con las presentadas en la última página de la unidad. En caso de no acertar, tener alguna duda, consulte con sus compañeros o con el asesor de su centro local.

7.1.- Un bloque de 5,0 kg descansa sobre una superficie horizontal sin

fricción. Un cordón sujeto al bloque pasa por una polea fija de 0,12 m de diámetro y se ata a un bloque colgante de 8,0 kg. El sistema se libera del reposo y los bloques se mueven 3,0 m en 2,0 s.

a) ¿Qué tensión hay en cada parte del cordón? b) ¿Qué momento de inercia tiene la polea sobre su eje de rotación? 7.2.- Un estudiante sentado en un banquito que gira libremente sostiene dos

pesas, cada una de las cuales tiene una masa de 3,0 kg. Cuando sus brazos se extienden de manera horizontal, las pesas están a 1,0 m del eje de rotación y él gira a una rapidez angular de 0,75 rad/s. El momento de inercia del estudiante más el banquillo es de 3,0 kg.m2 y se supone constante. El estudiante lleva las pesas hacia su cuerpo horizontalmente a una posición de 30 cm del eje de rotación. Con base en lo expuesto, encuentre:

a) La nueva rapidez angular del estudiante. b) La energía cinética del estudiante antes y después de que lleve las

pesas hacia su cuerpo.

López Frontado, R. (2011). Física general I, Texto UNA. 2da edición. Caracas: UNA

Page 264: Fisica General i Una

277

7.3.-

Figura 7.11

La figura muestra una cuerda ligera conectada en un extremo a un bloque, de masa m = 3,0 kg, que puede deslizarse sobre un plano inclinado; tiene su otro extremo enrollado alrededor de un cilindro, de masa M = 30 kg y de radio R = 0,2 m, que descansa en una depresión en la parte superior del plano. El coeficiente de fricción entre todas las superficies es µ = 0,035. Con base en lo antes expuesto, determine la rapidez del bloque después que ha descendido una distancia d = 1,8 m a lo largo del plano inclinado, partiendo del reposo.

7.4.-

Figura 7.12

La figura muestra una puerta de una cochera, uniforme y de peso 800 N, que está montada sobre un riel superior. Las ruedas de la puerta en A y B se oxidaron, de modo que no ruedan, sino que deslizan sobre el riel (el coeficiente de fricción dinámico entre las ruedas y el riel µd = 0,45). Cada rueda está a 0,5 m del borde vertical de la puerta. Una fuerza horizontal F

! aplicada sobre la puerta,

la empuja hacia la izquierda con una rapidez constante. Con base en lo expuesto:

a) Determine la componente vertical de la fuerza que ejerce el riel sobre cada rueda, si la distancia h = 1,50 m.

b) Calcule el valor máximo que puede tener h sin causar que una rueda se

levante del riel. 7.5.-

Figura 7.13

La figura muestra un anillo de 2,4 kg de masa, de 0,6 cm de radio interior y 0,8 cm de radio exterior, que sube rodando (sin deslizarse) por un plano cuya inclinación forma un ángulo θ = 37º con la horizontal. En el momento en que el anillo está en la posición d = 2,0 m al ascender por el plano, su rapidez es v = 2,8 m/s. El anillo continúa ascendiendo por el plano cierta distancia adicional y después rueda hacia abajo. En base a lo antes expuesto, determine:

López Frontado, R. (2011). Física general I, Texto UNA. 2da edición. Caracas: UNA

Page 265: Fisica General i Una

278

a) La distancia x que asciende por el plano antes de detenerse.

b) La rapidez con que pasa de regreso por el punto más bajo del plano en A.

El momento de Inercia del anillo es: IC = M(Ri

2 + Re2)/2.

RESPUESTA A LAS PREGUNTAS Estas respuestas se corresponden con las preguntas presentadas en el

desarrollo de la unidad. Respuesta a la pregunta 7.1 El movimiento angular del cuerpo es retardado, debido a que los vectores de

velocidad angular y de aceleración angular antiparalelos, esto quiere decir que la aceleración angular hace que la rapidez angular vaya decreciendo.

Respuesta a la pregunta 7.2 El sistema de la plataforma, Pedro y Ana es un cuerpo rígido, de forma que

todos los puntos sobre el cuerpo tienen la misma rapidez angular, y la rapidez tangencial depende de sus distancias radiales desde el eje de rotación.

Respuesta a la pregunta 7.3 El tubo hueco, debido a que tiene mayor momento de inercia que el tubo

sólido. Respuesta a la pregunta 7.4 El momento de inercia de la esfera sólida A es menor que la de esfera hueca

B, por lo tanto el centro de masa de B tiene menor rapidez y la esfera A llega primero.

Respuesta a la pregunta 7.5 El cuerpo está en equilibrio de fuerzas pero no en el par de torsión, ya que se

produce sobre él un movimiento de rotación tomando como eje el centro de masa del mismo.

López Frontado, R. (2011). Física general I, Texto UNA. 2da edición. Caracas: UNA

Page 266: Fisica General i Una

279

RESPUESTA A LOS EJERCICIOS PROPUESTOS Estas respuestas corresponden con los ejercicios propuestos en el desarrollo

de la unidad. Respuesta al ejercicio propuesto 7.1 DATOS: τ = 20.0 N.m, t1 = 8,0 s, ωo = 0 1

2 rad=100 rpm =20 60 s!

" !

t2 = 70,0 s SOLUCIÓN: En este problema se estudia el movimiento de rotación con aceleración

angular variable bajo el efecto del momento de torsión debido a la fuerza de fricción que actúa sobre el cuerpo, el momento de inercia y la velocidad angular y el desplazamiento angular.

a) Cuando un cuerpo gira alrededor de un eje, su C=I! "

!! .

Pero 11 1 1 1 1 2

1

20 rad= t , = = =2,5 t 8 sω πω α α α π→

El momento de Inercia 2

C C1

20I = = I =2,55 kg.m2,5

τα π

El momento de torsión de fricción, es F C 2=I

µτ α , pero se sabe que al detenerse

la rueda f 1 2 2=0= - t ! ! " , 12 2 2

2

20 rad= = =0,898 t 70 sω πα α→

F F=2,55 0,898 =2,29 N.mµ µ

τ τ× ⇒ b) Para determinar el número de revoluciones dada por la rueda durante el

lapso de 70 s, se tiene

2 22 2

1 2t 0,898(70)= t - =20 70- =2198,1 rad

2 2!

" # $ "% &

2198,1=2 n, n= = n=349,8 vueltas2 2!

! "" "

#

Respuesta al ejercicio propuesto 7.2 DATOS: R =0,6 m, M = 15 kg, ω = 12 rad/s,

Δt = 0,6 s, n = 20 rev.

López Frontado, R. (2011). Física general I, Texto UNA. 2da edición. Caracas: UNA

Page 267: Fisica General i Una

280

SOLUCIÓN: En este problema se analiza el movimiento producido por un motor sobre un

volante, el cual actúa durante cierto tiempo y le proporciona un movimiento circular uniformemente acelerado; y a continuación deja de actuar el motor, y desde este instante el volante se mueve con un movimiento uniformemente retardado debido al momento de torsión proporcionado por la fuerza de fricción que actúa sobre él.

Por esa razón se estudia el movimiento de este volante en dos etapas: una es

la etapa de aceleración y la otra es la desaceleración. Para en última instancia determinar el porcentaje de potencia desarrollada por el motor.

Figura 7.14

Durante la aceleración (entre 0 y 0,6 s),

1 1 2

neto M F C 1 C

12-0 rad= = =20 t 0,6 s

= - =I =20Iµ

ωα α

τ τ τ α

Δ →Δ

Durante la desaceleración (después de 0,6 s hasta detenerse),

2 2f o 2

2 2 2f o

2 2 2

F C 2 F C

= +2 , donde =2 n=40

- 0-(12) rad= = =-0,573 2 2 40 s

=I =- 0,573Iµ µ

ω ω α θ θ π πω ωα α

θ πτ α τ

Δ Δ

→Δ ×

Sea PF = potencia disipada por fricción, F FP =

tµ µ

θτ ΔΔ

Sea PM = potencia generada por el motor, MP = tM

!"

##

El porcentaje de la potencia generada por el motor empleada para vencer la

fricción, será

F F FC

M M C M

P P0,573I 100100= 100= 100=2,79%

P (20+0,573)I Pµ µ µ

ττ

×× × ⇒ ×

López Frontado, R. (2011). Física general I, Texto UNA. 2da edición. Caracas: UNA

Page 268: Fisica General i Una

281

Respuesta al ejercicio propuesto 7.3 DATOS: m = 2,0 kg, R = 0,05 m, θ = 30º , d = 3 m.

Figura 7.15 SOLUCIÓN: En este problema se determina el momento de inercia del cilindro aplicando el

teorema de ejes paralelos de Steiner; también se usa la ecuación de Newton relacionada con el movimiento de rotación de un cuerpo, para determinar la aceleración lineal del centro de masa del cuerpo; y por último se usa el teorema de trabajo y energía para obtener la velocidad del centro de masa del cilindro.

a) El momento de inercia con respecto a la generatriz, aplicando el

teorema de Steiner, está dado por:

2 22 2

A C

23 2

A A

MR 3MRI I MR MR2 2

3 2,0 (0,05)I I 7,5 10 kg.m2

= + = + =

× ×= ⇒ = ×

b) Para determinar la aceleración del centro de masa del disco en el

descenso, se aplica la 2ª ley fundamental de la dinámica, esto es:

A AdL Idt

τ α= =∑r

rr

Así se tiene que,

A AA

2

3 2

a MgRsenMgsen I I aR I

2 9,8 (0,05) 0,5 ma a 3,27(7,5 10 ) s

θθ α

= = → =

× × ×= ⇒ =×

López Frontado, R. (2011). Física general I, Texto UNA. 2da edición. Caracas: UNA

Page 269: Fisica General i Una

282

c) Para determinar la velocidad del disco después de recorrer una distancia, se puede tomar en cuenta el principio de conservación de la energía mecánica, tomando en consideración que el roce por deslizamiento, esto es, EA = EB

2 2CB CB

A CB A

2 2 2 2 22CB CB CB CB CB

2

Mv vMgh I dondeh dsen , y

2 2 RMv v Mv Mv 3MvMRMgdsen2 2 2 4 42R

ω θ ω

θ

= + = =

⎛ ⎞= + = + =⎜ ⎟

⎝ ⎠

Despejando vCB, se tiene

2CB CB

4gdsen 4 9,8 3 0,5 mv v 4,433 3 s

θ × × ×= = ⇒ =

Respuesta al ejercicio propuesto 7.4 DATOS: m1 = 512 g, m2 = 463 g, R = 4,9 cm, d = 76,5 cm, t = 5,11 s. SOLUCIÓN: En este problema se analiza el movimiento de un sistema formado por un

polea y dos cuerpos que cuelgan de él a través de una cuerda. Para ello se usan las condiciones de rotación de la polea y de traslación de los cuerpos, obteniendo así el momento de inercia de la polea.

Figura 7.16

López Frontado, R. (2011). Física general I, Texto UNA. 2da edición. Caracas: UNA

Page 270: Fisica General i Una

283

A partir del DCL, podemos escribir las ecuaciones de movimiento de cada uno de los cuerpos, esto es,

Para m1 Para m2 Para la polea

y 1 1 y 2 2 y C C

1 1 1 1 2 2 2 2 s 1 2

1 2 C

F =m a F =m a F =0, =I

m g-T =m a (1), T -m g=m a (2), F -Mg-T -T =0 (3) TR-T R=I

! "

"

# # # #

(4)

También se tiene que a1 = R α, y a2 = R α, → a1 = a2 = a

2

2 2 2

at 2y 2 0,765 my= a= = a=0,0586 2 t (5,11) s

× →

Despejando T1 de la ecuación (1), T2 de la ecuación (2), la sustituimos en la

ecuación (4), y despejando entonces IC, se tiene

[ ]

[ ]

21 2 1 2

C

2

C

-2 2C

(m -m )g-(m +m )a RI =

a(0,512-0,463)9,8-(0,512+0,463)0,0586 (0,049)

I =0,0586

I =1,73 10 kg.m⇒ ×

Respuesta al ejercicio propuesto 7.5 DATOS: m1 = 15 kg, m2 = 20 kg, a = 2 m/s2, θ = 37º ,R = 0.25 m SOLUCIÓN:

Figura 7.17

López Frontado, R. (2011). Física general I, Texto UNA. 2da edición. Caracas: UNA

Page 271: Fisica General i Una

284

Las ecuaciones de movimiento aplicando las leyes fundamentales de la mecánica, son:

Para m1 Para la polea Para m2

1 1 1 1 x 1 2 2 2 2

1 1 y 1 2

2 1 c

T -m gsen37º=m a F -Tcos37º=0 m g-T =m aN -m gcos37º=0 F -T sen37º-Mg-T =0

T R-TR=I !

Pero a1 = Rα = a2 = a. a) Despejando las tensiones, se tiene

1 1 1 1

2 2 2 2

T =m a+m gsen37º=15(2+9,8sen 37º) T =118,5 NT =m g-m a=20(9,8-2) T =156,0 N

!

!

b) Para determinar el momento de inercia de la polea, se tiene

22 1

2 1 c c

22

c c

(T T )Ra(T T )R I IR a

(156,0 118,5)(0,25)I I 1,17kg.m2

−− = → =

−= ⇒ =

Respuesta al ejercicio propuesto 7.6 DATOS: ro = 1,5 cm, Ro = 26,0 cm, θ = 35º SOLUCIÓN: a) Para determinar la rapidez de la esfera cuando abandona la pista en el

punto B, consideremos el movimiento de ella desde A hasta B, aplicando el principio de conservación de energía, debido a que la esfera rueda en la pista y no hay fricción de deslizamiento, esto es:

( )

22eB

A B o B

2o

e B o

ImvE =E , Esto es mgR = + mgh ,

2 22mr

donde I = y h =R 1-cos2

ω

θ

López Frontado, R. (2011). Física general I, Texto UNA. 2da edición. Caracas: UNA

Page 272: Fisica General i Una

285

Sustituyendo, se tiene:

( )θ

θ

θθ

⎛ ⎞⎜ ⎟⎝ ⎠

⎛ ⎞⎜ ⎟⎝ ⎠

22 2oB B

o o2o

2 2B B

o o o

2 oB o B

o

B B

2mrmv v1mgR = + +mgR 1-cos2 2 5 r

v vgR = + +gR -gR cos

2 510gR cos7v =gR cos v =

10 7

10×9,8×0,26cos45 mv = v =1,604 7 s

b) Para determinar la distancia D, consideremos el movimiento de la

esfera en el aire, desde el punto B hasta el punto C. Aplicando las condiciones iniciales para este movimiento, las ecuaciones paramétricas son:

o o

B B B o2

oB B B B

x=x +v tcos45 , donde x =R sen45

gty=y +v tsen45 - y =h2

El tiempo que tarda la esfera desde B hasta el punto C (o tiempo de vuelo tv), se obtiene conociendo que yC = 0, esto es:

( )( )

2o o v

C o B v

2v v

2v v v

gty =0=R 1-cos45 +v t sen45 -

24,9t -1,604×0,707t -0,26 1-0,707 =0

4,9t -1,134t -0,0762=0 t =0,282 s→

Entonces, o o

C o B vD=x =R sen45 +v t cos45D=0,26×0,707+1,604×0,282×0,707 D=0,504 m!

Respuesta al ejercicio propuesto 7.7 DATOS: Mg = 1200 N, mg = 2000 N

Figura 7.18

López Frontado, R. (2011). Física general I, Texto UNA. 2da edición. Caracas: UNA

Page 273: Fisica General i Una

286

SOLUCIÓN: Aplicando las condiciones de equilibrio de un cuerpo rígido, se tiene: Para el pescante Para el cuerpo

x x 1

y y 1 2 2

A 1 2

F =0, F -T cos25º=0

F =0, F +T sen25º-Mg-T =0 T -mg=0

3L Lτ =0, T -Mg sen25º-T Lsen25º=04 2

⎛ ⎞ ⎛ ⎞⎜ ⎟ ⎜ ⎟⎝ ⎠ ⎝ ⎠

∑∑∑

a) Para obtener las tensiones, se despeja T2, y se sustituye en la última

ecuación. Esto es,

T2 = mg ⇒ T2 = 2000 N

[ ]

1

1

4 Mg 4T mg sen25º 600 2000 sen25º3 2 3

T 1465N

⎡ ⎤= + = +⎢ ⎥⎣ ⎦⇒ =

b) Para determinar la fuerza que ejerce el perno, es necesario obtener Fx

y Fy.

x 1 x

y 1 2

y

F Tcos25º 1465cos25º F 1327,8NF Tsen25º Mg T 1465sen25º 1200 2000

F 2580,9N

= = → == − + + = − + +

→ =

ˆ ˆ F=(1327,8i+2580,9j) N⇒!

Respuesta al ejercicio propuesto 7.8 DATOS: L = 4 m, µe = 0,5, θ = 37º SOLUCIÓN:

Figura 7.19

López Frontado, R. (2011). Física general I, Texto UNA. 2da edición. Caracas: UNA

Page 274: Fisica General i Una

287

Aplicando las condiciones de equilibrio, tanto para la varilla como al bloque de peso extra, se tiene, Para la varilla Para el bloque

x x C

y y 2 C 2

A C 2

F 0, F T cos 0 (1)

F 0, F T W T sen 0, (2) T W 0 (3)

L0, T Lsen W T x 0 (4)2

θ

θ

τ θ

= − =

= − − + = − =

= − − =

∑∑∑

Pero y eF =Fµ . Por definición, e e e xF = N= Fµ µ µ , entonces se tiene que y e xF = Fµ , y de la ecuación (3) se tiene que T2 = W. Despejando Fx en la ecuación (1) , se tiene x CF =T cos .θ Sustituyéndolos en la ecuación (2), se obtiene: e C CT cos37°+T sen37°-2W=0µ

Al despejar TC, se tiene

Ce

2WTcos37º sen37ºµ

=+

Sustituyendo este resultado en la ec. (3), y despejando x, se obtiene

e

2Lsen37º L 2 4 0,6x 2cos37º sen37º 2 0,5 0,8 0,6

x 2,81mµ

× ×= − = −+ × +

⇒ =

Respuesta al ejercicio propuesto 7.9 DATOS: WAB = 200 N, WP = 200 N SOLUCIÓN:

Figura 7.20

López Frontado, R. (2011). Física general I, Texto UNA. 2da edición. Caracas: UNA

Page 275: Fisica General i Una

288

a) Usando el DCL, aplicando las leyes fundamentales de la mecánica a un cuerpo rígido, se tiene

En el pivote AB Objeto P

x x 2

y y AB 1 2 1 P

A 2 2 AB 1 1 3

F =0, F -T cos30°=0

F =0, F -W -T -T sen30°=0 T -W =0

=0, T b -W b -Tb =0!

"""

Los brazos de fuerzas b1, b2 y b3 son:

1 2 3Lb = cos45°, b =Lsen15° y b =Lcos45°2

Sustituyendo sus valores, y despejando se obtiene el valor de T2, esto es,

ABABPP

2

WW L W + cos45W Lcos45°+ cos45° 22T = =Lsen15° sen15°

⎛ ⎞ °⎜ ⎟⎝ ⎠

2 2

1 P 1

(200+100)0,707T = T =818,9 N0,259

T =W T =200,0 N

!

!

b) Para determinar la fuerza ejercida sobre el puntal por el pivote, es

necesario obtener los valores de Fx y Fy. Para ello, se tiene

x 1 x

y 1 2 AB y

F =Tcos30°=200 0,866 F =173,2 NF =T sen30°+T +W =200 0,5+818,9+200 F =1118,9 N

! "

! "

La magnitud de FA es

2 2 2 2 2A x y AF =F +F =(173,2) +(1118,9) F =1135,3 N⇒

La dirección está dada por

y-1 -1

x

F 1118,9=tan =tan =81,2°F 173,2

! !" # " #

$% & % &' (' (

López Frontado, R. (2011). Física general I, Texto UNA. 2da edición. Caracas: UNA

Page 276: Fisica General i Una

289

Respuesta al ejercicio propuesto 7.10 DATOS: m1 = 30 kg, m2 = 70 kg, mv = 20 kg, L = 4 m, d = 2,5 m. SOLUCIÓN:

DCL

Figura 7.21

Aplicando la 2ª ley de Newton, se tiene:

y A 1 v C 2

1 v A 2

F : F -m g-m g+F -m g=0

=0

m g(2,5)+m g(0,5)-F (2,5)-m g(x)=0C!

""

El equilibrio de la viga se rompe cuando FA=0, así se tiene que

1 v2 1 v

2

m g(2,5)+m g(0,5)m g(x)=m g(2,5)+m g(0,5) x=

m g2,5 30+0,5 20x= x=1,1286 m

70! !

"

O sea que la masa m2 se puede colocar a una distancia máxima de 1,1286 m a la derecha del punto C, para que no se rompa el equilibrio. Respuesta al ejercicio propuesto 7.11 DATOS: m1 = 3 kg, m2 = 7,0 kg, M = 2,0 kg, R = 0,2 m SOLUCIÓN:

Figura 7.22

López Frontado, R. (2011). Física general I, Texto UNA. 2da edición. Caracas: UNA

Page 277: Fisica General i Una

290

a) Suponiendo que el sistema se mueve en sentido horario, al aplicar las ecuaciones de la dinámica, tenemos,

Para m1 Para la polea Para m2

1 1 1 1 2 2 2 2

s 1 22

o 1 2

T m g m a F 0 m g T m aF Mg T T 0

MRI , TR T R2ατ α

− = = − =

− − − =

= − + =

r

(1)

Pero: a1 = R α, y a2 = R α → a1 = a2.

Despejando T1 y T2, se obtiene: T1 = m1g + m1a y T2 = m2g - m2a Sustituyéndola en la ec. (1), se tiene

1 1 2 2Mam g m a m g m a (2)2

− − + − =

de esta ecuación se obtiene una expresión para la aceleración, y sustituyendo sus valores, se tiene

2 12

1 2

(m m )g (7 3)9,8 ma a 3,56M 3 7 1 sm m2

− −= = ⇒ =+ ++ +

Como a > 0, entonces concluimos que el sistema gira en sentido horario. b) El momento angular del sistema está dado por o oL I ω=

r r

Pero 2

2 2o 1 2

MR vI mR m R y2 R

ω= + + =

El valor del momento angular:

2

1 2M kg.mL (m m )Rv (3 7 1)0,2 5 L 11,02 s

= + + = + + × ⇒ =

La dirección del momento angular L es hacia dentro del plano del papel, de

acuerdo con la regla de la mano derecha.

López Frontado, R. (2011). Física general I, Texto UNA. 2da edición. Caracas: UNA

Page 278: Fisica General i Una

291

RESPUESTA A LOS EJERCICIOS DE AUTOEVALUACIÓN

Respuesta al ejercicio de autoevaluación 7.1 a) T1 = 7,5 N, T2 = 66,4 N; b) IC = 0,14 kg.m2 Respuesta al ejercicio de autoevaluación 7.2 a) ω2 = 1907 rad/s; b) K1 = 2,53 J, K2 = 2,13 J

Respuesta al ejercicio de autoevaluación 7.3 v = 1,66 m/s. Respuesta al ejercicio de autoevaluación 7.4 a) NA = 130 N, NB = 670 N; b) h = 2,22 m. Respuesta al ejercicio de autoevaluación 7.5 a) x = 1,19 m; b) vA = 4,59 m/s

López Frontado, R. (2011). Física general I, Texto UNA. 2da edición. Caracas: UNA

Page 279: Fisica General i Una

293

UNIDAD 8

MOVIMIENTO GRAVITATORIO

En esta unidad se estudia la ley básica que gobierna las atracciones

gravitacionales. Como se conoce, la Tierra y todos los astros pertenecientes al sistema solar realizan un movimiento de traslación alrededor del Sol, describiendo una órbita elíptica, en uno de cuyos focos se encuentra éste. La forma de las trayectorias elípticas se explica por la existencia de una fuerza de atracción, expresada por la Ley de Gravitación Universal, establecida por Isaac Newton a finales del siglo XVII.

La ley de atracción gravitatoria es una ley universal, que define el campo de

fuerzas en que actúa, y por consiguiente la aceleración que adquieren los cuerpos en dicho campo, llamada la aceleración de la gravedad. Esta atracción actúa fundamentalmente de la misma manera entre la Tierra y nuestro cuerpo, entre el Sol y cualquier planeta y entre un planeta y sus satélites.

Se aplica la ley de gravitación universal a fenómenos como la variación del

peso con las diferentes alturas, las órbitas de los satélites terrestres y la de los planetas alrededor del Sol.

Con anterioridad a la ley de Gravitación Universal, se establecieron las leyes

de Kepler, las cuales, a partir del análisis de innumerables datos astronómicos, permiten estudiar el movimiento de los planetas que pertenecen al sistema solar, desde un punto de vista de la cinemática.

CONOCIMIENTOS PREVIOS: Para la correcta comprensión de esta unidad, el estudiante debe tener los

siguientes conocimientos: 1.- Movimiento circular uniforme (Unidad 2). 2.- Masa (Unidad 3). 3.- Fuerza, campos de fuerza (Unidad 3). 4.- Campo potencial. Potencial (Unidad 3). 5.- Leyes de la conservación de la energía (Unidad 4). 6.- Leyes de la conservación del momento lineal y del momento angular

(Unidad 5).

López Frontado, R. (2011). Física general I, Texto UNA. 2da edición. Caracas: UNA

Page 280: Fisica General i Una

294

8.1 OBJETIVO

Aplicar los conceptos de la dinámica de sistemas de partículas al estudio del movimiento gravitatorio universal. 8.2 RECOMENDACIONES PARA EL ESTUDIO DEL CONTENIDO

En esta unidad, el estudiante debe comprender claramente cómo se aplican

los principios y leyes fundamentales en el estudio del comportamiento de los sistemas en el campo gravitatorio, usando la ley de Gravitación Universal.

Para una mejor comprensión del contenido de la unidad, realice la lectura de

los conceptos relacionados con el movimiento de los cuerpos del Universo, aplicando las ideas estudiadas en el movimiento de sistemas de partículas y las leyes fundamentales de la mecánica en el estudio de los sistemas de esta unidad, y los ejemplos presentados. Utilice las técnicas de lectura que le faciliten la comprensión del tema. Dé respuesta a las preguntas propuestas, y consulte a los profesores que administran y asesoran en su Centro Local o, en su defecto, a través del correo electrónico (suministrado por la carrera).

8.3 CONTENIDO 8.3.1 LEY DE GRAVITACIÓN UNIVERSAL. 8.3.2 LEYES DE KEPLER. 8.3.3 APLICACIÓN DE LA LEY DE GRAVITACIÓN UNIVERSAL.

8.3.1 LEY DE GRAVITACIÓN UNIVERSAL La ley de Gravitación Universal establece que: “Cuando dos cuerpos interactúan entre sí, ellos se atraen con una fuerza

que es proporcional al producto de sus masas e inversamente proporcional al cuadrado de la distancia que las separa”.

Figura 8.1

Consideremos dos partículas que interactúan entre sí, siendo sus masas m1 y m2, y la separación entre ellas está dada por r12. En este caso estudiamos el efecto que la partícula 1 sobre la partícula 2.

López Frontado, R. (2011). Física general I, Texto UNA. 2da edición. Caracas: UNA

Page 281: Fisica General i Una

295

Esta interacción la podemos expresar por la siguiente fórmula:

1 212 122

12

mm ˆF α r r

!

Una expresión similar se obtiene para expresar el efecto que la partícula 2

produce sobre la partícula 1, y esto se debe a la aplicación de la Tercera ley de Newton, o sea el principio de acción y reacción.

Matemáticamente, la ley de Gravitación se puede expresar usando la

siguiente fórmula: i j

G ij2ij

2-11

2

GmmˆF =- r (8.1)

r

N.mG=6,67×10 , en sistema unidades SI.kg

r

G es una constante de proporcionalidad, llamada Constante de Gravitación Universal.

Cuando un cuerpo está a una altura h sobre la superficie de la Tierra, como se muestra en la figura (8.2), la fuerza de interacción que ejerce la Tierra sobre el cuerpo de masa m, aplicando la ley de Gravitación Universal, está dada por:

TG 2

T

GM m ˆF = r (8.2)(R +h)

!

Figura 8.2

La fuerza de interacción que ejerce la Tierra sobre el cuerpo es igual al peso

del mismo, esto es, GF =mgr r

. Igualando las magnitudes de la fuerza peso con la fuerza de atracción que ejerce la Tierra, se tiene

T

2T

GM mmg=(R +h)

De esta ecuación, se obtiene

T2

T

GMg= (8.3)(R +h)

Esta expresión nos proporciona la aceleración de gravedad g

r, donde MT y

RT son la masa y radio de la Tierra. De esta ecuación se deduce que el valor de la aceleración de la gravedad depende de la altura a la cual se encuentre el cuerpo.

López Frontado, R. (2011). Física general I, Texto UNA. 2da edición. Caracas: UNA

Page 282: Fisica General i Una

296

8.3.2 LEYES DE KEPLER

El astrónomo alemán Johannes Kepler publicó un análisis exhaustivo de las

mediciones de Tycho Brake, cuyas conclusiones constituyen las tres leyes de Kepler, usadas con gran precisión para predecir las posiciones planetarias con mucha anticipación al futuro. Estas leyes pueden expresarse como sigue:

1ª ley de Kepler (o ley de las órbitas) Esta ley establece que todos los planetas que forman el sistema solar

describen un movimiento a lo largo de una órbita elíptica, con el Sol en uno de los focos.

2ª ley de Kepler (o ley de las áreas) Esta ley establece que el radio vector de un planeta (o recta que va del

centro del Sol al del planeta) describe áreas iguales en intervalos iguales de tiempo, en todas partes de la órbita.

3ª ley de Kepler (o ley de los periodos). Esta ley establece que el cuadrado del periodo o tiempo necesario para

que un planeta recorra su órbita alrededor del Sol es proporcional al cubo del radio medio de la órbita.

Para darnos una idea del significado de las leyes de Kepler, consideremos lo

que se entiende por una órbita elíptica, mencionada como la órbita descrita por los planetas pertenecientes al sistema solar, cuando se mueven alrededor del Sol. La geometría de una elipse se muestra en la figura (8.3). Una elipse es una figura que se define matemáticamente al escoger dos puntos S1 y S2, cada uno de los cuales se llama foco, y luego trazar una curva, de tal forma que pase por los puntos para los cuales la suma de las distancias r1 y r2 sea constante.

La distancia más grande a se denomina semieje mayor; la distancia b se denomina semieje menor; los puntos focales están situados a una distancia c desde el centro de la elipse, denominada distancia focal, de tal forma que a2=b2+c2. La excentricidad de una elipse se define como la relación entre la distancia focal y el semieje mayor, esto es:

distancia focal ce= , e= (8.4)semieje mayor a

Figura 8.3

Geometría de una elipse. La suma de r1 y r2 es la misma para todos los puntos de la elipse.

López Frontado, R. (2011). Física general I, Texto UNA. 2da edición. Caracas: UNA

Page 283: Fisica General i Una

297

De acuerdo con la 1ª ley de Kepler, en la órbita elíptica descrita por un planeta alrededor del Sol, el Sol está en uno de los focos de la elipse. El punto de la elipse más cercana del Sol se denomina perihelio, (para un cuerpo que gire alrededor de la Tierra, este punto se llama perigeo). El punto más alejado del Sol se denomina afelio (para un cuerpo que gire alrededor de la Tierra, este punto se llama apogeo).

Figura 8.4

El planeta P se mueve alrededor del Sol en una órbita elíptica.

La 2ª ley de Kepler (o ley de las áreas), se puede considerar como una consecuencia de la conservación del momento angular. Matemáticamente se puede expresar por:

P

p

dA L= =constantedt 2M

L dA= dt (8.4)2M

!

Figura 8.5

Cuando un planeta gira alrededor del Sol, las áreas barridas por el radio vector en intervalos de tiempos iguales son iguales, esto es que A = A’.

3ª ley (o ley de los periodos).- El cuadrado del periodo o tiempo necesario

para que un planeta recorra su órbita alrededor del Sol es proporcional al cubo del radio medio de la órbita.

La 3a ley, matemáticamente se expresa por

!

2 3S m

S2 2

-19S 3

S

T =K R , (8.5)

donde K es una constante dada por

4 s K = =2,97x10 GM m

López Frontado, R. (2011). Física general I, Texto UNA. 2da edición. Caracas: UNA

Page 284: Fisica General i Una

298

8.3.3 APLICACIÓN DE LA LEY DE GRAVITACIÓN UNIVERSAL CAMPO GRAVITACIONAL Si un cuerpo de masa m experimenta una fuerza gravitatoria dada por

GF =mgr r

Se dice entonces que en esa región existe un campo gravitatorio g

r.

Debido a que la fuerza gravitacional que actúa sobre la masa m tiene una magnitud

TG 2

GM mF =r

, el campo gravitatorio gr

, a una

distancia r del centro de la Tierra está dado por:

G T2

F GM ˆg= =- r (8.6)m r

!!

En la superficie de la Tierra, la magnitud de q es

2

N mg=9,80 =9,80 kg s

Figura 8.6

El campo gravitatorio gr

sobre un cuerpo de masa m.

ENERGÍA POTENCIAL GRAVITACIONAL El cambio en la energía potencial gravitacional está dada por:

f i Tf i

1 1ΔU=U -U =GM m - (8.7)r r⎡ ⎤⎢ ⎥⎣ ⎦

La energía potencial de una partícula m2, cuando se coloca en una región

donde existe un campo gravitacional generado por una partícula m1, es:

1 2Gm mU=- (8.8)r

López Frontado, R. (2011). Física general I, Texto UNA. 2da edición. Caracas: UNA

Page 285: Fisica General i Una

299

Cuando se tienen tres partículas que interactúan entre si, la energía potencial de ese sistema de partículas es:

1 3 2 31 2Total 12 13 23

12 13 23

i jTotal

i¹j ij

m m m mm mU =U +U +U =-G + +r r r

mm U =-G (8.9)

r

! "# $% &

' (

Si un cuerpo de masa m se mueve con rapidez v, cerca de un cuerpo de masa

M (M>>m, y M se supone en reposo), la energía mecánica total del sistema formado por los dos cuerpos es

21 GMmE=K+U= mv - (8.10)

2 r

ENERGÍA EN EL MOVIMIENTO PLANETARIO Y DE SATÉLITES Consideremos un sistema formado por dos cuerpos, uno de masa m que se

mueve alrededor de un cuerpo de masa M, tal que M>>m. El sistema podría ser un planeta alrededor del Sol, o el de un satélite alrededor de la Tierra. Suponiendo que el cuerpo de masa M esté en reposo respecto a un sistema de referencia inercial, entonces la energía mecánica total E del sistema formado por los dos cuerpos separados una distancia r es la suma de la energía cinética del cuerpo de masa m y la energía potencial del sistema, esto es:

2

2

1 GMmE=K+U= mv - (8.11)2 r

Si suponemos que el cuerpo de masa m orbita circularmente alrededor del

cuerpo de masa M, aplicando la 2ª de Newton al cuerpo de masa m, se tiene

22

2

GMm mv 1 GMm=ma= , mv =r r 2 2r

Al sustituir en la ecuación (.11), se tiene

GMm GMm GMmE= - E=- (8.12)2r r 2r

La ecuación (8.12) nos indica que la energía mecánica total es negativa en

el caso de órbitas circulares. Además nos dice que la energía cinética es igual a la mitad del valor absoluto de la energía potencial.

López Frontado, R. (2011). Física general I, Texto UNA. 2da edición. Caracas: UNA

Page 286: Fisica General i Una

300

Cuando las órbitas son elípticas, la energía mecánica total también es negativa, y su expresión matemática está dada por:

GMmE=- (8.13)

2a

Donde a es la longitud del semieje mayor. Si el sistema está aislado, la energía total es constante. Así se tiene que si el

cuerpo de masa m se mueve desde un punto inicial A hasta un punto final B, la energía total permanece constante, entonces se puede escribir que:

2 2i f

i f

1 GMm 1 GMmE= mv - = mv - (8.14)2 r 2 r

RAPIDEZ DE ESCAPE La rapidez de escape es la velocidad mínima que un cuerpo de masa m,

debe tener en la superficie de cualquier planeta de masa M y de radio R, para alcanzar una distancia infinita de separación desde dicho planeta. Esto es:

esc2GMv = (8.15)

R

Es importante notar que la velocidad de escape es independiente de la masa del cuerpo, es decir que una nave espacial tiene la misma rapidez de escape que un cuerpo cualquiera por pequeño que sea.

A continuación se presentan unos ejemplos de problemas donde Ud., podrá

visualizar las aplicaciones de los conceptos tratados en esta unidad. Pero antes vea las Sugerencias y las Recomendaciones presentadas para resolver estos problemas.

RECOMENDACIONES PARA RESOLVER PROBLEMAS DE LAS LEYES

DE GRAVITACIONES UNIVERSALES. La estrategia para resolver problemas relacionados con las Leyes de

Gravitación Universal es muy similar a la presentada para resolver problemas del movimiento de partículas.

IDENTIFICAR los conceptos relevantes: decida si se aplica o no la Ley de

Gravitación Universal; también, si se aplican o no las leyes de Kepler.

López Frontado, R. (2011). Física general I, Texto UNA. 2da edición. Caracas: UNA

Page 287: Fisica General i Una

301

PLANTEAR el problema utilizando las etapas siguientes: 1.- Haga un dibujo que muestre la ubicación de las partículas con masa y

rotule cada partícula con su masa. Esta etapa es muy importante, sobre todo si están presentes más de dos partículas con masa.

2.- Si están presente más de tres partículas y no todas se encuentran

sobre la misma recta, construya un sistema de coordenadas xy. 3.- Cuando sea necesario hallar la fuerza gravitatoria que se ejerce sobre

una sola partícula, identifique esa partícula. EJECUTAR la solución como sigue: 1.- Con respecto a cada partícula que ejerce una fuerza sobre la partícula

de interés, calcule la magnitud de esa fuerza usando la ecuación i j

G 2ij

GmmF =-

r, donde

2-11

2

N.mG=6,67×10 kg

, es la constante de Gravitación

Universal en el sistema SI. 2.- Trace los vectores de fuerzas gravitatorias que actúan sobre las

partículas de interés, debido a la acción de las otras partículas. 3.- Calcule la fuerza gravitatoria total sobre las partículas de interés,

aplicando el algebra vectorial para ello. EVALUAR la respuesta: Compruebe que sus resultados numéricos sean

razonables, y que la dirección de la fuerza gravitatoria resultante concuerda con la ley de Gravitación Universal, de que las fuerzas gravitatorias entre dos cuerpos son siempre atractivas.

Recuerde que el manejo eficiente del contenido de esta unidad es

importante, por su repercusión en el contenido de las subsiguientes asignaturas del Plan de Estudios en Ingeniería y otras ciencias.

Ejemplo 8.1.- Un sistema consta de tres partículas con una masa de 5 kg cada una,

colocadas en las esquinas de un triángulo equilátero con lados de 30 cm. a) Determine la magnitud y dirección de la fuerza resultante de dos de las

masas sobre la otra.

López Frontado, R. (2011). Física general I, Texto UNA. 2da edición. Caracas: UNA

Page 288: Fisica General i Una

302

b) Calcule la energía potencial del sistema

DATOS: m = 5 kg, a = 0,3 m G = 6,67 x 10-11 N m2 /kg2. SOLUCIÓN: IDENTIFICAR Y PLANTEAR: Para determinar la fuerza resultante que se

ejerce sobre una partícula, se aplica le ley de Gravitación Universal. Para determinar la energía potencial del sistema se usa la ecuación (8.9).

EJECUTAR la solución del problema:

Figura 8.7

a) Para determinar la fuerza gravitacional resultante, se determina la magnitud de la fuerza gravitatoria ejercida por cada una de las partículas sobre la partícula de interés; luego se obtiene la suma vectorial usando componentes a lo largo de los ejes mostrados en la figura (8.7).

La fuerza de interacción gravitatoria está dada por:

1 2G 2

Gmm ˆF rr

= −r

En este caso, m1 = m2 = m3= m, y r = a, por

lo tanto 2

1 2 2

GmF Fa

= =

De la figura, se observa que

( )

g 1 2 2 1

2 -11 2

g 2 2

8g

ˆ ˆ ˆ ˆF =(F +F cos60º)i-F sen60ºj=F (1+cos60º)i-sen60ºj

Gm 6,67 10 (5)ˆ ˆ ˆ ˆF = (1+cos60º)i-sen60ºj = 1,5i-0,87ja (0,3)

ˆ ˆ F = 2,78i-1,61j 10 N−

⎡ ⎤⎣ ⎦⎛ ⎞ ⎛ ⎞×⎡ ⎤ ⎡ ⎤⎜ ⎟ ⎜ ⎟⎣ ⎦ ⎣ ⎦⎝ ⎠ ⎝ ⎠

⇒ ×

r

r

r

La magnitud de la fuerza gravitacional es Fg =3,21x10-8 N, y su dirección es

-1 -1,61=tan =-30,08º

2,78! !" #

$% &' (

López Frontado, R. (2011). Física general I, Texto UNA. 2da edición. Caracas: UNA

Page 289: Fisica General i Una

303

b) La energía potencial está dada por la ecuación siguiente:

21 3 2 31 2

12 13 23

11 28

mm m mmm 3mU G Gr r r a

6,67 10 3 (5)U U 1,67 10 J0,3

−−

⎡ ⎤= − + + = −⎢ ⎥

⎣ ⎦× × ×= − ⇒ = − ×

EVALUAR: Estos resultados nos demuestran que tanto la fuerza gravitacional resultante como la energía potencial del sistema es bastante pequeña.

Ejemplo 8.2.- ¿Cuál será el período de revolución de un satélite artificial de 100 kg de masa,

que circunda la Tierra siguiendo una órbita circular de 8000 Km de radio? DATOS: m = 100 kg, R = 8000 Km = 8 x 106 m,

MT = 5,98x1024 kg, G = 6,67 x10-11 N.m2/kg2. SOLUCIÓN: IDENTIFICAR Y PLANTEAR el problema: Se aplica la relación entre la Ley

de Gravitación Universal y el movimiento periódico del satélite alrededor de la Tierra. EJECUTAR la solución del problema: Así se tiene que la fuerza de

Gravitación Universal está dada por: T

G 2

GM m ˆF rR

= −!

Como la órbita es circular, la fuerza gravitatoria que ejerce la Tierra sobre el

satélite es la fuerza centrípeta, esto es,

2 2TG 2

GM mF mR mR

Rω ω= → =

Despejando la velocidad angular, se obtiene 2 T3

GMR

ω =

López Frontado, R. (2011). Física general I, Texto UNA. 2da edición. Caracas: UNA

Page 290: Fisica General i Una

304

Pero se tiene que el período está relacionado con la velocidad angular por medio de la siguiente expresión;

π πω

π −

= =

×= ⇒ =× × ×

3

T

6 3

11 24

2 RT 2GM

(8 10 )T 2 T 7118,7 s6,67 10 5,98 10

EVALUAR la respuesta: El período de revolución del satélite en la órbita

circular alrededor de la Tierra es de 7118,7 s, o sea el equivalente a 1,9774 horas.

Ejemplo 8:3.- Sabiendo que G = 6,67 x 10-11 N.m2 / kg2, el radio de la Tierra RT = 6400

Km, y la aceleración de la gravedad g = 9,8 m/s2, calcule: a) La densidad media de la Tierra.

b) ¿A qué altura, sobre la superficie terrestre, el valor de g se reduce a la

mitad? DATOS: RT = 6,4x106 m; g = 9,8 m/s2. SOLUCIÓN: IDENTIFICAR Y PLANTEAR el problema: Se usan los conceptos de densidad

de un cuerpo y la ley de Gravitación Universal. Además se aplica la expresión matemática que relaciona el concepto de peso con la ley de Gravitación Universal.

EJECUTAR la solución del problema: a) Para determinar la densidad media de la Tierra, por definición, se tiene

que: MV

ρ =

Pero para un cuerpo cualquiera en la superficie de la Tierra, su peso es igual a la fuerza gravitatoria terrestre, esto es :

2

2

GMm gRmg MGR

= → =

López Frontado, R. (2011). Física general I, Texto UNA. 2da edición. Caracas: UNA

Page 291: Fisica General i Una

305

Además, considerando a la Tierra como una esfera, se tiene que el volumen de ella está dado por 34V R

3π= .

Al sustituir en la ecuación de la densidad, se tiene:

2

3

11 6 3

gR 3 3gG4 RG4 R3 9,8 kg5480,6

6,67 10 4 6,4 10 m

ρππ

ρ ρπ−

= =

×= ⇒ =× × × ×

b) Para determinar la altura donde la aceleración de gravedad se reduce a

la mitad, se tiene que

22 2T T

2

2 2 2 2 2

GM 2GMg 2G gR(R h) 2R2 g g G(R h)h 2Rh R 2R h 2Rh R 0

= → + = = =+

+ + = → + − =

2 6 6 2

2 6 13 6

h 2 6,4 10 h (6,4 10 ) 0,h 12,8 10 h 4,096 10 0 h 2,65 10 m

+ × × − × =+ × − × = ⇒ = ×

EVALUAR la respuesta del problema: Como su nombre nos lo indica, la

densidad media de la Tierra es igual a 5480,6 kg/m3, conociéndose que la masa de la Tierra no está distribuida uniformemente en todo su volumen.

Ejercicio propuesto 8.1.- Se conoce que la masa de la Luna es ML = 7,36 x1022 kg y su radio es RL =

1,74 x106 m. a) Calcule el valor de la aceleración de gravedad en la superficie de la

Luna gL. b) ¿Cuál es el período de un péndulo de segundos (período = 2,00 s en la

Tierra) en la superficie de la Luna?

c) ¿Cuánto pesaría un objeto en la superficie de la Luna si pesa 100 N en la superficie de la Tierra?

Tome como masa de la Tierra MT = 5,98 x1024 kg, el radio medio de la Tierra

RT = 6,37 x106 m, y la constante de Gravitación Universal G = 6,67 x10-11 N m2/kg2.

López Frontado, R. (2011). Física general I, Texto UNA. 2da edición. Caracas: UNA

Page 292: Fisica General i Una

306

Ejercicio propuesto 8.2.- Una esfera uniforme de 0,6 kg es mantenida con su centro en el origen, y una

segunda esfera uniforme de 0,8 kg es mantenida con su centro en x = 0,3 m, y = 0. ¿Qué magnitud y dirección tiene la fuerza gravitatoria neta de estas esferas sobre una tercera esfera uniforme de 0,04 kg de masa colocada en x = 0, y = 0,4 m?

Ejercicio propuesto 8.3.- En un experimento con la balanza de Cavendish se observó que una esfera

de 0,8 kg atraía a otra de 4,0 x10-3 kg con una fuerza de 1,3x10-10 N, cuando la distancia entre sus centros era de 0,04 m.

a) ¿Cuál es el valor de la constante de Gravitación G?

b) ¿Cuál es el valor de la masa de la Tierra? Suponga g = 9,8 m/s2 y el radio de la Tierra RT = 6380 km.

Ejemplo 8.4.- Una partícula de masa m está situada a una distancia b de una barra

infinitamente larga de densidad de masa lineal λ. Determine la fuerza gravitatoria ejercida por la barra sobre la partícula.

DATOS: dm=

d!

!

SOLUCIÓN: IDENTIFICAR los conceptos pertinentes Se aplica la ley de Gravitación

Universal para determinar la fuerza gravitatoria producida por una distribución de masa continua, donde suponiendo que la densidad de masa sea constante, o mejor dicho que la distribución de masa sea uniforme.

López Frontado, R. (2011). Física general I, Texto UNA. 2da edición. Caracas: UNA

Page 293: Fisica General i Una

307

PLANTEAR Y EJECUTAR la solución del problema: Considerando que la barra es infinita, se puede suponer que el punto O está

situado en la mitad de la misma. Para determinar la fuerza sobre la partícula de masa m situada en el punto P, consideremos el efecto que un elemento diferencial de masa dm de la barra produce sobre la masa m, usando la ley de Gravitación Universal.

Figura 8.8

Esto es,

G 2

Gm dmdF = , donde dm= dxr

λ .

Por simetría, la fuerza gravitatoria solo tiene componente en dirección perpendicular a la barra, esto es,

G G 2 2

Gm dm Gm dx cosF = dF cos = cos =r r

λ θθ θ∫ ∫ ∫

De la figura, se tiene que

2xtan = , x=btan , dx=bsec db

! ! ! !"

Entonces,

2

G 20

Gm bsec cos dF =2 (1)r

! " " "#

$

De la figura, también se tiene que 2 rsec =

bθ . Si multiplicamos y dividimos a la

ecuación (1) por b, ésta queda:

/ 2

G G0

2Gm 2GmF = cos F = (2)b b

d!" "

# # $%

EVALUAR el resultado: La fuerza gravitatoria ejercida por la masa contenida

en la barra infinitamente larga sobre la partícula de masa m está dada por la ecuación (2)

López Frontado, R. (2011). Física general I, Texto UNA. 2da edición. Caracas: UNA

Page 294: Fisica General i Una

308

Ejercicio propuesto 8.4.- Una esfera sólida uniforme de 500 kg tiene un radio de 0,40 m. Encuentre la

magnitud de la fuerza gravitacional ejercida por la esfera sobre una partícula de 50,0 g localizada:

a) A 1,50 m del centro de la esfera.

b) En la superficie de la esfera.

Ejercicio propuesto 8.5.- ¿Cuál es el peso aparente de un astronauta de 75 kg de masa, que está

a 4100 km del centro de la Luna en un vehículo espacial? a) Si su vehículo se mueve a velocidad constante.

b) Si su vehículo está acelerando hacia la Luna a 2,6 m/s2 Tome como masa de la Luna ML = 7,35x1022 kg; el radio de la Luna RL =

1,74x103 km; la constante de Gravitación Universal G = 6,67x10-11 N.m2/kg2.

Ejemplo 8.5.- Calcule la energía total que tiene un satélite artificial de 200 kg de masa, que

describe una órbita circular a 400 km de altura sobre la superficie terrestre, siendo RT = 6400 km, MT = 5,98 x 1024 kg, g = 9,8 m/s2, G = 6,67 x 10-11 N.m2 / kg2.

DATOS: mS = 200 kg, h = 400 km. SOLUCIÓN: IDENTIFICAR los conceptos pertinentes: Se aplica el concepto de energía

mecánica total como la suma de la energía cinética y la energía potencial.

López Frontado, R. (2011). Física general I, Texto UNA. 2da edición. Caracas: UNA

Page 295: Fisica General i Una

309

PLANTEAR Y EJECUTAR la solución del problema: La energía total está dada por:

Total cinética potencial TotalE =E +E , E =K+U (1) Cuando el satélite se encuentra a una altura h sobre su superficie, la fuerza

peso del satélite es igual a la fuerza gravitacional ejercida por la tierra sobre él, pero como el movimiento del satélite es circular, el peso también es igual a la fuerza centrípeta, así se tiene que

2

2T T2

T TT

GM m GMvmg=- =-m , v =R +h R +h(R +h)

!

La energía mecánica total es,

2

T

T T T

T T T

mvE=K+U= +mg(R +h)2

mGM GM m GM mE= - , E=-

2(R +h) R +h 2(R +h)!

11 24

93

6,67 10 5,98 10 200E E 5,87 10 J2(6400 400) 10

−× × × ×= − ⇒ = − ×+ ×

EVALUAR la respuesta del problema: La energía mecánica total es

igual a -5,87x109 J.

Ejercicio propuesto 8.6.- Un satélite terrestre de masa mS = 1000 kg, se mueve en una órbita circular

con rapidez orbital de 5200 m/s. Con base en lo planteado, calcule: a) El período de rotación del satélite. b) La aceleración radial del satélite. c) ¿Cuánto trabajo adicional se necesitaría para que el satélite escapara

de la Tierra? Tome: Masa de la Tierra MT = 5,97x1024 kg, y el radio promedio de la Tierra

RT = 6,38x106 m.

López Frontado, R. (2011). Física general I, Texto UNA. 2da edición. Caracas: UNA

Page 296: Fisica General i Una

310

Ejercicio propuesto 8.7.- En el momento de orbitar la Luna, la nave espacial Apolo II tenía una masa

de 9,979x103 kg, su periodo era de 119 min y su distancia media desde el centro de la Luna era de 1,849x106 m. Suponga que su órbita era circular y considere a la Luna como una esfera uniforme, calcule:

a) La masa de la Luna. b) La velocidad orbital de la nave. c) La energía mínima necesaria para que la nave abandone la órbita y

escape del campo gravitacional lunar. La constante de Gravitación Universal G = 6,67x10-11 N.m2/kg2.

Ejercicio propuesto 8.8.- Dos masas esféricamente simétricas tienen sus centros en las siguientes

coordenadas (x; y): m1 = 4,0 kg en (1,0 m;0), y m2 = 3,0 kg en (0;-0,5 m). a) ¿Qué magnitud y dirección tiene la fuerza gravitatoria sobre una masa

prueba mo = 0,01 kg, colocada en el origen?

b) ¿Qué trabajo mínimo tendría que efectuarse para mover la masa prueba desde el origen a un punto alejado de las otras masas?

Ejercicio propuesto 8.9.-

Figura 8.9

En la figura se muestran dos partículas de masas iguales M, que están separadas una distancia 2 a. Con base en lo antes expuesto, calcule la magnitud y dirección del campo gravitacional en el punto P, situado en el bisector perpendicular de las dos partículas.

López Frontado, R. (2011). Física general I, Texto UNA. 2da edición. Caracas: UNA

Page 297: Fisica General i Una

311

Ejercicio propuesto 8.10.- Un satélite de la Tierra que tiene una masa de 100 kg y está a una altura

de 2,0x106 m. a) ¿Cuál es la energía potencial del sistema satélite-Tierra? b) ¿Cuál es la magnitud de la fuerza gravitacional ejercida por la Tierra

sobre el satélite? c) ¿Qué fuerza ejerce el satélite sobre la Tierra? Constante de Gravitación Universal G = 6,67x10-11 N.m2/kg2.

Ejercicio propuesto 8.11.- Dos esferas, de masas m1 = 2,53 kg, y m2 = 7,16 kg, están fijas a una

distancia de 1,56 m entre sus centros. Una tercera esfera de masa m3 = 212 g, se coloca a una distancia de 42,0 cm de la masa m2, a lo largo de la línea que une a las esferas. ¿Cuánto trabajo deberá efectuar un agente externo para mover a la esfera m3 a lo largo de la línea que une las esferas y situarla a 42,0 cm de la esfera de m1?

RESUMEN:

La ley de Newton de la gravitación universal nos dice que la fuerza gravitacional de atracción entre dos cuerpos cualesquiera de masa m1 y m2 separadas por una distancia r, está dada por:

2

-111 2G 2 2

Gm m N.mˆF =- r, donde G=6,67 10 r kg

!r

López Frontado, R. (2011). Física general I, Texto UNA. 2da edición. Caracas: UNA

Page 298: Fisica General i Una

312

Un cuerpo a una distancia h sobre la superficie de la Tierra, experimenta una fuerza gravitacional de magnitud mg, donde g es la aceleración en caída libre desde esa altura, dada por:

T2

T

GMg=(R +h)

Las leyes de Kepler del movimiento planetario. Establecen que: 1.- Todos los planetas del sistema solar se mueven en órbitas

elípticas con el Sol en un foco. 2.- El radio vector trazado desde el Sol a un planeta barre

áreas iguales en intervalos de tiempos iguales. 3.- El cuadrado del período orbital de cualquier planeta es

proporcional al cubo del semieje mayor de la órbita elíptica. Esto es,

2

2 3S

S

4T = a , donde M es la masa del Sol.GM

!" #$ %& '

El campo gravitacional en un punto en el espacio, se define

como la fuerza gravitacional que actúa sobre cualquier partícula situada en ese punto, dividida entre la masa de la partícula. Esto es,

GFg=m

rr

La energía potencial gravitacional de un sistema formado por

dos partículas separadas por una distancia r es

1 2Gm mU=- , donde U=0 cuando r

r→∞

Si un sistema, formado por un cuerpo de masa m y un cuerpo de

masa M (M>>m), está aislado, la energía total del sistema es

21 GMmE= mv -2 r

López Frontado, R. (2011). Física general I, Texto UNA. 2da edición. Caracas: UNA

Page 299: Fisica General i Una

313

La rapidez de escape para un cuerpo lanzado desde la superficie de un planeta de masa M y de radio R está dado por:

esc2GMv =R

EJERCICIO DE AUTOEVALUACIÓN

En su libreta de apuntes desarrolle los siguientes ejercicios. Luego de resolver

los ejercicios, compare sus respuestas con los presentados en la última página de la unidad. En caso de no acertar o de tener alguna duda consulte con un compañero o con el asesor de su centro local.

8.1.- Dos cuerpos se atraen entre sí con una fuerza gravitacional de

magnitud 1,0x10-8 N cuando están separados 20,0 cm. Si la masa total de los dos cuerpos es de 5,0 kg, ¿cuál es la masa de cada uno?

8.2.- Tres esferas uniformes de masa 4,0 kg, 6,0 kg y 8,0 kg se colocan en

los puntos (0; 3,0) m, ((0; 0) m, y (4,0; 0) m, respectivamente de un sistema de coordenadas rectangulares xy. Calcule la fuerza gravitacional resultante sobre el cuerpo de 6,0 kg, suponiendo que las esferas están aisladas del resto del universo.

8.3.- El asteroide Dactilo, descubierto en 1993, tiene un radio de sólo 700 m

y una masa aproximada de 3,6x1012 kg. Calcule la rapidez de escape de un objeto en la superficie de Dactilo. ¿Podría una persona alcanzar esta velocidad caminando?

8.4.- Un satélite de masa 200 kg se coloca en órbita terrestre a una altura de

200 km sobre la superficie. Con base en lo antes descrito, determine: a) Con una órbita circular, ¿cuánto tarda el satélite para completar una

órbita? b) ¿Cuál es la rapidez del satélite? c) ¿Cuál es la mínima energía de entrada necesaria para poner este

satélite en órbita? Desprecie la resistencia del aire pero incluya el efecto de la rotación

diaria del planeta.

López Frontado, R. (2011). Física general I, Texto UNA. 2da edición. Caracas: UNA

Page 300: Fisica General i Una

314

RESPUESTA A LOS EJERCICIOS PROPUESTOS Estos resultados se corresponden con los de los ejercicios propuestos en el

desarrollo de la unidad. Respuesta al ejercicio propuesto 8.1 DATOS: ML = 7,36 x1022 kg, RL = 1,74 x106 m, MT = 5,98 x1024 kg,

RT = 6,37 x106 m. SOLUCIÓN: Se aplica la ley de Gravitación Universal, suponiendo que el movimiento del

cuerpo alrededor de la Luna y de la Tierra sean circulares. a) La ley de Gravitación Universal nos dice que la fuerza entre dos

cuerpos está dada por:

G 2

GMmF =r

Si el Peso de un cuerpo lo igualamos a la Fuerza Gravitatoria, se tiene,

2

GMmmg=r

de donde se tiene que 2

GMg=r

.

Aplicando este resultado para un cuerpo en la Luna se tiene

-11 22L

L L2 6 2 2L

GM 6,67 10 7,36 10 mg = = g =1,622 R (1,74 10 ) s

× × × ⇒×

La aceleración en la Luna es igual a 1,622 m/s2. b) La expresión del período T en función de g está dado por

L TL T

L LT =2 (1) y T =2 (2)g g

π π

Dividiendo (1) entre (2), y despejando TL, se tiene

22 2 TL T L

L

g 2 9,81T =T = T =4,92 sg 1,622

!"

López Frontado, R. (2011). Física general I, Texto UNA. 2da edición. Caracas: UNA

Page 301: Fisica General i Una

315

c) Se tiene que T100(mg) =100 N m= =10,19 kg9,81

! .

El peso en la Luna será (mg)L = 10,19 x 1,622,

⇒ (mg)L = 16,53 N

El peso del cuerpo en la Luna es igual a 16,53 N.

Respuesta al ejercicio propuesto 8.2 DATOS: m1 = 0,6 kg, m2 = 0,8 kg, m3 = 0,04 kg. SOLUCIÓN:

Figura 8.10

Aplicando la ley de Gravitación Universal, se obtienen las fuerzas ejercidas por las partículas 1 y 2 sobre la 3, así se tiene, la magnitud de cada una de ellas, esto es,

-11

1 313 2 2

13-11

13

Gm m 6,67×10 ×0,6×0,04F = =r (0,4)

F = 1,0005×10 N

-11

2 323 2 2

23-11

23

Gm m 6,67×10 ×0,8×0,04F = =r (0,5)

F = 0,8538×10 N

De la figura (8.7), se obtiene el valor del ángulo θ, así se tiene

-1 0,4= tan = 53º0,3

! !" #$% &

' (

θ θ⎡ ⎤⎣ ⎦

r

r

r

3 23 13 23

-113

-113

La fuerza resultante ˆ ˆF = F cos i- (F +F sen ) j

ˆ ˆF = 0,8538cos53º i- (1,0005 + 0,8538sen53º ) j ×10 N

ˆ ˆF = (5,1383 i-16,824 j)×10 N

López Frontado, R. (2011). Física general I, Texto UNA. 2da edición. Caracas: UNA

Page 302: Fisica General i Una

316

La magnitud y la dirección de esta fuerza son:

2 2 -11 -113 3

-1

F = (5,1383) + (-16,824) ×10 F = 17,59×10 N-16,824= tan = -73º= 286º5,1383

! !

"

# $"% &

' (

Respuesta al ejercicio propuesto 8.3 DATOS: m1 = 0,8 kg, m2 = 4,0x10-3 kg, FG = 1,3x10-10 N R = 0,04 m, RT = 6380x103 m. SOLUCIÓN: a) Aplicando la ley de Gravitación Universal, se tiene

2

G1 2G 2

1 2-3 2 2

-11-3 2

F RGm mF = , G =

m mR1,3×10 (0,04) N.mG = G = 6,5×10 0,8×4,0×10 kg

!

"

b) El peso es la fuerza con que la Tierra atrae a un objeto, esto es,

2

T 1 T1 T2

T3 2

24T T-11

GM m gRm g = M =

GR

9,8(6380×10 )M = M = 6,137×10 kg6,5×10

!

"

Respuesta al ejercicio propuesto 8.4 DATOS: M = 500 kg, R = 0,40 m, m = 50,0 g, G = 6,67x10-11 N.m2/kg2 SOLUCIÓN: La ley de Gravitación Universal establece que G 2

GMm ˆF =- rr

!.

a) La fuerza gravitatoria sobre m, cuando está situada a 1,50 m del centro

de la esfera, es

-11-10

G G2 2

GMm 6,67 10 500 0,05 ˆF = = F =-7,41 10 r Nr (1,5)

× × × ⇒ ×r

López Frontado, R. (2011). Física general I, Texto UNA. 2da edición. Caracas: UNA

Page 303: Fisica General i Una

317

b) Cuando r = R = 0,40 m, la fuerza gravitatoria es

-11

-8G G2 2

GMm 6,67 10 500 0,05 ˆF = = F =-1,042 10 r Nr (0,4)

× × × ⇒ ×!

Respuesta al ejercicio propuesto 8.5 DATOS: ma = 75 kg, r = 4100 km, ar = 2,6 m/s2 SOLUCIÓN: a) El peso aparente cuando su vehículo se mueve a velocidad constante

está dado por su peso en la luna, esto es

( )-11 22

L aa L 2 23

a L

GM m 6,67×10 ×7,35×10 ×75m g = =r 4100×10

m g =21,87 N⇒

b) Su peso aparente cuando su vehículo está acelerando hacia la Luna:

a L a r a L a rm g -P'=m a P'=m g -m a

P'=21,87-75×2,6 P'=-173 N⇒

Alejándose de la Luna.

Respuesta al ejercicio propuesto 8.6 DATOS: ms = 1000 kg, vs = 5200 m/s, MT = 5,97x1024 kg,

RT = 6,38x106 m. SOLUCIÓN: a) El movimiento del satélite es circular, por lo tanto se tiene que la fuerza

centrípeta es igual a la fuerza de Gravitación Universal, esto es:

( )

2S T s T

2 2

-11 247

2

mv GM m GM= r=

r r v6,67×10 ×5,97×10r= r=1,473×10 m

5200→

López Frontado, R. (2011). Física general I, Texto UNA. 2da edición. Caracas: UNA

Page 304: Fisica General i Una

318

El período de rotación es:

( )74

2 1,473×102 2 rT= = = T=1,779×10 sv 5200

ππ πω

b) La aceleración radial del satélite es

( )22s

r r7 2

5200v ma = = a =1,836 r 1,473×10 s

c) El trabajo necesario es la diferencia entre la energía mecánica total E2

(cuando el satélite está en órbita), y la energía mecánica E1 (cuando el satélite está en la plataforma de lanzamiento), esto es

2 1

T S T S1 i i 2

T

W=E -E , GM m GM m

donde E =K +U =- y E =-R 2r

( )

T S T ST S

T T

-11 24 37 6

17 -7 10

GM m GM m 1 1W=- - - =-GM m -2r R 2r R

1 1W=-6,67 10 (5,97 10 )(10 ) -2×1,473 10 6,38 10

W=-3,982 10 -1,2279 10 W=4,889×10 J

⎛ ⎞ ⎛ ⎞⎜ ⎟ ⎜ ⎟⎝ ⎠ ⎝ ⎠

⎛ ⎞× × ⎜ ⎟× ×⎝ ⎠× × ⇒

Respuesta al ejercicio propuesto 8.7 DATOS: m = 9,979x103 kg, T = 119x60 = 7140 s, RL = 1,849x106 m. SOLUCIÓN: a) La masa ML de la Luna se obtiene considerando que su movimiento es

circular, por lo tanto la fuerza de atracción gravitatoria que ejerce la Luna sobre la nave espacial es la fuerza centrípeta que actúa sobre ella, esto es

2 2 2

-4L2L

GM m mv mR 2 2 rad= = , donde = = =8,8 10 R R T 7140 sR

ω π πω ω→ ×

Entonces se tiene que

2 3 -4 2 6 322

L L-11

R (8,8 10 ) (1,849 10 )M = = M =7,339 10 kg.G 6,67 10

ω × × ⇒ ××

López Frontado, R. (2011). Física general I, Texto UNA. 2da edición. Caracas: UNA

Page 305: Fisica General i Una

319

b) La velocidad orbital de la nave está dada por:

6 -4 mv=R =(1,849 10 )(8,8 10 ) v=1627 s

! " " #

La energía está dada por 2esc Lmv GM m

E=K+U= - ,2 R

2 Lesc

GMdonde v =

R

L L L

-11 2210

6

GM m GM m GM m E= - =-

2R R 2R(6,67 10 )(7,339 10 )(99979)E=- E=- 1,321 10 J

2(1,849 10 )

!

" "# "

"

Respuesta al ejercicio propuesto 8.8 DATOS: m1 = 4,0 kg, en (1,0 m; 0), m2 = 3,0 kg, en (0; -0,5 m) mo = 0,01 kg, en (0; 0), G=6,67x10-11 N.m2/kg2

SOLUCIÓN:

Figura 8.11

La ley de Gravitación Universal nos dice:

1 2

G 2

Gmm ˆF = rr

r

a) La fuerza sobre mo es

( ) ( )( )

-11 -11

1 2 2 2

-12

6,67×10 ×4×0,01 6,67×10 ×3×0,01ˆ ˆ ˆ ˆF=F i+F j= i - j1,0 0,5

ˆ ˆ F= 2,668i-8,004j 10 N⇒

!

!

Su magnitud es

2 2 -12 -12F= (2,668) +(-8,004) 10 F=8,437 10 N! " !

La dirección es

-1 o-8,004 =tan =-71,572,668

! !" #$% &

' (

López Frontado, R. (2011). Física general I, Texto UNA. 2da edición. Caracas: UNA

Page 306: Fisica General i Una

320

b) El trabajo mínimo es

o

-121 2o o

1 2

-12

W=-ΔU=U -U , U =0

m m 4 3W=U =Gm + =6,67×10 ×0,01 +r r 1,0 0,5

W=6,67×10 J

! !

" # " #$ % $ %

& '& '

(

Respuesta al ejercicio propuesto 8.9 SOLUCIÓN:

El campo gravitatorio ejercido por las partículas de masas M en el punto P, se obtiene a partir de la figura siguiente. El campo gravitatorio está EG está dado por:

GG 2

o

F GM ˆE = =- rm r

rr

Los campos gravitatorios EG, producidos por las dos masas tienen igual

magnitud. El campo gravitatorio EG, como cantidad vectorial que es, se descomponen en un componente a lo largo del eje x y otra en la dirección del eje y.

Las componentes de los campos en la dirección del eje y, se anulan uno a

otro, entonces el campo gravitatorio resultante, sólo tiene componentes en la dirección del eje x. Esto es

( )

2 2G 2

G 32 2 2

2GM xˆE =- cos i, r= a +x , y cosθ =rr

2GM.x ˆ E =- ia +x

θ

r

r

Respuesta al ejercicio propuesto 8.10 DATOS: ms = 100 kg, hs = 2,0x106 m, MT = 5,98x1024 kg,

RT = 6,37x106 m. SOLUCIÓN: a) La energía potencial del sistema Satélite-Tierra es

-11 24

T s6

T s9

GM m 6,67x10 x5,98x10 x100U=- =-R +h 8,37x10

U=-4,77x10 J⇒

López Frontado, R. (2011). Física general I, Texto UNA. 2da edición. Caracas: UNA

Page 307: Fisica General i Una

321

b) La fuerza gravitacional que ejerce la Tierra sobre el Satélite es

( ) ( )-11 24

T sG 2 26

T s

G

GM m 6,67x10 x5,98x10 x100F = =R +h 8,37×10

F =569,4 N⇒

Respuesta al ejercicio propuesto 8.11 DATOS: m1 = 2,53 kg, m2 = 7,16 kg, m3 = 0,212 kg, d = 1,56 m,

d’ = 0,42 m, G = 6,67x10-11 N.m2/kg2. SOLUCIÓN:

Figura 8.12

El trabajo realizado para mover la esfera m3 desde A hasta B;

W = - ΔU = UA - UB

Las energías potenciales en los puntos A y B son:

1 3 2 3 1 3 2 3A B

Gm m Gm m Gm m Gm mU = + , y U = +

(d- d') d' d' (d - d')

Entonces

[

]

1 3 2 3

-11

1 1 1 1W = G m m - +m m -d- d' d' d' d - d'

1 1 1 1W = G 2,53×0212 - +7,16×0,212 -1,14 0,42 0,42 1,14

W = 6,67×10 2,283 - 0,807 W

⎡ ⎤

⎛ ⎞ ⎛ ⎞

⎢ ⎥⎜ ⎟ ⎜ ⎟

⎝ ⎠ ⎝ ⎠⎣ ⎦

⎡ ⎤

⎛ ⎞ ⎛ ⎞

⎢ ⎥⎜ ⎟ ⎜ ⎟

⎝ ⎠ ⎝ ⎠⎣ ⎦

López Frontado, R. (2011). Física general I, Texto UNA. 2da edición. Caracas: UNA

Page 308: Fisica General i Una

322

RESPUESTA A LOS EJERCICIOS DE AUTOEVALUACIÓN Respuesta al ejercicio de autoevaluación 8.1 m1 = 3,003 kg, m2 = 1,997 kg. Respuesta al ejercicio de autoevaluación 8.2

R -10 R -10ˆ ˆF =(2,001i+1,7787j) 10 N; F =2,6773 10 N; =41,63º!" "!

Respuesta al ejercicio de autoevaluación 8.3 ve =0,83 m/s; si Respuesta al ejercicio de autoevaluación 8.4 a) t = 5,30x103 s, b) vS = 7,79x103 m/s, c) Emin = 6,43x109 J

López Frontado, R. (2011). Física general I, Texto UNA. 2da edición. Caracas: UNA

Page 309: Fisica General i Una

323

UNIDAD 9

MOVIMIENTO OSCILATORIO

En esta unidad se estudian aquellos movimientos que se repiten una y otra

vez, entre los cuales existen muchos ejemplos en la naturaleza, tales como el péndulo oscilante de un reloj, las vibraciones sonoras producidas por un instrumento musical, el movimiento de nuestro propio planeta alrededor de su propio eje y el que realiza alrededor del Sol y el movimiento periódico de los pistones de un motor de automóvil. A este tipo de movimiento se le denomina movimiento periódico u oscilación.

En esta unidad se estudiará con más énfasis en las llamadas oscilaciones

armónicas, por presentar la particularidad que pueden ser expresadas como función seno o coseno del tiempo, o como una combinación de estas dos funciones. También se tiene que la característica de un cuerpo que tiene un movimiento periódico es que su posición es estable, esto es que al alejarlo de esa posición de equilibrio y soltarlo, actúa sobre él una fuerza o un momento de torsión que lo lleva nuevamente a su posición de equilibrio. Además se tiene que estos pequeños movimientos periódicos de un sistema dado alrededor de su estado de equilibrio son oscilaciones armónicas.

También es importante señalar las oscilaciones armónicas libres y las

forzadas, y las oscilaciones con disipación. Un sistema mecánico puede oscilar más efectivamente a una frecuencia

determinada. Este fenómeno se conoce con el nombre de resonancia, el cual es de gran importancia para la Física y la Ingeniería, en posteriores estudios, tales como el de las ondas, el sonido, las corrientes eléctricas, la luz, vibraciones estructurales, etc.

CONOCIMIENTOS PREVIOS: Para la comprensión de esta unidad, el estudiante debe poseer el siguiente

conocimiento: 1.- Ecuación del movimiento de un sistema de partículas, aplicada a un

sistema rígido. 2.- Trabajo y energía para un sistema de partículas. 3.- Momento lineal y momento angular de un sistema de partículas.

López Frontado, R. (2011). Física general I, Texto UNA. 2da edición. Caracas: UNA

Page 310: Fisica General i Una

324

9.1 OBJETIVO Aplicar los conceptos de la dinámica de sistemas de partículas al

estudio de las oscilaciones mecánicas.

9.2 RECOMENDACIONES PARA EL ESTUDIO DEL CONTENIDO En esta unidad, el estudiante debe comprender claramente cómo se aplican

los principios y leyes fundamentales de la mecánica en el estudio del comportamiento de sistemas con Movimiento Periódico u Oscilatorio.

Para una mejor comprensión del contenido de la unidad, realice la lectura de

los conceptos relacionados con el movimiento oscilatorio de los cuerpos, aplicando las ideas estudiadas en el movimiento de sistemas de partículas y las leyes fundamentales de la mecánica en el estudio de los sistemas de esta unidad, y los ejemplos presentados en el texto, utilice las técnicas de lectura que le facilite la comprensión del tema.

9.3 CONTENIDO 9.3.1 MOVIMIENTO ARMÓNICO SIMPLE. 9.3.2 ENERGÍA EN MOVIMIENTO ARMÓNICO SIMPLE. 9.3.4 MOVIMIENTO AMORTIGUADO. 9.3.4 MOVIMIENTO FORZADO. 9.3.1 MOVIMIENTO ARMÓNICO SIMPLE En general, si una partícula se mueve a lo largo del eje x, se dice que el

movimiento es armónico simple si el desplazamiento de la partícula, desde el punto de equilibrio x, varía de acuerdo a la expresión:

x=Acos(ωt+φ) (9.1)donde A, ω, y φ son constante.

A, representa la amplitud del movimiento, y es el desplazamiento máximo de

la partícula desde su posición de equilibrio. ω, es la frecuencia angular del movimiento, y sus unidades son rad/s.

López Frontado, R. (2011). Física general I, Texto UNA. 2da edición. Caracas: UNA

Page 311: Fisica General i Una

325

ϕ , constante de fase (o ángulo de fase), y está determinado por el

desplazamiento y las velocidades iniciales de la partículas. (ωt + ϕ ) se conoce como fase del movimiento. Como en la ecuación (9.1), la función trigonométrica es periódica y se repite

así mismo cada vez que ωt se incrementa en 2π. El movimiento de la partícula es periódico.

Por lo tanto es conveniente definir lo que se entiende por el Período T del

movimiento, el cual se define como el tiempo que tarda la partícula en completar un ciclo. El período se expresa en segundos (s). Matemáticamente se expresa por:

tiempo tperiodo T= T= (9.2)

n de oscilaciones n!

La frecuencia f del movimiento se define como el número de oscilaciones

que efectúan los cuerpos en la unidad de tiempo. La frecuencia se expresa en rev ciclos ó ó en Hertz (Hz).s s

Matemáticamente se expresa por la siguiente

ecuación: n de oscilaciones nfrecuencia= f= (9.3)Unidad de tiempo t

!

De las ecuaciones (9.2) y (9.3), se puede observar que el período es el

inverso de la frecuencia, esto es 1T=f

La relación entre la velocidad angular y el período, y/o la frecuencia está

dada por: 2= =2 f (9.4)Tπω π

La velocidad lineal de una partícula que experimenta un movimiento armónico

simple es

dxv= =-ωAsen(ωt+φ) (9.5)dt

López Frontado, R. (2011). Física general I, Texto UNA. 2da edición. Caracas: UNA

Page 312: Fisica General i Una

326

La aceleración de la partícula es

2

2

dva= =-ω Acos(ωt+φ)dt

a=-ω x (9.6)

La ecuación (9.1) describe el movimiento armónico simple de una partícula en

general. Ahora veamos cómo se obtienen las constantes del movimiento. La frecuencia angular natural se evalúa a partir de la ecuación (9.4). Las constantes A y φ se obtienen a partir de las condiciones iniciales, esto es cuando el oscilador en t = 0.

Suponiendo que en t = 0, se tiene que x(0) = A y v(0) = 0, esto es

x(0)=Acos =A,v(0)=- Asen =0

ϕω ϕ

Estas condiciones se satisfacen sabiendo que sen φ=0, de donde se tiene φ =

0, dando que cos φ = 1. Se tiene entonces que la ecuación de posición y de la velocidad de la partícula están dada por:

x(t)=Acos t,v(t)=- Asen t

!! !

Suponiendo ahora que en t = 0, la partícula pasa por la posición de equilibrio

con una velocidad vo. Así se tiene que a partir de las condiciones iniciales x(0) = 0, v(0) = vo, se tiene que

o

x(0)=Acos =0v(0)=- Asen =v

!" !

La primera de estas condiciones se satisfacen para =2πϕ ± , debido a que

cos =0! . La segunda condición nos indica que ovA=ω

± . Como la velocidad inicial es

positiva, la amplitud debe ser positiva, y por lo tanto =-2!

" . Se tiene entonces que la

expresión para la posición y la velocidad de la partícula están dada por:

o

o

vx(t)= cos( t- ),2

v(t)=-v sen( t- )2

!"

"!

"

López Frontado, R. (2011). Física general I, Texto UNA. 2da edición. Caracas: UNA

Page 313: Fisica General i Una

327

A partir de esta ecuación se puede observar que en los sistemas que

presentan un movimiento armónico simple, sus aceleraciones son proporcionales a su desplazamiento y está dirigido en dirección opuesta al desplazamiento, a partir de alguna posición de equilibrio.

MOVIMIENTO DE UN CUERPO UNIDO A UN RESORTE

Como un ejemplo de movimiento armónico simple se tiene el caso del sistema formado por un bloque, libre de moverse sobre una superficie horizontal sin fricción, unido a un resorte, como se muestra en la figura (9.1). Se supone que el resorte tiene masa despreciable y está sujeto en un extremo de un bloque de masa m y su otro extremo sujeto a una pared fija. Como se muestra en la figura (9.1a) el bloque se desplaza hacia la derecha estirando al resorte desde la posición de equilibrio (x>0), la fuerza ejercida por el resorte actúa hacia a la izquierda. En la figura (9.1b) el bloque está en su posición de equilibrio, o sea que el resorte no está ni estirado ni comprimido.

Figura 9.1

Bloque unido a resorte que se mueve en una superficie sin fricción.

En la figura (9.1c) el bloque se desplaza hacia la izquierda (x<0), la fuerza

ejercida por el resorte actúa a la derecha. Observamos que, cuando el bloque se desplaza una distancia x desde su posición de equilibrio, el resorte ejerce una fuerza que es proporcional a la distancia, y está dada por la ley de Hooke, esto es:

eF =kx Esta fuerza ejercida por el resorte se denomina fuerza restauradora del

resorte, porque siempre está dirigida hacia la posición de equilibrio, y por lo tanto, opuesta al desplazamiento desde su posición de equilibrio.

Al aplicar la 2ª ley de Newton al movimiento del bloque, se tiene:

x x r

x x

F =ma F =ma

k -kx=ma a =- x (9.7)m

x!

López Frontado, R. (2011). Física general I, Texto UNA. 2da edición. Caracas: UNA

Page 314: Fisica General i Una

328

Comparando la ecuación (9.6) con la ecuación (9.7), se tiene que la velocidad natural de oscilación para el sistema bloque resorte está dado por:

2 kω = (9.8)

m

La ecuación (9.7) también se puede escribir en la siguiente forma

22

2

d x +ω x=0 (9.9)dt

Una de las soluciones de la ecuación diferencial (9.9), puede ser

x=Acos(ωt+φ)

El período y la frecuencia de oscilación, para el sistema bloque resorte, son:

La velocidad y la aceleración del bloque, en su movimiento sujeto al resorte,

se obtienen a partir de las ecuaciones (9.5) y (9.6)

dxv= =-ωAsen(ωt+φ) (9.11)dt

2dva= =-ω Acos(ωt+φ) (9.12)dt

De acuerdo con las ecuaciones (9.11), y (9.12), las funciones senos y

cosenos varían en 1± , los valores máximos de la velocidad son Aω± , y los valores máximos de la aceleración son 2Aω± . Por lo tanto los valores máximos de las magnitudes de la velocidad y la aceleración son

max

2max

kv = A= A (9.13)mka = A= A (9.14)m

!

!

López Frontado, R. (2011). Física general I, Texto UNA. 2da edición. Caracas: UNA

Page 315: Fisica General i Una

329

EL PÉNDULO Otro sistema mecánico que representa un ejemplo del movimiento armónico

simple lo constituye el movimiento del péndulo simple. El péndulo simple consiste de una plomada parecida a una partícula de masa m suspendida por una cuerda ligera de longitud L, donde el extremo libre de la cuerda está fijo, como se muestra en la figura (9.2).

El movimiento se produce en un plano

vertical, y se impulsa por la fuerza gravitacional. Las fuerzas que actúan sobre la plomada son la tensión T

! ejercida por la cuerda y la fuerza

gravitacional mg!

. La componente tangencial de la fuerza gravitacional (mg sen θ) actúa siempre hacia θ = 0, opuesto al desplazamiento. Por lo tanto, la fuerza restauradora es la componente tangencial de la fuerza gravitacional ( gF =-mgsenθ ).

Aplicando la segunda ley de Newton para

el movimiento en la dirección tangencial, se tiene:

2

t 2

d sF=-mgsenθ=m (9.15)dt∑

Figura 9.2 Péndulo Simple.

El movimiento del péndulo simple oscila alrededor de θ = 0, se considera que es un movimiento armónico simple sólo si el ángulo θ que forma la cuerda con la vertical es pequeño (<10º).

Donde s es el desplazamiento de la masa m a lo largo del arco. Como s = Lθ,

y L es constante, la ecuación (9.11) se puede escribir como :

2

2

d θ g+ senθ=0 (9.16)dt L

El segundo término de la ecuación (9.16) es proporcional a sen θ en lugar de

θ, por tanto, el movimiento no es armónico simple. Sin embargo, si se supone que θ sea pequeño (<10º), se tiene que sen θ ≈ θ, entonces la ecuación de movimiento se puede escribir:

2

2

d θ g+ θ=0 (9.17)dt L

Donde la velocidad natural de oscilación está dada por ω2 = g/L. La solución

para la ecuación diferencial (9.13) es:

maxθ=θ cos(ωt+φ).

López Frontado, R. (2011). Física general I, Texto UNA. 2da edición. Caracas: UNA

Page 316: Fisica General i Una

330

El período del movimiento y la frecuencia de oscilación son:

PÉNDULO FÍSICO Si ahora se tiene un cuerpo, por ejemplo, un cuadro que cuelga de un clavo

en la pared del cual puede oscilar libremente sin fricción, al desplazar el cuadro de su posición de equilibrio, y soltarse, se observa que el cuadro oscila respecto a su posición de equilibrio. Estos tipos de cuerpos que pueden oscilar libremente, alrededor de su posición de equilibrio, se denominan péndulo físico.

Consideremos un cuerpo rígido que hace pivote

en un punto O, que está a una distancia d de su centro de masa CM, como se muestra en la figura (9.3). La fuerza gravitatoria gF =mg

r r produce un

momento de torsión alrededor de un eje que pasa por el punto O, siendo la magnitud de ese momento de torsión igual a mgd sen! , donde θ es el ángulo que forma la distancia d con el eje vertical. Aplicando la forma rotacional de la 2ª ley de Newton, =I ,! "# se tiene:

Figura 9.3

Péndulo físico con pivote en O.

2

2

d-mgdsen =Idt

!!

Si suponemos que θ es pequeño, entonces se tiene que senθ θ≈ ; entonces

la ecuación de movimiento se puede escribir como

2 22

2 2

d mgd d+ = + =0 (9.19)dt I dt

! !! " !# $

% &' (

La ecuación (9.19) nos indica que el movimiento del cuerpo es un movimiento

armónico simple. La solución a la ecuación diferencia (9.19) está dada por

máx= cos( t+ ),! ! " #

donde máx! es la máxima posición angular y ω mgd=I

, es la velocidad

angular.

López Frontado, R. (2011). Física general I, Texto UNA. 2da edición. Caracas: UNA

Page 317: Fisica General i Una

331

El período está dado por

2 IT= =2 (9.20)mgd

!!

"

9.3.2 ENERGÍA EN MOVIMIENTO ARMÓNICO SIMPLE Considerando el sistema bloque-resorte, visto anteriormente, y suponiendo

que no existe fuerza de fricción, se tiene que la energía mecánica del sistema es E = K + U = Constante. La energía cinética del sistema es

2 2 2 21 1K= mv = mω A sen (ωt+φ) (9.21)2 2

La energía potencial del sistema es

2 2 21 1U= kx = kA cos (ωt+φ) (9.22)2 2

La energía mecánica del sistema es

2 2 2 2 2

2 2 2

2

1 1E=K+U, E= mω A sen (ωt+φ)+ kA cos (ωt+φ)2 2

1E= kA sen (ωt+φ)+cos (ωt+φ)2

1 E= kA (9.23)2

! "# $

%

La energía mecánica total de un oscilador armónico simple es una constante

del movimiento, y es proporcional al cuadrado de la amplitud. 9.3.3 MOVIMIENTO AMORTIGUADO Hasta los momentos se han considerado problemas oscilatorios ideales,

donde la amplitud del movimiento se mantiene constante. Sin embargo, la realidad nos enseña que si un bloque que está sujeto a un resorte se ponen a vibrar, la amplitud de ese movimiento después de cierto tiempo va disminuyendo hasta hacerse igual a cero, cuando se detiene. Lo mismo sucede cuando ponemos a oscilar un péndulo: al cabo de cierto tiempo se detiene en su posición de equilibrio.

López Frontado, R. (2011). Física general I, Texto UNA. 2da edición. Caracas: UNA

Page 318: Fisica General i Una

332

En consecuencia, la energía mecánica del sistema disminuye en el tiempo, y se dice que el movimiento es amortiguado. Para ilustrar estas situaciones, consideremos el movimiento de un sistema bloque resorte, introduciendo previamente al bloque en un recipiente contenedor de un líquido. Como se muestra en la figura (9.4):

Figura 9.4

El bloque sostenido por resorte y sumergido en un líquido viscoso constituye un ejemplo de movimiento amortiguado.

Cuando el cuerpo se mueve a través de un medio líquido, se produce una fuerza retardadora que es proporcional a la rapidez del objeto en movimiento y actúa en la dirección opuesta al movimiento. La fuerza retardadora proporcionada por el líquido se puede escribir por

RdyF =-λv=-λ (9.24)dt

Donde λ es una constante llamada coeficiente de amortiguamiento. Aplicando la segunda ley de Newton, se tiene

2

y 2

dy d yF =-ky-λ =m (9.25)dt dt!

La solución de la ecuación (9.25) es

λ- t

2my=Ae cos(ωt+φ) (9.26) donde la frecuencia angular de oscilación es

2 2

2o

k λ λω= - = ω - (9.27)m 2m 2m

⎛ ⎞ ⎛ ⎞⎜ ⎟ ⎜ ⎟⎝ ⎠ ⎝ ⎠

y ωo

2 = k/m es la frecuencia natural de oscilación del sistema.

9.3.4 MOVIMIENTO FORZADO Para compensar la pérdida de energía en un sistema amortiguado, se aplica

una fuerza externa que efectúe trabajo positivo en el sistema, manteniendo constante de esta manera la amplitud del movimiento. El movimiento resultando se denomina oscilaciones forzadas.

López Frontado, R. (2011). Física general I, Texto UNA. 2da edición. Caracas: UNA

Page 319: Fisica General i Una

333

Un ejemplo muy común para obtener un movimiento forzado es aplicarle a un movimiento amortiguado una fuerza externa senoidal, como la representada por la ecuación siguiente:

ext oF =F cos ωt,

donde ω es la frecuencia angular de la fuerza externa Fext,y Fo es un valor constante. La ecuación del movimiento, aplicando la 2ª ley de Newton es

o2

o2

F cosωt-kx-λv=maF k λ dx d xcosωt- x- = (9.28)m m m dt dt

La solución de esta ecuación, cuando el sistema está en estado estable, es

ext

22 2

o

x=Acos (ωt+φ)donde la amplitud es

FmA= (9.29)

λω(ω -ω )+m

⎛ ⎞⎜ ⎟⎝ ⎠

y ωo

2 = k/m es la frecuencia angular del oscilador no amortiguado. De acuerdo con esta ecuación, el sistema oscila en la frecuencia angular ω de la fuerza externa. La amplitud aumenta considerablemente cerca de la frecuencia natural ωo, esto se conoce como resonancia, y por eso algunas veces ωo recibe el nombre de frecuencia de resonancia del sistema.

A continuación se presentan unos ejemplos de problemas donde Ud. podrá

visualizar las aplicaciones de los conceptos tratados en esta unidad. Pero antes vea las Sugerencias y las Recomendaciones presentadas para resolver estos problemas.

RECOMENDACIONES PARA RESOLVER PROBLEMAS DE DINÁMICA

DEL SISTEMAS DE PARTÍCULAS EN MOVIMIENTOS PERIÓDICOS u OSCILATORIOS.

La estrategia para resolver problemas que se refieran a Sistemas Periódicos

u Oscilatorios, consiste en tener presente la diferencia existente entre las

López Frontado, R. (2011). Física general I, Texto UNA. 2da edición. Caracas: UNA

Page 320: Fisica General i Una

334

cantidades que representan las propiedades físicas del sistema, y las que describen lo que ocurre cuando el sistema se pone en movimiento de una forma específica.

IDENTIFICAR los conceptos relevantes: Un sistema que oscila o vibra tiene un movimiento armónico simple (MAS) sólo si la fuerza de restitución es directamente proporcional al desplazamiento. Es conveniente comprobar previamente que esto se cumple, antes de aplicar cualquiera de los resultados de esta sección.

PLANTEAR el problema siguiendo estos pasos: 1.- Identifique las cantidades conocidas y desconocidas, e indique cuáles

son las incógnitas. 2- Es importante tener presente la diferencia que existe entre las

cantidades que representan las propiedades físicas básicas del sistema (donde se incluyen la masa m, la constante de fuerza k, el período T, la frecuencia f y la frecuencia angular ! ) y las que describen lo que ocurre cuando el sistema se pone en movimiento de una forma específica (tales como la amplitud A, la velocidad máxima vmax, el ángulo de fase ! , los valores de la posición, velocidad y la aceleración en un instante dado).

3.- De ser necesario defina un eje de coordenada para describir el movimiento.

EJECUTAR la solución, como sigue: 1.- Use las ecuaciones correspondiente, ya sea aplicando la 2ª ley de

Newton o el principio de conservación de energía, para obtener las incógnitas.

2.- Si necesita realizar operaciones con el ángulo de fase, tenga cuidado de expresarlo en radianes, al igual que la cantidad t! .

3.- Tenga presente que al dársele la posición inicial xo y la velocidad inicial vo, es necesario determinar el ángulo de fase y la amplitud A. Para ello tome en cuenta que si el cuerpo tiene un desplazamiento inicial xo pero velocidad inicial cero (vo=0), la amplitud es A=xo y el ángulo de fase es ! =0. Si el cuerpo tiene velocidad inicial positiva pero ningún desplazamiento inicial (xo=0), la amplitud es oA=v /ω y el ángulo de fase es =- /2! " .

EVALUAR la respuesta: Compruebe que sus resultados son congruentes. Si supone que ha usado la posición y la velocidad inicial para obtener ecuaciones generales para x y v en el instante t; si sustituye t=0 en estas expresiones, deberá obtener los valores correctos de xo y vo.

López Frontado, R. (2011). Física general I, Texto UNA. 2da edición. Caracas: UNA

Page 321: Fisica General i Una

335

Recuerde que el manejo eficiente del contenido de esta unidad es importante por su repercusión en el contenido de las subsiguientes asignaturas del Plan de Estudios en Ingeniería y en otras ciencias.

Ejemplo 9.1.- Una partícula de 12,3 kg se halla en movimiento armónico simple con una

amplitud de 1,86 mm. La aceleración máxima de la partícula es de 7,93 km/s2. a) Halle el período del movimiento.

b) ¿Cuál es la velocidad máxima de la partícula?

c) Calcule la energía mecánica total del oscilador armónico simple. DATOS: m = 12,3 kg, A = 1,86 x10-3 m, amax = 7,93 Km/s2

SOLUCIÓN: IDENTIFICAR los conceptos pertinentes: Como el movimiento del cuerpo es

armónico simple, se pueden usar aplicar la 2ª ley de Newton y el principio de conservación de energía, para determinar el período T, la velocidad máxima y la energía total de la partícula.

PLANTEAR el problema: Para determinar el período y la velocidad máxima se

usan las ecuaciones que provienen de la 2ª ley de Newton, y para determinar la energía mecánica total se usa el principio de energía.

EJECUTAR la solución del problema: a) Para determinar el período, es necesario conocer la frecuencia angular

de la partícula, así se tiene que la aceleración máxima está dada por

2 2 maxmax

32 3

-3

33

aa = A =

A7,63×10= = 2,025×10 rad1,86×10

2 2pT = = T = 3,102 10 s2,025×10

ω ω

ω ω

πω

⇒ ×

López Frontado, R. (2011). Física general I, Texto UNA. 2da edición. Caracas: UNA

Page 322: Fisica General i Una

336

b) La velocidad máxima de la partícula se obtiene conociendo que maxv =Aω , esto es

-3 3max max

mv = 1,86×10 ×2,025×10 v = 3,77 s

c) La energía mecánica total está dada por:

2 2mv 12,3×(3,77)U = = U = 87,41 J2 2

!

EVALUAR la respuesta: El período del movimiento de la partícula es de 3,102

ms; su velocidad máxima es 3,77 m/s; y la energía mecánica total es de 87,41 J.

Ejemplo 9.2.- Un objeto de masa m está suspendido de un resorte vertical de constante de

fuerza k = 1800 N /m. Cuando se hala de él 2,5 cm y se suelta, oscilando a 5,5 Hz. Calcule:

a) La masa m.

b) El alargamiento correspondiente a la posición de equilibrio del resorte.

c) Las expresiones para x(t), v(t) y a(t). DATOS: k = 1800 N /m, xo = 0,025 m, f = 5,5 Hz. SOLUCIÓN IDENTIFICAR los conceptos pertinentes: Considerando que el movimiento es

armónico simple se aplica la 2ª ley de Newton para su resolución, para determinar la masa del objeto, el alargamiento del resorte con respecto a su posición de equilibrio, y la posición, velocidad y aceleración en el objeto en cualquier instante.

PLANTEAR Y EJECUTAR la solución del problema: a) Para determinar la masa del cuerpo, es necesario obtener previamente

la rapidez angular del mismo, la cual se obtiene considerando la relación entre la frecuencia con que se mueve el cuerpo y la rapidez angular del mismo se tiene:

o orad2 f 2 5,5 34,6s

ω π π ω= = × → =

López Frontado, R. (2011). Física general I, Texto UNA. 2da edición. Caracas: UNA

Page 323: Fisica General i Una

337

La masa se obtiene a partir de 2okω =m

. Entonces la masa del cuerpo

está dado por:

2 2o

k 1800m= = m=1,51 kg.ω (34,6)

!

b) Para determinar el alargamiento del resorte, correspondiente a la

posición de equilibrio, consideremos lo que se muestra en la figura (9.5).

Figura 9.5 Así se tiene que:

3mg 1,51 9,8k mg 8,21 10 mk 1800

−×Δ = → Δ = = ⇒ Δ = ×! ! !

c) Las expresiones de x(t), v(t) y a(t), tomando en cuenta las condiciones

iniciales, están dada por

o o

o o o2o o o

x(t)=x sen( t+ ),v(t)= x cos( t+ )

a(t)=- x sen( t+ )

ω δω ω δω ω δ

-2 -2o

o o

-2o

x(t=0)=2,5 10 , 2,5 10 =x senCond. inic. en t=0

v(t=0)=0, 0= x cos

= y x =2,5 10 2

!

" !

#!

$ % %&'&(

%¡√√√√√√√√√√√√√√√√√√ ¬√√√√√√√√√√√√√

2

2

2

x(t) 2,5 10 sen(34,6 t )2

v(t) 86,5 10 cos(34,6 t )2

a(t) 2992,9 10 sen(34,6 t )2

π

π

π

⎧ = × +⎪⎪⎪⇒ = × +⎨⎪⎪ = − × +⎪⎩

López Frontado, R. (2011). Física general I, Texto UNA. 2da edición. Caracas: UNA

Page 324: Fisica General i Una

338

EVALUAR la respuesta: Las ecuaciones correspondientes a la posición x(t), la velocidad v(t) y la aceleración a(t), se obtienen tomando en cuenta las condiciones iniciales.

Ejercicio propuesto 9.1.- Un peso de 40 N se suspende de un resorte con constante elástica de 200

N/m. El sistema no está amortiguado y se impulsa por una fuerza armónica de frecuencia 10 Hz, dando por resultado un movimiento armónico de amplitud 2 cm. Determine el máximo valor de la fuerza aplicada.

Ejercicio propuesto 9.2.- Una masa de 0,5 kg sujeta a un resorte de constante elástico de 8 N /m, vibra,

sobre una superficie horizontal lisa, con un movimiento armónico simple con una amplitud de 10 cm. Calcule:

a) El valor de la rapidez y aceleración máxima.

b) La rapidez y la aceleración cuando la masa se encuentra en x = 6 cm

de la posición de equilibrio.

c) El tiempo que le toma a la masa en moverse de x = 0 hasta x = 8 cm.

Ejercicio propuesto 9.3.- Una masa de 2,0 kg se une a un resorte y se coloca sobre una superficie

horizontal lisa. Se necesita una fuerza horizontal de 20,0 N para mantener la masa en reposo cuando se jala 20 cm a partir de su posición de equilibrio. La masa se suelta después desde el reposo con un desplazamiento inicial de 30 cm y subsecuentemente experimenta oscilaciones armónicas simples.

Con base en lo antes expuesto, encuentre: a) La constante elástica del resorte.

b) La frecuencia de las oscilaciones.

López Frontado, R. (2011). Física general I, Texto UNA. 2da edición. Caracas: UNA

Page 325: Fisica General i Una

339

c) La rapidez máxima de la masa, ¿dónde ocurre la rapidez máxima?

d) La energía total del sistema oscilante.

Ejercicio propuesta 9.4.-

Figura 9.6

En la figura se muestra un bloque de masa M, que descansa sobre una superficie horizontal lisa y está conectado a un resorte horizontal con una constante de fuerza k. El otro extremo del resorte está fijo a una pared. Un segundo bloque de masa m está sobre el primero. El coeficiente de fricción estática entre los bloque es µe. Determine la amplitud de oscilaciones máxima que no hace que el bloque superior resbale.

Ejercicio propuesto 9.5.- Una masa de 1,62 kg estira un resorte vertical 0,315 m. El mismo resorte vibra

con una amplitud de 22 cm cuando 0,25 kg cuelgan de él. En base a lo antes expuesto:

a) Exprese la ecuación que describe este movimiento en función del

tiempo.

b) Calcule cuando tendrá el resorte sus extensiones máximas y mínimas, suponiendo que la masa pasa hacia abajo a través del punto de equilibrio cuando t = 0. (Considere y positiva hacia arriba y negativa hacia abajo).

Ejercicio propuesto 9.6.- Un objeto de 0,2 kg cuelga de un resorte ideal con masa insignificante. Si se

tira del objeto para bajarlo 0,1 m respecto de su posición de equilibrio y se suelta, vibra con un periodo de 1,8 s.

a) ¿Qué rapidez tiene al pasar por su posición de equilibrio?

López Frontado, R. (2011). Física general I, Texto UNA. 2da edición. Caracas: UNA

Page 326: Fisica General i Una

340

b) ¿Qué aceleración tiene cuando está a 0.05 m por encima de dicha posición?

c) Cuando está subiendo, ¿qué tiempo tarda en moverse desde un punto

0,05 m por debajo de la posición de equilibrio a un punto 0,05 m por encima de él?

Ejemplo 9.3.- A un péndulo simple que tiene una longitud de 2,23 m y una masa de 6,74 kg,

se le imprime una velocidad inicial de 2,06 m/s en su posición de equilibrio. Suponga que experimenta un movimiento armónico simple, y determine:

a) Su periodo.

b) Su energía total.

c) Su máximo desplazamiento angular. DATOS: L = 2,23 m, m = 6,74 kg, vo = 2,06 m/s SOLUCIÓN: IDENTIFICAR los conceptos pertinentes: En este problema se considera el movimiento de una partícula sujeta por

medio de una cuerda a un punto O, con respecto al cual está oscilando de una forma armónica, lo que nos permite determinar el período, la energía de la partícula y el máximo desplazamiento angular.

PLANTEAR Y EJECUTAR la solución del problema:

Figura 9.7

a) El período del péndulo simple se obtiene usando la siguiente expresión:

L 2,23T=2 =2 g 9,8

T=2,997 s

! !

"

b) La energía total E = K + U. Por lo tanto, la energía total en el punto más bajo de su trayectoria (o sea cuando pasa por su posición de equilibrio) es:

2 2omv 6,74(2,06)E= = E=14,30 J

2 2⇒

López Frontado, R. (2011). Física general I, Texto UNA. 2da edición. Caracas: UNA

Page 327: Fisica General i Una

341

b) El máximo desplazamiento angular se obtiene, comparando la energía

en los puntos A y B. Como no se desprecia la fuerza de fricción, entonces la energía mecánica se mantiene constante, esto es, que la energía en el punto A y en el punto B son iguales. Esto es, EA = EB. Así se tiene que:

2omv

=E=mgh donde h=L(1-cos )2

!

Sustituyendo, se tiene Ecos =1-mgL

!

!

!

" # " #$ % $ %& &' (' (

)

-1 -1E 14,30=cos 1- =cos 1-mgL 6.74 9,8 2,23

=25,46°=0,444 rad

EVALUAR el resultado obtenido: Los resultados obtenidos, tanto del período

como del desplazamiento se corresponden a los valores proporcionados con los valores dados, ya que para un péndulo de 1,o m de longitud, tanto el período como la desviación tendrían valores menores.

Ejercicio propuesto 9.7.- Una manzana pesa 1,0 N. Si la colgamos del extremo de un resorte largo

con k = 2,0 N/m y masa insignificante, rebota verticalmente con un MAS. Si detenemos el rebote y dejamos que la manzana oscile de lado a lado con un ángulo pequeño, la frecuencia de este movimiento es la mitad de la del rebote. ¿Qué longitud tiene el resorte no estirado?

Ejemplo 9.4.-

Figura 9.8

Una varilla delgada tiene una masa M y una longitud L = 1,5 m. Uno de los extremos de la varilla se sujeta en un pivote fijo y la varilla oscila alrededor del pivote con oscilaciones pequeñas. Encuentre la frecuencia de estas oscilaciones. Si se agrega una partícula de masa M al extremo final de la varilla, ¿en que factor cambiará el período? El momento de inercia de la varilla I = ML2 / 3 DATOS: L = 1,5 m

López Frontado, R. (2011). Física general I, Texto UNA. 2da edición. Caracas: UNA

Page 328: Fisica General i Una

342

SOLUCIÓN: IDENTIFICAR los conceptos pertinentes: El problema se refiere al movimiento de un péndulo físico, cumpliéndose la

ecuación de movimiento para un cuerpo sólido, dada por z=Izτ α∑ , lo cual permite obtener la frecuencia de oscilaciones de la varilla y el factor de cambio del período.

PLANTEAR Y CALCULAR la solución del problema: a) Para determinar la frecuencia con que oscila la varilla, se aplica en la

figura (1) la ecuación de movimiento, así se tiene,

2

o o o 2

L dI Mg sen I .2 dt

θτ α θ= → − =!!

Para pequeñas oscilaciones, se tiene que sen θ ≈ θ, por lo tanto:

θ θθ θ

ω

⎛ ⎞− = → + =⎜ ⎟

⎝ ⎠

= = =

2 2

o 2 2o

2o 2

o

MgL d d MgLI 02 2Idt dt

MgL MgL3 3gSe tiene que2I 2L2ML

También se tiene que oo

1 3g2 f f2 2 2Lωω ππ π

= → = = ,

1 3 9,8 f 0,498Hz2 2 1,5π

×= ⇒ =×

b) Para hallar el factor de cambio del período, se tiene 2

1

TK

T=

Pero 1 11 1T T 2,007 sf 4,98

= = → =

Para determinar el período T2, usamos la figura (2), y se tiene entonces que al aplicar el torque con respecto a O, obtenemos,

2

o 1 1 2

L dI Mg sen MgLsen I2 dt

θτ α θ θ= → − − =!!

Pero 2 2

21 1ML 4MLI ML I3 3

= + → =

Al sustituir en la ecuación del momento, se tiene , 2

222

d 9g 9g08L 8Ldt

θ θ ω⎛ ⎞+ = =⎜ ⎟⎝ ⎠

2 22

2 8L 8 1,5T 2 2 T 2,32 s9g 9 9,8

π π πω

×= = = → =×

López Frontado, R. (2011). Física general I, Texto UNA. 2da edición. Caracas: UNA

Page 329: Fisica General i Una

343

El factor de cambio del período es

2

1

T 2,32K K 1,154T 2,01

= = ⇒ =

EVALUAR los resultados obtenidos: Se observa como varía el período del péndulo físico cuando al mismo se le

agrega una masa M en el extremo del péndulo en una factor igual a 1,154.

Ejercicio propuesto 9.8.-

Figura 9.9

Considere una barra delgada con masa M = 4 kg y de longitud L = 1,2 cm, pivoteada en un eje horizontal libre de fricción en el punto P que está situado a una distancia L/4 desde un extremo, como se muestra en la figura. Haga el diagrama de cuerpo libre correspondiente. a) Obtenga una ecuación que dé la aceleración angular α en función del ángulo θ. a) Determine el período para pequeñas amplitudes de oscilación respecto de la vertical. El momento de inercia de la barra respecto al centro de masa es IC = ML2 / 12

Ejercicio propuesto 9.9.- Un adorno navideño con forma de esfera sólida de masa M = 0,015 kg y radio

R = 0,05 m, es colgado de una rama con un trozo de alambre unido a la superficie de la esfera. Si el adorno se desplaza una distancia corta y se suelta, oscila como un péndulo físico. Calcule el período de la esfera en su movimiento oscilatorio. El momento de inercia de la esfera respecto al pivote es la rama es 7MR2 / 5.

López Frontado, R. (2011). Física general I, Texto UNA. 2da edición. Caracas: UNA

Page 330: Fisica General i Una

344

Ejercicio propuesto 9.10.- Un aro circular de 65,3 cm de radio y 2,16 kg de masa está suspendido de un

clavo horizontal. Entonces: a) Halle la frecuencia de oscilación para desplazamientos pequeños

desde el equilibrio.

b) ¿Cuál es la longitud del péndulo simple equivalente?

Ejercicio propuesto 9.11.- La figura muestra,

Figura 9.10

Un disco de madera contrachapada de radio R = 20,0 cm y masa M = 3,0 kg, que tiene un pequeño agujero taladrado a una distancia d = 2,0 cm de su borde. El disco cuelga de la pared por medio de un pasador metálico que pasa a través del agujero en P, y se usa como péndulo. Con base en lo antes expuesto, determine: a) El período de este péndulo para oscilaciones pequeñas. b) La longitud de un péndulo simple equivalente. DATOS: R = 0,2 m, M = 3,0 kg, d = 0,02 m.

Ejercicio propuesto 9.12.-

Figura 9.11

La figura muestra un pequeño disco delgado de radio r y masa m que está rígidamente unido a la cara de un segundo disco delgado de radio R y masa M. El centro del disco pequeño se localiza en el borde del disco grande, el cual está montado en su centro sobre un eje sin fricción. Si el arreglo se hace girar un ángulo θ a partir de su posición de equilibrio y se suelta, determine: a) La velocidad del centro del disco pequeño cuando pasa por la posición de equilibrio. b) El periodo de movimiento del sistema mostrado.

López Frontado, R. (2011). Física general I, Texto UNA. 2da edición. Caracas: UNA

Page 331: Fisica General i Una

345

RESUMEN

Cuando un cuerpo se mueve con movimiento armónico simple, su

posición varía periódicamente con el tiempo, de acuerdo a la siguiente expresión:

x(t)=A cos( t+ )! " Donde A es la amplitud del movimiento, ω es la frecuencia angular,

( t+ )! " es el ángulo de fase, ! es el ángulo de fase inicial, la cual depende de su posición inicial y su velocidad inicial.

El período T se define como el intervalo de tiempo

necesario para una oscilación completa, esto es:

2T= πω

El inverso del período es la frecuencia f del movimiento, la cual se define

como el número de oscilaciones por segundo. La velocidad y aceleración de un movimiento armónico simple son:

22

2

2 2

dxv= =- Asen( t+ )dtd xa= =- Acos( t+ )dt

v= A -x

ω ω ϕ

ω ω ϕ

ω±

La máxima velocidad es A! , y la máxima aceleración es 2A! . La

rapidez es cero cuando está en su punto de inflexión (x = ±A), y es máxima cuando el oscilador está en la posición de equilibrio (x = 0). La aceleración es máxima en los puntos de inflexión, y cero en la posición de equilibrio.

La energía cinética y la energía potencial para un oscilador armónico

están dada por 2 2 2 2

2 2 2

1 1K= mv = m A sen ( t+ )2 21 1U= kx = kA cos ( t+ )2 2

! ! "

! "

López Frontado, R. (2011). Física general I, Texto UNA. 2da edición. Caracas: UNA

Page 332: Fisica General i Una

346

La energía total de un oscilador armónico simple es una constante del

movimiento y está dada por 21E= kA

2

La energía potencial del oscilador es máxima cuando está en sus puntos

de inflexión, y es cero cuando está en su posición de equilibrio. La energía cinética del oscilador es máxima en la posición de equilibrio, y es mínima en sus puntos de inflexión.

El sistema bloque resorte se mueve con movimiento armónico simple

sobre una superficie sin fricción con un período

2 mT= =2k

π πω

Un péndulo simple de longitud L se mueve con movimiento armónico

simple cuando los desplazamientos angulares desde la vertical son pequeños. Su período está dado por.

2 LT= =2g

π πω

Un péndulo físico tiene un movimiento armónico simple cuando tiene

pequeños desplazamientos angulares desde la vertical alrededor de un pivote que no pasa por el centro de masa. El período de este movimiento está dado por

2 IT= =2mgd

!!

"

donde I es el momento de inercia respecto al pivote, y d es la distancia desde el pivote al centro de masa.

Para un oscilador que experimenta una fuerza amortiguadora aF =-bv

! !, su

posición para un pequeño amortiguamiento está dado por

2b- t2m k bx=Ae cos( t+ ), donde = -

m 2mω ϕ ω ⎛ ⎞

⎜ ⎟⎝ ⎠

Cuando un oscilador está sometido a una fuerza de excitación senoidal

oF(t)=F sen t! , exhibe resonancia, y esto sucede por que la amplitud es máxima cuando la frecuencia de excitación es igual a la frecuencia natural del oscilador.

López Frontado, R. (2011). Física general I, Texto UNA. 2da edición. Caracas: UNA

Page 333: Fisica General i Una

347

EJERCICIOS DE AUTOEVALUACIÓN

En su libreta de apuntes, desarrolle los siguientes ejercicios. Luego de

resolver los ejercicios, compare sus respuestas con los presentados al final de la unidad. En caso de no acertar o de tener alguna duda sobre los mismos, consulte con sus compañeros o con el asesor de su centro local.

9.1.- Una partícula cuya masa es de 1 g vibre con movimiento armónico

simple de 2 mm de amplitud. Su aceleración en el extremo de recorrido es de 8,0x103 m/s2. Calcule: a) la frecuencia del movimiento; b) la velocidad de la partícula cuando pasa por la posición de equilibrio, y cuando la elongación es de 1,2 mm; c) Escriba la ecuación que expresa la fuerza que actúa sobre la partícula en función de la posición y del tiempo.

9.2.- Una partícula cuya masa es de 0,5 kg se mueve con movimiento

armónico simple. Su período es de 0,1 s y la amplitud de su movimiento es de 10 cm. Calcule la aceleración, la fuerza, la energía potencial y la energía cinética cuando la partícula está a 5 cm de la posición de equilibrio.

9.3.- Una varilla de 1 m de longitud está suspendida de uno de sus extremos

de tal manera que constituye un péndulo físico. Encuentre el período de oscilación si la varilla se cuelga de un eje situado a una distancia de uno de sus extremos igual a la longitud del péndulo equivalente previamente determinada.

9.4.- Una partícula se desliza hacia delante y hacia atrás entre dos planos

inclinados un ángulo θ sin fricción. a) Encuentre el período del movimiento si h es la altura inicial; b) ¿Es el movimiento oscilatorio? ¿Es armónica simple?

RESPUESTA A LOS EJERCICIOS PROPUESTOS: Estas respuestas se corresponden con los ejercicios propuestos presentados

en el desarrollo de la unidad. Respuesta al ejercicio propuesto 9.1 DATOS: k = 200 N/m, mg = 40 N, f = 10 Hz, A = 2 x 10-2 m

López Frontado, R. (2011). Física general I, Texto UNA. 2da edición. Caracas: UNA

Page 334: Fisica General i Una

348

Figura 9.12

SOLUCIÓN: La fuerza aplicada está dada por la expresión Fa = Fo cos ωt El peso mg = 40 N → m = 40 / g = 40 / 9,8 → m = 4,08 kg Tomando en cuenta el diagrama de cuerpo libre (DCL), la ecuación de

movimiento es 2

a 2

d xF kx mdt

− =

La solución de la ec. de movimiento x=Acos( t+ )! " , donde A es la amplitud

de la oscilación, y está dada por

2oo2 2

o

F kA , dondemm( )

ωω ω

= =−

Al despejar Fo se obtiene el valor de fuerza aplicada, esto es

2 2o

o

k 200F Am( ) 2 10 4,08(2 10 )m 4,08

F 318N

ω π−= − = × × × −

⇒ =

Respuesta al ejercicio propuesto 9.2 DATOS: m = 0,5 kg, k = 8 N/m, A = 0,1 m, SOLUCIÓN:

Figura 9.13

a) El movimiento de la masa está dado por:

o

o

x(t) Asen t

k 8 raddonde 4m 1,5 s

ω

ω

=

= = =

López Frontado, R. (2011). Física general I, Texto UNA. 2da edición. Caracas: UNA

Page 335: Fisica General i Una

349

Entonces la ecuaciones de posición, velocidad y aceleración están dadas por: → x(t) = 0,1 sen 4 t (1) → v(t) = 0,4 con 4 t (2) → a(t) = - 1,6 sen 4 t (3) Los valores máximos de la rapidez y de la aceleración, son:

max max 2

m m v =0,4 y a =1,6 s s

b) Despejando el tiempo t de la ecuación (1), se tiene que

11 xt sen (4)4 0,1

− ⎛ ⎞= ⎜ ⎟⎝ ⎠

Cuando x = 0,06 m, 11 0,06t sen t 0,161s4 0,1

− ⎛ ⎞= → =⎜ ⎟⎝ ⎠

Y los valores de la velocidad y aceleración en este momento son:

2

m v=0,4cos(4 0,161) v=0,32 sm a=-1,6sen(4 0,161) a=-0,96 s

! " #

! " #

c) En x = 0 → t1 = 0, y en x = 0,08 m → t2 = 0,232 s 2 1t=t -t t=0,232 sΔ ⇒ Δ Respuesta al ejercicio propuesto 9.3 DATOS: m = 2,0 kg, Fe = 20,0 N, x1 = 0,20 m, x2 = 0,30 m SOLUCIÓN: a) Para obtener la constante elástica k del resorte, se tiene:

e

e 11

F 20,0 NF =kx k= = k=100 x 0,2 m

! "

b) La frecuencia de las oscilaciones es

1 k 1 100f= = = f=1,125 Hz2 2 m 2 2,0!" " "

#

López Frontado, R. (2011). Física general I, Texto UNA. 2da edición. Caracas: UNA

Page 336: Fisica General i Una

350

c) Para determinar la rapidez máxima, se tiene que la energía mecánica E = K + U = constante, debido a que no existe roce entre las superficies.

2 2

f f i i 21 1K +U =K +U mv +0=0- kx2 2

2 2k mv= x = x =7,07×0,3 v=2,121 m s

! "

La velocidad máxima de la masa ocurre cuando pasa por su posición de

equilibrio, esto es cuando el resorte está en su longitud natural. d) La energía total del sistema oscilante es

( )2

2Total 2 Total

100× 0,31E = kx = E =4,5 J2 2

Respuesta al ejercicio propuesto 9.4 DATOS: m = masa del bloque pequeño, M = masa del bloque grande,

µe = coeficiente de fricción entre los bloques.

SOLUCIÓN:

Aplicando la ley de Newton, a los dos cuerpos, de tiene. Para M Para m

x

y 2 1 1

F : F -kx=-Ma -F =-ma

F : N -N -Mg=0 N -mg=0

µ µ∑∑

Pero Fµ = µe N1 = µemg, de donde µe mg = ma e a= gµ→ eg(m+M)(m+M)akx=ma+Ma=(m+M)a, x= =

k kµ

La amplitud A = a la elongación máxima (xmax ) eg(m+M)

A<k

µ⇒

López Frontado, R. (2011). Física general I, Texto UNA. 2da edición. Caracas: UNA

Page 337: Fisica General i Una

351

Respuesta al ejercicio propuesto 9.5 DATOS: m1 = 1,62 kg, Δx1 = 0,315 m, A = 22 cm, m2 = 0,25 kg SOLUCIÓN: a) Para escribir la ecuación que expresa este movimiento en función del

tiempo, es necesario conocer primero el valor de la constante elástica k del resorte. Para obtener la constante elástica k del resorte, se tiene,

1

1m g 1,62×9,8 NkΔy=m g k= = k=50,4 Δy 0,315 m

Aplicando la 2ª ley de Newton, se tiene

2 2

22 2 2

2 2

d y d y k k-ky=m , + y=0 =m mdt dt

! 2

22

d y + y=0dt

!"

( )( )

La ecuación de movimiento es y=Acos t+ ,

y su velocidad es v=-A sen t+

ω φω ω φ

Cuando el desplazamiento de la masa m2 es igual a la amplitud, la velocidad es cero, esto es:

2

v=0=-A sen , sen =0 =00,22=Acos A=0,22 m

50,4 radademás ω = ω=14,2 0,25 s

ω φ φ φφ

→→

La ecuación de movimiento es ( ) y=0,22cos 14,2t⇒ b) Para calcular el instante cuando el resorte tenga sus extensiones

máximas y mínimas, es necesario el período del movimiento de la masa m2, esto es

1

2

2 2T= = T=0,44 s14,2

La extensión máxima (y=-22 cm) en t =0,11 sLa extensión mínima (y= 22 cm) en t =0,33 s

π πω

Respuesta al ejercicio propuesto 9.6 DATOS: m = 0,2 kg, Δx = 0,1 m T = 1,8 s

López Frontado, R. (2011). Física general I, Texto UNA. 2da edición. Caracas: UNA

Page 338: Fisica General i Una

352

SOLUCIÓN: a) Aplicando la 2ª ley de Newton a un cuerpo sujeto a un resorte, se tiene

k x =maΔ

El período de vibración está dado

2 2 2 radT = , = = = 3,49 T 1,8 s

! ! !" "

"#

La velocidad angular, también está dada por

2 2 2k N= , k = m = 0,2×(3,4907) k = 2,44 m m

ω ω →

Aplicando el teorema de trabajo y energía, se tiene

2 2 22

22

k x mv k x= , v =2 2 m

2,44× (0,1) mv = v = 0,35 0,2 s

Δ Δ→

b) La ecuación que describe el movimiento del objeto está dada por:

x = A cos (ωt + δ),

y las ecuaciones de la velocidad y aceleración son v = - ωA sen (ωt + δ) a = - ω2 A cos (ωt + δ) = - ω2 x

De las condiciones iniciales, cuando t = 0, x = 0,1 m y v = 0, se tiene que A = 0,1 y δ = 0, por lo tanto las ecuaciones de movimiento se expresan x = 0,1 cos (3,49 t). Entonces la aceleración a = ω2x = (3,4907)2 0,05

2

m a = 0,61 s

c) Para determinar el intervalo de tiempo que tarda en ir desde un punto

situado a 0,05 m por debajo hasta un punto situado a 0,05 m por encima de la posición de equilibrio, se determina el tiempo que tarda en pasar por esos puntos, esto es:

López Frontado, R. (2011). Física general I, Texto UNA. 2da edición. Caracas: UNA

Page 339: Fisica General i Una

353

Para el punto por debajo,

-11 1

1 1 1

0,050,05 = 0,1 cos(3,49 t ), 3,49 t = cos0,1

1,0473,49 t = 60º= 1,047 rad, t = t = 0,3 s3,49

⎛ ⎞∴ ⎜ ⎟⎝ ⎠

Para el punto por encima,

-12 2

-0,053,49t = cos = 120º= 2,094 t = 0,6 s0,1

! "#$ %

& '

Entonces, el intervalo Δt = t2 – t1 = 0,6 – 0,3

⇒ Δt = 0,3 s

Respuesta al ejercicio propuesto 9.7 DATOS: Fg = mg = 1,0 N, k = 2,0 N/m, SOLUCIÓN:

La frecuencia angular de un cuerpo sujeto a un resorte está dado por k=m

! .

La frecuencia de oscilación del resorte está dada por 11 kf = =

2 2 mωπ π

.

La frecuencia de oscilación del péndulo está dada por 21 gf = =

2 2ωπ π l

.

Considerando las condiciones del problema, f2 = f1/2, esto es

1 g 1 k g k 4mg 4×1,0= = = = =2 m2 4 m 4m k 2,0! !

" "! !! !

La longitud original o! del resorte es, omg 1,0= -Δx, donde Δx= = =0,5 mk 0,2

! !

o o=2,0-0,5 =1,55 m!! !

López Frontado, R. (2011). Física general I, Texto UNA. 2da edición. Caracas: UNA

Page 340: Fisica General i Una

354

Respuesta al ejercicio propuesto 9.8 DATOS: M = 4 kg, L = 1,2 m, d = L/4. SOLUCIÓN a) La ecuación de movimiento que hace que oscile la barra, está dada por

la componente tangencial de la fuerza, esto es:

2 2

p 2 2

2 2 22

p c

d dMgdsen I (1), Perodt dtML ML 7MLy I I Md12 16 48

θ θθ α− = =

= + = + =

Sustituyendo en la ec. (1) y despejando la aceleración angular α, se tiene:

12gsen 12 9,8sen 14sen

7L 7 1,2θ θα α θ×= − = − ⇒ = −

×

b) Para pequeñas amplitudes de oscilaciones, sen θ ≈ θ,

2 2

2 2

d d14 14 0dt dtθ θθ θ= − → + =

Donde o

o

2 214 T T 1,68s14

π πωω

= → = = ⇒ =

Respuesta al ejercicio propuesto 9.9 DATOS: M = 0,015 kg, R = 0,05 m, IP = 7 MR2 / 5 SOLUCIÓN:

Figura 9.14

El periodo de un cuerpo sólido está dado por: 2

p

7MRI2 7R5T = = 2 = 2 = 2

MgR MgR 5g

7×0,05T = 2 T = 0,531 s5×9,8

π π π πω

π ⇒

López Frontado, R. (2011). Física general I, Texto UNA. 2da edición. Caracas: UNA

Page 341: Fisica General i Una

355

Respuesta al ejercicio propuesto 9.10 DATOS: M = 2,16 kg, R = 0,653 m SOLUCIÓN:

Figura 9.15

a) Supongamos que el aro oscile alrededor del punto P, razón por la cual desplazamos el centro del aro desde C hasta C’. El período del cuerpo para pequeños desplazamientos, está dado por:

2PP C

2 2P

IT=2π , donde I =I +Md

Mgd

I =MR +MR , siendo d=R

22MR 2R 2×0,653 1T=2 =2 =2 =2,2937 s f= =436 mHz

MgR g 9,8 T! ! ! "

b) La longitud del péndulo simple equivalente se obtiene comparando el

período de un péndulo simple con el período de un péndulo físico. Esto es

P P PI I IL L2π =2π , = , L=

g Mgd g Mgd Md! "

22MRL= =2R=2×0,653 L=1,306 mMR

Respuesta al ejercicio propuesto 9.11 DATOS: R = 0,2 m, M = 3,0 kg, d = 0,02 m. SOLUCIÓN:

a) Para oscilaciones pequeñas, el período está dado por

( )2 22

2 2C

IT=2 , donde h=R-d=0,2-0,02mgh

h=0,18 m

M R +2hMRI=I +Mh = +Mh =2 2

π

López Frontado, R. (2011). Física general I, Texto UNA. 2da edición. Caracas: UNA

Page 342: Fisica General i Una

356

Entonces se tiene ( ) ( ) ( )

( )

2 22 2M R +2h 0,2 +2 0,18T=2 =2

2Mgh 2×9,8× 0,18

T=1,083 s

π π

b) La longitud equivalente del péndulo simple se obtiene comparando el

período para el cuerpo de este problema y el período de un péndulo simple, esto es

( ) ( )

( ) ( )( )

π π ∴

⎡ ⎤⎣ ⎦ ⇒

2 2 2 2

S P

2 2

M R +2h M R +2hLT =2 , T =2 L=Mg 2Mgh 2h

3,0 0,2 +2 0,18L= L=0,603 m

2 0,18

Respuesta al ejercicio propuesto 9.12 DATOS: M, y R = masa y radio del disco grande. m y r = masa y radio del

disco pequeño. SOLUCIÓN: a) Considerando que no existe roce, entonces podemos considerar que

o o f f

2C

K+ U=0 K +U =K +UI

mgh= (1)2ω

Δ Δ →

Donde 2 2

2C

v MR mr= , I = + mR , y h=R-Rcos =R(1-cos )r 2 2

ω θ θ+

Al sustituir en la ecuación (1), se tiene

2 2 22

2

1 MR mr vmgR(1-cos )= +mR2 2 2 R

θ⎡ ⎤

+⎢ ⎥⎣ ⎦

Despejando la velocidad v, se tiene 2

2

mgR(1-cos θ ) v=M mr m+ +4 24R

!" #$ %& '

López Frontado, R. (2011). Física general I, Texto UNA. 2da edición. Caracas: UNA

Page 343: Fisica General i Una

357

b) El periodo de movimiento del sistema es

2 2 2

C

T cm

I MR +mr +mRT=2π T=2πM gd 2mgR

RESPUESTAS A LOS EJERCICIOS DE AUTOEVALUACIÓN Respuesta al ejercicio de autoevaluación 9.1

3 3m ma) f=3,183 10 Hz; b) v=4,0 , v=3,2 ; c) 8sen(2 10 t+ ) Ns s

!" "

Respuesta al ejercicio de autoevaluación 9.2

2 2 2 22

ma=-20 ; F=-10 N; U=0,25 J; K=0,75 Js

! ! ! !

Respuesta al ejercicio de autoevaluación 9.3 T=1,64 s Respuesta al ejercicio de autoevaluación 9.4

4 2ha) T= ; b) si, no.sen gφ

López Frontado, R. (2011). Física general I, Texto UNA. 2da edición. Caracas: UNA

Page 344: Fisica General i Una

359

UNIDAD 10

FLUIDOS

La mayoría de las sustancias en el mundo se presenta en una de estas tres

fases: sólido, líquido o gas. Cuando sobre estas fases actúan fuerzas externas, los sólidos son sistemas de partículas que tienden a mantener su forma y su volumen. Los líquidos son sistemas de partículas que tienen la particularidad de no tener una forma definida, sino que adquieren la del recipiente que los contienen. También se caracterizan por su resistencia a variar su volumen, aun a presiones muy altas. Los gases son aquellos sistemas de partículas que tienen la particularidad de no tener forma ni volumen propio, ya que adquieren el tamaño y la forma del recipiente que los contiene; además tienen la propiedad de variar con gran facilidad su volumen.

En esta unidad se estudia la mecánica de los fluidos, o sea aquellas

sustancias que pueden fluir, entre los cuales se encuentran los líquidos y los gases. Se comienza el estudio con la estática de los fluidos, llamada también

Hidrostática, es decir, cuando los fluidos se encuentran en estado de reposo en situación de equilibrio, así se estudiarán los conceptos clave de densidad, presión y flotación. Luego se continúa con la dinámica de los fluidos, llamada también Hidrodinámica, o sea cuando los fluidos están en movimiento (o flujo de un fluido). Ésta es una de las ramas más complejas de la mecánica, ya que muchos de los efectos producidos por una fuerza que actúa sobre un cuerpo son trasmitidos a través de los fluidos, como es el caso de las bombas hidráulicas; mientras que soportan el efecto de un golpe a través de los fluidos, como es el caso de los amortiguadores.

CONOCIMIENTOS PREVIOS: Para comprender el contenido de esta unidad, el estudiante debe poseer los

siguientes conocimientos: 1.- Ecuación de movimiento de un sistema de partículas. 2.- Trabajo y energía. 3.- Tipos de equilibrio (Unidad 7).

López Frontado, R. (2011). Física general I, Texto UNA. 2da edición. Caracas: UNA

Page 345: Fisica General i Una

360

10.1 OBJETIVO Aplicar los conceptos de la dinámica de sistemas de partículas al

estudio de los fluidos.

10.2 RECOMENDACIONES PARA EL ESTUDIO DEL CONTENIDO En esta unidad, el estudiante debe comprender claramente cómo se aplican

los principios y las leyes fundamentales en el estudio del comportamiento de sistemas fluidos.

Para una mejor comprensión del contenido de la unidad, realice lectura de los

conceptos relacionados con el movimiento de los cuerpos, aplicando las ideas estudiadas en el movimiento de sistemas de partículas y las leyes fundamentales de la mecánica en el estudio de los sistemas de esta unidad, y los ejemplos presentados. Utilice las técnicas de lectura que le facilite la comprensión del tema.

10.3 CONTENIDO 10.3.1 ESTADO DE LA MATERIA. 10.3.2 ESTÁTICA DE FLUIDOS. 10.3.3 DINÁMICA DE FLUIDOS. 10.3.1 ESTADOS DE LA MATERIA Los tres estados comunes (o fases) de la materia son: el sólido, el líquido y el

gaseoso. Un sólido tiene la característica de mantener su forma propia, y un tamaño definido; aun si una fuerza grande es aplicada a un sólido, éste no cambia fácilmente de forma o volumen. Un líquido no mantiene una forma fija (toma la forma del recipiente que lo contiene), pero no es fácilmente compresible, y su volumen sólo puede cambiarse aplicando una fuerza muy grande. Un gas no tiene ni forma ni volúmenes fijos, sino que se expande hasta llenar un recipiente.

Como los líquidos y gases no mantienen una forma fija, tienen la capacidad de

fluir, por eso se les llama fluidos.

López Frontado, R. (2011). Física general I, Texto UNA. 2da edición. Caracas: UNA

Page 346: Fisica General i Una

361

10.3.2 ESTÁTICA DE FLUIDOS (Ó HIDROSTÁTICA) DENSIDAD Una propiedad importante de cualquier material es su densidad ρ , la cual se

define como una masa por unidad de volumen. y se representa por la letra ρ . Si el cuerpo es homogéneo, es decir su masa se distribuye uniformemente en todo su volumen, la densidad de ese cuerpo tiene el mismo valor en cada uno de los puntos que constituyen dicho cuerpo, entonces la densidad se puede expresar como

m= (10.1)V

!

Algunos materiales tienen la particularidad de ser heterogéneos, es decir que

su masa no se distribuye uniformemente en todo su volumen; entonces se debe considerar un elemento unitario de masa dm contenido en un elemento unitario de volumen dV, así se tiene que la densidad se expresa por:

dmρ= (10.2)dV

La unidad de la densidad en el sistema de unidades Internacional (SI) es

kg/m3. Si en el cuerpo predominan dos dimensiones sobre la tercera, se tiene el caso

de una densidad superficial del mismo, esto es:

dmσ= (10.3)dA

donde dA es el elemento unitario de superficie. La unidad SI de la densidad superficial es kg/m2.

Si en el cuerpo, predomina una dimensión sobre las otras dos, se tiene el

caso de una densidad lineal del mismo, esto es:

dmλ= (10.4)d!

donde d! es el elemento diferencial de longitud. La unidad SI de la densidad lineal es kg/m.

López Frontado, R. (2011). Física general I, Texto UNA. 2da edición. Caracas: UNA

Page 347: Fisica General i Una

362

TABLA No. 10.1 Tipos de densidades en los cuerpos

CUERPOS CUERPOS

HOMOGÉNEOS CUERPOS

HETEROGÉNEOS* UNIDADES (SI)

Volumétricos m=V

! dm=dV

ρ 3

kgm

Superficiales m=A

! dm=dA

! 2

kgm

Longitudinales m=λl

dm=d

!l

kgm

*Nota: Cuando se trata de un cuerpo heterogéneo, la fórmula mostrada en la

Tabla 10.1 corresponde a la de un elemento diferencial de masa. PESO ESPECÍFICO Se define como peso específico de una sustancia a la razón de la densidad

de esa sustancia a la densidad del agua a 4º C. El peso específico es un número sin unidades o dimensiones.

sustancia

sustanciaagua a 4ºC

densidadPeso específico = (10.5)densidad

10.3.2 ESTÁTICA DE FLUIDOS (o HIDROSTÁTICA) PRESIÓN EN UN FLUIDO Cuando un fluido está en reposo, ejerce una fuerza perpendicular a cualquier

superficie en contacto con él, sea la pared del recipiente o un cuerpo sumergido en él.

Considere una superficie pequeña de área dA, centrada en un punto del fluido; la fuerza normal producida por el fluido sobre cada lado es dF, como se muestra en la figura (10.1). La presión p en ese punto se define como la fuerza por unidad de área, esto es:

dFp= (10.6)dA

Si la presión es la misma en todos los puntos de

la superficie plana, se tiene:

Fp= (10.7)A

Figura 10.1

La presión que actúa sobre ambos lados de un área pequeña dentro de un fluido.

López Frontado, R. (2011). Física general I, Texto UNA. 2da edición. Caracas: UNA

Page 348: Fisica General i Una

363

La unidad SI de la presión es el Pascal (Pa), donde se tiene que:

52

N1 Pa=1 , 1 atm=1,013×10 Pam

PRESIÓN A DIFERENTES ALTURAS Consideremos el efecto de la fuerza gravitatoria de

la Tierra en la variación vertical de la presión en un fluido. Como se muestra en la figura (10.2), sea dV un elemento de volumen de un fluido. Como el fluido está en equilibrio, la suma de las fuerzas que actúan sobre él es cero, así se tiene que:

ii

F=0, pA-(p+dp)A-dmg=0!r

pero dmg=( dV)g= gAdyρ ρ . De donde se obtiene que:

dp=- gdy (10.8)!

Figura 10.2

Fuerza sobre un volumen elemental de fluido en equilibrio.

Esta diferencia de presión es la que soporta el peso del volumen elemental de

fluido. Al integral la ecuación (10.8) se obtiene la diferencia de presión entre dos

puntos situados a diferentes alturas, esto es:

2 2 2

1 1 1

p y y

2 1p y ydp=- gdy, p -p =- gdyρ ρ∫ ∫ ∫

Si p1 y p2, son las presiones en las alturas y1 y y2, donde ρ y g son constantes,

se tiene:

2 1 2 1p -p =-ρg(y -y ) (10.9)

López Frontado, R. (2011). Física general I, Texto UNA. 2da edición. Caracas: UNA

Page 349: Fisica General i Una

364

La presión en un punto situado a una profundidad h bajo la superficie de un líquido, donde po es la presión en la superficie del líquido, donde ρ es constante, se tiene:

op=p +ρgh (10.10) PRINCIPIO DE PASCAL Nos dice que: “la presión aplicada a un fluido encerrado se

trasmite sin disminución a todas las partes del fluido y a las paredes del recipiente”.

Figura 10.3

La presión p a una profundidad h en un fluido es mayor que en la superficie por ghρ .

Una aplicación de la ley de Pascal es

el principio del elevador hidráulico. La

presión aplicada 1

1

Fp=A

, se trasmite al pistón

mayor de área A2. Así se tiene que:

1 2 22 1

1 2 1

F F Ap= = , F = F (10.11)A A A

Figura 10.4

El principio del elevador hidráulico.

PRESIÓN MANOMÉTRICA La presión manométrica es el exceso de la presión atmosférica. En el caso

de la ecuación (10.10), la presión manométrica está dada por: op-p =ρgh (10.12) La presión absoluta es la presión total, esto es op=p +ρgh (10.13) El medidor de presión más sencillo es el manómetro de tubo abierto. Existe

una gran variedad de tipos de manómetros, entre los que se encuentra el barómetro de mercurio, el esfigmomanómetro, lleno de mercurio.

López Frontado, R. (2011). Física general I, Texto UNA. 2da edición. Caracas: UNA

Page 350: Fisica General i Una

365

FLOTACIÓN La flotación, fenómeno muy conocido, se refiere a que un cuerpo sumergido

en agua parece pesar menos que en el aire. Cuando se trata de hundir un cuerpo dentro de un depósito de agua, se siente cierta resistencia a ese hundimiento; esto se debe a la fuerza ejercida por el agua sobre ese cuerpo. La fuerza ejercida hacia arriba por un líquido (fluido) sobre cualquier cuerpo sumergido recibe el nombre de empuje hidrostático o fuerza de empuje (también se denomina fuerza flotante).

La magnitud de esta fuerza de empuje es igual al peso del líquido desalojado

por el cuerpo, por lo tanto es la misma para todos los cuerpos que tengan el mismo volumen. La fuerza flotante es la fuerza resultante debida a todas las fuerzas ejercida por el líquido que rodea al cuerpo.

Como consecuencia de estas observaciones, donde la fuerza flotante actúa

sobre cualquier cuerpo sumergido en un líquido, Arquímedes enuncia su principio, el cual dice así:

“Si un cuerpo está parcialmente o totalmente sumergido en un fluido,

éste ejerce una fuerza hacia arriba sobre el cuerpo igual al peso del fluido desplazado por el cuerpo”.

La fuerza de empuje (fuerza flotante), matemáticamente se expresa por: E fluidoF = gV (peso del fluido) (10.14)ρ A continuación analicemos situaciones

que se presentan, cuando un cuerpo está sumergido en un líquido, como el ilustrado en la figura (10.5).

Las fuerzas que actúan sobre el cuerpo

son EF!

, y el peso del cuerpo gF =mg! !

, donde

E fluido cuerpoF = gV!! !

, y g cuerpo cuerpoF =mg= gV!! ! !

. La fuerza neta sobre el cuerpo es:

neta E g fluido cuerpo cuerpoF =F -F =( - )V g (10.15)! !! ! ! !

Figura 10.5

Las fuerzas que actúan sobre el cubo líquido son la fuerza gravitacional (mg)

!

y la fuerza de empuje E(F )r

.

De la ecuación (10.15) se observa que: Si fluido cuerpoρ ρ> , el cuerpo sube. Si fluido cuerpoρ ρ< , el cuerpo se hunde.

López Frontado, R. (2011). Física general I, Texto UNA. 2da edición. Caracas: UNA

Page 351: Fisica General i Una

366

Si fluido cuerpo=ρ ρ , el cuerpo se mantiene en equilibrio dentro del líquido, no sube ni se hunde.

Como conclusión se puede establecer que: el movimiento de un cuerpo sumergido en un líquido está determinado sólo por las densidades del cuerpo y del fluido.

A continuación se presentan unos ejemplos de problemas, donde Ud., podrá visualizar las aplicaciones de los conceptos tratados en esta unidad. Pero antes vea las Sugerencias y las Recomendaciones presentadas para resolver estos problemas.

RECOMENDACIONES PARA RESOLVER PROBLEMAS DE DINÁMICA

DEL SISTEMA DE PARTÍCULAS EN FORMA DE FLUIDOS. Cuando el problema se refiera a Fluidos, se debe tener en cuenta si el

sistema está en equilibrio o en movimiento. IDENTIFICAR los conceptos pertinentes: Si el sistema está en equilibrio es

necesario tener en cuenta el comportamiento de la densidad y de la presión del fluido. Si el sistema está en movimiento se debe tener presente que la ecuación de Bernoulli se deduce de las relaciones de trabajo y la energía.

PLANTEAR el problema: Es conveniente seguir estos pasos: 1.- Siempre es conveniente identificar los puntos 1 y 2 a los que se refiere

el problema. 2.- Defina un sistema de coordenadas, indicando el nivel donde y=0. 3.- Haga una lista de las cantidades conocidas y desconocidas, indicando

cuales son las variables y cuales las constantes. Indique las incógnitas a determinar.

CALCULAR la solución siguiendo, tomando en cuenta que en algunos

problemas debe tomar en cuenta las ecuaciones de continuidad, o la ecuación de Bernoulli. Además en algunos problemas se puede considerar la razón de flujo de volumen.

EVALUAR la respuesta: Como siempre, las unidades deben ser congruentes.

En el SI, la presión está en Pa, la densidad en kg/m2, y la rapidez en m/s. También debe tener presente que las presiones todas deben ser absolutas o todas manométricas.

López Frontado, R. (2011). Física general I, Texto UNA. 2da edición. Caracas: UNA

Page 352: Fisica General i Una

367

Recuerde que el manejo eficiente del contenido de esta unidad es

importante, por su repercusión en el contenido de las subsiguientes asignaturas del Plan de Estudios en Ingeniería y otras ciencias.

Ejemplo 10.1.- Una pieza de aluminio con 1,0 kg de masa y 2700 kg/m3 de densidad, está

suspendida de un resorte y luego se sumerge por completo en un recipiente lleno de agua. Calcule la tensión en el resorte:

a) Antes de sumergirse en el agua.

b) Después de sumergirse en el agua. DATOS: mAl = 1,0 kg, ρAl = 2700 kg/m3. SOLUCIÓN: IDENTIFICAR los conceptos pertinentes: El problema se refiere a un sistema en equilibrio, donde se aplican los

conceptos de fuerza, flotación del cuerpo, tanto en el aire como dentro de un fluido. PLANTEAR Y CALCULAR la solución del problema: Se aplican las condiciones de equilibrio de la mecánica clásica, para el cuerpo

cuando está en el aire y dentro de un fluido, haciendo uso del diagrama de cuerpo libre (DCL), presentado en la figura (10.6), tomando en cuenta todas las fuerzas que actúan sobre el cuerpo.

(a) (b)

Figura 10.6

López Frontado, R. (2011). Física general I, Texto UNA. 2da edición. Caracas: UNA

Page 353: Fisica General i Una

368

a) La figura (10.6a) representa el bloque suspendido de la cuerda, aplicando la ley de Newton, así se tiene que la tensión T de la cuerda está dada por:

T- mg = 0, T = mg = 1,0×9,8 T = 9,8 N⇒

b) La figura (10.6b) represente el bloque suspendido de la cuerda, pero

dentro de un envase lleno de agua, ahora se tiene que la tensión de la cuerda T’, está dada por:

agua s S aguaAl Al

agua

Al

m mT'+B- mg = 0, donde B = gV , V = B = g

1000T' = mg-B = mg 1- = 1,0×9,8 1- T' = 6,17 N2700

! !! !

!

!

"

# $ # $ %& ' & '( )( )

EVALUAR los resultados del problema: Se observa que la tensión de la

cuerda, cuando el cuerpo está dentro del fluido es menor que cuando está fuera de él, y esto se debe a la fuerza de empuje que el fluido hace sobre el mismo.

Ejercicio propuesto 10.1.- Un objeto de masa de 180 g, pero densidad desconocida se “pesa” en agua

(densidad ρa = 1,0 g/cm3), y el peso así obtenido corresponde a una masa equivalente de 150 g. Al “pesarlo” de nuevo en un líquido de densidad desconocida ρl, resulta que se necesita una masa equivalente de 144 g. Calcule:

a) La densidad del segundo líquido ρl.

b) La densidad del objeto ρob.

Ejercicio propuesto 10.2.- Un bloque de metal de 10 kg y medidas de 12 cm x 10 cm x 10 cm, se cuelga

de un dinamómetro y se sumerge en agua, como se muestra en la figura. La dimensión vertical es de 12 cm y la parte superior del bloque está a 5 cm de la superficie del agua.

López Frontado, R. (2011). Física general I, Texto UNA. 2da edición. Caracas: UNA

Page 354: Fisica General i Una

369

a) ¿Cuáles son las fuerzas sobre la parte superior y sobre la parte inferior del bloque? (Tome Pa = 1,013 x105 N/m2).

b) ¿Cuál es la lectura en la escala del dinamómetro?

c) Demuestre que la fuerza de empuje es igual a la diferencia entre las

fuerzas que actúan sobre la parte superior y sobre la parte inferior del bloque.

Ejercicio propuesto 10.3.- Se tiene una piscina de 22,0 m por 8,5 m cuya profundidad uniforme es de 2,0

m. Considerando lo antes expuesto obtenga: a) La fuerza total y la presión absoluta sobre el fondo de la piscina.

b) La presión contra el lado de la piscina cerca del fondo.

Ejemplo 10.2.- En una bomba hidráulica el émbolo pequeño tiene un diámetro de 3,73 cm, y

el émbolo grande uno de 51,3 cm. a) ¿Qué peso sobre el émbolo pequeño soportará 18,6 kN (por ejemplo,

un automóvil) sobre el émbolo grande?

b) ¿Qué distancia debe moverse el émbolo pequeño para que el automóvil se eleve 1,65 m?

DATOS: D1 = 3,73 cm, D2 = 51,3 cm, F2 = 18,6 N, d2 = 1,65 m SOLUCIÓN: IDENTIFICAR los conceptos pertinentes: En este problema se consideran las

condiciones de equilibrio establecida en la mecánica clásica, usando los conceptos de presión, propagación de fuerza a través del fluido.

PLANTEAR Y CALCULAR la solución del problema: Se aplica los conceptos

de presión, y el principio de continuidad, cuando se aplica una fuerza en un punto y

López Frontado, R. (2011). Física general I, Texto UNA. 2da edición. Caracas: UNA

Page 355: Fisica General i Una

370

la propagación de su efecto a través del fluido. Obteniéndose luego los valores de las incógnitas del problema.

Figura 10.7

a) La presión sobre una superficie está dada por Fp=A

, y el área está dada

por 2DA=4

π . Como la presión es la misma en toda la superficie, se

tiene, 2

1 2 1 2 11 2

1 2 2 2

23

1 1

F F A F D= F = = F

A A A D

0,0373F = (18,6 10 ) F =98,3 kN0,513

⎛ ⎞→ ⎜ ⎟

⎝ ⎠

⎛ ⎞ × ⇒⎜ ⎟⎝ ⎠

b) Por el principio de continuidad, se tiene que A1d1 = A2d2.

2 2

2 2 21 2 1

1 1

A d D 0,513d = = d = (1,65) d =312,11 mA D 0,0373

! " ! "#$ % $ %

& '& '

EVALUAR la respuesta: En la primera parte del problema se obtiene que la

fuerza necesaria que debe aplicarse en el émbolo más pequeño es de 98,3 kN, y la distancia que se movería el émbolo pequeño es de 312,11 m.

Ejemplo 10.3 Si el dique de una presa tiene una altura de 35 m y una anchura de 60 m,

determine: a) la componente de fuerza horizontal total Fh que ejerce el agua de la presa sobre el dique; b) el momento de fuerza ! ejercido por el agua sobre el dique, alrededor de un eje que de un lado al otro lado de la base del dique.

DATOS: H = 35 m, L = 60 m, ρag =103 kg/m3, g=0,8 m/s2.

López Frontado, R. (2011). Física general I, Texto UNA. 2da edición. Caracas: UNA

Page 356: Fisica General i Una

371

SOLUCIÓN: IDENTIFICAR los conceptos pertinentes: La presión atmosférica actúa a

ambos lados del dique, además la presión proporcionada por el agua varía con la profundidad.

PLANTEAR Y CALCULAR la solución del problema:

Figura 10.8

a) Como la presión del agua sobre el dique varía con la profundidad, es necesario hacer uso de la presión diferencial del agua para calcular la fuerza Fh . El valor de componente de la fuerza dFh sobre el elemento diferencial de superficie situado a una profundidad h con anchura L está dado por:

h dH

2h

0

dF =p dA=( gh)(Ldh)1F = gLhdh= gLH2

!

! !"

Al sustituir los valores, se tiene que la fuerza

3 2

8h h

10 9,8 60 (35)F = F =3,60 10 N2

× × × ⇒ ×

c) El momento de fuerza alrededor de un eje que pasa a través de O, está

dado por:

3H

03 3

9

gLHd =(H-h)dF=(H-h)( gh)(Ldh) = gL hdh=6

10 9,8 60 (35)= =4,20 10 N.m6

ρτ ρ τ ρ

τ τ

× × × ⇒ ×

EVALUAR la respuecta: La fuerza total que ejerce el agua sobre el dique es

de 360 MN, y el momento de fuerza que ejerce es de 4,20 GN.m

López Frontado, R. (2011). Física general I, Texto UNA. 2da edición. Caracas: UNA

Page 357: Fisica General i Una

372

10.3.3 DINÁMICA DE FLUIDOS (o HIDRODINÁMICA) Hasta este momento, el estudio de los fluidos se ha restringido al de fluidos en

reposo. Ahora centraremos nuestra atención en el estudio del movimiento de fluidos. Para ello consideremos antes lo que se entiende por línea de flujo (o línea de corriente), que es la trayectoria descrita por una partícula individual en un fluido en movimiento.

Flujo laminar o estable, que es aquel

donde cada partícula del fluido sigue una trayectoria lisa, de modo que las trayectorias de diferentes partículas nunca se cruzan entre ellas. La velocidad de la partícula es siempre tangente a la línea de flujo.

Cuando las partículas del fluido tienen

una rapidez por encima de la rapidez crítica, entonces el flujo se hace turbulento, o sea que el flujo es irregula,r constituido por pequeñas regiones semejantes a remolinos.

Figura 10.9

Una partícula en flujo laminar sigue una

línea de flujo. En cada punto de su trayectoria, la velocidad de cada

partícula es tangente a la línea de flujo.

Un fluido es viscoso cuando el flujo del fluido presenta cierta fricción interna,

que se manifiesta por la resistencia entre dos capas adyacentes del fluido al moverse una con respecto a la otra. Se dice que un fluido es incompresible, cuando la densidad del mismo es constante.

Los líquidos por lo general son aproximadamente incompresibles; mientras

que los gases se pueden tratar como incompresibles si las diferencias de presión de una región a otra no son muy grandes.

Ahora se estudia el movimiento de un fluido, e imaginemos que se tiene un

fluido ideal, el cual es incompresible, estable, irrotacional, y no es viscoso.

ECUACIÓN DE CONTINUIDAD La masa de un fluido en movimiento no cambia al fluir, esto nos proporciona

una importante relación cuantitativa llamada ecuación de continuidad.

Consideremos el caso de un fluido incompresible, cuya densidad ρ tiene el mismo valor en todos los puntos. La masa dm1 que fluye al interior del tubo por A1 en el tiempo dt es dm1 = ρA1v1. De igual forma, la masa dm2 que sale por A2 en el mismo intervalo de tiempo dt es dm2 = ρA2v2.

López Frontado, R. (2011). Física general I, Texto UNA. 2da edición. Caracas: UNA

Page 358: Fisica General i Una

373

En flujo estable, la masa total en

el tubo es constante, así que:

1 2

1 1 2 2

1 1 2 2

dm =dm y ρA v dt=ρA v dt o seaA v =A v =const. (10.16)

La ecuación (10.16) es la

ecuación de continuidad para un fluido incompresible, la cual nos dice que: el producto del área y la rapidez el fluido en todos los puntos a lo largo de un tubo es constante.

Figura 10.10

El volumen de fluido que circula por el área A1 en un intervalo Δt debe ser igual al volumen que circula por el área A2 en el mismo intervalo.

La ecuación (10.16) es la ecuación de continuidad para un fluido

incompresible, la cual nos dice que: el producto del área y la rapidez el fluido en todos los puntos a lo largo de un tubo es constante.

El producto Av es la razón de flujo de volumen (o gasto volumétrico)

dV( )dt

, o sea la rapidez con que el volumen cruza una sección del tubo,

dV =Av (10.17)dt

La razón de flujo de masa dm( )dt

, es el flujo de masa por unidad de tiempo a

través de una sección transversal, y es:

dm dV=ρ (10.18)dt dt

La ecuación de continuidad para un fluido no incompresible es 1 1 1 2 2 2ρ A v =ρ A v (10.19)

donde ρ1 y ρ2 son las densidades del fluido en las secciones trasversales 1 y 2.

López Frontado, R. (2011). Física general I, Texto UNA. 2da edición. Caracas: UNA

Page 359: Fisica General i Una

374

ECUACIÓN DE BERNOULLI De acuerdo a la ecuación de continuidad, la rapidez de flujo de un fluido

puede variar a lo largo de las trayectorias del fluido. La presión también puede variar, depende de la altura y de la rapidez de flujo. La ecuación de Bernoulli relaciona la presión, la rapidez de flujo y la altura para el flujo de un fluido ideal.

Cuando un flujo estable de un

fluido estable incompresible sin viscosidad recorre un tubo de sección transversal variable, se tiene:

2 2

1 2 2 1 2 1

2 21 1 1 2 2 2

1p -p = ρ(v -v )+ρg(y -y ) (10.20)2

ó,1 1p +ρgy + ρv =p +ρgy + ρv (10.21)2 2

La ecuación (10.20) es la ecuación de Bernoulli, la cual dice que:

Figura 10.11

Fluido en flujo laminar en un tubo con constricción. El volumen del punto 1 es igual al volumen del punto 2.

“el trabajo efectuado por el fluido circulante sobre un volumen unitario

de fluido es igual al trabajo a la suma de los cambios de energías cinética y potencial por unidad de volumen que ocurren durante el flujo”.

La ecuación (10.21) es la ecuación de Bernoulli para los puntos 1 y 2

cualesquiera del tubo de flujo. Se tiene además que

21p+ρgy+ ρv =constante (10.22)2

Ejemplo 10.4.- En la figura mostrada, la presión diferencial es de 75 kPa y la velocidad del

agua que fluye en esta cañería de 50 mm de diámetro es de 1,7 m/s. La cañería se desdobla en dos más pequeñas, cada una de ellas con un diámetro de 25 mm.

a) ¿Cuál es la rapidez de flujo en el punto A, y cuál en el punto B?

b) ¿Cuál es la presión diferencial en el punto B?

López Frontado, R. (2011). Física general I, Texto UNA. 2da edición. Caracas: UNA

Page 360: Fisica General i Una

375

Suponga un flujo incompresible de líneas de corriente. DATOS: pA = 75 kPa, vA = 1,7 m/s, DA=50 x 10-3 m, DB = 25 x10-3 m.

Figura 10.12

SOLUCIÓN: IDENTIFICAR los conceptos pertinentes: El problema se refiera a la dinámica

del fluido, donde aplicaremos la ecuación de Bernoulli, para determinar la rapidez de flujo y la presión diferencial en B.

PLANTEAR Y CALCULAR la solución del problema: Considerando el

principio de continuidad se tiene que la rapidez de flujo que pasa por la sección A es igual a rapidez de flujo que pasa por la sección B, esto es QA = 2QB. También se obtiene el diferencial de presión a través de la rama más delgada a partir de la ecuación de Bernoulli.

a) Para determinar la rapidez de flujo que pasa por la sección A es igual a

la rapidez de flujo que pasa por las secciones B, se tiene:

3

2 3 2 3A A A A A A

mQ v A v r 1,7 (25 10 ) Q 3,34 10s

π π −= = = × ⇒ = ×

3 3

3AB B

Q 3,34 10 mQ Q 1,67 102 2 s

−−×= = ⇒ = ×

b) Para determinar el diferencial de presión en el punto B, usamos la

ecuación

2 2B A B Ap p (v v ) (1)

2ρ= + −

Previamente se determina la velocidad del agua en la sección B. Así,

3B

B B B B 3 2B

B

Q 1,67 10Q v A vA (12,5 10 )

mv 3,46s

π

×= → = =×

→ =

López Frontado, R. (2011). Física general I, Texto UNA. 2da edición. Caracas: UNA

Page 361: Fisica General i Una

376

Sustituyendo los valores en la ec. (1), se tiene el diferencial de presión en B. Así se tiene

5 2 2B B

1000p 75 10 (3,46) (1,7) p 70,5kPa.2

⎡ ⎤= × + − ⇒ =⎣ ⎦

EVALUAR la respuesta: La rapidez de flujo en los puntos A y B, son 3,34x10-3

m3/s y 1,62x10-3 m3/s, respectivamente. El diferencial de presión en B es de 70,5 kPa.

Ejercicio propuesto 10.4.- Fluye agua a través de un tubo que tiene una contracción a un gasto

uniforme. En un punto, donde la presión es 2,5 x 104 Pa, el diámetro es de 8,0 cm. En otro punto que está 0,5 m más arriba, la presión es de 1,5 x 104 Pa, y el diámetro es 4,0 cm.

a) Calcule la rapidez de flujo en las secciones inferior y superior del tubo.

b) Determine el gasto a través del tubo.

Ejemplo 10.5.-

Figura 10.13

La figura representa un barril de agua de lluvia sobre una plataforma de altura H = 4 m. Si se perfora un pequeño orificio en la pared lateral cerca del fondo del barril, se observa que la corriente resultante de agua choca contra el suelo a una distancia D = 2,5 m del recipiente. a) Calcule la velocidad del agua al salir por el orificio. b) Calcule la altura h del nivel del agua en el barril por encima de la pequeña abertura del fondo. DATOS: H = 4 m, D = 2,5 m

López Frontado, R. (2011). Física general I, Texto UNA. 2da edición. Caracas: UNA

Page 362: Fisica General i Una

377

SOLUCIÓN:

IDENTIFICAR los conceptos pertinentes: En este problema, consideramos el movimiento del chorro de agua en forma

similar al movimiento de una partícula bajo el efecto de la aceleración de gravedad, por lo tanto el chorro de agua describe una trayectoria parabólica cuando sale por el orificio en A. En el interior del tanque se aplica la ecuación de Bernoulli, para la altura del nivel desagua por encima de la abertura A.

PLANTEAR Y CALCULAR la resolución del problema: Para describir el movimiento del chorro de agua fuera de la abertura,

apliquemos la ecuación de una partícula bajo el efecto de una aceleración constante (la gravedad), tomando en cuenta las condicione iniciales correspondientes.

a) La velocidad con que sale el agua por el orificio en A, lo determinamos

usando la ecuación de movimiento, esto es, x =vAt (1), y

2gty H2

= −

Pero 2

Bgt 2Hy 0 H t2 g

= = − → =

Sustituyendo en la ecuación (1), se tiene 2Hx vg

= . Al despejar la

velocidad, se tiene A Ag 9,8 mv D 2,5 v 2,772H 2 4 s

→ = = ⇒ =×

b) Para hallar la altura h del nivel del agua en el barril, se usa la ec. de

Bernoulli, entre los puntos C y A. Así se tiene:

22CA

at at C

2 2A

vvp p gh pero v 0

2 2v (2,77)h h 0,39 m2g 2 9,8

ρρ ρ+ = + + =

→ = = ⇒ =×

EVALUAR los resultados del problema: Los valores de la velocidad del agua en el orificio es de 2,77 m/s y la altura del

nivel de agua es de 0,39 m.

López Frontado, R. (2011). Física general I, Texto UNA. 2da edición. Caracas: UNA

Page 363: Fisica General i Una

378

Ejercicio propuesto 10.5.- Un tanque sellado que contiene agua hasta una altura de 12,0 m contiene

también aire a una presión manométrica de 5,0 atm. Fluye agua a través de un agujero pequeño en el fondo. Calcule la rapidez de salida del agua.

Ejercicio propuesto 10.6.- El suministro de agua de un edificio se alimenta a través de una tubería

principal de 6,0 cm de diámetro. Se observa que una llave de 2,0 cm de diámetro localizada 2,0 m arriba de la tubería principal llena un recipiente de 25 litros en 30,0s.

a) ¿Cuál es la velocidad a la cual el agua sale de la llave?

b) ¿Cuál es la presión manométrica en la tubería principal de 6,0 cm?

(Suponga que la llave es la única “fuga “ en el edificio).

Ejercicio propuesto 10.7.- Se bombea agua desde el río Caroní hasta Ciudad Bolivar a través de una

tubería de 15,0 cm de diámetro. El río está a 564 m de altura y la ciudad a 2096 m. a) ¿Cuál es la presión mínima con que debe bombearse el agua para que

llegue a la población? b) Si se bombea 4500 m3 diarios, ¿cuál es la velocidad del agua en la

tubería?

Ejercicio propuesto 10.8.- Una cubeta cilíndrica, abierta por arriba, tiene 0,2 m de altura y 0,1 m de

diámetro. Se hace un agujero circular con área de 1,0 cm2 en el centro del fondo de la cubeta. Fluye agua hacia la cubeta de un tubo que está por encima de ella a razón de 1,30x10-4 m3/s. ¿A qué altura subirá en agua en la cubeta?

López Frontado, R. (2011). Física general I, Texto UNA. 2da edición. Caracas: UNA

Page 364: Fisica General i Una

379

Ejercicio propuesto 10.9.- Un líquido fluye por una tubería horizontal cuyo radio interior es de 2,52 cm.

La tubería se dobla hacia arriba hasta una altura de 11,5 m, donde se ensancha y se une con otra tubería horizontal de 6,14 cm de radio interior. Determine entonces:

¿Cuál debe ser el flujo volumétrico si la presión en las dos tuberías

horizontales es la misma?

RESUMEN:

La presión p en un fluido es la fuerza por unidad de área ejercida por

el fluido sobre una superficie: Fp=A

La unidad de la presión en el sistema de unidades internacional (SI) es 1 Pascal = 1 N/m2.

La presión en un fluido en reposo a una profundidad h está dada por:

op=p + gh! , donde po es la presión en h = 0.

El principio de Pascal dice que al aplicarse una presión a un

fluido encerrado, la presión se trasmite sin disminución a todos puntos del fluido y a todos los puntos sobre las paredes del recipiente.

Cuando un cuerpo se sumerge parcial o totalmente en un fluido, el

fluido ejerce sobre el cuerpo una fuerza hacia arriba, llamada fuerza de empuje (o fuerza flotante).

De acuerdo con el principio de Arquímedes, la fuerza de empuje es

igual al peso del fluido desalojado por cuerpo:

E fluidoF = gVρ La ecuación de continuidad de fluidos dice que el caudal (flujo de

volumen) que pasa por un tubo es constante. La ecuación de continuidad se expresa por:

1 1 2 2A v =A v =constante.

López Frontado, R. (2011). Física general I, Texto UNA. 2da edición. Caracas: UNA

Page 365: Fisica General i Una

380

La ecuación de Bernoulli establece que: la suma de la presión,

energía cinética por unidad de volumen, y la energía potencial gravitacional por unidad de volumen tiene el mismo valor en todos los puntos a lo largo de una línea de flujo. Esta ecuación se expresa matemáticamente por:

21p+ v + gy=constante

2! ! .

EJERCICIOS DE AUTOEVALUACIÓN

En su libreta de apuntes desarrolle los siguientes ejercicios. Luego de resolver

los ejercicios, compare sus respuestas con los presentados en la última página de la unidad. En caso de no acertar o de alguna duda pregunte a un compañero o al asesor de su centro local

10.1.- Encuentre la presión total, a 130 m bajo la superficie del océano. La

densidad del agua de mar es de 1,024 kg/m3 y la presión atmosférica al nivel del mar es de 1,013x105 Pa.

10.2.- Los miembros de una tripulación tratan de escapar de un submarino

averiado que está a una profundidad de 115 m bajo la superficie. ¿Cuánta fuerza deberán aplicar contra la escotilla que abre hacia fuera, la cual tiene 1,22 m por 0,69 m, para poder abrirla?

10.3.- Un bote que flota en agua dulce desaloja 35,6 kN de agua. Si el bote

estuviese flotando en agua salada de 1024 kg/m3, determine: a) ¿Qué peso de agua salada desalojaría este bote?; b) ¿Cambia el volumen del agua desalojada? Si cambia, ¿en cuánto?

10.4.- Las ventanas de un edificio de oficinas tienen 5,35, m por 6,26 m. En

un día tempestuoso, el aire sopla a razón de 35,0 m/s al pasar por una ventana en el piso 55. Calcúlese la fuerza neta sobre la ventana. La densidad del aire es de 1,23 kg/m3.

López Frontado, R. (2011). Física general I, Texto UNA. 2da edición. Caracas: UNA

Page 366: Fisica General i Una

381

RESPUESTA A LOS EJERCICIOS PROPUESTOS Estos resultados corresponden con los ejercicios propuestos en el desarrollo

de la unidad. Respuesta al ejercicio propuesto 10.1 DATOS: m = 180 g, m1 = 150 g, m2 = 144 g,

ρa = 1,0 g/cm3 = 1000 kg/m3. SOLUCIÓN:

Figura 10.14

Sean T = tensión de la cuerda, Fb = fuerza boyante (o de empuje), mg = peso del objeto, V= volumen del cuerpo. a) Aplicando la ley de Newton, en las dos pesadas, se tienen:

1 b1 b1 a

2 b2 b2 l

T +F -mg=0 (1), donde F = gVT +F -mg=0 (2), donde F = gV

ρρ

Sustituyendo en (1) y en (2) las fuerzas boyantes Fb2 y Fb2, se tiene:

a 1 l 2gV=mg-T (3), y gV=mg-T (4)ρ ρ

dividiendo (4) entre (3),

2 2a

a 1 1

3

mg T mg Tmg T mg T1000 (0,180 0,144) kg1200

0,180 0,150 m

ρ ρ ρρ

ρ ρ

⎛ ⎞− −= → = ⎜ ⎟− −⎝ ⎠

× −= ⇒ =−

!!

! !

b) De la ecuación (3), se tiene 1

a

mg TV

gρ−

= .

Entonces la densidad del objeto está dado por

aob ob 3

1

mg 1000 0,180 9,8 kg6000mg T (0,180 0,150) 9,8 mρρ ρ× ×= = ⇒ =

− − ×

López Frontado, R. (2011). Física general I, Texto UNA. 2da edición. Caracas: UNA

Page 367: Fisica General i Una

382

Respuesta al ejercicio propuesto 10.2 DATOS: m = 10 kg, ρag = 1000 kg / m3, h1 = 5 x10-2 m,

h2 = 17 x 10-2 m SOLUCIÓN:

Figura 10.15

a) La fuerza que ejerce el agua sobre la tapa superior es Fs = Ps A A = 10 x10 cm2 = 100 cm2 → A = 100 x10-4 m2

s at ag

5 -2S

5s

p =p +ρ gh

p =1,013×10 +1000×9,8×5×10

p =1,017×10 Pa.!

Fs = 1,0179 x105 x 10-2 ⇒ Fs = 1,0179 x103 N

La fuerza en la tapa inferior es Fi = Pi A

5 2i at ag 2

5i

p p gh 1,013 10 1000 9,8 17 10

p 1,0297 10 Pa.

ρ −= + = × + × × ×

→ = ×

Fi = 1,0297 x105 x10-2 ⇒ Fi = 1,0297 x103 N b) La lectura del dinamómetro está dada por la Fuerza Fd.

Del diagrama de cuerpo libre, se tiene

Fd - mg + Fb = 0 → Fd = mg - Fb,

Fb = 1000x 9,8x 0,12x0,1x0,1 = 11,8 N

Fd = 10x 9,8 - 11,8 ⇒ Fd = 86,24 N c) Para demostrar que la fuerza de empuje es igual a la diferencia a la

fuerza ejercida enn la superficie inferior y la fuerza ejercida en la superficie superior, se tiene que:

Fb’ = Fi - Fs = 1,0297x103 - 1,0179 x103 Fb’ = 11,8 N

López Frontado, R. (2011). Física general I, Texto UNA. 2da edición. Caracas: UNA

Page 368: Fisica General i Una

383

Lo cual nos demuestra que Fb’ = Fb, lqqd. Respuesta al ejercicio propuesto 10.3 DATOS: a = 22 m, b = 8,5 m, c = 2,0 m, ρagua = 1000 kg/m3

SOLUCIÓN: a) La presión absoluta en el fondo de la piscina está dada por,

5abs atm

5abs

p =p + gh=1,013×10 +1000×9,8×2,0

p =1,209×10 Pa

ρ⇒

La fuerza total en el fondo de la piscina es

5atm

5

p=p +ρgh=1,013×10 +1000×9,8×2,0

p=1,209×10 Pa!

b) La presión contra el lado de la piscina cerca del fondo es

5atm

5

p=p +ρgh=1,013×10 +1000×9,8×2,0

p=1,209×10 Pa!

Respuesta al ejercicio propuesto 10.4 DATOS: p1 = 2,5 x104 Pa, D1 = 8,0 x10-2 m, h = 0,5 m,

p2 = 1,5 x104 Pa, D2 = 4,0 x 10-2 m, ρagua = 1000 kg/m3, 1 Pa = 1 N/m2

SOLUCIÓN:

Figura 10.16

a) Aplicando la ecuación de Bernoulli, se tiene:

2 21 2

1 1 2 2ρv ρv

p + +ρgy =p + +ρgy (1)2 2

La ecuación de continuidad 1 1

1 1 2 2 22

A vA v A v v (2)

A= → =

López Frontado, R. (2011). Física general I, Texto UNA. 2da edición. Caracas: UNA

Page 369: Fisica General i Una

384

Reordenando la ec. (1) se tiene:

2 21 2 2 1 2 1 2 1

2 2 22 21 1 1 2 11 12 2

2 2

(v v ) (p p ) g(y y donde h y y ,2

A v A 2(p p )(v ) v (1 ) 2gh,

A A

ρ ρ

ρ

− = − + − = −

−− = − = +

42 1

21 2 2

122

1 2

2(p p ) 2(1,5 2,5) 102gh 2 9,8 0,51000v

A 0,081 1A 0,04m mv 0,26 y v 1,04s s

ρ− − ×+ + × ×

= =⎛ ⎞− − ⎜ ⎟⎝ ⎠

⇒ = =

b) El gasto de flujo = A1v1 = A2v2.

El gasto de flujo = 2(0,08) 0,26

4π⎡ ⎤

×⎢ ⎥⎣ ⎦

⇒ El gasto de flujo = 3

3 m1,307 10s

!"

Respuesta al ejercicio propuesto 10.5 DATOS: H = 12,0 m, pmanom = 5 atm(1,01x105) = 5,05x105 Pa SOLUCIÓN:

Figura 10.17

Aplicando la ec. de Bernoulli, se tiene:

2 21 2

o a

2 o a2

v vp + + gH=p +

2 22(p -p )

v = +2gH

ρ ρρ

ρ→

Donde v2 es la rapidez de salida del agua, y (po – pa), es la presión manométrica. Sustituyendo los valores, se tiene

522 23

2 5,05 10 mv = +2 9,8 12,0 v =35,29 s10

× × × × ⇒

López Frontado, R. (2011). Física general I, Texto UNA. 2da edición. Caracas: UNA

Page 370: Fisica General i Una

385

Respuesta al ejercicio propuesto 10.6 DATOS: D = 6,0 cm, d = 2,0 cm, h = 2,0 m, V = 25 lt, t = 30 s SOLUCIÓN: a) Para determinar la velocidad con que sale el agua de la llave, se tiene

La razón de flujo dV 1 dV=Av, de donde v= .dt A dt

2

2 dPero A= r =4

π π .

3

2 -2 2

4 V 4 25 10 mv= = v=2,65 t 30 sd (2 10 )! !

"# $% &'( )% &* +

b) Aplicando la ley de Bernoulli, se tiene 2 21 2

1 2v v

p + =p + gh+2 2

! !! ,

de donde se obtiene que la presión manométrica está dada por

2 22 1

1 2(v -v )

p -p = + gh (1)2

ρ ρ

De la ecuación de continuidad, se tiene A1v1 = A2 v2

2

2 21 12

1

A v 2 mv = = (2,65) v =0,295 A s6

ππ

⎛ ⎞→⎜ ⎟

⎝ ⎠

Sustituyendo los valores en la ecuación (1), se tiene

-3 2 2

2 1

2 1

1 10 (2,65) -(0,295)p -p = +1000 9,8 2

2 p -p =0,2307 atm

⎡ ⎤× ⎣ ⎦ × ×

Respuesta al ejercicio propuesto 10.7 DATOS: D = 15,0 cm, h1 = 564,0 m, h2 = 2096,0 m SOLUCIÓN:

Figura 10.18

a) La presión mínima con que debe bombardearse el agua, se obtiene aplicando la ecuación de Bernoulli, así se tiene que:

2 21 2

1 1 2 2v v

p + + gh =p + + gh2 2

ρ ρρ ρ

López Frontado, R. (2011). Física general I, Texto UNA. 2da edición. Caracas: UNA

Page 371: Fisica General i Una

386

Pero debido al principio de continuidad, Av = constante, y como A1 = A2, se

tiene que v1 = v2. Además se tiene también que Δh = h2 – h1, y p2 = patm = 1,01x105 Pa.

Así se tiene que: 5

1 27

1

p =p + g h=1,013 10 +1000 9,8(2096-564)

p =1,5225 10 Pa.

ρ Δ × ×

⇒ ×

b) Si se bombea 4500 m3 diarios, se tiene que:

2 2 22

2 22

4500 4500A v = , v =24 3600 24 3600A

4500 mv = v =2,95 s24 3600 (0,15)!

" "

#" "

Respuesta al ejercicio propuesto 10.8 DATOS: H = 0,2 m, D = 0,1 m, A2 = 1,0 cm2 = 1,0x10-4 m2,

dV/dt = 1,3x10-4 m3/s. SOLUCIÓN:

Figura 10.19

La ecuación de Bernoulli, aplicado en los puntos 1 y 2 de la (Figura 10.18), está dada por:

2 21 2

1 2 1 2 atm

21 2 2

v vp + + gh = p + , además p = p = p

2 2y v << v , v = 2gh

! !!

"

La razón de flujo:

2

2 2 2 22

-4 2-2

-4 2

dv 1 dv= A v = A 2gh, h =dt dt2gA

(1,3×10 )h = h = 8,62×10 m2×9,8×(1,0×10 )

⎛ ⎞⎜ ⎟⎝ ⎠

López Frontado, R. (2011). Física general I, Texto UNA. 2da edición. Caracas: UNA

Page 372: Fisica General i Una

387

Respuesta al ejercicio propuesto 10.9 DATOS: r1 = 2,52x10-2 m, r2 = 6,14x10-2 m, ΔH = H2 – H1 = 11,5 m. SOLUCIÓN:

Figura 10.20

El flujo volumétrico a través de una tubería está dada por el producto del área transversal por la velocidad del flujo a través de esa sección, esto es: Flujo volumétrico = Av, y como el líquido es el mismo, entonces este flujo volumétrico es constante, o sea que A1v1 = A2v2

De acuerdo con la ecuación de Bernoulli, se tiene

( ) ( )

2 21 2

1 1 2 2 1 2

2 21 2 2 1

v vp + + gH = p + + gH pero p = p ,

2 2

v - v = g H - H2

ρ ρρ ρ

ρ ρ∴

( ) ( )

21 1 2 2 1 2

1

2 22 2 22 22 2 2 1 2 2 12 2

1 1

AAdemás A v = A v v = v

A

A A v - v = 2g H -H v -1 = 2g H -H

A A

⎛ ⎞ ⎛ ⎞→⎜ ⎟ ⎜ ⎟

⎝ ⎠ ⎝ ⎠

( )2 12

2 22 2 4 42 2

42 2 41 1

2g H -H 2×9,8×11,5 225,4 mv = = = v = 2,57 sA r 6,14 -1-1 -1

2,52A rππ

→⎛ ⎞ ⎛ ⎞ ⎛ ⎞⎜ ⎟ ⎜ ⎟ ⎜ ⎟

⎝ ⎠⎝ ⎠ ⎝ ⎠

Entonces, el flujo volumétrico es:

3

-2 2 -22 2 2 2

mA v = (6,14×10 ) ×2,57 A v = 3,04×10 s

! "

López Frontado, R. (2011). Física general I, Texto UNA. 2da edición. Caracas: UNA

Page 373: Fisica General i Una

388

RESPUESTA A LOS EJERCICIOS DE AUTOEVALUACIÓN

Respuesta al ejercicio de autoevaluación 10.1 14,06x105 Pa. Respuesta al ejercicio de autoevaluación 10.2 1057x103 N. Respuesta al ejercicio de autoevaluación 10.3 a) 35,6 kN; b) Si, disminuye en 0,0851 m3. Respuesta al ejercicio de autoevaluación 10.4 25.23 kN.

López Frontado, R. (2011). Física general I, Texto UNA. 2da edición. Caracas: UNA

Page 374: Fisica General i Una

389

BIBLIOGRAFÍA

López F, R.A., (2008), Física General I, UNA, Venezuela. Resnick, Halliday, Krane, (1997), FÍSICA, Tomo I, 6ª Edic, Compañía Editorial

Continental S. A. de C. V. (CECSA), México. R. A. Serway, y Jewett Jr, J.W., (2005), FÍSICA para Ciencias e Ingeniería, Tomo I,

R. 6ª Edic,Thomson, México. Young H.D. y Freedman R.A.,(Sears y Zemansky), (2004), Física Universitaria,

Tomo I, 11ª Edic, Pearson, Addison Wesley, México. Alonso M y Finn E, (1976), FÍSICA, Volumen I, Fondo Educativo Interamericano, S.

A., Colombia. Fishbane, Gasiorowicz y Thornton, (1994), Física para Ciencias e Ingeniería,

volumen I, Prentice Hall Hispanoamericana, S. A., México.

Eisberg y Lerner, (1990), FÍSICA, Fundamento y Aplicaciones, Vol. I, McGraw Hill, México.

FÍSICA, Feynman, (1971), Vol. I, Fondo Educativo Interamericano S.A., Gartenhaus (1979), FÍSICA, Tomo I, Interamericana, México. Lahera, (1985), Introducción a la Didáctica de la Física, Editorial Vicens-Vives. Tipler, (1992), FÍSICA, Tomo I, Editorial Reverté S.A. D.C. Giancoli, (2002), FÍSICA para Universitarios, 3ª Edic, Pearson Educación,

México. Getty, Keller, Skove, (1991), FÍSICA, Clásica y Moderna, 1ª Edic, McGraw-

Hill/Interamericana de España S.A. España. F.J.Bueche, (1990), FÍSICA PARA ESTUDIANTES DE CIENCIAS E INGENIERÍA,

3ª Edic, McGraw Hill/Interamericana de México, S.A.

López Frontado, R. (2011). Física general I, Texto UNA. 2da edición. Caracas: UNA

Page 375: Fisica General i Una

391

APÉNDICE A Sistemas de medidas

Hasta hace cierto tiempo, en el mundo han predominado, dos sistemas de

medidas: el sistema inglés y el sistema métrico. Cada sistema tiene sus propias unidades o patrones de longitud, masa y tiempo; las cuales al ser seleccionadas, casi todas las demás cantidades pueden medirse en función de ellas. Por esta razón, a las unidades de longitud, masa y tiempo algunas veces se le da el nombre de unidades fundamentales. Sistema inglés

Utiliza el pie como la unidad de longitud, la libra como la unidad de peso (o de fuerza) y el segundo como la unidad de tiempo. Este sistema desarrollado en Inglaterra, se ha usado en los países de habla inglesa para muchos fines comerciales y de ingeniería. Sistema métrico

El sistema métrico fue creado por los franceses después de la Revolución Francesa (1791). Presenta una lógica que lo hace muy útil para el trabajo científico, y es empleado por los hombres de ciencia de todo el mundo. Éste sistema tiene la característica de usar sistema decimal, de tal modo que todas las unidades están relacionadas con las más pequeña o las más grandes al dividir entre 10 o multiplicar por esta cantidad. El sistema métrico se subdivide en dos sistemas de unidades: el cgs y el mks. En el sistema cgs, la unidad de longitud es centímetro (cm), la unidad de la masa es gramo (g) y la del tiempo es el segundo; mientras que en el sistema mks, la unidad de longitud es el metro (m), la de la masa es el kilogramo (kg) y la del tiempo es el segundo (s). Tabla A.1.- Conversión entre diferentes unidades.

UNIDAD DE

LONGITUD

Kilómetro (km)

Metro (m)

Centímetro (cm)

Pulgada Pie Milla

1 km = 1 = 1000 = 105 = 39.370,00 = 3280,83 = 0,62140 1 m = 10-3 = 1 = 100 = 39,37 = 3,28083 = 6,21x10-4

1 cm = 10-5 = 0,01 = 1 = 0,3937 = 0,032808 = 6,21x10-6

1 pulgada = 2,54x10-5 = 0,0254 = 2,54 = 1 = 0,08333 = 1,58x10-5

1 pie = 3,05x10-4 = 0,3048 = 30,48 = 12 = 1 = 1,89x10-4

1 milla = 1,60934 = 1609,34 = 160,934 = 63.360,00 = 5280,00 = 1

Debido al desarrollo creciente de la ciencia y de la cooperación internacional, se hizo necesario el uso uniforme de unidades y símbolos. Así se tiene que en el año 1960, durante la Conferencia Internacional sobre Pesas y Medidas, celebrada en París, se definieron las unidades SI (Sistema Internacional) y se les dio vigencia. Las unidades y símbolos del Sistema Internacional (SI) se presentan en la tabla A.2.

López Frontado, R. (2011). Física general I, Texto UNA. 2da edición. Caracas: UNA

Page 376: Fisica General i Una

392

Tabla A.2.- Unidades SI Cantidad Unidad Símbolo Longitud metro m Masa kilogramo kg Tiempo segundo s Fuerza newton N Energía joule J Corriente ampere A Temperatura kelvin K

Los números grandes y pequeños, se expresan por medio de ciertos prefijos, algunos de los cuales se muestran en la tabla A.3. Tabla A.3.- Prefijos para unidades SI

Símbolo Nombre Valor T tera 1012

G giga 109

M mega 106

k kilo 103

↓ ↓ ↓ d deci 10-1

c centi 10-2

m mili 10-3

µ micro 10-6

n nano 10-9

p pico 10-12

Tabla A.4.- Datos Físicos que a menudo se utilizan en Física

Datos Físicos Valores Rapidez de la luz en el vacío, c = 2,9979x108 m/s Rapidez del sonido (20ºC, 1 atm)

=343 m/s

Unidad astronómica (UA) (Distancia promedio Tierra-Sol)

= 1,50x1011 m

Distancia promedio Tierra-Luna = 3,84x108 m Radio promedio del Sol = 6,96x108 m Radio promedio de la Tierra = 6,37x106 m Radio promedio de la Luna = 1,74X106 m Radio promedio del átomo de hidrógeno

= 5x10-11 m

Masa del Sol = 1,99x1030 kg Masa de la Tierra = 5,98x1024 kg Masa de la Luna = 7,36x1022 kg Masa del protón, mp = 1,6726x10-27 kg Masa del neutrón, mn = 1,6749x10-27 kg Masa del electrón, me = 9,1x10-31 kg Presión atmosférica normal = 1,01x105 Pa Carga fundamental e = 1,602x10-19 C Constante gravitacional, G = 6,67x10-11 m3/kg.s2

Constante de Plank, h = 6,62608x10-34 j.s = 4,13567x10-15 eV.s

Número de Avogadro, NA = 6,022x1023 /mol

López Frontado, R. (2011). Física general I, Texto UNA. 2da edición. Caracas: UNA

Page 377: Fisica General i Una

393

En la tabla A.5 se presentan algunos factores de conversión. Tabla A.5.- Factores de conversión

Longitud y volumen Presión 1 pulgada = 2,54 cm (exacto) 1 Pa = 1 N/m2

1 pie = 0,3048 m (exacto) 1 atm = 1,01325x105 Pa 1 m = 39,37 pulgada (in) 1 lb/in2 = 6895 Pa 1 mi = 1609,34 m 1 litro = 103 cm3 = 10-3 m3 Energía y potencia 1 cal = 4,184 J Tiempo 1 kWh= 3,6x106 J 1 año = 365 días = 3,1558x107 s 1 eV = 1,602x10-19 J 1 d = 86.400,00 s 1 u = 931,5 MeV 1 h = 3600 s 1 hp = 746 W Masa Rapidez 1 kg = 1000 g 1 m/s = 3,6 km/h = 2,24 mi/h 1 kg peso = 2,205 lb 1 km/h = 0,621 mi/h 1 uma = 1,6605x10-27 kg Fuerza 1 lb = 4,448 N

López Frontado, R. (2011). Física general I, Texto UNA. 2da edición. Caracas: UNA

Page 378: Fisica General i Una

395

APÉNDICE B Como trabajar con Unidades en Física

Para sumar y restar varias cantidades, todas deben tener la misma unidad, de tal forma que la suma o la resta debe poseer la misma unidad de los sumandos. Para ilustrar esto, veamos los siguientes ejemplos: Ejemplo B.1 4 m + 10 km + 210 cm = 4 m + 10 000 m + 2,10 m

10 006,10 m⇒

Ejemplo B.2

20 s + 30 min + 3,0 h = 20 s + 1800 s + 108 000 s 109 820 s⇒

Ejemplo B.3

(3 min)+(6 h)+(12 s)=21 s (Incorrecto) Para multiplicar o dividir varias cantidades, todas deben estar en el mismo sistema de unidad. Veamos así los siguientes ejemplos: Ejemplo B.4 (3 N) (12 m) 36 N.m× ⇒ Ejemplo B.5

km(3 km)/(2 h) 1,5 h

m(3 km)/(2 h)=(3000 m)/(7200 s) 0,4167 s

Ejemplo B.6

(3m) (20 cm) (3 mm)=(3 m) (0,2 m) (0,003 m) 0,0018 m

! ! ! !

" Ejemplo B.7

3 km(3 km)/(2 km)= 1,52 km

! Ejemplo B.8

22

m.cm2(9,8 m/s ) (300 cm)=5880,0 (Incorrecto)s

×

Para usar números grandes y pequeños es conveniente utilizar una abreviatura matemática, mejor dicho emplear una notación científica. Por ejemplo: Ejemplo B.9

5

-4

5 -4 1

400 000,0=4,0 100,0005=5,0 10

4,0 10 5,0 10 20,0 10 =200

!

!

" ! ! ! # !

López Frontado, R. (2011). Física general I, Texto UNA. 2da edición. Caracas: UNA

Page 379: Fisica General i Una

397

GLOSARIO Aceleración. Rapidez de cambio de la velocidad de un cuerpo en un tiempo. El cambio de la velocidad puede ser de su magnitud, de su dirección o en ambas. Año luz. Distancia recorrida por la luz en el vacío durante un año. Adhesión. Atracción molecular entre dos superficies diferentes en contacto. Átomo. La partícula más pequeña de un elemento identificado con dicho elemento. El átomo está compuesto por protones y neutrones en un núcleo rodeado por electrones. Amplitud. En un cuerpo que describe un movimiento armónico simple, la amplitud es el desplazamiento máximo hacia cualquiera de los dos lados de su posición de equilibrio. Angstrom (ºA). Unidad de longitud igual a 10-10 m, Los átomos tienen radio de 1 a 2 ºA. Avogadro, principio de Volúmenes iguales de todos los gases, a la misma temperatura y presión, contiene el mismo número de moléculas, e igual a 6,025x1023 en un mol. Barómetro Se denomina así cualquier aparato que mida la presión atmosférica. Bernoulli, principio de La presión en un fluido es inversamente proporcional a la velocidad de éste. Boyle, ley de Para una masa de gas confinado, el producto de la presión y el volumen es una constante, mientras no se cambie la temperatura. Brazo de palanca Se denomina así la distancia perpendicular entre un eje y la línea de acción de una fuerza, cuando dicha fuerza tiende a producir un movimiento de rotación alrededor de dicho eje. Caída libre Movimiento de un cuerpo en el que sólo influye la fuerza gravitacional.

López Frontado, R. (2011). Física general I, Texto UNA. 2da edición. Caracas: UNA

Page 380: Fisica General i Una

398

Calor Energía térmica que fluye de un cuerpo que tiene una temperatura mayor a uno que tiene temperatura menor. El calor se mide en calorías o joules. Calor específico Se define como la cantidad de calor por unidad de masa necesario para elevar la temperatura de una sustancia en un grado centígrado, o equivalente en un kelvin. Caloría Cantidad de energía requerida para elevar la temperatura de un gramo de agua en un grado Celsius. Campo gravitatorio Espacio que rodea a un cuerpo masivo en el que se ejerce una fuerza de atracción sobre otro cuerpo masivo. Cantidad escalar Cantidad que se especifica completamente por su magnitud. Como ejemplo podemos mencionar la masa de un cuerpo, el volumen de un cuerpo, la temperatura. Cantidad vectorial Cantidad que se especifica completamente cuando se da su magnitud y su dirección. Por ejemplo la velocidad, la fuerza, la aceleración, el campo eléctrico, el campo magnético.. Centro de masa Posición promedio de la masa de un cuerpo. Colisión elástica Colisión en la que no se disipa energía. Densidad Masa de una sustancia por unidad de volumen. Energía Capacidad que tiene un sistema para producir trabajo. Energía cinética Energía que se debe al movimiento, de un cuerpo o sistema. Energía potencial Energía almacenada que posee un cuerpo en virtud de su posición respecto a otros cuerpos.

López Frontado, R. (2011). Física general I, Texto UNA. 2da edición. Caracas: UNA

Page 381: Fisica General i Una

399

Equilibrio Es aquel estado en el cual un cuerpo no experimenta el efecto de fuerza alguna ni de momento torsional netos. El cuerpo en equilibrio puede estar en reposo o en movimiento uniforme. Equivalencia masa-energía Relación entre la masa y la energía expresada por la ecuación E=mc2. Frecuencia Para un cuerpo cuyo movimiento es armónico simple, la frecuencia es el número de vibraciones que realiza por unidad de tiempo. Fricción Es la fuerza que se opone al movimiento de un cuerpo sobre otro, con el cual está en contacto. Fuerza Es la interacción entre dos cuerpos y que hace que adquieran una aceleración. Se mide en newtons. Fuerza centrípeta Fuerza dirigida hacia el centro de curvatura y que hace que el objeto describa una trayectoria circular. Fuerza de empuje Fuerza que ejerce un fluido sobre un objeto sumergido en él. Gas Estado de la materia, cuya característica es que no tiene forma definida y sus moléculas ocupan todo el espacio del que disponen. Hertz (Hz) Unidad de frecuencia, donde un hertz es un ciclo (o vibración) por segundo. Hoyo negro Configuración de una estrella masiva que ha pasado por un colapso gravitacional, donde la gravitación es tan intensa que ni su propia luz puede escapar. Inercia Resistencia aparente opuesta por un cuerpo a los cambios en su estado de movimiento. Joule (J) Unidad en el sistema internacional (SI) de trabajo o de energía en cualquiera de sus formas.

López Frontado, R. (2011). Física general I, Texto UNA. 2da edición. Caracas: UNA

Page 382: Fisica General i Una

400

Kepler, leyes del movimiento planetario de Las cuales establecen: Primera ley, Cada planeta se mueve en una órbita elíptica, teniendo al Sol como uno de sus focos. Segunda ley, La línea recta que va desde el Sol a cualquiera de los planetas, barre áreas de espacios iguales en intervalos de tiempos iguales. Tercera ley, Los cuadrados de los tiempos de revolución (en años) de los planetas son proporcionales a los cubos de sus distancias promedio al Sol. Kilogramo (kg) Unidad de la masa, la cual se define como la cantidad de masa a la que una fuerza de un newton proporciona una aceleración de un metro/segundo2. Ley de gravitación universal La cual establece que cada uno de los cuerpos en el universo atrae a cada uno de los demás cuerpos, con una fuerza que, para los dos cuerpos, es proporcional a sus masas e inversamente proporcional al cuadrado de la distancia que los separa. Líquido Es el estado de la materia que posee un volumen definido, pero que no tiene forma definida sino que toma la del recipiente que lo contiene. Marco de referencia Punto de referencia con respecto al cual pueden describirse la posición y el movimiento de un cuerpo. Marco de referencia inercial Marco de referencia no acelerado en el cual se cumplen con exactitud las leyes de Newton. Masa Cantidad de materia de un cuerpo; medida de la inercia o resistencia que un cuerpo opone a cualquier esfuerzo que se haga para ponerlo en movimiento, o cambiar de cualquier manera su estado de movimiento. Más específicamente, es la medida de la cantidad de energía contenida en él. Momento torsional Es el producto de la fuerza por la distancia del brazo de palanca, lo cual tiende a producir rotación. Movimiento armónico simple Movimiento vibratorio o periódico, en el que la fuerza que actúa sobre el cuerpo vibrante, es proporcional a su desplazamiento respecto a su posición central de equilibrio, y actúa hacia esa posición. Newton (N) Unidad en el sistema SI de la fuerza que imparte a una masa de un kilogramo una aceleración de un metro por segundo al cuadrado.

López Frontado, R. (2011). Física general I, Texto UNA. 2da edición. Caracas: UNA

Page 383: Fisica General i Una

401

Parábola Trayectoria curva seguida por un proyectil que sólo recibe la influencia de la gravedad. Peso Fuerza de atracción gravitacional ejercida sobre un cuerpo como se observa sobre la Tierra en rotación o en situaciones semejantes. Planck, constante de Constante fundamental h, que relaciona la energía y la frecuencia de los cuantos de luz, y es igual a 6,6x10-34 J.s Potencia Razón de cambio del trabajo realizado; o medida de la eficiencia de un trabajo realizado. Presión Razón de magnitud de una fuerza entre área sobre la que se distribuye. Principio de correspondencia Establece que una teoría nueva es válida siempre y cuando, al superponerla con la teoría anterior, concuerda con los resultados verificados de ésta. Principio de equivalencia Las observaciones locales realizadas en un marco de referencia acelerado no pueden distinguirse de las que se realiza en un campo gravitacional newtoniano. Principio de flotación Un cuerpo flotante desplaza un peso de fluido igual a su propio peso. Proyectil Cualquier cuerpo que se lanza por la acción de alguna fuerza y que continúa en movimiento debido a su propia inercia. Rapidez Razón respecto al tiempo con que un cuerpo cubre una distancia en movimiento. Resultante Es la suma geométrica de dos o más vectores. Sólido Estado de la materia que tienen volumen y forma definidos. Vector Representación en forma de flecha de una cantidad vectorial.

López Frontado, R. (2011). Física general I, Texto UNA. 2da edición. Caracas: UNA

Page 384: Fisica General i Una

402

Velocidad Especificación de la rapidez con que se mueve un cuerpo y la dirección de su movimiento. Es una cantidad vectorial. Vibración forzada Producción de vibraciones en un cuerpo por medio de una fuerza vibrante. Volumen Cantidad de espacio que ocupa un cuerpo. Watt (W) Unidad SI de potencia, y es igual a 1J/s.

López Frontado, R. (2011). Física general I, Texto UNA. 2da edición. Caracas: UNA